CA Inter Income Tax AY 2020-21 Material by RSN Sir

You might also like

Download as pdf or txt
Download as pdf or txt
You are on page 1of 404

CA Intermediate (New & Old

Syllabus)
Paper 4
Taxation

Part-A Income Tax Law


AY 2020-21

By
CA. R. SOUMYANARAYANAN
FCA, GRAD CWA.
Preface

Dear friends

It is a great pleasure in presenting this Book on Direct Tax Laws &


International. This Book is compiled from ICAI study material,
Revision Test Papers and past examination suggested answers. It
can be used by students as preparatory tool for the theory part in
CA Final Direct Tax Laws & International Taxation.

I am thankful to my friend Prof. S. S. Rajagopal FCA, Grad CMA, for


helping me in compiling this work book.

I wish the students all the best for their future endeavors. Students
can approach me for any queries on the subject at the mail id given
below.

R. Soumyanarayanan FCA, Grad CMA

Chennai,

EMAIL: krsn5976@yahoo.co.in

reachssr@yahoo.co.in
A. Components of Paper-4 & their weightage in the examination:

SN Components Marks Handled by


1 Income Tax Law (Section-A) 60 RSN
2 Indirect Taxes (Section-B) 40 Tarun Raj

B. Relevant Income Tax Act for May 2020 & Nov 2020 Examinations:

Exam Relevant Act Relevant PY &


AY
May Income Tax Act, 1961 as amended by the Finance Act, PY 19-20
2020 2019
Nov 2020 AY 20-21

C. Study segments in Section-A:

SN Study segments
1 Basic concepts
2 Residential status and scope of total income
3 Income which do not form part of total income
4 Heads of income – (i) Salaries (ii) IFHP (iii) PGBP (iv) Capital gains (v) IFOS
5 Clubbing provisions
6 Aggregation of income, set-off, or carry forward and set-off of losses
7 Deductions from gross total income
8 Computation of total income and tax liability of individuals
9 Advance tax, TDS and introduction to TCS
10 Provisions for filing return of income and self-assessment

D. About our subject:


1 Ever changing
2 Voluminous
3 Involves interpretation
4 Requires memory skills & application abilities

E. Approach required for success:


1 Section by section approach should be followed.
2 Thorough understanding and visualisation of the each & every section is to be obtained.
3 Our understanding of the sections should be tested by answering computational problems
and case study type questions.
4 Time bound writing practice should be there because even for knowledgeable students, time
proves to be a limiting factor in examination.
5 Systematic revision is required because human brain tends to forget things as time passes.
This is even more pertinent in our subject because so many provisions are to be remembered
for answering case questions tested in examination.
6 Pick and Choose approach is to be avoided. Efforts should be made for covering the full
syllabus.
F. Sources for preparation:

1 Class room discussion supported by class room notes and handouts given by RSN.
2 Latest ICAI study material (Module-1 to 3) [AY 2020-21].
3 Latest practice manual prepared by ICAI.

G. What we expect from you?

1 Do not skip the lectures, since no source can substitute the class room discussion.
2 Report to the class in time.
3 Maintain silence and discipline in the class room.
4 Stay focused when lectures are on.
5 Have three note books - one for writing class room notes, another for solving class
room problems and the last for solving assignment questions.
6 For every class, bring calculator, note books and the relevant handouts.
7 Do not demand break during class hours.
8 Get all your doubts clarified in the time allotted for it.
9 Do systematic revision of provisions covered in the class.
10 Practice independently problems solved in the class.
11 Sincerely solve the assignment questions given by us.
12 Take the tests conducted seriously.
Income Tax Law
Index
S.No Chapter Name Page
No
1 Basic concepts 1-27

2 Residential Status 28-40

3 Exemptions 41-46

4 Income from House Property 47-62

5 Income from Salaries 63-110

6 Income from Capital Gains 111-205

7 Income from Other Sources 206-217

8 Clubbing of Income 218-235

9 Set off & Carry Forward 236-250

10 Deductions under Chapter VI A 251-277

11 Income from Profits and Gains of Business or Profession 278-328

12 Return of Income 329-342

13 Advance Tax 343-345

14 Tax Collected at Source 346-350

15 Tax Deducted at Source 351-391

16 Alternative Minimum Tax (ATM) 392-394


Chapter-1: Basic concepts

Contents of this chapter:

1 Meaning of tax
2 Purpose of levy of tax
3 Basis for levy of tax
4 Classification of tax
5 Constitutional back ground behind Income Tax law
6 Administering authorities for tax laws
7 Components of Income Tax law
8 Charging section (S. 4)
9 Concept of PY & AY
10 Structure of Annual Finance Act
11 Meaning of ‘Person’
12 A brief introduction to different categories of persons
13 Tax rates applicable for different categories of persons for the AY 20-21 (as per
Finance Act 2019)
14 Applicability of surcharge and Health and education cess.
15 Rebate U/s 87A
16 Rounding off of Total income and tax
17 Concept of marginal relief & its necessity
18 Computation of marginal relief for different categories of persons.
19 Concept of income in common parlance – Capital receipts Vs Revenue receipts
20 Concept of income under the Income Tax Act
21 Basic principles clarifying the concept of income – (including irrelevance of illegality
of income, real income theory, application of income, diversion of income by over-
riding title, concept of mutuality…..).
22 Heads of income – brief introduction.
23 Relevance of method of accounting & ICDS
24 Clubbing, adjustment of losses & Deductions under Chapter-VIA – a brief introduction
25 Applicability special rates in case of specified income.
26 Scheme of partial integration
27 Issues relating to agricultural income

Additional discussion:

1 Relevance of definitions in taxation laws


2 Meaning of Section, Subsection, Clause, Sub-clause, Proviso and Explanation.
3 Meaning of ‘India’
4 Meaning of ‘assessee’
5 Income from undisclosed sources
6 Cases when income of a PY will be assessed in the PY itself

MT Educare – CA Inter Income Tax Classes – R. SOUMYANARAYANAN.FCA. GRAD CWA. Page-1


Chapter-1: Basic concepts

1. Fundamentals:

1 Meaning of ‘tax’ It is a compulsory fee charged by a Government on its subjects


2 Purpose of levy of Taxes constituted basic source of revenue to the Government.
tax Revenue raised is utilised for meeting the expenses of
Government in (a) provision of education & health care (b)
meeting the defence requirements (c) creation of infrastructure
facilities like roads, railways, dams, bridges, highways, airport,
ports, power, water, sanitation etc (d) carrying out various social
welfare measures and schemes.
3 Basis for levy of Income, sales, purchase, provision of service, manufacture,
tax import/export etc.
4 Classification of Taxes could be classified into (a) direct taxes (b) indirect taxes.
taxes
5 Direct tax If tax is levied directly on the income or wealth of a person, then it
is a direct tax. The incidence and impact is on the same person
(who earns income or possesses wealth)
6 Examples of direct (a) Income tax (b) Tax on undisclosed foreign income and assets
taxes (levied under Black money and imposition of tax (undisclosed
foreign income and assets) Act 2015.
7 Indirect tax If tax is levied on the price of good or service, then it is an indirect
tax. In case of indirect taxes, the person paying taxes passes on
the burden to another person.
8 Examples of (a) customs duty (b) GST
indirect taxes
9 Constitutional Article 265 of the Constitution provides that no tax could be levied
background or collected without the authority of law.
Income Tax Act Entry 82 of list I of 7th Schedule to Article 246 of the constitution
empowers the CG to levy taxes on income (other than agricultural
income).
In exercise of such powers, the CG has enacted the Income Tax Act
1961 for levying taxes on income.
Agricultural income is excluded from the purview, since it is a
state subject. Vide Entry-46 of list II.
Article 270 of the constitution provides for sharing of income tax
collection with the SG.
10 Administering For direct tax laws – CBDT
authority for tax For indirect tax laws – CBEC
laws

2. Components of Income Tax laws:


Components of income tax law

The Income Tax The Income Tax Finance Acts Circulars/Notification Court decisions
Act, 1961 Rules, 1962 from CBDT

MT Educare – CA Inter Income Tax Classes – R. SOUMYANARAYANAN.FCA. GRAD CWA. Page-2


Chapter-1: Basic concepts

SN Components Details
1 The Income tax Levy of income tax is governed by this Act. This Act came into force
Act, 1961 on 1st April 1962. It contains 298 sections and XIV schedules.
It contains provisions for determination of taxable income,
determination of tax liability, procedure for assessment, collection &
recovery, appeals, penalties and prosecutions.
It also lays down the powers and duties of various income-tax
authorities.
Income tax Act is amended through the annual finance Act and also
through the amendment Acts and ordinances.
2 Finance Act The Provisions of income tax Act are amended every year through
the Finance Act.
The Finance Act consists of tax rates applicable for the assessment
year, TDS rates and Advance tax rates (applicable for the current
financial year).
It also lays down the manner of computation of tax on non-
agricultural income where the assessee has got agricultural income.
3 The Income tax For carrying out the purposes this Act, CBDT is empowered U/s 295
rules, 1962 to frame rules. These rules are collectively called the Income tax
rules.
These Rules deal with procedural aspects and valuation aspects.
These rules, being subordinate legislation, can’t supersede the
provisions of the Act.
Along with the Act, these rules should also be studied.
4 Circulars These circulars are issued by CBDT (in exercise of powers conferred
U/s 119) from time to time to deal with certain specific problems
and to clarify doubts regarding the scope and meaning of the
provisions.
These circulars are issued for the guidance of officers and assessees.
Moreover, in order to avoid stringency of law, the CBDT, in exercise
of its powers U/s 119, may relax the provisions of the Act through its
circulars.
The department is bound by the circulars. However, such circulars
are not binding on assessees. But they can take advantage of
beneficial circulars.
5 Notifications Notifications are issued by the CG to give effect to the provisions of
the Act. The CBDT is also empowered to make and amend rules for
the purposes of the Act by issue of notifications.
6 Case laws It is not possible for parliament to conceive and provide for all
possible issues that may arise in the implementation of the Act. So
judiciary will hear the disputes between the assessees and the
department and give decision on various issues.
Supreme Court decisions are binding on all since it is regarded as law
of the land.
High court decisions will apply in the respective states in which such
High courts have jurisdiction.

MT Educare – CA Inter Income Tax Classes – R. SOUMYANARAYANAN.FCA. GRAD CWA. Page-3


Chapter-1: Basic concepts

3. Charging section – S. 4:

There shall be charged income tax for every AY in respect of the total income of the PY of
every person at such rate(s) as may be specified in the annual Finance Act as applicable to
the relevant AY.

4. Important aspects of charging section:

A. Previous year & Assessment year:

Previous It is a period during which income is earned.


year Usually it is a financial year.
But where in a financial year, a new business is commenced or a new source
of income comes into existence, it starts from the date of commencement of
business or date on which new source of income comes into existence and
ends with the last day of that financial year.
Different sources of income can have different previous years.
Assessment It is a financial year immediately succeeding the previous year.
year Unlike previous year, it is always a financial year.
Income earned during the previous year is assessed to tax in the assessment
year.

B. Parameters for determination of tax liability for the AY:

1 Total income of the corresponding PY


2 Tax rates applicable for the AY (specified in the Annual Finance Act)

C. Provisions relevant for computation of taxable income of the PY:

1 Generally, the total income of the PY shall be computed in accordance with the
provisions of Income tax Act as they stand on 1st April of the assessment year.
2 Any amendment thereafter is not relevant for computation of total income of the PY.

D. An overview of relevant parts of First schedule of Finance Act:

Part Content Illustration


I It contains tax rates applicable Part I of the first schedule to the Finance Act 2019
for different categories of gives the tax rates for different categories of
persons for the current AY. persons for the AY 19-20.
III It contains tax rates applicable Part III of the first schedule to the Finance Act
for different categories of 2019 gives tax rates for different categories of
persons for the next AY. (For persons for the AY 20-21. (For payment of
payment of advance tax advance tax during the FY 19-20).
purposes)

MT Educare – CA Inter Income Tax Classes – R. SOUMYANARAYANAN.FCA. GRAD CWA. Page-4


Chapter-1: Basic concepts

Note: Part III of first schedule of a particular Finance Act, generally, becomes Part I of first
schedule of the subsequent Finance Act. For example, Part III of first schedule of Finance Act
2019 becomes Part I of First Schedule of Finance Act 2020.

E. Definition of Person – S. 2 (31):

Person includes

Company Every other artificial


Individual HUF Firm AOP/BOI Local authority juridicial person

Individual:

1. Individual means only natural persons.


2 It includes both males and females.
3 Individual includes a minor/person of unsound mind (But assessment is made U/s
S.161 (1) on guardian or manager)
4 Individual includes even a deceased person (But the assessment would be made
on the legal representative U/s 159)

HUF:

1 Family means group of persons related to each other through blood relationship,
marital tie or adoption.
2 If all the members of a family are Hindus, such family is a Hindu family.
3 A Hindu family having some common property remaining undivided is called HUF.
4 In reality, a family is not distinct and separate from its members. It has no separate
existence.
5 However, for the purposes of this Act, HUF is regarded as distinct and separate from its
members. This is because, the income arising from common undivided property can’t
be assessed to tax in the hands of members severally until partition. Until partition,
share of each member is definite but not ascertainable. Therefore, it has to be assessed
to tax jointly.
6 HUF is represented by karta, senior most co-parcener.
7 In the hands of karta, there will be two assessments in two different capacities. His
personal income will be assessed to tax in his hands in individual capacity, whereas as
the income of his HUF is assessed to tax in representative capacity.
8 Members of HUF are divided into two categories – (a) Co-parceners (b) Non-
coparceners.
9 Co-parceners have right to demand partition and get a share in the common property.
However, the non-coparceners can’t demand partition. They will get a share if partition
takes place.

MT Educare – CA Inter Income Tax Classes – R. SOUMYANARAYANAN.FCA. GRAD CWA. Page-5


Chapter-1: Basic concepts

10 Daughter (married or unmarried) of a coparcener is also a coparcener. However, the


wife or daughter-i n-law of a coparcener.
11 For the purpose of this Act, Jain undivided family and Sikh undivided family would be
assessed only as HUF.
12 Any sum received by a member from his HUF out of income accruing or arising from
common properties is exempt in the hands of such member. [S. 10 (2)]. S. 10 (2)
ensures that the income which got taxed in the hands of HUF doesn’t get taxed in the
hands of the member again.

Firm:

1 As per S. 2 (23), the terms Partnership, Partner and Firm are to have the same meaning
as assigned in the Indian Partnership Act.
2 Partnership has been defined under the Indian partnership Act to mean the
relationship between persons who have agreed to share the profits of a business
carried on by all or any of them acting for all.
3 The persons between whom partnership subsists are called as partners.
4 The partners are collectively referred to as Firm. It’s a compendious way of describing
partners.
5 Mere co-ownership does not amount to partnership. Common title in a property
coupled with production of income followed by sharing thereof is not sufficient.
Carrying on of a business is a pre-requisite.
6 There is mutual agency among partners. Even unauthorized or independent acts of a
partner bind others.
7 S. 2 (31) has created a fiction recognizing firm as a distinct tax entity i.e.; distinct and
separate from partners, since profits arising from a single profit-yielding venture
carried on jointly shall be taxed jointly and not severally.
8 Thus, profits arising from business or profession carried on by the firm (i.e. partners
jointly) and the income from the assets of the firm (in which partners have joint
interest) are taxed in the hands of the firm.
9 Personal income of partners gets taxed in their hands respectively.
10 Share income of a partner from his firm is exempt in his hands. [S. 10 (2A)]. This is to
avoid over-lapping taxation.

Points requiring attention:

1 As per the Partnership Act, partnership agreement may be written or oral. However, S.
184 requires an instrument evidencing partnership, in the absence of which a
disadvantage is caused to the firm. (i.e. disallowance of interest and remuneration paid
to partners is made in the hands of the firm).
2 As per the Partnership Act, there is no necessity that the partners should have a definite
profit sharing ratio. In the absence of understanding among partners regarding the profit
sharing ratio, it is regarded as equal. However, S. 184 requires mentioning of individual
share of each partner in the instrument evidencing the partnership.
3 As per the Partnership Act, a minor can’t be a partner. But he could be admitted to the
benefits of partnership. However, S. 2 (23) defines ‘partner’ to include even a minor.

MT Educare – CA Inter Income Tax Classes – R. SOUMYANARAYANAN.FCA. GRAD CWA. Page-6


Chapter-1: Basic concepts

AOP:

1 When two or more persons come together voluntarily to carry out some income
generating activity and to share the income arising therefrom (without constituting
partnership), such association is called AOP.
2 Essentials of AOP: (a) Meeting of minds (b) common design for making income (c) joint
enterprise (d) joint act of management (e) joint & several liability (f) sharing of income.
3 Mere co-ownership does not make the co-owners constitute an AOP. There should be
income generating activity.
4 AOP is a distinct tax entity. Logic behind the fiction treating AOP as distinct and separate
from its members is same as that in case of a firm.
5 Unlike in case of firm, there is no restriction on number of members which an AOP can
have.
6 Even a firm can be a member of AOP. However, a firm can’t be a partner in a firm.
7 The individual share of member of AOP can be kept indeterminate (that is left open so
that it could be determined at the end of each year).

BOI:

1 Concept of BOI could be understood with the help of an illustration.


2 Mrs. X, after the demise of her husband, continued the business on her behalf and on
behalf of her three minor children.
3 In the business aforesaid, Mrs. X and her children have unity of interest. Income from a
single profit yielding venture shall be assessed to tax jointly and not severally. They can’t
be treated as AOP since there is no common design, joint enterprise or joint act of
management.
4 By operation of law, this group becomes a separate tax entity. This is called BOI.

Note: Efforts made in distinguishing BOI from AOP shall go waste, since exactly the same set
of provisions of the Act are applicable to both.

Company:

1 Company is a body corporate.


2 It has separate legal existence from its shareholders in the eyes of law.
3 It has perpetual succession.
4 Its liability is, generally, limited.
5 It can own property in its own name.
6 It can sue and get sued in its own name.
7 It functions through human agencies viz. (a) BOD (b) shareholders.
8 BOD is vested with day to day administration. Decisions are taken by BOD through
meeting and circular resolution.
9 Major strategic decisions are made by the shareholders in AGM and EGM.
10 S. 2 (17) defines company to include (a) Indian company (b) Body corporate
incorporated under laws of a country outside India.

MT Educare – CA Inter Income Tax Classes – R. SOUMYANARAYANAN.FCA. GRAD CWA. Page-7


Chapter-1: Basic concepts

11 S. 2 (26) defines Indian company to mean a company formed and registered under any
law relating to companies prevalent in India and whose registered office is situated in
India. It also includes statutory corporations (like LIC, GIC, IDBI etc)
12 For the purpose of this Act, companies are classified into (a) domestic companies (b)
foreign companies.
13 A company which is not a domestic company is foreign company. [S. 2 (23A)].
14 All Indian companies are domestic companies. Any other company which has made
prescribed arrangement for declaring dividend out of income chargeable to tax in India
is also regarded as domestic company. [S. 2 (22A)].

Company in which public is substantially interested (S.2 (18)):

Following companies are regarded as companies in which public is substantially interested:

1 A company owned by Government or RBI.


2 A company in which not less than 40% of the shares are held by Government or RBI or a
corporation owned by RBI (either singly or jointly).
3 A company registered U/s 8 of companies Act.
4 A company whose principal business is accepting deposits from its members and which
is declared by Government U/s 620A of the companies Act to be Nidhi or Mutual Benefit
society.
5 A company in which at least 50% or more equity shares have been held by one or more
cooperative societies throughout the PY.
6 A public company whose equity shares are listed on a recognized stock exchange as on
the last day of the PY.
7 A company in which equity shares carrying not less than 50% of the voting power are
held throughout the PY by Government, a statutory corporation, a company in which
public is substantially interested or its wholly owned subsidiary.

LLP:
1 LLP is a hybrid form of business organisation. It partakes the characters of a company as
well as traditional partnership firm.
2 Since it is a body corporate it enjoys the advantages of a corporate form of business
organisation (such as separate legal existence, perpetual succession, limited liability etc).
However, at the same time, it has flexibility in operations which a traditional partnership
firm has.
3 Unlike for companies, there are no statutory provisions relating to meetings, registration of
charges, remuneration to partners, maintenance of various registers and records etc.
4 Liability of LLP is limited to the extent of assets possessed by it. Liability of partner of LLP is
limited to the extent of agreed contribution in the LLP agreement (except in case of fraud).
5 There is no restriction on number of partners which an LLP can have.
6 LLP can follow either cash system of accounting or mercantile system.
7 LLP formed and registered under LLP Act 2008 (Indian LLP) = Firm (S. 2 (23).
All the provisions which are applicable to a traditional partnership firm shall equally apply
to Indian LLP.
8 Foreign LLP = company [S. 2 (17)].

MT Educare – CA Inter Income Tax Classes – R. SOUMYANARAYANAN.FCA. GRAD CWA. Page-8


Chapter-1: Basic concepts

Local authority:

1 It means municipal corporation, municipality, municipal committee, town committee,


notified area committee or cantonment board.
2 A local authority gets exemption U/s 10 (20) in respect of (a) income from house
property (b) gain arising from transfer of capital asset (c) income from trade or business
carried on by it within its own territorial jurisdictional area (d) Income from other
sources.
3 Also income from supply of water or electricity within or outside its jurisdictional area is
exempt.

Artificial juridical person:

1 This is a residual category. Any entity not covered in any of the specific categories in S. 2
(31) shall come under AJP.
2 Deities are to be regarded as artificial juridical person.
3 Bar council, Political parties, Trade union, Chamber of commerce and Universities are
also regarded as artificial juridical person.

Note: CG and SG do not come within the ambit of the term ‘person’ and hence, not liable to
income tax. [Dredging corporation of India (AP)].

F. Tax rates applicable for AY 20-21:

(a) Individual (not covered by (b) & (c)) /HUF/ AOP & BOI (not covered by S. 167B) and
every AJP:

Total income Rate of income-tax


Where the TI does not exceed Rs. Nil
2,50,000
Where the TI exceeds Rs. 2,50,000 but 5% of the amount by which the TI exceeds
does not exceed Rs. 5,00,000 Rs. 2,50,000
Where the TI exceeds Rs. 5,00,000 but Rs. 12,500 plus 20% of the amount by which
does not exceed Rs. 10,00,000 the TI exceeds Rs. 5,00,000
Where the TI exceeds Rs. 10,00,000 Rs. 1,12,500 plus 30% of the amount by
which the TI exceeds Rs. 10,00,000
Note: AOP/BOI covered by S. 167B shall suffer tax @ MMR/Higher rate/combination
thereof.

MT Educare – CA Inter Income Tax Classes – R. SOUMYANARAYANAN.FCA. GRAD CWA. Page-9


Chapter-1: Basic concepts

(b) Resident individuals aged 60 years or more but less than 80 years at any time during
the PY:

Total income Rate of income-tax


Where the TI does not exceed Rs. Nil
3,00,000
Where the TI exceeds Rs. 3,00,000 but 5% of the amount by which the TI exceeds
does not exceed Rs. 5,00,000 Rs. 3,00,000
Where the TI exceeds Rs. 5,00,000 but Rs. 10,000 plus 20% of the amount by which
does not exceed Rs. 10,00,000 the TI exceeds Rs. 5,00,000
Where the TI exceeds Rs. 10,00,000 Rs. 1,10,000 plus 30% of the amount by
which the TI exceeds Rs. 10,00,000

Note: The aforesaid slab rates apply to those resident individuals who attained the age of 60
years on or before 01.04.2020.

(c) Resident individuals of the age of 80 years or more at any time during the PY:

Total income Rate of income-tax


Where the TI does not exceed Rs. Nil
5,00,000
Where the TI exceeds Rs. 5,00,000 but 20% of the amount by which the TI exceeds
does not exceed Rs. 10,00,000 Rs. 5,00,000
Where the TI exceeds Rs. 10,00,000 Rs. 1,00,000 plus 30% of the amount by
which the TI exceeds Rs. 10,00,000

Note: The aforesaid slab rates apply to those resident individuals who attained the age of 80
years on or before 01.04.2020.

(d) Co-operative society:

Total income Rate of tax


Where the TI does not exceed Rs. 10000 10% of TI
Where the TI exceeds Rs. 10000 but does Rs. 1000 plus 20% of the amount by which
not exceed Rs. 20000 the TI exceeds Rs. 10000
Above Rs. 20,000 Rs. 3000 plus 30% of the amount by which
the TI exceeds Rs. 20000.

(e) Firm/LLP:

Tax rate is 30% on the whole of the TI of the firm. This rate would apply to an LLP also.

(f) Local authority:

The rate of tax for a local authority is 30% on the whole of the total income.

MT Educare – CA Inter Income Tax Classes – R. SOUMYANARAYANAN.FCA. GRAD CWA. Page-10


Chapter-1: Basic concepts

(g) Company:

(i) Domestic company whose turnover or gross receipt in the PY 17- 25% of the total
18 ≤ Rs. 400 Crores income
(ii) Any other domestic company 30% of the total
income
(iii) Foreign company 40% of the total
income

Applicability of surcharge:

(i) Individual/HUF/AOP/BOI/AJP Total income doesn’t exceed No surcharge


Rs. 50L
Total income exceeds Rs. 50L Surcharge rate
but doesn’t exceed Rs. 100L = 10%
Total income exceeds Rs. Surcharge rate
100L but doesn’t exceed Rs. = 15%
200L
Total income exceeds Rs. Surcharge rate
200L but doesn’t exceed Rs. = 25%
500L
Total income exceeds Rs. Surcharge rate
500L = 37%
(ii) Co-operative society / Local authority Total income doesn’t exceed No surcharge
/ Firm / LLP Rs. 100L
Total income exceeds Rs. Surcharge rate
100L = 12%
(iii) Domestic company Total income doesn’t exceed No surcharge
Rs. 100L
Total income exceeds Rs. Surcharge rate
100L but doesn’t exceed Rs. = 7%
1000L
Total income exceeds Rs. Surcharge rate
1000L = 12%
(iv) Foreign company Total income doesn’t exceed No surcharge
Rs. 100L
Total income exceeds Rs. Surcharge rate
100L but doesn’t exceed Rs. = 2%
1000L
Total income exceeds Rs. Surcharge rate
1000L = 5%

Note: Surcharge rate shall be applied on income tax.

MT Educare – CA Inter Income Tax Classes – R. SOUMYANARAYANAN.FCA. GRAD CWA. Page-11


Chapter-1: Basic concepts

Applicability of Health and education cess:

(a) Health and education cess is leviable in case of all assessees (irrespective of the
amount of total income)
(b) It shall be quantified by applying 4% on tax plus surcharge.

G. Rebate U/s 87A:

1 Person eligible for Resident individual whose total income does not exceed Rs.
rebate 500000.
2 Quantum of rebate Income tax payable or Rs. 12500 (whichever is less)
3 Treatment of rebate It should be reduced against income-tax.

H. Rounding off total income & tax: [S. 288A & S. 288B]:

1 Ignore decimal.
2 Last digit < 5, make it zero.
3 Last digit ≥ 5, round it off to the next multiple of 10.

I. Marginal relief:

Computation of marginal relief in case of Individual /HUF/AJP/AOP & BOI (not covered by S.
167B):

Total income > Rs. 50L but doesn’t exceed Rs. Total income > Rs. 100L but doesn’t exceed Rs.
100L 200L
1 Tax on total income + surcharge @ ***** 1 Tax on total income + Surcharge @ *****
10% 15%
2 Tax on Rs. 50L (****) 2 Tax on Rs. 100L + Surcharge @ 10% (****)
3 Total income – Rs. 50L (****) 3 Total income – Rs. 100L (****)
4 Marginal relief (1-2-3) ***** 4 Marginal relief (1-2-3) *****

Total income > Rs. 200L but doesn’t exceed Rs. Total income > Rs. 500L
500L
1 Tax on total income + surcharge @ ***** 1 Tax on total income + Surcharge @ *****
25% 37%
2 Tax on Rs. 200L + Surcharge @15% (****) 2 Tax on Rs. 500L + Surcharge @ 25% (****)
3 Total income – Rs. 200L (****) 3 Total income – Rs. 500L (****)
4 Marginal relief (1-2-3) ***** 4 Marginal relief (1-2-3) *****

Computation of marginal relief in case of Co-operative society / Local authority / Firm / LLP:

1 Tax on total income + surcharge @ 12% *****


2 Tax on Rs. 100L (****)
3 Total income – Rs. 100L (****)
4 Marginal relief (1-2-3) *****

MT Educare – CA Inter Income Tax Classes – R. SOUMYANARAYANAN.FCA. GRAD CWA. Page-12


Chapter-1: Basic concepts

Computation of marginal relief in case of domestic companies:

Total income > Rs. 100L but doesn’t exceed Rs. Total income > Rs. 1000L
1000L
1 Tax on total income + surcharge @ ***** 1 Tax on total income + Surcharge @ *****
7% 12%
2 Tax on Rs. 100L (****) 2 Tax on Rs. 1000L + Surcharge @ 7% (****)
3 Total income – Rs. 100L (****) 3 Total income – Rs. 1000L (****)
4 Marginal relief (1-2-3) ***** 4 Marginal relief (1-2-3) *****

Computation of marginal relief in case of foreign companies:

Total income > Rs. 100L but doesn’t exceed Rs. Total income > Rs. 1000L
1000L
1 Tax on total income + surcharge @ ***** 1 Tax on total income + Surcharge @ *****
2% 5%
2 Tax on Rs. 100L (****) 2 Tax on Rs. 1000L + Surcharge @ 2% (****)
3 Total income – Rs. 100L (****) 3 Total income – Rs. 1000L (****)
4 Marginal relief (1-2-3) ***** 4 Marginal relief (1-2-3) *****

Points requiring attention:

1 Marginal relief is reduced from the sum total of tax and surcharge.
2 If marginal relief is a negative figure, take it as zero.
3 Benefit of marginal relief is available only in respect of surcharge and not in respect of education
cess.

J. Concept of income:

1 All receipts are not income. Whether a receipt is income are not depends on its nature.
2 Receipts are of two types: (a) Revenue receipts (b) Capital receipts.
3 Revenue receipts are income and are taxable unless specifically exempted under the Act.
4 Capital receipts are not income and are not taxable unless the Act provides otherwise.
5 Any return (monetary or otherwise) coming with some sort of regularity from a definite source
is revenue receipt.
6 Examples of revenue receipt: (a) Rent (b) Salary (c) Dividend (d) Interest (e) Business profits etc.
7 Examples of revenue receipts which are exempt under this Act: (a) Dividend from domestic
company – exempt U/s 10 (34); (b) Income from units of mutual fund – exempt U/s 10 (35); (c)
Rent from letting out of agricultural land in India for carrying out agricultural operations –
exempt U/s 10 (1).
8 Any receipt not being a revenue receipt is a capital receipt.
9 Examples of capital receipts:
(a) Receipts in lieu of Consideration for sale of house property or shares in a company (not held
source as stock in trade)
(b) Windfall receipts (i) Lottery winnings (ii) horse race winnings (iii) winning from cross word
or casual receipts puzzles (iv) card game winnings (v) winnings from betting (vi) winnings
from gambling (vii) winnings from game shows etc.

MT Educare – CA Inter Income Tax Classes – R. SOUMYANARAYANAN.FCA. GRAD CWA. Page-13


Chapter-1: Basic concepts

(c) Grants or subsidies received from (i) Subsidy from Government for setting up an industrial
Government for creation of profit undertaking in a backward area; (ii) subsidy to enterprises
making apparatus or for improving in sugar industry for repayment of loan taken for setting
the profit making ability of profit new sugar mill or for substantially expanding the capacity
making apparatus. of existing sugar mill.
(d) Benefit on account of cessation or Waiver by a nationalize bank, of a part of loan taken for
remission of liabilities (not being acquisition of fixed asset, pursuant to one time
trading liability) settlement scheme designed to reduce NPA.
(e) Forfeiture of advance on account of failed negotiation relating to transfer of a capital asset
(f) Compensation for sterilization or (a) Non-compete fees; (b) Fee for exclusivity of rights; (c)
impairment of source Retrenchment compensation; (d) Voluntary retirement
compensation; (e) Compensation paid for termination of
agency or modification of the terms of agency to the
detriment of the agent; (f) Compensation to managerial
personnel for loss of office
(g) Insurance compensation for destruction of machines in the factory due to fire accident.
(h) Liquidated damage received for delay in supply of machinery
(i) Income from trial run production
(j) Receipts incidental to acquisition of asset
(k) Gifts (monetary as well as non-monetary)
(l) Receipts during the construction period
10 Capital receipts referred to in 9 (a), (b), (c), (e), (f), (g) and (k) are included in income definition
contained in S. 2 (24) and are, hence, taxable.

Points requiring attention:

1 Relevant portion of definition of the term ‘income’ contained in S. 2 (24) (to the extent not
covered here) will be discussed in Salary, PGBP and IFOS.
2 Assistance in the form of subsidy or grant or cash incentive or duty drawback or waiver or
concession or reimbursement, by whatever name called, by the CG, or a SG or any authority or
body or agency in cash or kind to the assessee is included in the definition of income.
However, any asset specific subsidy or grant or reimbursement which is adjusted against the
actual cost of the asset in accordance with Explanation-10 to S. 43 (1) shall stand excluded from
the ambit of the term ‘income’.
Similarly, any subsidy or grant given by the CG for the purpose of corpus of a trust established by
the CG or SG shall also stand excluded from the purview of the term ‘income’.
3 Only gifts covered by S. 56 (2) (x) come within the ambit of the term ‘income’. [Detailed
discussion is made in IFOS chapter].
4 Profits and gains arising from transfer of capital asset are included in income definition.
The term ‘capital asset’ is defined in S. 2 (14). It means property of any kind held by the assessee,
whether or not connected with his business or profession.
It takes within its fold movable assets and immovable assets. It covers tangible properties as well
as intangible properties.
It excludes (a) stock in trade (not being securities held by FII); (b) rural agricultural land; (c)
personal effects (i.e. movable properties held for the personal use of the assessee or his
dependent family member) (d) Deposit certificates issued under Gold monetisation scheme,
2015 notified by the CG.
Capital assets could be classified into (a) LTCA and (b) STCA. Gain arising on account of transfer of
LTCA is called LTCG and the gain arising on account of transfer of STCA is called STCG. (Detailed
discussion is made in the chapter ‘Capital gains’).

MT Educare – CA Inter Income Tax Classes – R. SOUMYANARAYANAN.FCA. GRAD CWA. Page-14


Chapter-1: Basic concepts

K. Definition of income (S. 2(24)):

Income includes
1 Profits and gains
2 Dividend
3 Voluntary contributions received by charitable or religious trusts or by Universities or
Educational Institutions and Hospitals referred to in S. 10 (23C) or by an Electoral trust.
4 Special allowance or benefit specifically granted either to meet personal expenses or for the
performance of duties of an office or an employment of profit.
5 Perquisite or profit in lieu of salary given to employees.
6 Compensation or other payments due to or received by the agent at the time of termination
of his agency
7 Surplus derived by a trade, professional or similar association from the specific services
performed for its members.
8 Export incentives
9 Value of any non-monetary benefit or perquisite arising from business or the exercise of a
profession
10 Any interest, salary, bonus, commission or remuneration received by a partner from a firm.
11 Any sum received for not carrying out any activity in relation to any business or not to share
any know-how, patent, copyright, trademark, etc.
12 Deemed profit taxable u/s. 41 or 59
13 Any capital gains chargeable u/s. 45
14 a. Winnings from lottery
b. Winnings from crossword puzzles
c. Winnings from races including horse races
d. Winnings from card game and other games of any sort
e. Winnings from gambling
f. Winnings from betting
15 Any sum received by the assessee from his employees towards welfare fund contributions
such as provident fund, superannuation fund, ESI fund or any other welfare fund.
16 Any sum received under a Keyman Insurance policy including the sum allocated by way of
bonus on such policy
17 Any sum of money or value of property referred to in S. 56 (2) (x) (received without
consideration or for inadequate consideration by any person).
18 Value of property referred to in S. 56 (2) (viia).
19 Any consideration received for issue of shares as exceeds the FMV of shares referred to in S.
56 (2) (viib).
20 Any sum of money received as advance, if such sum is forfeited consequent to failure of
negotiation for transfer of a capital asset (referred to in S. 56 (2) (ix)).
21 Grants or subsidy (as aforesaid)
22 Profits and gains of any business of banking carried by a co-operative society with its
members.
23 FMV of inventory which is converted into, or treated as a capital asset. (Vide S. 28 (iva)).
24 Any compensation or other payment, due to or received by any person, in connection with
termination of his employment or the modification of the terms and conditions relating
thereto. [S. 56 (2) (xi)].

MT Educare – CA Inter Income Tax Classes – R. SOUMYANARAYANAN.FCA. GRAD CWA. Page-15


Chapter-1: Basic concepts

L. Special rates of tax in respect of specified income prescribed under the Act:

SN Income Section Special rate Remarks


1 LTCG 112 20% (flat) Detailed
discussion in
capital gains
chapter
2 LTCG arising on account of transfer of 112A 10% (flat) on (LTCG Detailed
equity shares (provided the assessee exceeding Rs. 1L) discussion in
has suffered STT at the time of capital gains
acquisition as well as at the time of chapter
transfer)
LTCG arising on account of transfer of 10% (flat) (LTCG
equity oriented units or units of exceeding Rs. 1L)
business trust (provided the assessee
has suffered STT at the time of transfer)
3 STCG arising on account of transfer of 111A 15% (flat) Detailed
equity shares or equity oriented units or discussion in
units of business trust (provided the capital gains
assessee has suffered STT) chapter
4 Casual income (i.e. windfall receipts) 115BB 30% (flat) Detailed
(taxed on gross basis) discussion in
IFOS chapter
5 Dividend (not being one referred to in S. 115BBDA 10% (flat) Detailed
2 (22) (e)) from domestic companies discussion in
exceeding Rs. 10L received by a resident IFOS chapter
being specified assessee (see below)
(taxed on gross basis).
6 Income from undisclosed sources 115BBE 60% (flat) + Surcharge Discussed
(referred to in S. 68, 69, 69A, 69B and S. @ 25% + 4% Health below
69C) (gross basis) and education cess
(Effective rate = 78%)

Points requiring attention:


1 Specified assessee Specified assessee means any person other than (a) a domestic company;
(for the purpose of(b) entities referred to in S. 10 (23C) (iv)/(v)/(vi)/(via); (c) a trust or
S. 115BBDA) institution registered U/s 12AA.
2 Income referred to Where any sum is found credited in the books of an assessee maintained
in S. 68for any PY, and the assessee offers no explanation about the nature and
(unexplained cash source thereof or the explanation offered by him is not, in the opinion of
credit) the AO, satisfactory, the sum so credited may be charged to income-tax as
the income of the assessee of that PY.
3 Income referred to Where in the FY immediately preceding the AY the assessee has made
in S. 69 investments which are not recorded in the books of account, if any,
(unexplained maintained by him for any source of income, and the assessee offers no
investment) explanation about the nature and source of the investments or the
explanation offered by him is not, in the opinion of the AO, satisfactory,
the value of the investments may be deemed to be the income of the
assessee of such FY.

MT Educare – CA Inter Income Tax Classes – R. SOUMYANARAYANAN.FCA. GRAD CWA. Page-16


Chapter-1: Basic concepts

4 Income referred to in Where in any FY the assessee is found to be the owner of any money,
S. 69A (unexplained bullion, jewellery or other valuable article and such money, bullion,
money etc) jewellery or valuable article is not recorded in the books of account, if
any, maintained by him for any source of income, and the assessee
offers no explanation about the nature and source of acquisition of the
money, bullion, jewellery or other valuable article, or the explanation
offered by him is not, in the opinion of the AO, satisfactory, the money
and the value of the bullion, jewellery or other valuable article may be
deemed to be the income of the assessee for such FY.
5 Income referred to in Where in any FY the assessee has made investments and the AO finds
S. 69B (Investments that the amount expended on making such investments exceeds the
not fully disclosed in amount recorded in this behalf in the books of account maintained by
the books) the assessee for any source of income, and the assessee offers no
explanation about such excess amount or the explanation offered by
him is not, in the opinion of the AO, satisfactory, the excess amount
may be deemed to be the income of the assessee for such FY.
6 Income referred to in Where in any FY an assessee has incurred any expenditure and he
S. 69C (unexplained offers no explanation about the source of such expenditure or part
expenditure) thereof, or the explanation, if any, offered by him is not, in the opinion
of the AO, satisfactory, the amount covered by such expenditure or
part thereof, as the case may be, may be deemed to be the income of
the assessee for such FY.

M. Basic principles clarifying the concept of income:

1 Income tainted with The income tax law does not make any distinction between income arisen
illegality – taxable from a legal source and the income tainted with illegality. Even income
from smuggling is taxable.
2 Real income theory Unless the Act provides otherwise, what could be taxed is real income
and not notional or fictional income.
3 Application of Every income that accrues or arises is liable to be taxed, irrespective of
income Vs Diversion what happened to it afterwards. How the income earned is spent or
of income by over- applied is not relevant. Amount applied or spent can’t be deducted in
riding title determining the taxable income.
Where by an obligation, income is diverted before it reaches the
assessee, it is “diversion of income” and not taxable. Conversely, after
earning the income, if it is required to be applied to discharge an
obligation, it is merely an “application of income” and income is
chargeable to tax.
In order to decide whether a particular payment is a diversion of income
or application of income, it has to be determined whether the
disbursement of income made by the assessee is a result of fulfillment
of an obligation on him or whether income has been applied to
discharge an obligation after it reaches the assessee.
4 Surplus arising from It is not that persons come together only for earning profit. Persons come
mutual activities ≠ together even for other purposes like entertainment (Example: Social
income clubs), professional development (Example: Bombay Chartered
Accountant society), advancement of common interest etc (Example:
Resident welfare association, trade association, trade union). Such
associations are called mutual concern.
Following are the features of a mutual concern: (a) Common purpose; (b)

MT Educare – CA Inter Income Tax Classes – R. SOUMYANARAYANAN.FCA. GRAD CWA. Page-17


Chapter-1: Basic concepts

Contribution to a common fund; (c) Expending out of common fund for


achieving the common purpose; (d) Complete identity between persons
who contribute to the common fund and persons who participate in the
surplus in the common fund; (e) No transaction with outsiders; (f) No
profit motive and activities are not tainted with commerciality.
The surplus arising from the mutual activities carried on by the mutual
concern is not income.
Bankipur club (SC) If a club (Non-profit organisation) receives any surplus arising from sale of
drinks, refreshments etc, or amount by way of rent for letting out the
building or amount by way of admission fees, periodical subscription,
then these are not taxable since these were only charges for the
privileges, conveniences and amenities provided to members, which they
are entitled as per the rules and regulations of the club.
Though all members didn’t take advantage of facilities provided by the
club, still members as a class contribute and as a class participate in it.
That is sufficient.
If the object of the assessee claiming to be a mutual concern or club is to
carry on a particular business and money is realized both from members
and from non-members, for provision of same or similar facilities to all
alike, the dealings as a whole disclose profit earning motive and are alike
tainted with commerciality. In other words, the activity carried on by the
assessee in such cases, claiming to be a mutual concern or club is a trade
and the resultant surplus is profit liable to tax.
5 Pin money Pin money received by wife for her dress/personal expenses and small
savings made by a woman out of money received from her husband for
meeting household expenses is not treated as her income.

N. Heads of income – S. 14:

For the purpose of levy or charge of income-tax and for computation of total income, every income
of the assessee shall be classified under any of the following heads:

Head Chapter Sections Charging section


Salary IV-A 15-17 15
House property IV-C 22-27 22
Profits & Gains from Business or Profession IV-D 28-44DB 28 & 41
Capital gains IV-E 45-55A 45 & 46 (2)
Other sources IV-F 56-59 56

Note:

1 Whether a particular income is chargeable under a particular head or not is decided by the
charging section. In other words, the scope of particular head is decided by the concerned
charging section.
2 The Act has self-contained provisions in respect of each head of income. Each head has unique
set of rules for computation of income chargeable to tax.
3 An income chargeable under a particular head cannot again be charged under any other head.
4 If a particular non-exempted income is not covered by the first four heads, it automatically falls
U/H IFOS.
5 The aggregate of income under these heads is termed as “Gross Total Income”.

MT Educare – CA Inter Income Tax Classes – R. SOUMYANARAYANAN.FCA. GRAD CWA. Page-18


Chapter-1: Basic concepts

O. Brief introduction to each head of income:

SN Head Brief introduction


1 Salary Benefits arising from employment are charged to tax U/H salaries.
The pre-requisite for invoking this head is employer-employee relationship
between the person earning income and the person paying it. That is, there
should exist a contract of services between them.
Salary income is taxed on due basis or receipt basis, whichever is earlier.
Vide charging section 15.
While computing income chargeable to tax U/H Salaries, deductions
contemplated U/s 16 are allowed.
2 IFHP Annual value of a house property of which the assessee is the owner is
charged to tax U/H IFHP. Vide charging section 22.
Annual value means the sum for which the property is expected to be let
from time to time. That is, the inherent rent-earning capacity of the house
expressed in monetary terms.
Even if the house is let for a sum which is less than its annual value, what is
taken into consideration while computing income chargeable U/H IFHP is
the annual value and not the rent actually earned. Further, even if the
property is not let out, still annual value of the property could be brought
to tax under this head. Since a notional income is charged to tax under this
head, it is an exception to real income theory.
However, if the house property is used by the assessee for the purpose of
business or profession carried on by him, then S. 22 exempts the annual
value from taxation.
Further, if the assessee occupies his house for his own residence, then the
annual value of such house is regarded as nil. However, where the assessee
occupies more than one house for his residential purposes, then the annual
value of any one house of his choice is regarded as nil. This is called self-
occupied property benefit.
Income from subletting is not brought to tax under this head, since the
element of ‘ownership’ is missing.
Also, income from letting of vacant land is outside the ambit of this head
since what is taxed under this head is only the annual value of house
property.
Deductions contemplated in S. 24 are allowed while computing income
chargeable under this head.
3 PGBP Profits and gains from business or profession carried on by the assessee
during the PY are charged to tax U/H PGBP. The charging section for this
head is S. 28.
4 Capital gains Profits and gains arising from transfer of capital asset effected during the
PY shall be charged to tax U/H Capital gains in the PY inn which transfer is
effected. The charging sections for this head are S. 45 and S. 46 (2).
S. 48 lays down the manner of computing capital gains.
Exemptions are provided in respect of capital gains U/s 54, 54B, 54D, 54EC,
54EE, 54F, 54G, 54GA, 54GB, 10 (37), 10 (38) etc. What is brought to tax
under this head is only the capital gains which is not exempted under these
sections.

MT Educare – CA Inter Income Tax Classes – R. SOUMYANARAYANAN.FCA. GRAD CWA. Page-19


Chapter-1: Basic concepts

5 IFOS Charging section 56 (1) provides for taxing under the head IFOS any income
which is not exempt under this Act and which is not getting fitted under the
first four heads. This head is a residuary head.
Apart from this, the incomes specified U/s 56 (2) are to be taxed only under
the head IFOS.
While computing income chargeable under this head, deductions
contemplated in S. 57 are to be allowed.

P. Relevance of method of accounting and ICDS:

1 S. 145 (1) permits the assessees to follow either mercantile system of accounting or cash system
of accounting.
2 For the head ‘salaries’, method of accounting has no relevance since S. 15 provides for taxing
salary on due basis or receipt basis, whichever is earlier.
3 Even for the head IFHP, method of accounting has no relevance since S. 22 provides for taxing
annual value on a notional basis.
4 For the head CG, method of accounting has no relevance since S. 45 provides for taxing capital
gains in the PY of transfer.
5 However, the income U/H PGBP and IFOS shall be computed in accordance with the method of
accounting regularly employed by the assessee (subject to statutory exceptions).
6 In exercise of powers given U/s 145 (2), the Central Government has come with ICDS which are
to be followed by the assessees (following mercantile system of accounting) in computing
income chargeable under the head PGBP and IFOS.

Q. Introduction to clubbing provisions:

1 Generally, income earned by a person is taxed only in his hands.


2 However, S.60 to S. 64 of the Act contemplates certain circumstances in which income earned by
some other person is also included in the income sheet of the assessee and is charged to tax in
his hands. (Example: Income of a minor child is clubbed in the hands of parent as per S. 64 (1A)).
3 Incomes under these five heads are to be computed after applying the clubbing provisions.

R. Introduction to adjustment of losses:

1 For the purpose of computing the taxable income, incomes from all sources are aggregated.
2 Where the assessee has loss from a source, it shall be adjusted against income from other
sources within the same head. This is called inter-source adjustment.
3 If the loss is still remaining unabsorbed, it shall be adjusted against income chargeable under
other heads. This is called inter-head adjustment.
4 If the loss subsists even thereafter, it shall be carried forward to the next year for adjustment
against next year’s income. This is called carried forward and set off.
5 The provisions relating to adjustment of losses are contained in S. 70 to S. 80.

S. Introduction to deductions under Chapter VI-A:


1 Chapter VI-A starts with S.80C and ends with S. 80U.
2 It provides deduction against gross total income. Aggregate of income under all heads after
applying clubbing provisions and after making adjustment of losses is called gross total income.
3 S. 80C to S. 80GGB provides deduction in respect of specified payments, contributions,
donations, subscriptions and investments made by the assessee during the relevant PY.

MT Educare – CA Inter Income Tax Classes – R. SOUMYANARAYANAN.FCA. GRAD CWA. Page-20


Chapter-1: Basic concepts

4 S. 80 IA to S. 80 TTB provides deduction to the assessee for earning specified income.


5 S. 80U provides deduction to the assessee who suffers from specified disabilities referred to
therein.
6 No deduction is allowed under Chapter VI-A in respect of that portion of gross total income
which represents LTCG, STCG referred to in S. 111A, casual income, income from undisclosed
sources etc.
7 The sum total of deductions under Chapter VIA cannot cross the gross total income.

T. Format for computation of taxable income:

1 Salaries ****
2 IFHP ****
3 PGBP ****
4 Capital gains ****
5 IFOS ****
6 Total (1+2+3+4+5) ****
7 Adjustment of losses as per S. 70 to S. 80 (***)
8 Gross total income ****
9 Deductions under Chapter VI-A (***)
10 Total income (8-9) ****

U. Scheme of partial integration (SOPI):

Introduction:

1 Entry 82 to List I of VII schedule to the Constitution gives power to the Union Government to levy
tax only on income other agricultural income. The power to levy tax on agricultural income is
vested only with the State Government. The CG cannot impose tax on agricultural income.
Hence, agricultural income is exempted u/s. 10(1).
2 However, S. 10 (1) does not exempt foreign agricultural income. It is very much taxable. In other
words, it is treated on par with non-agricultural income.
3 Though agricultural income is not taxable as such, it has the effect of enhancing the tax liability
on non-agricultural income. In other words, it is included for determining the rate at which non-
agricultural income is chargeable to tax.
4 The computation mechanism employed in computing the tax liability on non-agricultural income
where the assessee is having agricultural income is called the scheme of partial integration. The
scheme of partial integration is provided for in the Annual Finance Act – First schedule – Part IV.

Steps involved in SOPI:

Step 1 Find out the tax payable on Ʃ (Agricultural income + non-agricultural income)
Step 2 Find out the tax payable on Ʃ (Agricultural income + Basic Exemption)
Step 3 Tax payable on Non-agricultural income = (Tax in Step 1) – (Tax in Step 2)
Step 4 Add surcharge (if the non-agricultural income is more than Rs. 50L)
Step 5 Add Health and education Cess @ 4%.
Step 6 Tax liability = Step 3 + Step 4 + Step 5.

Note: If the assessee is a resident individual and his non-agricultural income doesn’t exceed Rs.
500000, then rebate U/s 87A could be claimed against tax on non-agricultural income.

MT Educare – CA Inter Income Tax Classes – R. SOUMYANARAYANAN.FCA. GRAD CWA. Page-21


Chapter-1: Basic concepts

When SOPI does not apply?:

1 If the assessee is a firm or a company or a local authority or a co-operative society or an AOP or


BOI (covered by S. 167B), the scheme of partial integration does not apply. That is, simply the
agricultural income has to be ignored and it has no relevance in computing the tax liability on
non-agricultural income.
2 If the non-agricultural income doesn’t exceed the basic exemption limit, then also the scheme of
partial integration does not apply.
3 If the net agricultural income is less than or equal to Rs. 5000, then also the scheme of partial
integration does not apply.
4 If the non-agricultural income purely consists of income suffering at flat rate (like winnings from
lottery etc), then also the scheme of partial integration does not apply.

Disintegration of composite income into agricultural and non-agricultural:

Certain businesses generate composite income which has both agricultural and non-agricultural
elements. Example – Tea business, Rubber business and coffee business. For applying the scheme of
partial integration, we need to segregate the composite income into agricultural and non-
agricultural. For this we have guidance in the Income tax Rules which is summarized as follows:

Nature of business Non- Agricultural Relevant


agricultural Income Rule
Income
Growing and manufacturing tea in India 40% of composite 60% of composite Rule 8
income income
Growing and manufacturing rubber in India 35% of composite 65% of composite Rule 7A
income income
Sale of coffee grown and cured by seller 25% of composite 75% of composite Rule 7B (1)
income income
Sale of coffee grown, cured, roasted and 40% of composite 60% of composite Rule 7B
grounded by seller in India with or without income income (1A)
mixing chicory or other flavoring ingredients

Note: The composite income is to be computed in accordance with chapter IV-D (S. 28 to S. 44DB).

Illustration: Mr. C manufactures latex from the rubber plants grown by him in India. These
are then sold in the market for Rs. 30L. The cost of growing rubber plants is Rs. 10L and that
of manufacturing latex is Rs. 8L. Compute his total income.

Composite income from rubber business = Rs. 30L – Rs. 10L – Rs. 8L = Rs. 12L.

Agricultural income = 65% of Rs. 12L = Rs. 7.80L.

Non-agricultural income = 35% of Rs. 12L = Rs. 4.20L.

MT Educare – CA Inter Income Tax Classes – R. SOUMYANARAYANAN.FCA. GRAD CWA. Page-22


Chapter-1: Basic concepts

Manner of disintegration of composite income from other businesses- R. 7:

For disintegrating the composite income from other business, there are no specified
percentages available. The manner of such disintegration can be illustrated as follows:

Mr. B grows sugarcane and uses the same for the purpose of manufacturing sugar in his
factory. 30% of sugarcane produce is sold for Rs. 10L, and the cost of cultivation of such
sugarcane is Rs. 5L. The cost of cultivation of the balance sugarcane (70%) is Rs. 14L and the
market value of the same is Rs. 22L. After incurring Rs. 1.5L in the manufacturing process on
the balance sugarcane, the sugar was sold for Rs. 25L. Compute B’s business income and
agricultural income.

Income from sale of sugarcane gives rise to agricultural income and from sale of sugar gives
rise to business income.

Business income = Sales (-) Market value of 70% of sugarcane produce (-) Manufacturing
expenses = Rs. 25L – Rs. 22L – Rs. 1.5L = Rs. 1.5L.

Agricultural income = Market value of sugarcane produce – cost of cultivation = [Rs. 10L +
Rs. 22L] – [Rs. 5L + Rs. 14L] = Rs. 13L.

Note: Salary and interest received by a partner from a firm (engaged in tea business or
rubber business or coffee business) is taxable only to the extent of 40 or 35 or 25 or 40%
and the balance is treated as agricultural income.

Points requiring attention:

1 Suppose Mr. X owns a tea estate R. 8 could be pressed into operation only when the assessee
in India. During the relevant PY, he grows tea leaves and manufactures tea in India. In the instant
made an income of Rs. 30L from case, assessee is not engaged in manufacturing tea. He just
sale of tea leaves in India. Discuss sells tea leaves. Since, there is no non-agricultural activity, the
the tax implications entire income is agricultural in nature. Hence, it is exempt U/s
10 (1). However, for determination of tax on non-agricultural
income, this is integrated with the non-agricultural income.

2 Mr. X owns a rubber estate in R. 7A applies only when the activity of growing and
Malaysia. Income from growing manufacturing rubber is carried out in India. In the instant
and manufacturing rubber is Rs. case, it is carried out in Malaysia. Hence, R. 7A doesn’t apply.
100L. Discuss the tax implications. Entire income of Rs. 100L is taxable. Exemption U/s 10 (1)
doesn’t apply to foreign agricultural income.
3 Dividend is declared by a company It is non-agricultural income in the hands of shareholders. This
out of its agricultural income. is because the proximate source for this dividend is investment
Discuss the tax implications in the in shares and not the agricultural activity. If the company is a
hands of shareholders. domestic company, it is exempt in the hands of shareholders
U/s 10 (34). However, if the company is a foreign company,
then it taxable in the hands of shareholders.

MT Educare – CA Inter Income Tax Classes – R. SOUMYANARAYANAN.FCA. GRAD CWA. Page-23


Chapter-1: Basic concepts

Test your knowledge:

1. Mr. X, a resident, has provided the following particulars of his income for the PY 19-20:

1 Income from salary (computed) Rs. 180000


2 Income from house property (computed) Rs. 200000
3 Agricultural income from a land in Jaipur Rs. 280000
4 Expenses incurred for earning agricultural income Rs. 170000

Compute his tax liability assuming his age is (a) 45 years; (b) 70 years.

2. Mr. X, a resident aged 30 years, furnishes the following particulars:

1 Lottery winnings Rs. 3.50L


2 Agricultural income Rs. 6L
3 Contribution to PPF Rs. 1L

Determine his tax liability for the AY 20-21.

3. A partnership firm having to equal partners X and Y, furnishes the following details
pertaining to the PY 19-20:

Business income details Agricultural income details


1 Business income (before Rs. 50L 1 Gross agricultural Rs. 60L
deducting interest and income
remuneration paid to
partners)
2 Interest paid to X @ 10% (Rs. 3L) 2 Interest paid to X @ (Rs. 6L)
10%
3 Interest paid to Y @ 105 (Rs. 5L) 3 Interest paid to Y @ 105 (Rs. 10L)
4 Remuneration paid to X (Rs. 10L) 4 Remuneration paid to X (Rs. 10L)
5 Remuneration paid to Y (Rs. 10L) 5 Remuneration paid to Y (Rs. 10L)
6 Taxable income (1-2-3-4-5) Rs. 22L 6 Net agricultural income Rs. 24L
(1-2-3-4-5)

Details about the partners are also provided:

SN Particulars Mr. X Mr. Y


1 Age 30 years 32 years
2 Personal agricultural income (India) Rs. 5L Rs. 5L
3 Personal agricultural income (abroad) Rs. 5L Rs. 5L
4 Interest on FD Rs. 4L -
5 Salary from ABC Ltd - Rs. 3L
6 Contribution to PPF Rs. 1L Rs. 1L
7 Dividend from XYZ Ltd (Agro company) Rs. 2L Rs. 3L

MT Educare – CA Inter Income Tax Classes – R. SOUMYANARAYANAN.FCA. GRAD CWA. Page-24


Chapter-1: Basic concepts

8 Lottery winnings Rs. 10L -


9 LTCG - Rs. 6L
10 STCG (S. 111A) Rs. 5L -
11 Dividend from foreign company - Rs. 4L

Determine the tax liability of the firm and partners for the AY 20-21.

Meaning of agricultural income: (S. 2 (1A)):

Following incomes are regarded as agricultural income:


1 Rent or revenue derived from land situated in India and used for agricultural purposes.
(a) The amount received in money or in kind, by one person from another for right to use
land is termed as ‘Rent’.
(b) The rent can either be received by the owner of the land or by the original tenant from
the sub-tenant. It implies that ownership of land is not necessary. Thus, the rent received
by the original tenant from sub-tenant would also be agricultural income.
(c) The scope of the term ‘Revenue’ is much broader than rent. It includes income other than
rent. For example, fee received for renewal for grant of land on lease would be revenue
derived from land.
2 Income derived from such land through agriculture
(a) The term ‘agriculture’ has not been defined in the Act. However, cultivation of a field
involving human skill and labour on the land can be broadly termed as agriculture.
(b) ‘Agriculture’ means tilling of the land, sowing of the seeds and similar operations. It
involves basic operations and subsequent operations.
(c) Basic operations means operations carried out by agriculturalists which are necessary for
the purpose of effectively raising produce from the land.
(d) Subsequent operations are operations which are to be performed after the produce
sprouts from the land (example: weeding, digging etc).
(e) The subsequent operations would be regarded as agricultural operations only when these
are taken in conjunction with and as a continuation of the basic operations.
(f) The term ‘agriculture’ cannot be extended to all activities which have some distant
relation to land like dairy farming, breeding and rearing of live stock, butter and cheese
making and poultry farming.
3 Income derived from such land by the performance of a process ordinarily employed by
a cultivator or receiver of rent in kind to render the produce fit to be taken to the
market.
(a) Sometimes, to make the agricultural produce a saleable commodity, it becomes
necessary to perform some kind of process on the produce. The income from the process
employed to render the produce fit to be taken to the market would be agricultural
income.
(b) However, it must be a process ordinarily employed by the cultivator or receiver of rent in
kind and the process must be applied to make the produce fit to be taken to the market.
(c) The ordinary process employed to render the produce fit to be taken to market includes
(a) thrashing (b) winnowing (c) cleaning (d) drying (e) crushing etc.
(d) For example, the process ordinarily employed by the cultivator to obtain the rice from
paddy is to first improve the hay from the basic grain, and thereafter to remove the chaff

MT Educare – CA Inter Income Tax Classes – R. SOUMYANARAYANAN.FCA. GRAD CWA. Page-25


Chapter-1: Basic concepts

from the grain. The grain then has to be properly filtered to remove stones etc and finally
the rice has to be packed in gunny bags for sale in the market.
(e) After such process, the rice can be taken to the market for sale. This process of making
the rice ready for the market may involve manual operations or mechanical operations.
(f) All these operations constitute the process ordinarily employed to make the product fit
for the market. The produce must retain its original character inspite of the processing
unless there is no market for selling it in that condition.
4 Income derived from such land through sale of such agricultural produce in the market.
(a) Any income from the sale of any produce to the cultivator or receiver of rent-in-kind is
agricultural income provided it is from the land situated in India and used for agricultural
purposes.
(b) However, if the produce is subjected to any process other than process ordinarily
employed to make the produce fit for market, the income arising on sale of such produce
would be partly agricultural income and partly non-agricultural income.
(c) Similarly, if other agricultural produce like tea, cotton, tobacco, sugarcane etc are
subjected to manufacturing process and the manufactured product is sold, the profit on
such sale will consist of agricultural income as well as business income. That portion of
the profit representing agricultural income will be exempted.
5 Income from farm building
(a) Income from the farm building which is owned and occupied by the receiver of rent of
any such land or occupied by the cultivator or receiver of rent in kind, of any land (supra)
would be agricultural income.
(b) However, the income from such farm building would be agricultural income only if the
following conditions are satisfied: (a) The building should be on or in the immediate
vicinity of the land; and (b) the receiver of the rent or the cultivator or the receiver of
rent in kind should, by reason of his connection with such land require it as a dwelling
house or as a store house.
(c) In addition to the above conditions, any one of the following conditions should also be
satisfied: (i) The land should either be assessed to land revenue in India or be subject to a
local rate assessed and collected by the officers of the Government as such; or (ii) The
land should not be with urban limits.

Urban limits:
The following lands are regarded as those situated within urban limits:
(a) Land situated in any area comprised within the jurisdiction of a municipality or a
cantonment Board and which has a population of not less than 10000.

(b) Land situated in any area within such distance, measured aerially, in relation to the
range of population according to the last preceding census as shown hereunder:
Shortest aerial distance from the local Population according to the last preceding
limits of Municipality or Cantonment census of which the relevant figures have been
board referred to in (a) published before the first day of the PY
2Km > 10000 ≤ 1L
6Km >1L ≤10L
8Km >10L

MT Educare – CA Inter Income Tax Classes – R. SOUMYANARAYANAN.FCA. GRAD CWA. Page-26


Chapter-1: Basic concepts

Would income arising from transfer of agricultural land situated within urban limits be
regarded as agricultural income?

No, as per Explanation to S. 2 (1A), the capital gains arising from the transfer of such urban
agricultural land would not be treated as agricultural income. No exemption is available U/s
10 (1). It shall be taxable U/s 45 (CG head).

Note: Gain arising on account of transfer of agricultural land not situated in urban limits
shall not be brought to tax U/H CG, since it is not a capital asset.

Whether income from nursery constitute agricultural income?

Yes, as per Explanation 3 to S. 2 (1A), income derived from saplings or seedlings grown in
nursery would be deemed to agricultural income, whether or not the basic operations were
carried out on land.

Points requiring attention:

1 Income from breeding of livestock, income from poultry farming, income from fisheries,
income from dairy farming, income from allowing cinema shooting etc ≠ agricultural
income. [Indirect connection with agricultural land is not enough to regard an income as
agricultural income].
2 Income from the sale of forest trees of spontaneous growth does not constitute
agricultural income. These trees grow on the soil unaided by any human skill and labour.

Therefore, there is no cultivation of soil at all. Even though operations in the nature of
forestry performed by the assessee may have the effect of nursing and fostering the
growth of such forest trees, it cannot constitute agricultural operations.
3 Income from sale of seeds, income from growing of flowers and creepers, income from
growing of bamboos, rent received from land used for grazing of cattle required for
agricultural activities etc = Agricultural income.
4 X was the managing agent of a company. He was entitled for a commission of 10% on
the annual net profits of the company. A part of the company’s income was agricultural
income. X claims that since his remuneration was calculated with reference to income of
the company, part of which was agricultural income, such part of commission as was
proportionate to the agricultural income was exempt from income tax. Is he justified?
No. X received remuneration under a contract for personal service calculated on the
amount of profits earned by the company. Such remuneration does not constitute
agricultural income.

MT Educare – CA Inter Income Tax Classes – R. SOUMYANARAYANAN.FCA. GRAD CWA. Page-27


Chapter-2: Residence & Scope of Total Income

Learning objectives:

1 Rules for determination of residential status of Individual


2 Rules for determination of residential status of HUF
3 Rules for determination of residential status of Companies
4 Rules for determination of residential status of other persons
5 Scope of total income
6 Circumstances in which income is deemed to accrue or arise in India

Introduction:

The scope of total income of an assessee depends upon the following three important
considerations:

1 The residential status of the assessee


2 The place of accrual or receipt of income
3 The point of time at which the income had accrued to or received by the assessee

A. Residential status – S. 6:

1. Rules for determination of residential status of Individual – S. 6 (1) & S. 6 (6) (a):

Stage-1: Determining whether an individual is resident or non-resident: S. 6 (1) (a) & (c):

An Individual is said to be resident in India in any PY, if he is:

(i) In India in that PY for atleast 182 days (or)


(ii) In India in that PY for atleast 60 days and in India for atleast 365 days during 4 PYs
preceding that PY

Explanation-1: Condition-1 alone shall apply for certain persons: For the following persons,
condition-1 alone is relevant for determination of residential status:

(a) Indian citizen who leaves India during the PY as a member of crew of an Indian ship
India citizen who leaves India during the PY for the purpose of employment outside
India
(b) Indian citizen or a person of Indian origin (POIO) who, being outside India, comes on a
visit to India in the PY

Note: A person is said to be of Indian origin if he or either of his parents or either of his
grandparents were born in undivided India.

MT Educare –CA Inter Income Tax Classes – R. SOUMYANARAYANAN. FCA. GRAD CWA. Page-28
Chapter-2: Residence & Scope of Total Income

Points requiring attention:


1 Residential status of a person is to be determined in respect of each PY. A person who
is resident for the PY 2018-19 may become non-resident for the PY 019-20.
2 Citizenship is different from residential status. An Indian citizen may be non-resident
and a foreign citizen may be resident. Residence of an Individual for income-tax
purpose has nothing to do with citizenship, place of birth or domicile.
3 Period of stay in India need not be continuous.
4 It is also not necessary that the stay should be at one place (i.e. the usual place of
residence, business or employment of Individual).
5 The purpose of stay is immaterial.
6 S. 2 (25A) defines India to mean (a) the territory of India as referred to in Article-1 of
the Constitution; (b) its territorial waters, seabeds and subsoil underlying such waters;
(c) air space above its territory and territorial waters; (d) exclusive economic zones &
maritime zones.
Even the stay in a ship or boat moored in the territorial waters of India would be
sufficient to make the individual resident in India.
7 For the purpose of counting the number of days stayed in India, both the date of
departure as well as the date of arrival are considered to be in India.
8 For the same PY, an individual can be resident of India as well as resident of a foreign
country. The mere fact that a person is resident of a foreign country will not per se lead
to a conclusion that he is a non-resident in India.
9 For the purpose of Explanation-1, going abroad for the purpose of employment means
going abroad to take up employment or any vocation, which takes in self-employment
like business or profession. [O. Abdul Razak (2011) (ker)].
10 If a person is resident in a PY in respect of any source of income, he shall be deemed to
be resident in India in the PY in respect of each of his other sources of income. [S. 6 (5)].

Explanation-2: How to determine period of stay in India for an Indian citizen, being a crew
member? In the case of an individual, being a citizen of India and a member of the crew of a
foreign bound ship leaving India, the period or periods of stay in India shall, in respect of
such voyage, be determined in the prescribed manner and subject to the prescribed
conditions.

R. 126: In case of an individual, being a citizen of India and a member of the crew of a ship,
the period or periods of stay in India shall, in respect of an eligible voyage, not include the
period commencing from the date entered into the Continuous Discharge Certificate in
respect of joining the ship by the said individual for the eligible voyage and ending on the
date entered into the Continuous Discharge Certificate in respect of signing off by that
individual from the ship in respect of such voyage.

Eligible voyage: A voyage undertaken by a ship engaged in the carriage of passengers or


freight in international traffic where- (i) for the voyage having originated from any port in
India, has as its destination any port outside India; and (ii) for the voyage having originated
from any port outside India, has as its destination any port in India.

MT Educare –CA Inter Income Tax Classes – R. SOUMYANARAYANAN. FCA. GRAD CWA. Page-29
Chapter-2: Residence & Scope of Total Income

Stage-2: Determining whether a resident individual is ordinarily resident or not – S.6 (6)
(a):

A resident individual is said to be resident and ordinarily resident if he satisfies both the
following conditions:

1 He is a resident in any 2 out of the last 10 PYs preceding the relevant PY; and
2 His total stay in India in the last 7 PYs preceding the relevant PY is 730 days or more.

Latest from Judiciary:

Mr. X was assessed as non-resident consistently from 1994-95 and visited India routinely,
but consciously did not intend treatment as resident Indian. However, during the PY 2019-
20, his stay in India exceeded 182 days because his passport had been impounded by CBI
rendering him unable to travel from India.

The Courts found this impounding of passport to be wrongful and the assessee had been
continuously trying to get passport released so that he could travel outside India to maintain
his NRI status. The assessee had virtually become an unwilling resident on Indian soil
without his consent and against his will. This involuntary stay of the assessee in India is to be
excluded for determining his residential status U/s 6. [Suresh Nanda (2015) (Del)].

2. Rules for determination of residential status of HUF:

Stage-1: Determining whether HUF is resident or not: S. 6 (2):

Control and management


of the affairs of HUF is

Wholly in India Wholly outside


Partly in India and
India
partly outside India

Resident
Non-resident
Resident

Points requiring attention:

1 For determining whether HUF is resident or not, the POS in India of karta has no
relevance.
2 There are two types of control- Dejure control & Defacto control. Dejure controller
means the person who is supposed to have control. De Facto controller means the
person who actually controls. For determination of residential status the situs of defacto
control is only relevant.
3 Although, it is normally the karta who has the control and management of the affairs of
HUF, yet another coparcener can control and manage the affairs. Therefore, the mere
fact of absence of karta from India does not make the family non-resident.
4 The expression ‘control and management’ refers to the central control and management
and not to the carrying on of day-to-day business by servants, employees or agents.

MT Educare –CA Inter Income Tax Classes – R. SOUMYANARAYANAN. FCA. GRAD CWA. Page-30
Chapter-2: Residence & Scope of Total Income

5 The control and management of HUF lies at the place where the vital decisions regarding
the affairs of HUF are taken. In other words, control and management of a business is
said to be situated at a place where the head and brain of the adventure is situated.

State-2: Determining whether a resident HUF is ordinarily resident or not: S. 6 (6) (b):

If karta complies with the following cumulative conditions, a resident HUF is regarded as
ordinarily resident:

Condition 1 Karta had been resident in India for atleast any 2 PY out of 10 PYs
immediately preceding the relevant PY.
Condition 2 Karta had stayed in India for atleast 730 days during 7 PYs immediately
preceding the relevant PY.

3. Rules for determination of residential status of companies – S. 6 (3):

A company would be resident in India in any PY, if

(i) It is an Indian company; or


(ii) Its place of effective management (POEM), in that PY, is in India.

Note: POEM means a place where key management and commercial decisions that are
necessary for the conduct of the business of an entity as a whole are, in substance made.
[Explanation to S. 6 (3)].

Question
ABC Inc., a Swedish company headquartered at Stockholm, not having a permanent
establishment in India, has set up a liaison office in Mumbai in April, 2019 in compliance
with RBI guidelines to look after its day to day business operations in India, spread
awareness about the company’s products and explore further opportunities. The liaison
office takes decisions relating to day to day routine operations and performs support
functions that are preparatory and auxiliary in nature. The significant management and

MT Educare –CA Inter Income Tax Classes – R. SOUMYANARAYANAN. FCA. GRAD CWA. Page-31
Chapter-2: Residence & Scope of Total Income

commercial decisions are, however, in substance made by the Board of Directors at Sweden.
Determine the residential status of ABC Inc. for AY 2020-21.

Solution:
ABC Inc. has only a liaison office in India through which it looks after its routine day to day
business operations in India. The place where decisions relating to day to day routine
operations are taken and support functions that are preparatory or auxiliary in nature are
performed are not relevant in determining the place of effective management. Hence, ABC
Inc., being a foreign company is a non-resident for AY 2020-21, since its place of effective
management is outside India in the PY 2019-20.

4. Rules for determination of residential status of other persons – S. 6 (2) & S. 6 (4):

Control and management


of affairs is situated

Wholly outside
Wholly in India Partly in India
India
and partly
outside India

Resident Non-Resident Resident

B. Scope of total income:

1. Classification of income based on the place of accrual and receipt:

Income

Indian Income Foreign Income

Whether income is Whether income


Nature of the
received in India during accrues in India during
Income
the PY the PY
Yes Yes Indian Income
Yes No Indian Income
No Yes Indian Income
No No Foreign Income

3. Incidence of tax for assessees (other than individual & HUF):


Resident in India Non-resident
India income Taxable in India Taxable in India
Foreign income Taxable in India Not taxable in India

MT Educare –CA Inter Income Tax Classes – R. SOUMYANARAYANAN. FCA. GRAD CWA. Page-32
Chapter-2: Residence & Scope of Total Income

3. Incidence of tax for individual and HUF:

Resident & Resident but not


Non-resident in
Ordinarily resident in ordinarily resident in
India
India India
Indian Income Taxable in India Taxable in India Taxable in India
Foreign income
If it is business income and
the business is controlled Taxable in India Taxable in India Not taxable in India
wholly or partly from India
If it is income from
profession which is set up Taxable in India Taxable in India Not taxable in India
in India
Any other foreign income Taxable in India Not taxable in India Not taxable in India

Points requiring attention:

1 ‘Accrue’ refers to the right to receive income, whereas ‘due’ refers to the right to
enforce payment of the same.
2 Example-1: salary for work done in December will accrue throughout the month, day to
day, but will become due for payment on salary bill being passed on 31st December or 1st
January.
3 Example-2: Interest on Government securities is usually payable on specified dates, say
on 1st January and 1st July. In all such cases, the interest would be said to accrue from 1 st
July to 31st December and on 1st January, it will fall due for payment.
4 Receipt of income refers to only the first occasion when the recipient gets the money
under his control. Therefore, when once an amount is received as income, remittance or
transmission of that amount from one place or person to another does not constitute
receipt of income in the hands of the subsequent recipient or at the place of subsequent
receipt.
5 To come within the scope of total income of the relevant PY, the income should have
either accrued during the relevant PY or should have been received during the relevant
PY. Where neither accrual nor receipt has taken place during the PY, such income would
not come within the scope of total income of the relevant PY.
6 Explantion-1 to S. 5: An item of income accruing outside India shall not be deemed to be
received in India merely because it is taken into account in a balance sheet prepared in
India.
7 Explanation-2 to S. 5: Once an item of income is included in the assessee’s total income
and subjected to tax on the ground of its accrual or receipt, it cannot again be included
in the person’s total income and subjected to tax either in the same or in a subsequent
year on the ground of receipt.

MT Educare –CA Inter Income Tax Classes – R. SOUMYANARAYANAN. FCA. GRAD CWA. Page-33
Chapter-2: Residence & Scope of Total Income

C. Circumstances in which income is deemed to accrue or arise in India – S. 9:

S. 9 (1) (i) Income from It is deemed to accrue or arise in India. Therefore, it is an Indian
Limb-1 business income. Hence, it is taxable irrespective of the residential status.
connection in [Detail discussion is at the end].
India
S. 9 (1) (i) Income from Any income which arises from any property (movable, immovable,
Limb-2 & property, asset tangible or intangible) would be deemed to accrue or arise in India.
Limb-3 or source in India Therefore, it is an Indian income. Hence, it is taxable irrespective of
the residential status.
Examples (a) Hire charges or rent paid outside India for the use of machinery or
building situated in India; (b) Interest on deposits with an Indian
company which is received outside India.
S. 9 (1) (i) Income through Income accruing or arising through transfer of capital asset situated in
Limb-4 transfer of India is deemed to accrue or arise in India. Therefore, it is an Indian
capital asset income. Hence, it is taxable irrespective of the residential status.
situated in India The capital asset may be movable or immovable; tangible or
intangible. The place of payment of the consideration for transfer may
be within India or outside India. These are not relevant.
Shares in foreign companies are deemed to be located in India if they
derive their value substantially from assets located in India.
However, dividend declared and paid by such foreign companies
outside India shall not be deemed to accrue or arise in India. [CBDT
circular 4/2015].
S. 9 (1) Salary earned in Income, which falls U/H Salaries, is deemed to accrue or arise in India,
(ii) India if it is earned in India.
Salary payable for service rendered in India would be treated as
earned in India.
Where contract of employment was entered into is not relevant;
where the employment is exercised is relevant. That is, where the
services are rendered is relevant.
Any income by way of salaries payable for the rest period or leave
period which is preceded and succeeded by services rendered in
India, and forms part of the service contract of employment, shall be
regarded as income earned in India.
Even pension paid outside India by the former employer for the past
services rendered in India falls within the ambit of S. 9 (1) (ii).
However, S. 9 (2) provides that the pension payable outside India by
the Government to its officials and Judges who permanently reside
outside India shall not be deemed to accrue or arise in India.
S. 9 (1) Salary payable by Income by way of salaries which is payable by the Government to a
(iii) Government for citizen of India for services rendered outside India would deemed to
services accrue or arise in India.
rendered outside However, allowances and perquisites paid outside India by the
India Government is exempt U/s 10 (7).

MT Educare –CA Inter Income Tax Classes – R. SOUMYANARAYANAN. FCA. GRAD CWA. Page-34
Chapter-2: Residence & Scope of Total Income

S. 9 (1) Dividend paid by All dividend paid by an Indian company must be deemed to accrue or
(iv) Indian company arise in India.
outside India However, such dividends are exempt in the hands of shareholders U/s
10 (34).
But the company has to pay DDT U/s 115-O.

Interest – S. 9 (1) (v):

S. 9 (1) (v) (a) Interest payable by the Government is deemed to accrue or arise in India.
S. 9 (1) (v) (b) Interest payable by a resident is deemed to accrue or arise in India.
Exceptions Where it is payable in respect of money borrowed and used for the purposes of
business or profession carried on by him outside India or for the purpose of making or
earning any income from any source outside India, it will not be deemed to accrue or
arise in India.
S. 9 (1) (v) (c) Interest payable by a non-resident is deemed to accrue or arise in India provided it is
payable in respect of money borrowed and used for the purposes of business or
profession carried on in India by him.

Note: Interest on money borrowed by the non-resident for any purpose other than business
or profession will not be deemed to accrue or arise in India.

Example: If A (non-resident) borrows money from B (non-resident) and invests the same in
shares of an Indian company, interest payable by A to B will not be deemed to accrue or
arise in India.

Royalty: S. 9 (1) (vi):

S. 9 (1) (vi) (a) Royalty payable by the Government is deemed to accrue or arise in India.
S. 9 (1) (vi) (b) Royalty payable by a resident is deemed to accrue or arise in India.
Exceptions Where it is payable in respect of any right or property or information used for the
purpose of business or profession carried on by such resident outside India or for
the purpose of making or earning any income from any source outside India, it shall
not be regarded as accruing or arising in India.
S. 9 (1) (vi) (c) Royalty payable by a non-resident is deemed to accrue or arise in India if it is
payable in respect of any right or property or information used for the purpose of
business or profession carried on by such non-resident in India or for the purpose
of making or earning any income from any source in India.

Meaning of royalty: S. 9 (1) (vi) Explanation-2:

Royalty means consideration for


(a) Use of patent, invention, model, design, secret formula, process, trademark or similar property;
(b) Imparting of any information concerning the working of or use of patent, invention, model,
design, secret formula, process, trademark or similar property;
(c) Use of rights in copyright in any literary, artistic or scientific work;

MT Educare –CA Inter Income Tax Classes – R. SOUMYANARAYANAN. FCA. GRAD CWA. Page-35
Chapter-2: Residence & Scope of Total Income

(d) Imparting of any information concerning technical, industrial, commercial or scientific


knowledge, experience or skills;
(e) Use of industrial, commercial or scientific equipment
(f) Rendering of any services in connection with the activities listed above

Points requiring attention:

1 Payments made for securing the use of computer software amounts to royalty. [Explanation-4 to
S. 9 (1) (vi)].
2 However, lumpsum royalty payments made by a resident for transfer of rights in respect of a
computer software supplied by a non-resident along with computer hardware under any scheme
approved by the Government under the policy on computer software export, software
development and training, 1986 shall not be deemed to accrue or arise in India.
3 ‘Process’ used in the definition (supra) includes transmission by satellite (including up-linking,
amplification, conversion for down-linking of any signal), cable, optic fiber or by any other similar
technology, whether or not such process is secret. [Explanation-6 to S. 9 (1) (vi)].
4 Royalty includes consideration in respect of any right, property or information, whether or not (a)
the possession or control of such right, property or information is with the payer; (b) such right,
property or information is used directly by the payer; (c) the location of such right, property or
information is in India. [Explanation-5 to S. 9 (1) (vi)].

Fee for technical services (FTS): S. 9 (1) (vii):

S. 9 (1) (vii) (a) FTS payable by the Government is deemed to accrue or arise in India.
S. 9 (1) (vii) (b) FTS payable by a resident is deemed to accrue or arise in India.
Exceptions Where it is payable in respect of services utilised in a business or profession
carried on by such resident outside India or for the purpose of making or earning
any income from any source outside India, it shall not be regarded as accruing or
arising in India.
S. 9 (1) (vii) (c) FTS payable by a non-resident is deemed to accrue or arise in India if it is
payable in respect of services utilised in a business or profession carried on by
such non-resident in India or for the purpose of making or earning any income
from any source in India.

Meaning of FTS: Explanation-2 to S. 9 (1) (vii):


(i) FTS means any consideration for rendering of any managerial, technical or consultancy services.
(ii) It also includes consideration for providing the services of technical or other personnel.
(ii) However, it excludes the following (a) any consideration which would be income of the
recipient chargeable U/H Salaries; (b) any consideration for any construction, assembly, mining
or like project undertaken by the recipient.

MT Educare –CA Inter Income Tax Classes – R. SOUMYANARAYANAN. FCA. GRAD CWA. Page-36
Chapter-2: Residence & Scope of Total Income

Territorial nexus not relevant for taxing interest, royalty & FTS – Explanation to S. 9:

Income by way of interest, royalty or FTS which is deemed to accrue or arise in India by
virtue of S. 9 (1) (v)/ (vi)/ (vii), shall be included in the TI of the non-resident, whether or not:

(a) the non-resident has a residence or place of business or business connection in India;
or
(b) the non-resident has rendered services in India.

Business connection – Explanation-2 to S. 9 (1) (i):

Business connection shall include any business activity carried out through a person acting
on behalf of the non-resident. For business connection to be established

(i) the person acting on behalf of the non-resident should have and must habitually
exercise authority to conclude contracts on behalf of the non-resident; or
(ii) Where he has no such authority, he must habitually maintain in India a stock of goods
or merchandise from which he regularly delivers goods or merchandise on behalf of
non-resident; or
(iii) He must habitually secure orders in India, mainly or wholly for the non-resident or for
that non-resident and other non-residents controlling, controlled by, or subject to the
same common control as that of non-resident;

Points requiring attention:

1 Where the activities of the person acting on behalf of non-resident are restricted to
purchase of goods or merchandise for the non-resident, it does not amount to business
connection.
2 Business activities carried out through broker or agent having an independent status,
who is acting in the ordinary course of his business, do not amount to business
connection. [Proviso-1 to Explanation-2 to S. 9 (1) (i)].
3 If a broker or agent works mainly or wholly on behalf of a non-resident or on behalf of
such non-resident and other non-residents controlling, controlled by, or subject to the
same common control as that of non-resident, he shall not be deemed to be broker or
agent of an independent status. [Proviso-2 to Explanation-2 to S. 9 (1) (i)].
4 In case of a business of which all the operations are not carried out in India, the income
of the business deemed to accrue or arise in India shall be only such part of income as is
reasonably attributable to the operations carried out in India. Therefore, it follows that
such part of income which cannot be reasonably attributed to the operations in India, is
not deemed to accrue or arise in India. [Explanation-1 (a) to S. 9 (1) (i)].

MT Educare –CA Inter Income Tax Classes – R. SOUMYANARAYANAN. FCA. GRAD CWA. Page-37
Chapter-2: Residence & Scope of Total Income

Gifts: S. 9 (1) (vi):


New clause (viii) is inserted in Section 9(1) – Section 9(1)(viii) - Deemed income in
respect of Gift [w.e.f. 05 July 2019 as inserted by Finance (No. 2) Act 2019]

Income arising outside India, being any sum of money (Only money) as referred to in
Section 56(2)(x), paid on or after the 5th day of July, 2019 by a person resident in India
to a non-resident, not being a company, or to a foreign company.

Example: Gift of Rs.60,000 received on 10th July 2019 by Non-resident outside India
from a Resident person is now an income deemed to accrue or arise in India and thus
taxable.

There is 366 days in PY 2019-20 as February 2020 has 29 days. Take care while
calculating stay period in India for PY 2019-20.

Following not to constitute business connection in India:

1 Purchase of goods In case of a non-resident, no income shall be deemed to accrue or arise in


in India for export India to him through or from operations which are confined to the purchase
of goods in India for the purpose of export. [Explanation-1 (b) to s. 9 (1) (i)].
2 Collection of news In case of a non-resident, being a person engaged in the business of running
and views in India a news agency or of publishing newspapers, magazines or journals, no
for transmission income shall be deemed to accrue or arise in India to him through or from
out of India activities which are confined to the collection of news and views in India for
transmission out of India. [Explanation-1 (c) to S. 9 (1) (i)].
3 Shooting of In case of non-resident, no income shall be deemed to accrue or arise in
cinematograph India through or from operations which are confined to the shooting of any
films in India cinematograph film in India.
If such non-resident is an individual, he should not be a citizen of India.
If such non-resident is a firm, it should not have any partner who is citizen
of India or who is resident in India.
If such non-resident is a company, it should not have any shareholder who
is citizen of India or who is resident in India. [Explanation-1 (d) to s. 9 (1) (i)].
4 Activities confined In case of a foreign company engaged in the business of mining of
to display of rough diamonds, no income shall be deemed to accrue or arise in India to it
diamonds in through or from the activities which are confined to display of uncut and
special notified unassorted diamonds in any special zone notified by the Central
zones Government in the official gazette in this behalf. [Explanation-1 (e) to s. 9
(1) (i)].

MT Educare –CA Inter Income Tax Classes – R. SOUMYANARAYANAN. FCA. GRAD CWA. Page-38
Chapter-2: Residence & Scope of Total Income

Summary of S. 9 (1):

Income from any Capital Gains on


Property, Asset or transfer of
Income from Salary for
Source of Income Capital Asset
Business services in
in India situated in India
Connection India
(Sec.9(1)(i)) (Sec.9(1)(i)
(Sec. 9(1)(i)) (Sec.9(1)(ii)

Income by way
Salary to Govt.
of technical fees
employee
(in certain Income deemed to accrue or arise in India
(citizen of India)
cases). (Sec.
for services
9(1)(vii)
outside India.
(Sec.9(1)(iii)
Income by
Income by way Dividend by
way of
of Royalty (in Indian
Interest (in
certain cases). Companies.
certain cases).
(Sec. 9(1)(vi) (Sec. 9(1)(iv)
(Sec 9(1)(v)

Nature of income Whether income is


deemed to accrue
or arise in India
Income from business connection in India (S. 9 (1) (i)) Yes
Income from any property, asset or source of income in India (S. 9 (1) (i)) Yes
Capital gain on transfer of a capital asset situated in India (S. 9 (1) (i) Yes
Income by way of salary for service rendered in India (S. 9 (1) (ii)) Yes
Income from salary (not being perquisite/allowance) if service is rendered Yes
outside India (provided the employer is Government and the employee is a
citizen of India) (S. 9 (1) (iii))
Dividend paid by the India company (S. 9 (1) (iv)) Yes [Exempt U/s 10
(34)]

Nature of From whom Remarks Accrues in


income income is received India
Interest [S. 9 Government of - Yes
(1) (v) (a)] India
Interest [S. 9 Resident in India Borrowed capital is used by the payer for No
(1) (v) (b)] carrying on business/profession outside
India or earning any income outside India
Interest [S. 9 Resident in India Borrowed capital is used by the payer for Yes
(1) (v) (b)] any other purpose
Interest [S. 9 Non-resident Borrowed capital is used by the payer for Yes
(1) (v) (c)] carrying on business/profession in India
Interest [S. 9 Non-resident Borrowed capital is used by the payer for No
(1) (v) (c)] any other purpose

MT Educare –CA Inter Income Tax Classes – R. SOUMYANARAYANAN. FCA. GRAD CWA. Page-39
Chapter-2: Residence & Scope of Total Income

Royalty [S. 9 (1) Government of - Yes


(vi) (a)] /FTS [S. India
9 (1) (vii) (a)]
Royalty [S. 9 (1) Resident in India Payment is relatable to a business of No
(vi) (b)] /FTS [S. profession or any other source carried on
9 (1) (vii) (b)] by the payer outside India
Royalty [S. 9 (1) Resident in India Payment is relatable to any other source of Yes
(vi) (b)] /FTS [S. income
9 (1) (vii) (b)]
Royalty [S. 9 (1) Non-resident Payment is relatable to a business or Yes
(vi) (c)] /FTS [S. profession or any other source carried by
9 (1) (vii) (c)] the payer in India
Royalty [S. 9 (1) Non-resident Payment is relatable to any other source of No
(vi) (c)] /FTS [S. income.
9 (1) (vii) (c)]

Income deemed to be received – S. 7:

1 Employer’s contribution to RPF in excess of 12% of Salary


2 Interest on RPF Credit Balance in excess of 9.5% p.a.
3 Amount transferred from URPF to RPF to the extent taxable as per rule 11(4) of part A of
the IV scheduled.
4 Contribution by Central Government or any other employer to the account of an
employee under a pension scheme referred to in S. 80CCD.
5 TDS is deemed to be received in the hands of the payee. [S. 198]

MT Educare –CA Inter Income Tax Classes – R. SOUMYANARAYANAN. FCA. GRAD CWA. Page-40
Chapter-3: Income which do not form part of total income:

Learning objectives:

1 Difference between exemption and deduction


2 Income which are exempted U/s 10
3 Disallowance of expenses attributable to exempted income – S. 14A & R. 8D
4 Deduction U/s 10AA

Exemption Vs Deduction:

1 There are various items of income which are excluded from the total income of the
assessee. These are referred to in S. 10. These are known as exempted income. These do
not enter into the computation of taxable income.
2 There are certain other incomes which are included in computation of gross total income
but are wholly are partly allowed as deductions under Chapter VI-A in computation of
total income.
3 The difference between deduction and exemption shall be properly appreciated.

Income which do not form part of total income: (S. 10):

SN Section Income exempted


1 S. 10 (1) Agricultural income is exempted. [Discussed already in Basic concepts].
2 S. 10 (2) Since HUF is taxed in respect of its income, the share income is exempt
from tax in the hands of member. [Discussed already in Basic concepts].
3 S. 10 (2A) Partner’s share in the total income of the firm or LLP is exempt from tax.
[Discussed already in Basic concepts].
CBDT Income of a firm is to be taxed in the hands of the firm only and the same
circular can under no circumstances be taxed in the hands of its partners.
8/2014 Therefore, the entire profit credited to the partner’s accounts in the firm
would be exempted from tax in the hands of such partners, even if the
income chargeable to tax becomes Nil in the hands of the firm on account
or deduction available under the provisions of the Act.
4 S. 10 (4) Income by way of interest on moneys standing to his credit in NRE
(ii) account, is exempt in the hands of an individual, being a person resident
outside India as per FEMA, 1999 or in the hands of an individual who has
been permitted by the RBI to maintain such account.
Joint holders of the NRE account do not constitute an AOP by merely
having these accounts in joint names. The benefit of exemption U/s 10 (4)
(ii) will be available to such joint account holders, subject to fulfillment of
others conditions contained in the section by each of the individual joint
account holders.
5 S. 10 (6) The remuneration received by a person for services as an official of an
(ii) embassy, high commission, legation, consulate or the trade
representation of a foreign State or as a member of the staff of any of
these officials is exempt subject to the following conditions.
Conditions: (a) The remuneration received by our corresponding
Government officers resident in such foreign countries should be exempt;

MT Educare –CA Inter Income Tax Classes – R. SOUMYANARAYANAN. FCA. GRAD CWA. Page-41
Chapter-3: Income which do not form part of total income:

(b) The above-mentioned officers should be the subjects of the respective


countries and should not be engaged in any other business or profession
or employment in India.
6 S. 10 (6) Remuneration received by a foreign national as an employee of a foreign
(vi) enterprise for service rendered by him during his stay in India is also
exempt from tax subject to the following conditions.
Conditions: (a) The foreign enterprise is not engaged in a business activity
in India; (b) The employee’s stay in India does not exceed 90 days in the
PY; (c) The remuneration is not liable to be deducted from the employer’s
income chargeable to tax under the Act.
7 S. 10 (6) Salary income received by or due to non-citizen of India who is also non-
(viii) resident for services rendered in connection with his employment on a
foreign ship is exempt where his total stay in India does not exceed 90
days in the PY.
8 S. 10 (6) Any remuneration received by employees of foreign Government from
(xi) their respective Government during their stay in connection with their
training in any establishment or office of the Government or any public
sector undertaking is exempt from tax.
The expression ‘Public sector undertaking’ will cover statutory
corporations, companies wholly owned by the CG or SG or jointly by the
CG and SG, subsidiaries of such companies and societies registered under
the Societies Registration Act, 1860 or any other similar law, which are
wholly financed by the CG or SG or jointly by the CG or SG.
9 S. 10 Any payment made to a person under Bhopal Gas Leak Disaster
(10BB) (Processing of claims) Act 1985 and any scheme framed thereunder will be
fully exempt.
10 S. 10 Any amount received or receivable as compensation by an individual or
(10BC) his legal heir on account disaster is exempt.
Such compensation should be granted by the CG or SG or a local authority.
However, exemption would not be available in respect of compensation
for alleviating any damage or loss, which has already been allowed as
deduction under the Act.
‘Disaster’ means a catastrophe, mishap, calamity or grave occurrence in
any area, arising from natural or manmade causes, or by accident or
negligence. It should have the effect of causing: (a) substantial loss of life
or human suffering; or (b) damage to, and destruction, of property; or (c)
damage to, or degradation of, environment.
It should be of such a nature or magnitude as to be beyond the coping
capacity of the community of the affected area.
11 S. 10 Any payment from an account opened in accordance with Sukanya
(11A) Samriddhi Account Rules, 2014 made under the Government Savings Bank
Act, 1873, shall not be included in the total income of the assessee.
Accordingly, the interest accruing on deposits in, and withdrawals from
any account under the said scheme would be exempt.
12 S. 10(12A) Any payment from the National Pension System Trust to an assessee on
closure of his account or on his opting out of the pension scheme referred

MT Educare –CA Inter Income Tax Classes – R. SOUMYANARAYANAN. FCA. GRAD CWA. Page-42
Chapter-3: Income which do not form part of total income:

to in sec. 80CCD, to the extent it does not exceed 60% of the total amount
payable to him at the time of such closure or his opting out of the scheme.
(Increased from 40% to 60% as amended by Finance (No. 2) Act, 2019.)
13 S. 10 (16) The value of scholarship granted to meet the cost of education would be
exempt from tax in the hands of the recipient irrespective of the amount
or source of scholarship.
14 S. 10 (17) The following incomes of MPs / MLAs will be exempt:
(a) Daily allowance received by any MP or MLA.
(b) Constituency allowance received by any MP or MLA.
15 S. 10 Any award instituted in the public interest by the CG or SG or any body
(17A) approved by the CG and a reward by CG or SG for such purposes as may
be approved by the CG in public interest, is exempt from tax.
16 S. 10 (18) Any income by way of pension received by an individual who has been
awarded ‘Param Vir Chakra’ or ‘Maha Vir Chakra’ or ‘Vir Chakra’ or such
other gallantry award as the CG may, by notification in the official gazette,
specify in this behalf.
In case of death of the awardee, any income by way of family pension
received by any member of the family of the individual shall also be
exempt U/s 10 (18).
Family, in relation to an individual means (a) spouse (b) Children (c)
parents, brothers and sisters of the individual or any of them wholly or
mainly dependent on the individual.
17 S. 10 (26) A member of a scheduled tribe residing in (i) any area (specified in the
Constitution); (ii) the states of Manipur, Tripura, Arunachal Pradesh,
Mizoram and Nagaland; (iii) ladakh region of the State of Jammu and
Kashmir is exempt from tax on his income arising or accruing (a) from any
source in the areas or states aforesaid; (b) by way of dividend or interest
on securities.

18 S. 10 The following income, which accrues or arises to a Sikkimese individual,


(26AAA) would be exempt from income tax: (a) income from any source in the
State of Sikkim; or (b) income by way of dividend or interest on securities.
However, this exemption will not be available to a Sikkimese woman who,
on or after 01.04.2008, marries a non-Sikkimese individual.
19 S. 10 (30) Any amount of subsidy received by any assessee engaged in the business
of growing and manufacturing tea in India through or from the Tea Board
will be wholly exempt from tax subject to the following conditions.
Conditions:

(a) The subsidy should have been received under any scheme for
replantation or replacement of the bushes or for rejuvenation or
consolidation of areas used for cultivation of tea, as notified by the CG;

(b) The assessee should furnish a certificate from the Tea Board, as to the
subsidy received by him during the PY, to the AO along with his return of
the relevant AY or within the time extended by the AO for this purpose.

MT Educare –CA Inter Income Tax Classes – R. SOUMYANARAYANAN. FCA. GRAD CWA. Page-43
Chapter-3: Income which do not form part of total income:

20 S. 10 (31) Amount of any subsidy received by an assessee engaged in the business of


growing and manufacturing rubber, coffee, cardamom or other specified
commodity in India, as notified by the CG, will be wholly exempt from tax
subject to the following conditions.
Conditions:

(a) The subsidies should have been received from or through the Rubber
Board, Spices Board or any other Board in respect of any other commodity
under any scheme for replantation or replacement of rubber, coffee,
cardamom or other plants or for rejuvenation or consolidation of areas
used for cultivation of all such commodities;

(b) The assessee should furnish a certificate from the Board, as to the
subsidy received by him during the PY, to the AO along with his return of
the relevant AY or within the time extended by the AO for this purpose.

20 S. 10 (35) Income from the units of mutual fund is exempt.

Restriction on allowability of expenditure – S. 14A:

S. 14A (1) Any expenditure incurred in relation to any exempt income is not allowed as
deduction while computing income under any of the five heads of income.
S. 14A (2) If the AO is not satisfied with the correctness of the claim of the assessee
& S. 14A regarding the expenditure incurred in relation to the exempt income, having
(3) regard to the accounts of the assessee or where the assessee claims that no
expenditure has been incurred in relation to exempt income, then the AO may
use the method prescribed in R. 8D for determining the expenditure to the
disallowed U/s 14A.

Quantification of disallowance U/R 8D:

The expenditure in relation to exempt income shall be the aggregate of the following and
shall be disallowed:

(i) The amount of expenditure directly relating to exempt income.


(ii) An amount equal to 1% of the annual average of the monthly averages of the opening
and closing balances of the value of investment, income from which does not form part
of total income.

Note: The amount to be disallowed U/R 8D shall not exceed the total expenditure claimed
by the assessee.

MT Educare –CA Inter Income Tax Classes – R. SOUMYANARAYANAN. FCA. GRAD CWA. Page-44
Chapter-3: Income which do not form part of total income:

CBDT Circular 5/2014:

1 The legislative intent is to allow only that expenditure which is relatable to earning of
taxable income.
2 Therefore, it follows that the expenses which are relatable to earning of exempt income
have to be considered for disallowance, irrespective of the fact whether such income has
been earned during the FY or not.
3 In effect, S. 14A read along with R. 8D provides for disallowance of expenditure even
where the taxpayer has not earned any exempt income in a particular year.

Tax holidays for newly established units in SEZ [S. 10AA]:

1 Person eligible for tax Any assessee who derives profits or gains from an undertaking
holiday U/s 10AA (located in SEZ) which is engaged in the export of article or
things (including computer software) or providing any service
is eligible for deduction U/s 10AA. [S. 10AA (1)].
Note: The undertaking should have commenced manufacture
or production on or after 01.04.2005 but before 01.04.2020.
2 Quantum of deduction and the period over which it is available (S. 10AA (1))
(i) First 5 AYs beginning 100% of profits and gains derived from export of articles or
with the AY relevant things.
to the PY in which
undertaking begins to
manufacture or
produce
(ii) Next 5 Consecutive 50% of profits and gains derived from export of articles or
AYs things.
(iii) Next 5 Consecutive Amount transferred to SEZ reinvestment reserve account
AYs (Ceiling: 50% of profits and gains derived from export of
articles or things).
3 Manner of utilization Amount transferred to SEZ re-investment reserve shall be
of amount lying in SEZ utilised for acquiring plant and machinery.
reinvestment reserve Such P&M shall be first put to use within 3 years from the end
of the PY in which the reserve was created.
Until the P&M is acquired, the amount lying in the reserve
account can be used for the purpose of business of the
undertaking.
However, it should not be used for (a) distribution by way of
dividend or profits; (b) acquisition of assets outside India.
Assessee shall furnish the particulars of P&M which was put to
use in Form-56FF, along with the ROI for the AY relevant to
the PY of usage. [S. 10AA (2)].
4 Consequences of mis- Amount mis-utilised or non-utilised shall be charged to tax in
utilisation or non- the PY succeeding the PY in which aforesaid 3 years expire. [S.
utilisation 10AA (3)].

MT Educare –CA Inter Income Tax Classes – R. SOUMYANARAYANAN. FCA. GRAD CWA. Page-45
Chapter-3: Income which do not form part of total income:

5 Conditions for (i) The undertaking is not formed by splitting up or


availing deduction reconstruction of a business which is already in existence.
U/s 10AA (ii) The undertaking is not formed by transfer to new business
of P&M previously used for any purpose. [S. 10AA (4) ].
6 Relaxation of If the aggregate value of all second hand machines used for
condition-2 setting up the undertaking does not exceed 20% of the
aggregate value of all P&M used for setting up the
undertaking, condition-2 is deemed to have been complied
with. [S. 10AA (4)].
7 Imported second Any machine which was put to use for any other purpose
hand machine – not earlier shall also be regarded as brand new machine if the
to be regarded as following conditions are satisfied:
second hand machine (a) it is imported from abroad;
(b) it was not put use in India before its installation by the
assessee;
(c) no deduction by way of depreciation was allowed under
this Act in respect of such machine to any person earlier. [S.
10AA (4)].
8 Continuance of Even if the undertaking in SEZ is transferred pursuant to a
deduction U/s 10AA scheme of amalgamation or demerger, the deduction U/s
even after 10AA shall continue in the hands of amalgamated company or
amalgamation or resulting company for the unexpired residual period beginning
demerger with the PY of amalgamation or demerger, as the case may be.
[S. 10AA (7A)].
9 Carry forward of If the business loss or capital loss of the undertaking remains
losses allowed unabsorbed, it could be carry forward U/s 72 or S. 74, as the
case may be. [S. 10AA (6)].
10 Profits and gains (Profits of undertaking in SEZ * Export turnover of undertaking
derived from export in SEZ)/Total turnover of undertaking in SEZ. [S. 10AA (7)].
of such undertaking in
SEZ
11 Report of CA The assessee should furnish, along with the ROI, the report (In
Form 56G) of a CA certifying that the deduction has been
correctly claimed. [S.10AA (5)].
12 Bar on overlapping Assessee claiming deduction U/s 10AA shall not be entitled to
deduction claim deduction under Chapter VI-A (Part-C) & S. 35AD in
respect of profits of the undertaking located in SEZ. (S. 80A (4)
+ S. 10AA (10)

MT Educare –CA Inter Income Tax Classes – R. SOUMYANARAYANAN. FCA. GRAD CWA. Page-46
Chapter-4: Income from house property

Segment-1: Charging section – S. 22:

Annual value of property consisting of building or land appurtenant thereto of which the
assessee is the owner other than such portion of such property as he may occupy for the
purpose of any business or profession carried on by him the profits of which are chargeable
to tax, shall be chargeable to tax U/H IFHP.

Points requiring attention:

1 The word ‘building’ is wide enough to include residential house (whether let out or self
occupied), building let out for office use or for storage or for use as factory, music halls,
dance halls, lecture halls, shopping complex, malls, public auditoriums used for cinema
and stage shows etc.
2 Structure in ruins cannot be regarded as building.
3 Incomplete structure (structure under construction) is also not to be regarded as
building.
4 ‘Land appurtenant to building’ means land adjacent to building which is necessary for
effective enjoyment of building such as approach road connecting public streets,
parking space, backyard, courtyard, kitchen garden, cattle shed, stable, motor garage
playground (tennis court) etc.
Appurtenant Land

W.r.t residential Building W.r.t non- residential Building

➢ Approach roads to/from ➢ Car-Parking


public streets Spaces
➢ Compounds ➢ Play Ground for
➢ Court yards benefit of
➢ Back Yard employees
➢ Play ground ➢ Road-Connecting
➢ Kitchen -garden one department
➢ Motor Garage with another
➢ Stable
➢ Coach-Home
➢ Cattle Shed
5 What is covered by S. 22 is land appurtenant to building and not vacant land.
Therefore, income from letting of vacant land doesn’t come U/H IFHP. It comes U/H
IFOS. However, if the assessee is engaged in the business of letting out plots of land on
lease, the income from such letting will be charged to tax U/H PGBP. [Chowdhury
Sharafat Hussain Vs CIT 29 ITR 759 (Pat)].
6 Ownership is vital for invoking IFHP. Income from subletting is not to be taxed U/H
IFHP. It should be taxed U/H IFOS.
Lets out HP
A to Sublets to
B C

Rent
Rent
Not chargeable to tax u/s 22
Chargeable
to tax u/s 22
It is chargeable either u/s 28/56

MT Educare –CA Inter Income Tax Classes – R. SOUMYANARAYANAN. FCA. GRAD CWA. Page-47
Chapter-4: Income from house property

7 The assessee who owns the building need not also be the owner of the land upon
which it stands. [Puttanna sons (p) Ltd Vs CIT 162 ITR 468 (Kar)]
8 If the issue of ownership over the property is in dispute in a Court of law, the decision
as to who will be the owner chargeable to income-tax U/s 22 will be of the IT
Department till the Court gives its decision to the suit filed in respect of such property.
9 In certain cases, though assessee is not the owner of the house property, he will
regarded as owner and the annual value of the house property will be taxed in his
hands. The cases of deemed ownership are covered U/s 27. [Discussed later].
10 Annual value means the sum for which the property is expected to be let from time to
time. That is, the inherent rent-earning capacity of the house expressed in monetary
terms.
11 Even if the house is let for a sum which is less than its annual value, what is taken into
consideration while computing income chargeable U/H IFHP is the annual value and not
the rent actually earned. Further, even if the property is not let out, still annual value of
the property could be brought to tax under this head. Since a notional income is
charged to tax under this head, it is an exception to real income theory. Accordingly,
the method of accounting is not relevant vis-à-vis the head IFHP.
12 However, if the house property is used by the assessee for the purpose of business or
profession carried on by him, then S. 22 exempts the annual value from taxation.

Pictorial presentation of Charging section:

Property = No
Income from such property is
Building/ not taxable under this head
appurtenant
land?
Yes

Assessee =
No
owner of such
property?

Yes

Such property used No


Income from such property
for the assessee’s is taxable under this head
business/profession?

Yes

Profits from such No


business/profession
chargeable to tax?

Yes
Income from such property is
not taxable under this head

MT Educare –CA Inter Income Tax Classes – R. SOUMYANARAYANAN. FCA. GRAD CWA. Page-48
Chapter-4: Income from house property

Note:

1 Where the main objective of the assessee-company as per its memorandum of


association is to acquire, hold and let out properties and earn rental income, the letting
of properties constitute business. Accordingly, the rental income shall be assessed to tax
U/H PGBP. [Chennai Properties and Investments Ltd (2015) (SC)].
2 The aforesaid decision holds good even if acquiring, holding and letting of properties is
one among many of the objectives listed in memorandum of association and not the
main objective. [Rayala corporation (P) Ltd (2016) (SC)].
3 Annual value of house property will be charged to tax U/H IFHP even if it is held by the
assessee as stock-in-trade of his business.

Deemed ownership – s. 27:

A person having legal title over the property is regarded as owner. In other words, any
person who gets a valid title legally conveyed to him after complying with the requirements
of law such as the transfer of property Act, the registration Act etc, is regarded as owner.

However, in certain circumstances, certain persons are deemed to be the owners of the
property though they don’t have legal title over it. Such circumstances are contemplated in
S. 27. In other words, S. 27 deals with deemed ownership.

Deemed owners:

(i) Individual transferring a property to his spouse for inadequate consideration – S. 27 (i):

An individual transferring a house property, is still, deemed to be the owner if the following
conditions are satisfied:

1 Transferee=spouse of the individual.


2 Transfer is not in connection with an arrangement to live apart.
3 Transfer is without adequate consideration.

Points requiring attention:

1 Spouse means legally wedded wife or husband.


2 Consideration which is not measurable in monetary terms cannot be regarded as
adequate consideration. Example: Love and affection, consent to marry, consent to
adopt a child etc.
3 Where an individual transfers cash to his spouse who uses the same for purchase of a
house property, transferor-spouse is not hit by S. 27 (i) and accordingly, he shall not be
regarded as the owner of the house property. However, the income from such house
property will still be clubbed in the hands of transferor-spouse as per S. 64 (1) (iv).

MT Educare –CA Inter Income Tax Classes – R. SOUMYANARAYANAN. FCA. GRAD CWA. Page-49
Chapter-4: Income from house property

(ii) Individual transferring a house property to his minor child for inadequate
consideration – S. 27 (i):

An individual transferring a house property, is still, deemed to be the owner of such house
property if the following conditions are satisfied:

1 Transferee=minor child not being a married minor daughter.


2 Transfer is without adequate consideration.

Points requiring attention:

1 The term child means only legitimate child.


2 It also includes adopted child and step child. [S. 2 (15B)].
3 Where an assessee transfers his house property for adequate consideration to his minor
child, he is not deemed to be the owner as per S. 27 (i). However, the income from such
property will still be clubbed in the hands of the assessee by virtue of S. 64 (1A).
However, the assessee is entitled to get exemption U/s 10 (32).
4 The position will be the same even if he transfers his house property to his minor
married daughter for an inadequate consideration.

(iii) Holder of impartible estate – S. 27 (ii):

As per S. 27 (ii), holder of an impartible estate is deemed to be the owner of house property.
However, after the enactment of the Hindu succession Act, 1956, this clause becomes
inoperative since, the impartible estate by custom does not have a legal sanction.

(iv) Member of a co-operative society etc – S. 27 (iii):

A member of a co-operative society, company or other AOP to whom a building or part


thereof is allotted or leased under a house building scheme of the society, company or
association, as the case may be, shall be deemed to be the owner of that building or part
thereof.

(v) Possessor of a property under a contract referred to in S. 53A of Transfer of Property


Act – S. 27 (iiia):

If the following conditions referred to in S. 53A of the Transfer of property Act are complied
with, the buyer will be regarded as owner even though the property is yet to be registered
in his name U/s 54 of the transfer of property Act read with S. 17 of the registration Act:

1 There is an agreement in writing between the buyer and the seller.


2 The buyer has paid the consideration or he is ready to pay the consideration.
3 The buyer has taken the possession of the property.

MT Educare –CA Inter Income Tax Classes – R. SOUMYANARAYANAN. FCA. GRAD CWA. Page-50
Chapter-4: Income from house property

(vi) Person having right in a property for a period of atleast 12 years - S. 27 (iiib):

1 Where the house property is given on a lease and the lease period is atleast 12 years,
then the lessee shall be deemed to be the owner of such property.
2 The position will be the same, even if the original lease period is less than 12 years but
along with the period of extension is not less than 12 years.
3 However, in the following cases, only the lessor will be regarded as owner: (a) Month to
month lease; (b) Lease for a period of less than one year.

Owner

Legal Owner Deemed Owner (sec.27)

Holder of Impartible Purchaser under Sec. 53A of


Estate Transfer of Property Act
Member of a co-operative
society /company/AOP- Person who has
Individual Transferring HP allotted a house under the acquired a right in a
to his spouse/Minor child house building scheme of the building u/s 269UA
(in certain cases) society/Co-operative /AOP.

Some issues in charging section:

Issue-1: Property owned by the partner and used by the firm for the purpose of its
business – whether annual value is taxable in the hands of partner?

1 If the firm carries on business, it shall be regarded as being carried on by all the partners.
2 Accordingly, the annual value of a building belonging to the assessee which is in the
occupation of a firm in which the assessee is a partner, is not assessable U/s. 22. [CIT Vs
K.M. Jagannathan 180 ITR 191 (Mad); CIT Vs Vaidhyanathan 180 ITR 198 (Mad); CIT Vs
P.T.Mannel 47 Taxman 108 (ker); CIT Vs Rabindranath Bhol 211 ITR 799 (Orissa); CIT Vs
Mustafa Khan 145 Taxman 522 (All); CIT Vs Syed Anwar Hussain 186 ITR 749 (Pat)].

Issue-2: Property owned by HUF and used by the firm in which members are partners –
whether annual value is taxable in the hands of HUF?

1 Where the premises were owned by HUF but used by the partnership firm in which karta
and the individual members were partners in their individual capacity, it was held by the
Allahabad HC in CIT Vs Shiv Mohanlal 202 ITR 60 that that it would not be open to the
HUF to claim that the relevant premises were occupied for the business carried on by
the HUF.
2 However, where the karta was a partner in the firm in his representative capacity and
the firm was occupying a portion of the house belonging to the HUF, the Delhi HC in CIT
Vs H.S.Singhal & Sons 253 ITR 653 held that the annual value of such property would not
assessable to tax U/s. 22. This same view was expressed by the Madras HC in CIT Vs
Champalal jeev Raj 215 ITR 289.

MT Educare –CA Inter Income Tax Classes – R. SOUMYANARAYANAN. FCA. GRAD CWA. Page-51
Chapter-4: Income from house property

Issue-3: Letting of building is subservient to the business carried on by the assessee –


whether annual value is taxable in the hands of the assessee?

1 The assessee-company, in order to avoid commutation problem to its workers and as a


measure of preserving the productivity level of its workers, constructed a residential
building with a number of residential units within its factory campus and charged a
nominal rent which was intended to be used for the maintenance of the building and for
providing some services to its workers.
2 The Punjab HC in CIT Vs Delhi cloth & General mills co. Ltd 59 ITR 152 held that the
annual value of this residential building is not assessable to tax in the hands of assessee-
company. The assessee-company let out the building not with an intention to earn rental
income by exercising ownership rights but to carry on the business smoothly, efficiently
and effectively. In other words, letting of building is subservient to the business carried
on by the assessee-company.
3 Accordingly, the building should be regarded as being used by the assessee-company for
the purpose of its business and hence, the annual value is exempt U/s 22.
4 The nominal rent recovered from the workers is a business receipt and shall be taken to
the head PGBP.
5 The assessee-company is also eligible for depreciation U/s 32 with respect to this
building.
6 Similar views were expressed by the Madhya Pradesh HC in CIT Vs National newsprint &
paper mills Ltd 114 ITR 388; Madras HC in CIT Vs V.S.T motors (P) Ltd 226 ITR 155;
Andhra Pradesh HC in CIT Vs Vazir sultan tobacco Ltd 173 ITR 290.

Issue-4: Location of the property Vis a Vis Chargeability:

House
Property

In India Outside India

Income deemed Rent received Rent received


to accrue/arise in in India outside India
India u/s 9(1)(i)

Foreign Income
Indian Income

Taxable only for


Taxable for all resident and
ordinarily resident

MT Educare –CA Inter Income Tax Classes – R. SOUMYANARAYANAN. FCA. GRAD CWA. Page-52
Chapter-4: Income from house property

Issue-5: Annual value of club buildings – exempt - Doctrine of mutuality:

1 The assessee-club provides recreational and refreshment facilities exclusively to its


members & their guests.
2 It is run on ‘no profit no loss’ basis in that the members pay for all their expenses and are
not entitled to any share in the profit.
3 Surplus, if any, arising from the operations of the assessee-club is used for the
maintenance and development of the club.
4 The Supreme Court in Chelmsford Club Vs CIT 243 ITR 89 held that since the activity of
the Club is governed by the doctrine of mutuality, not only the surplus from the activity
of the club but even the annual value of the club house will be outside the purview of
the levy of income tax.

Issue-6: Rules for converting rent earned in foreign currency into Indian rupees:
The rental income in foreign currency is to be converted into Indian rupees using the
telegraphic buying rate of such currency on the last day of the previous year. [R. 115].

Issue-7: Cases where the annual value of the building is exempt:

S. 10 (1) Annual value of farm house


S. 10 (9A) Annual value of any one palace in the occupation of an ex-ruler
S. 10 (20) Annual value of house property owned by a local authority
S. 10 (21) Annual value of buildings owned by Approved Scientific research association
S. 10 Annual value of buildings owned by a University or other educational
(22) institutions
S. 10 Annual value of buildings owned by an institution for the development of khadi
(23B) and village industries.
S. 10 Annual value of property of khadi and village industries board
(23BB)
S. 10Annual value of buildings owned by certain funds, educational institutions,
(23C) hospitals etc.
S. 10 (24) Annual value of buildings owned by any registered trade union
S. 10Annual value of buildings owned by a statutory corporation or an institution or
(26B) association financed by the Government for promoting the interests of
members of SC/ST.
S. 10 Annual value of buildings owned by a statutory corporation established for
(26BB) promoting the interests of members of minority community.
S. 10 Annual value of buildings owned by a statutory corporation established for
(26BBB) welfare and economic upliftment of ex-servicemen being citizen of India.
S. 10 (27) Annual value of buildings owned by a co-operative society formed for
promoting the interest of the members of SC/ST.
S. 11 Income from house property held for any charitable purpose.
S. 13A Property income of a political party

MT Educare –CA Inter Income Tax Classes – R. SOUMYANARAYANAN. FCA. GRAD CWA. Page-53
Chapter-4: Income from house property

Segment-2: Computation of income from let out property:

House property

Let out by the assessee Used by the assessee

For the purpose of For the purpose


hisbusiness/profession of his residence
House property can be classified into (i) Let out property and (ii) self occupied property.
Self-occupied property can be classified into (a) Self-occupied for residential purpose (b)
Deemed to be self-occupied for residential purpose (c) Self-occupied for the purpose of
business or profession.
In this segment, we are going to learn as to how to compute income from let out property.
The manner of computation of income from self occupied property will be learnt in the next
segment.

Format for computing income from a let out property:


1 Gross annual value (GAV) Xxx
2 Municipal taxes Xxx
3 Net annual value (NAV) (1-2) Xxx
4 Standard deduction [30% of NAV] (S. 24 (a)) Xxx
5 Interest on borrowed capital (S. 24 (b)) Xxx
6 Income from let out property (3-4-5) Xxx

Computation of GAV: [S. 23 (1) (a)/(b)]:


(a) Inputs required for computation of GAV: For computing the GAV, we need two inputs:
(i) Reasonable expected rent (RER) and (ii) Actual rent received or receivable (ARR).
(b) Inputs required for computation of RER: For computing RER, we need three inputs: (a)
Municipal value (MV) (b) Fair rent (FR) and (c) Standard rent (SR).
(c) Inputs required for computation of ARR: For computing ARR, we need three inputs: (a)
Annual rent (AR) (b) Unrealized rent (URR) and (c) Loss due to vacancy (LDV).

Basic inputs for


computation of GAV

Reasonable expected Rent actually received


rent of the property or receivable (d)

Basic inputs for computing the rent


Basic inputs for computing
actually received or receivable
the reasonable expected rent

Unrealised Lose of rent


Rent of previous
Municipal Fair rent (FR) rent (d2) because of
Standard Rent (SR) year for which the
value (MV) vacancy (d3)
property is available
for letting out (d1)

MT Educare –CA Inter Income Tax Classes – R. SOUMYANARAYANAN. FCA. GRAD CWA. Page-54
Chapter-4: Income from house property

(d) Computation of RER:

Compare MV and FR

X = Higher of MV and FR

Compare X and SR

RER = Lesser of X and SR

(e) Computation of ARR:

1 Rent of the PY for which the property is available for letting out (d1) Xxx
2 Unrealised rent (subject to certain conditions) (d2) Xxx
3 Rent of vacant period or Loss due to vacancy (d3) Xxx
4 Actual rent received/receivable (1-2-3) (d) Xxx

(f) Determination of GAV:

Situation Reason GAV


d > RER D
d < RER Purely due to vacancy D
Purely due to other factors RER
Partly due to vacancy and partly due to other factors RER – d3

Note: Other factors may be: (a) charging lower rate of rent (b) unrealised rent etc.

Yes
d>RER? GAV = d

No

D < RER - only D < RER - Partly because of vacancy D < RER - only because
because of vacancy and partly because of other factors of other factors
(such as unrealised rent letting out
property for lower rent)
GAV = RER
GAV = d
GAV = RER - loss
due to vacancy

MT Educare –CA Inter Income Tax Classes – R. SOUMYANARAYANAN. FCA. GRAD CWA. Page-55
Chapter-4: Income from house property

(g) How to determine as to why ARR is less than RER?

Situation Is the Narration of the situation Reason for d < RER GAV
property
vacant
during
the PY?
d1<RER Yes The assessee is charging a Charging of lower RER-d3
lower rent for the property rent and vacancy
than what it could reasonably
fetch. The property was vacant
for some period during the PY.

d1<RER No The assessee is not charging a Charging of lower RER


lower rent for the property rent
than what it could reasonably
fetch. The property was not at
all vacant during the PY.
d1>RER Yes The assessee is not charging a Unrealised rent and RER-d3
but (d1- lower rent than what the vacancy
d2)<RER property could reasonably
fetch. The house is vacant
during the PY for some period.
The assessee could not realise
some portion of the rent due to
him
d1>RER No The assessee is not charging a Unrealised rent RER
but (d1- lower rent than what the
d2)<RER property could reasonably
fetch. The house is not vacant
during the PY for some period.
The assessee couldn’t realise
some portion of rent due to
him.
d1>RER Yes The assessee is not charging a Vacancy d
and (d1- lower rent than what the
d2)>RER property could reasonably
fetch. The house is vacant
during the PY for some period.
The assessee couldn’t realise
some portion of the rent due to
him.

MT Educare –CA Inter Income Tax Classes – R. SOUMYANARAYANAN. FCA. GRAD CWA. Page-56
Chapter-4: Income from house property

(h) Meaning of MV – FR – SR:

Municipal Municipal authorities, normally, charge house taxes on the basis of the annual
value letting value of house property. For determining this, they will conduct survey
of all the buildings within their jurisdiction. The value so determined can be
taken as the basis for determining the earning capacity of the house property.
Fair rent It is the rent which a similar property can fetch in the same or similar locality, if
it is let for a year.
Standard It is fixed under the rent control Act. If standard rent is fixed with respect to a
rent property, the owner cannot be expected to get a rent higher than the standard
rent fixed under the rent control Act.

(i) Issues in computing ARR:

SN Situations Impact on ARR


1 Obligations of the Where the owner agrees to bear certain obligations of the tenant
tenant met by the such as, water bills, electricity bills etc, the amount incurred for
owner meeting such obligations shall be reduced from the rent received
or receivable.
2 Obligations of the Where the tenant, in addition to rent, also incurs expenses for
owner met by the meeting any obligation of the owner, the rent received or
tenant receivable shall stand increased by such amount incurred.
3 Municipal taxes Municipal taxes are to be borne by the occupier of the property. If
borne by the the property is let out, the occupier is tenant and he shall bear it.
tenant In computing the ARR, the municipal taxes borne by the tenant
shall not be added. (CIT Vs Gillanders Arbuthnot and co Ltd 142
ITR 598).
4 Repair expenses Where the repair expenses are contractually borne by the tenant,
borne by the the rent received or receivable should not be stepped up to
tenant calculate the defacto rent. (CIT Vs Parbutty Churn Law 57 ITR 609
(Cal))
5 Non-refundable This is a capital receipt and hence, cannot be treated as income.
deposit received
from the tenant
6 Refundable It cannot be included in rent received or receivable, since it is
deposit refundable and does not belong to the land lord.
7 Notional interest For determining ARR, the notional interest on the deposit
on the deposit received from the tenant shall not be considered. If the owner
received from actually earns interest on this deposit, it will be taxed U/H IFOS. If
tenant the owner doesn’t earn any interest on this deposit, there is no
need to tax it since, what is taxed is real income and not the
notional income. (CIT Vs Hemraj Mahabir prasad Ltd 279 ITR 522
(cal))
8 Advance rent It cannot be considered as rent received or receivable of the year
of receipt. (CIT Vs Prince rubber & plastics 1 SOT 85)

MT Educare –CA Inter Income Tax Classes – R. SOUMYANARAYANAN. FCA. GRAD CWA. Page-57
Chapter-4: Income from house property

9 Conditions for exclusion of The tenancy should be bonafide.


unrealised rent (Notification No. The defaulting tenant has vacated or steps have been
198/2001, dated 02-07-2001) taken to compel him to vacate the property.
The defaulting tenant is not in occupation of any
other property of the assessee.
The assessee has taken all reasonable steps to
institute legal proceedings for the recovery of the
unpaid rent or satisfies the AO that legal proceedings
would be useless.

Composite rent-tax treatment at a glance:

Composite rent

Composite rent = rent Composite rent = rent of


of building and charges letting out of building and rent
for different services of letting out of other assets

Rent of the Letting out of


Amount of
building building and
rendering services
other assets
Charged to Charged to
tax u/s 22 tax u/s 28/56 Are Are not
seperable seperable

Income is taxable
Income from Income from letting U/s 28/56
letting out of out of other assets
building charged charged to tax
to tax u/s 22 either u/s 28/56 Example. Furnished
guest accommodation
well equipped theatre
Air conditions
furnished lectured hall

Tax treatment of composite rent charged on account of rent for the property and service
charges for facilities provided:

1 The Calcutta HC in CIT Vs Kanak Investments (P) Ltd 95 ITR 419 held that where the
owner of the property charges a composite rent for letting out the property and for the
services provided along with the property such as lift, gas, electricity, water, air
conditioning, watch and ward, parking facility etc, that portion of the composite rent
attributable to the letting of the property shall be assessed to tax U/H IFHP and the
other portion which is attributable to the services shall be assessable to tax U/H IFOS.
2 But, where the assessee renders such services to the tenants through his activities
carried on continuously in an organized manner, with a set purpose and with a view to
earn profits, that portion of the composite which is attributable to the services shall be
assessed to tax under the head PGBP. This was the view expressed by the Supreme Court
in Karnani properties Ltd Vs CIT 82 ITR 547.

MT Educare –CA Inter Income Tax Classes – R. SOUMYANARAYANAN. FCA. GRAD CWA. Page-58
Chapter-4: Income from house property

Tax treatment of the composite rent charged on account of rent for the property and the
hire charges of machinery, plant or furniture belonging to the owner.
Letting of property is separable from letting of the Letting of property is inseparable
other assets from letting of the other assets
That portion of the rent attributable to the letting of The entire rent is to be assessed to
the premises shall be assessed to tax U/H IFHP. The tax either U/H PGBP or IFOS.
other portion traceable to letting of other assets
shall be assessable either U/H PGBP or IFOS.

In case of deemed to be let out property [Section 23(4)]

Where the assessee owns more than two properties for self-occupation, then the income
from any two properties, at the option of the assessee, shall be computed under the self-
occupied property category and their annual value will be nil.
The other self-occupied/unoccupied properties shall be treated as “deemed let out
properties”.
This option can be changed year after year in a manner beneficial to the assessee.
In case of deemed let-out property, the ER shall be taken as the GAV.
The question of considering actual rent received/receivable does not arise. Consequently,
no adjustment is necessary on account of property remaining vacant or unrealized rent.
Municipal taxes actually paid by the owner during the previous year, in respect of the
deemed let out properties, can be claimed as deduction.
However, maximum limit for deduction u/s 24(b), in aggregate (for one or two
house together), cannot exceed ` 30,000 or ` 2,00,000, as the case may be. [Section
24(b) is also amended to this extent]

In case of a house property held as stock-in-trade [Section 23(5)]

(a) In some cases, property consisting of any buildings or lands appurtenant thereto may be
held as stock-in-trade, and the whole or any part of the property may not be let out during
the whole or any part of the previous year.

(b) In such cases, the annual value of such property or part of the property shall be Nil.

(c) This benefit would be available for the period upto two years from the end of the
financial year in which certificate of completion of construction of the property is obtained
from the competent authority.

MT Educare –CA Inter Income Tax Classes – R. SOUMYANARAYANAN. FCA. GRAD CWA. Page-59
Chapter-4: Income from house property

Deduction with respect to municipal taxes: [S. 23]:

In computing the annual value, municipal taxes paid by the assessee during the PY shall be
allowed as a deduction.

Municipal No
taxes BORNE Not deductible
by owner?

Yes

Actually paid No
by the owner?

Yes

Deductible

Points requiring attention:

1 Municipal taxes This shall not be deducted from GAV. Only municipal taxes paid by
paid by tenant the owner shall be deducted from the GAV.
2 Deduction on Irrespective of the method of accounting followed by the owner,
payment basis the deduction with respect to municipal taxes is allowed only on
payment basis.
That is, the deduction is allowed only in the year of payment,
though the municipal taxes may relate to earlier years.
3 Advance payment Municipal taxes paid during the PY are allowed as deduction, even if
of municipal tax they relate to future years.
4 Foreign municipal In computing the annual value of the property situated outside
tax India, the taxes levied by local authorities of that country are
deductible. (CIT Vs Venugopala Reddiar 58 ITR 439 (Mad))

Deductions U/s 24:

Deduction u/s 24

Standard Interest on
deduction borrowed/capital

1 In computing the IFHP, the assessee is entitled to deductions contemplated in S. 24


against the NAV.
2 S. 24 contemplates two deductions: (a) Standard deduction @ 30% of NAV and (b)
Interest on borrowed capital.
3 No other deduction is allowed in computing IFHP. Indian city properties Ltd Vs CIT 55
ITR 262 (cal).

MT Educare –CA Inter Income Tax Classes – R. SOUMYANARAYANAN. FCA. GRAD CWA. Page-60
Chapter-4: Income from house property

Interest on borrowings: S. 24 (b):

Where an assessee borrows money for the following purposes, the interest on such
borrowings shall be allowed as a deduction in computing income U/H IFHP:

1 Purchase of house property


2 Construction of house property
3 Reconstruction of house property
4 Repairs of house property
5 Renewal of house property

If the assessee borrows for any other purpose, interest on such borrowings will not be
allowed as deduction U/s 24 (b).

Interest on funds borrowed for the aforesaid purposes can be divided based on the period
to which it relates into (a) Pre-construction period interest and (b) Post-construction period
interest. Pre-construction period interest is allowed as deduction in 5 equal annual
instalments beginning with the PY in which the construction of the property is completed.
Pre-construction period begins on the date of borrowing ends on 31st March immediately
preceding the date of completion of construction of the house property. Post construction
period interest is allowed as a deduction in the PY in which it is incurred. There is no ceiling
with respect to the amount of deduction U/s 24 (b).
Pictorial representation of deduction with respect to interest on borrowings:

Purpose of borrowing

Purchase of HP Construction of HP Reconstruction Repairs Renewals


of HP

Interest
Borrowing Date of completion
Date of Pre-construction Immediately
committment
of construction or
borrowing period peceeding 31st March
acquisition
Interest attributed
to this period

1/5th 1/5th 1/5th 1/5th 1/5th


deductible deductible deductible deductible deductible

In the year of
acquisition or In the
construction of subsequent
house years

MT Educare –CA Inter Income Tax Classes – R. SOUMYANARAYANAN. FCA. GRAD CWA. Page-61
Chapter-4: Income from house property

Interest on borrowing

Relating to post
Relating to pre-
construction period
construction period

Deductible on
Discussed below accrual basis

Points requiring attention:


1 Deduction on accrual Since S. 24(b) uses the word ‘Payable’, the interest on borrowing
basis is allowed as deduction on due basis. (CIT Vs. Devendra Bros &
Co 200 ITR 146 (All)).
2 Interest payable to Where the property is purchased on deferred payment basis and
vendor on the the assessee is liable to pay interest on the outstanding balance,
outstanding purchase even that is eligible for deduction U/s 24 (b). CIT Vs Sunil Kumar
consideration. Sharma 254 ITR 103 (P&H)
3 Interest on interest Any interest paid on outstanding amount of interest, will not be
allowed as deduction. (Shew Kissen Bhatter Vs CIT 89 ITR 61 (SC))
4 Fresh loan to repay Where a fresh loan has been raised to repay the original loan
original loan which was taken for construction, acquisition, reconstruction,
repairs or renewals of the house property, the interest on fresh
loan would also allowed as deduction u/s. 24. (CBDT circular no
28, dated 20-08-1969)
5 Brokerage or Any amount paid for brokerage or commission for arrangement
commission for of the loan will be allowed as deduction. (S. 2 (28A)).
arrangement of loan
6 Interest on borrowed Interest on borrowed money, which is payable outside India and
money payable outside which is taxable in India in the hands of recipient, shall not be
India (S. 25) allowed as deduction u/s 24(b), unless the tax on the same has
been paid or deducted at source.

MT Educare –CA Inter Income Tax Classes – R. SOUMYANARAYANAN. FCA. GRAD CWA. Page-62
Chapter-5: Salary

1. Fundamentals:
(a) Employer-employee relationship:

1 Every payment made by employer to his employee for service rendered would be
chargeable U/H Salary.
2 In order to invoke the head Salary, one has to verify as to whether there exists
employer-employee relationship between payer and payee.
3 It does not matter as whether the employee is a full-time employee or a part-time
employee.

Points requiring attention:


1 Though judges have no employer and are constitutional functionaries, still, the
remuneration paid to them shall be chargeable to tax U/H Salary. (Justice Deoki Nandan
Agarwal Vs UOI).
2 Salary paid to suspended employees shall also be charged to tax U/H salary, since there
continues the employer-employee relationship even during the period of suspension.
3 Salary paid by a firm to its partners shall not come U/H Salary. [Explanation-2 to S. 15]. It
shall be brought to tax U/H PGBP. [S. 28 (v)]. There is no employer-employee
relationship between the firm and its partners.
4 If an employee-director receives any commission from the company for giving guarantee
to a financial institution providing financial assistance to the employer-company, such
commission shall not be assessed to tax U/H ‘salaries’ but has to be assessed to tax U/H
‘other sources’. It is so because such commission is given to the director for undertaking
risk and not for rendering services.

(b) Foregoing of salary:


1 Once salary accrues, the subsequent waiver by the employee does not absolve him from
liability to income-tax.
2 Such waiver is only an application of income and hence, chargeable to tax.
3 However, if an employee surrenders his salary to the Central Government U/s 2 of the
Voluntary Surrender of Salaries (Exemption from Taxation) Act 1961, then the salary so
surrendered would be exempt while computing his taxable income.

Standard Deduction u/s 16(ia)


Standard deduction of Rs. 50,000 or salary income whichever lower, has been allowed while
computing income under the head “Salaries”. This limit from the AY 2020-21 previously it
was Rs. 40,000 or salary income

MT Educare –CA Inter Income Tax Classes – R.SOUMYANARAYANAN.FCA. GRAD CWA. Page-63
Chapter-5: Salary

(c) Pictorial representation of tax incidence on salary income:

Services rendered in

India Outside India

Taxable for all Payments made Payments


employees outside India made in India

Taxable for all


Indian Employer - employees
Government is the Any Other
employer person

Employee is Employee is Employee is


an Indian not an Indian Employee is Resident but
citizen citizen Resident and not ordinarily
Ordinarily Resident
Resident
Payment Employee is Non-resident
Employee is Resident but
Resident and not ordinarily
Allowances/ Ordinarily Resident
Salary Taxable Not Taxable
Perks Resident
Non-resident
Taxable Not Taxable
Sec. 10(7)
Taxable Not Taxable

Note-1: If an employee gets pension paid abroad in respect of services rendered in India, the
same will be deemed to accrue or arise in India.

Note-2: Similarly, leave salary paid abroad in respect of leave earned in India is deemed to
accrue or arise in India.

Note-3: If salary is earned in foreign exchange, it has to be converted into rupees. The
relevant exchange rate for conversion is, the telegraphic buying rate of such foreign
currency on the last day of the month immediately preceding the month in which the salary
is due or is paid in advance or in arrears.

2. Basis of charge of salary income (S. 15):

SN Situations Year of chargeability


1 Salary becomes due from the employer in the PY and is paid PY in which it became
during that PY. due and was paid.
2 Salary becomes due from the employer in the PY and is not paid PY in which it became
during that PY. due.
3 Salary is received from the employer during the PY but it has not PY in which it was
become due during that previous year received.

MT Educare –CA Inter Income Tax Classes – R.SOUMYANARAYANAN.FCA. GRAD CWA. Page-64
Chapter-5: Salary

4 Arrears of salary is received from the employer during the PY PY in which arrears were
which were not taxed in the earlier PYs on due basis. [Pay received.
commission was appointed by the CG and it recommends revision
of salaries of employees; arrears is received by the employee in
that connection]

Points requiring attention:

1 In nutshell, the salary income is chargeable to tax on due basis or receipt basis
whichever is earlier.
2 If salary is taxed on due or receipt basis, it will not be taxed once again on receipt/falling
due, as the case may be. [Explanation-1 to S. 15].
3 Advance salary is taxable in the year of receipt and not the advance against salary.
4 The method of accounting followed by the assessee has no relevance in computing the
income under the head ‘Salaries’.

3. Definition of salary – S. 17 (1):

Wages Annual accretion Fees


in any P.F to the credit balance of
an employee participating in a RPF Commission
-to the extent it is taxable. Perks
Annuity/ Profit in lieu of
Pension Salary
Salary
Gratuity Profit in addition
to salary
Advance of Contribution by CG
Salary to the account of
employee under
pension scheme U/s. Balance transferred
Leave Salary from URPF to RPF
80CCD

4. Format for computation of salary income:

SN Particulars Amount
1 Basic salary ******
2 Fees / commission ******
3 Bonus ******
4 Taxable allowances ******
5 Taxable perquisites ******
6 Uncommuted Pension ******
7 Taxable commuted pension ******
8 Taxable gratuity ******
9 Leave encashment ******

MT Educare –CA Inter Income Tax Classes – R.SOUMYANARAYANAN.FCA. GRAD CWA. Page-65
Chapter-5: Salary

10 Taxable leave travel concession ******


11 Taxable voluntary retirement compensation ******
12 Taxable retrenchment compensation ******
13 Profit in lieu of salary ******
14 Taxable portion of employer’s contribution to PF ******
15 Taxable portion of interest credited to PF account of employee ******
16 Employer’s contribution to pension fund referred to S. 80CCD ******
17 Gross salary (1 to 16) ******
18 Deduction w.r.t entertainment allowance (allowable only to Government (****)
employees) [S. 16 (ii)].
19 Deduction in respect of Professional tax paid [S. 16 (ii)]. (****)
20 Income U/H Salaries (17-18-19) ******

Note-1: The term ‘perquisite’ is defined in S. 17 (2). [Discussed later].

Note-2: The term ‘profit in lieu of salary’ is defined in s. 17 (3). [Discussed later].

5. Allowances:

(a) Meaning & taxability of allowances:

1 Allowance means fixed monetary payment made by the employer to his employees for
meeting (a) personal expenses; or (b) expenses incurred for performance of his official duties.
2 Allowances are generally taxable and are to be included in computing gross salary unless a
specific exemption has been provided in respect of any such allowances.

(b) Illustrative list of allowances which are taxable:

1 Dearness allowance (paid to employees to cope with increase in cost of living due to
inflation)
2 City compensatory allowance (to meet increased cost of living in cities)
3 Fixed medical allowance (to meet the medical treatment expenses of the employee or
his family members)
4 Lunch / dinner allowance
5 Tiffin allowance
6 Project allowance (paid to employees to meet project expenses)
7 Overtime allowance
8 Servant allowance (paid to employees to engage the services of a servant)
9 Non-practicing allowance (paid to doctors attached to clinical centers of various labs or
institutes)
10 Interim allowance (paid in lieu of final allowance)
11 Warden allowance (paid to employees working as warden (i.e. keeper in educational
institutions).
12 Entertainment allowance [paid to employees to meet the expenses towards hospitality
in receiving customers etc] [subject to deduction U/S 16 (ii)]

MT Educare –CA Inter Income Tax Classes – R.SOUMYANARAYANAN.FCA. GRAD CWA. Page-66
Chapter-5: Salary

(c) Deduction with respect to entertainment allowance- S. 16 (ii):

Employees

Government
Employees Non-Government employees
- including the employees of
statutory corporation, Local
Quantum of Exemption: authority

Least of the following


a. Rs. 5,000
b. 20% of Basic Salary
c. Amount of entertainment Taxable
allowance

(d) Allowances which are partly exempt:


(i) House rent allowance [Exemption provided U/s 10 (13A)].
(ii) Special allowances referred to in S. 10 (14).

(e) Allowances which are fully exempt:


(i) Allowances given to High Court Judges
(ii) Allowances given to Supreme Court Judges
(iii) Allowances paid by the United Nation Organisation
(iv) Allowance to Government employees serving outside India – S. 10 (7)

(f) House rent allowance


Meaning of house rent allowance:
HRA is a special allowance specifically granted to an employee by his employer towards
payment of rent for residence of the employee.

Conditions for availing exemption:


1 Assessee should not stay in his own house.
2 He should incur expenditure towards rent.

Employee Yes
HRA - Not
lives in his
exempted
own house?

No

Employee No
pays rent for
the house?

Yes

HRA -
exempted

MT Educare –CA Inter Income Tax Classes – R.SOUMYANARAYANAN.FCA. GRAD CWA. Page-67
Chapter-5: Salary

Format for computation of taxable HRA:


1 HRA received *****
2 Exemption U/S 10 (13A) (***)
3 Taxable HRA (1-2) *****

Quantum of exemption:

Exemption in respect of house rent allowance shall be quantified in the manner prescribed
below:

Situs of Residential
House

In Bombay, Calcutta, In
Delhi, Chennai other cities

Quantum of Exemption Quantum of Exemption

Least of the following: Least of the following:


a. Rent paid (-) 10% of a. Rent paid (-) 10% of
salary for the period salary for the period
of occupation. of occupation.
b. 50% of salary. b. 40% of salary.
c. Actual HRA received c. Actual HRA received
during the period of during the period of
occupation. occupation.

Salary for this purpose:

(a) Salary for the purpose of S. 10 (13A) = Basic pay + DA (if the terms of employment so
provide) + Fixed percentage commission on turnover achieved by the employee.
(b) Salary for this purpose shall be determined on due basis. That is, the advance salary
received by the employee shall be ignored.
Mode of computation of exemption:

Factors affecting Quantum of Exemption

Salary of Place where


HRA Rent paid house is
employee
situated

These factors These factors


were same were not the
through out same through
the year out the year

Exemption - computed Exemption - computed


on “Annual basis” on “Monthly basis”

MT Educare –CA Inter Income Tax Classes – R.SOUMYANARAYANAN.FCA. GRAD CWA. Page-68
Chapter-5: Salary

(g) Special allowances referred to in S. 10 (14):

Special
Allowance

Exemption depending upon Exemption not depending


actual expenditure by upon actual expenditure
employee by employee
Category I Category II

Quantum of exemption Quantum of exemption


(whichever is less) (whichever is less)

* Amount of allowance * Amount of allowance


* Amount utilized for * Amount specified in
specific purpose Rule 2BB

Category I
Name of Allowance Nature of Allowance
Travelling allowance Any allowance (by whatever name called) granted to meet the cost of travel on
/ Transfer allowance tour or on transfer (including any sum paid in connection with transfer, packing
and transportation of personal effects on such transfer).
Conveyance Conveyance allowance granted to meet the expenditure on conveyance in
Allowance performance of duties of an office (it may be noted that expenditure for
covering the journey between office and residence is not treated as expenditure
in performance of duties of the office and, consequently, such expenditure is
not exempt from tax).
Daily Allowance Any allowance whether granted on tour or for the period of journey in
connection with transfer, to meet the ordinary daily charges incurred by an
employee on account of absence from his normal place of duty.
Helper Allowance Any allowance (by whatever name called) to meet the expenditure on a helper
where such helper is engaged for the performance of official duties.
Research Allowance Any allowance (by whatever name called) granted for encouraging the academic
research and other professional pursuits.
Uniform Allowance Any allowance (by whatever name called) to meet the expenditure on the
purchase or maintenance of uniform for wear during the performance of duties
of an office.

Category II
Name of Nature of Allowance Exemption as specified in R.
Allowance 2BB
Special It includes any special compensatory allowance in Rs. 300 p.m. or Rs. 800 p.m or
Compensatory the nature of special compensatory (hilly areas) Rs. 7000 p.m depending upon
(Hill Areas)allowance or high altitude allowance or specified locations.
Allowance uncongenial climate allowance or snow bound area
allowance or avalanche allowance
Border Area It includes any special compensatory allowance in Rs. 1300 or 1100 or 1050 or
allowance the nature of border area allowance or remote 750 or 300 or 200 p.m.

MT Educare –CA Inter Income Tax Classes – R.SOUMYANARAYANAN.FCA. GRAD CWA. Page-69
Chapter-5: Salary

locality allowance or difficulty area allowance or depending on specified


disturbed area allowance. locations.
Tribal areas / Tribal areas allowance is given in (a) Madhya Rs. 200 p.m.
scheduled areas Pradesh (b) Tamilnadu (c) UP (d) Karnataka (e)
allowance Tripura (f) Assam (g) West Bengal (h) Bihar (i) Orissa
Allowance for It is an allowance granted to an employee working The amount of exemption is
transport in any transport system to meet his personal lower of–
employees expenditure during his duty performed in the a) 70% of such allowance
course of running of such transport from one place or
to another place provided that such employee is b) Rs. 10000 p.m.
not in receipt of daily allowance
Children This allowance is given for children’s education The amount exempt is limited
Education to Rs. 100 per month per child
Allowance up to a maximum of two
children.
Hostel This allowance is granted to an employee to meet It is exempt from tax to the
expenditure the hostel expenditure on his child. extent of Rs. 300 per month
allowance per child up to the maximum
of two children
Compensatory If this exemption is taken, the same employee Exemption is limited to Rs.
field area cannot claim any exemption in respect of border 2600 per month in some cases.
allowance area allowance mentioned above.
Compensatory If this exemption is taken, the same employee Exemption is limited to Rs.
modified area cannot claim any exemption in respect of border 1000 per month in some cases.
allowance area allowance mentioned above.
Counter It includes any special allowance in the nature of Exemption is limited to Rs.
insurgency counter-insurgency allowance granted to the 3900 per month in some cases.
allowance members of armed forces operating in areas away
from their permanent locations for a period of
more than 30 days. If this exemption is taken, the
same employee cannot claim any exemption in
respect of border area allowance mentioned above.
Transport Transport allowance is granted to an employee to It is exempt up to Rs. 1600 per
allowance meet his expenditure for the purpose of month (Rs. 3200 per month in
commuting between the place of his residence and the case of an employee who
the place of his duty. is blind or orthopaedically
handicapped or deaf and
dumb).
Underground Underground allowance is granted to an employee Exemption is limited to Rs. 800
allowance who is working in uncongenial, unnatural climate in per month.
underground coal mines.
High altitude It is granted to the members of armed forces It is exempt from tax up to Rs.
allowance operating in high altitude areas. 1060 per month (for altitude
of 9000 to 15000 feet) or Rs.
1600 per month (for altitude
above 15000 feet).

MT Educare –CA Inter Income Tax Classes – R.SOUMYANARAYANAN.FCA. GRAD CWA. Page-70
Chapter-5: Salary

Highly active This special allowance is granted to the members of It is exempt from tax up to Rs.
field area armed forces in the nature of special compensatory 4200 per month.
allowance highly active field area allowance

Island duty This special allowance is granted to the members of It is exempt up to Rs. 3250 per
allowance armed forces in the nature of island (duty) month.
allowance in Andaman and Nicobar and
Lakshadweep group of Island

(h) Exemption of specified allowances and perquisites paid to Chairman or a retired


Chairman or any other member or retired member of the Union Public Service
Commission (UPSC) – S. 10 (45):
(i) S. 10 (45) exempts specified allowances and perquisites paid to Chairman or a retired Chairman
or any other member or retired member of the Union Public Service Commission (UPSC).
(ii) The exemption would be available in respect of such allowances and perquisites as may be
notified by the CG in this behalf.
(iii) Accordingly, the CG has notified the following allowances and perquisites for serving Chairman
and members of UPSC, for the purpose of exemption U/s 10 (45):
(a) The value of rent free official residence,
(b) The value of conveyance facilities including transport allowance,
(c) The sumptuary allowance (generally paid to persons holding honorary posts) and
(d) The value of leave travel concession.
(iv) In case of retired Chairman and retired members of UPSC, the following have been notified for
exemption U/s 10 (45):
(a) A sum of maximum of Rs. 14000 p.m for defraying the service of an orderly (i.e. attendant) and
for meeting expenses incurred towards secretarial assistance on contract basis.
(b) The value of a residential telephone free of cost and the number of free calls to the extent of
Rs. 1500 p.m (over and above free calls per month allowed by the telephone authorities).

Note: Tax exemption is also available in respect of certain specified perquisites enjoyed by
the Chief Election Commissioner or Election Commissioner and Judges of Supreme Court on
account of enabling provisions in the respective Acts which govern their service conditions.

6. Annuity:
(i) Annuity is a sum payable in respect of a particular year. It is a yearly grant.
(ii) If a person invests some money entitling him to series of equal annual sums, such annual
sums are annuities in the hands of investor.
(iii) Annuity received from a past employer is to be taxed as salary. It does not matter whether it
is paid in pursuance of a contractual obligation or voluntarily.
(iv) Annuity received from a past employer is taxable as profit in lieu of salary.
(v) Annuity received from person other than an employer is taxable U/H IFOS.

MT Educare –CA Inter Income Tax Classes – R.SOUMYANARAYANAN.FCA. GRAD CWA. Page-71
Chapter-5: Salary

7. Pension:
(a) Meaning of pension & its classification:
(i) It means a periodic payment made especially by the Government or a company or other
employers to the employee in consideration of past service after his retirement.
(ii) Such pension is of two types: (a) commuted pension; (b) non-commuted pension.
(iii) Some employers may give an option to their employees to forgo a portion of their monthly
pension in consideration for a lump sum amount. This is called commutation of pension. The
lump sum amount received is called commuted pension.
(iv) The un-forgone monthly pension is called non-commuted pension.
(v) Suppose a person is entitled to receive a pension of say Rs. 2000 p.m for the rest of his life.
He may commute 1/4th of this amount and get a lumpsum of say Rs. 30000. This lumpsum
amount of Rs. 30000 is commuted pension. After commutation, his pension will now be the
balance 75% of Rs. 2000 p.m. (i.e. Rs. 1500 p.m).

(b) Tax treatment of commuted & uncommuted pension:


Pension

Commutted Uncommuted

Taxable as salary u/s


➢ Government 15 for all employees
Other Employees
Employee
➢ Judges of SC+HC
(CBDT Circular 623)
Employee received Employee not in receipt
Gratuity of Gratuity

Wholly Exempt 1/3 of Commuted full Pension 1/2 of Commuted full Pension
is exempted is exempted

Note: Commuted full pension = (Amount received on commutation / percentage of


commutation).

(c) Format for computation of pension:

SN Particulars Amount
1 Commuted pension received ******
2 Exemption under section 10 (10A) (*****)
3 Taxable portion of commuted pension (1-2) ******
4 Non-commuted pension received ******
5 Taxable pension (3+4) ******

MT Educare –CA Inter Income Tax Classes – R.SOUMYANARAYANAN.FCA. GRAD CWA. Page-72
Chapter-5: Salary

Points requiring attention:

1 Meaning of Government Employees of CG (including members of defence services), SG, Local


employee authority & Statutory corporations.
2 Pension from UNO Pension received by employees of UNO is fully exempt from tax.
3 Pension to Defence Pension received by Defence personnel who had received Gallantry
personnel awards is fully exempt from tax. (S. 10 (18))
4 Family pension Family pension means a regular monthly amount payable by the
employer to a person belonging to the family of an employee in the
event of his death. [Explanation to S. 57 (iia)].
It is taxable in the hands of the recipient U/H IFOS. Standard deduction
of Rs. 15000 or 1/3rd of the pension whichever is less, is available. [S. 57
(iia)].
Family pension received by family members of CG employees who had
received Gallantry awards is also fully exempt from tax. (S. 10 (18)).
Family pension received by the family members of member of Armed
forces, where the death of such member has occurred in the course of
discharge of operational duties is fully exempt from tax. (S. 10(19)).
Commuted pension out of Any commuted pension received by an individual out of annuity plan of
5
annuity plan of LIC LIC from a fund set up by LIC will be exempted.

8. Gratuity:

(a) Meaning of gratuity:

(i) It is a payment made by the employer to his employee in appreciation of services


rendered by the employee.
(ii) Generally, it is paid to the employee at the time of his superannuation or retirement or
resignation or disablement due to accident or disease.
(iii) If the employee dies, it is paid to his legal heirs.

(b) Tax treatment of gratuity:

(i) Any gratuity received during the period of service is fully taxable U/H Salary.
(ii) However, terminal gratuity is eligible for exemption U/s 10 (10). The unexempted
portion is taxable U/H Salary.

(b) Format for computation of taxable terminal gratuity:

SN Particulars Amount
1 Gratuity received at the time of superannuation or retirement or resignation ******
or death or disablement due to accident or decease
2 Exemption under section 10 (10) (*****)
3 Taxable portion of terminal gratuity (1-2) ******

MT Educare –CA Inter Income Tax Classes – R.SOUMYANARAYANAN.FCA. GRAD CWA. Page-73
Chapter-5: Salary

(c) Exemption U/s 10 (10):

Terminal gratuity received by the Government employees or Exemption is available U/s


member of defence services 10 (10) (i)
Terminal gratuity received by employees covered by Payment Exemption is available U/s
of Gratuity Act 10 (10) (ii)
Terminal gratuity received by employees not covered by Exemption is available U/s
Payment of Gratuity Act 10 (10) (iii)

Gratuity

Gratuity received on
Received during the
retirement/death/
period of Service
Resignation/Termination

Taxable Employees

Government Employees covered by Employee not covered by


Employees* payment of Gratuity Act payment of Gratuity Act

Wholly Least of the following exempt Least of the following exempt


exempt
➢ Rs. 1000000
➢ Rs. 1000000
➢ Completed year of Service
➢ Completed year of Service
X Average monthly salary
or Part thereof in excess
X 1/2
of 6 months X Last drawn
➢ Actual Amount Received
Salary X 15/26
➢ Actual Amount Received
BS + DA (term
of employment)
Salary = BS + DA Salary =
+ Commission
(% Turnover)

Government employees=CG Employees, SG Employees and Employees of local authority

Average monthly salary = Average of salary for 10 months immediately preceding the
month of retirement.

MT Educare –CA Inter Income Tax Classes – R.SOUMYANARAYANAN.FCA. GRAD CWA. Page-74
Chapter-5: Salary

Gratuity received from two or more employers – Ceiling on exemption:

Working with more than one employer

Receiving Gratuity from two/ Received Gratuity in earlier


more employers in the same year.
year
Receiving Gratuity in the
current year from some
other employers
The total exemption cannot
exceed Rs. 1000000/- Then
Ceiling = Rs. 10,00,000 -
Gratuity exempted in earlier
years.

Note: The exemption in respect of gratuity would be available even if it is received by the
widow, children or dependents of a deceased employee.

9. Leave salary or leave encashment:


(a) Meaning of leave salary:
1 Employees are entitled to various types of leave while they are in service.
2 The leave may either be availed by them or in case these are not availed of, these may
either lapse or these are allowed to be encashed every year or these are accumulated
and encashed after retirement or death.
3 The amount received on encashment of leave is called leave salary.

Leave (as per service rules)

Used Lapsed Accumulated Encashed if


for future use not used

During At the time of retirement


Service or Leaving Job

(b) Tax treatment of leave salary:

Leave encashed while in service is fully taxable in the hands of employees. However, the
terminal leave salary is eligible for exemption U/s 10 (10AA).
(c) Format for computation of taxable leave salary:

SN Particulars Amount Amount


1 Leave salary received from the employer at the time of retirement *****
2 Exemption under section 10(10AA) (****)
3 Taxable terminal leave salary (1-2) *****
4 Leave salary received while in service *****
5 Taxable leave salary (3+4) *****

MT Educare –CA Inter Income Tax Classes – R.SOUMYANARAYANAN.FCA. GRAD CWA. Page-75
Chapter-5: Salary

(d) Exemption U/s 10 (10AA):

Leave encashment

During At the time of retirement


service or leaving Job

-Taxable for all employees


Employees
-Sec 89 relief available

Government Non-
Employees Government
Employees

Fully exempt
from tax u/s Quantum of exemption
10(10AA)(i) =
Least of the following
Note: (i) Period of earned leave
i) Computation of leave standing to the credit of (in no. of months) to the
an employee at the time of retirement or leaving credit of the employee at
the Job: the time of his retirement
Leave to the credit (in months) = or leaving the job *
Average monthly Salary
[ Duration of service in years (full) * Leave (in (ii) 10 * Average monthly
days) credited for each year of service - as per Salary
service rules or 30 days (whichever is less)] - (iii) Rs.3,00,000
Leave taken or encashed during service (in days) (iv) Leave encashment
actually received at the
30 time of retirement

Average monthly salary = Average of salary drawn during the period of 10 months
immediately preceding retirement.

Salary = BS + DA (if terms of employment so provide) + Commission (fixed % of turnover


achieved by employee).

Earned leave encashment from two/more employers:

Working with more than one employer

Receiving earned leave Received earned leave


salary from two/more salary in earlier year.
employers in the same year
Receiving Leave salary in
the current year from some
Total amount of exemption other employers
cannot exceed Rs.3,00,000.
Then
Ceiling = Rs.3,00,000 -
Leave salary exempted in
earlier years.

MT Educare –CA Inter Income Tax Classes – R.SOUMYANARAYANAN.FCA. GRAD CWA. Page-76
Chapter-5: Salary

Points requiring attention:

1 For the purpose of this section, Government employee means Central and State
Government employees.
2 A Government servant dies in harness. Leave salary is paid to his family. It is not taxable
in the hands of recipient. (CBDT circular)
3 Leave salary paid to the legal heirs of a deceased employee in respect of privilege leave
standing to the credit of such employee at the time of his death is an ex-gratia payment
on compassionate grounds in the nature of gifts. Thus the payment is not in the nature
of salary. (CBDT Circular).

10. Retrenchment compensation:

(a) Meaning of retrenchment compensation:

1 It is a compensation paid to the employees whose services got terminated on account of


their retrenchment.
2 Any compensation received by a workman at the time of closing down of undertaking in
which he is employed, shall also be treated as retrenchment compensation.
3 Employer transfers the ownership or management of the undertaking in which the
employee is employed to a new employer. As a result the employee’s service gets
interrupted or the terms & conditions applicable to the workman after such transfer
becomes less favourable when compared with those before transfer. Any compensation
received by the employee in this regard, shall also be treated as retrenchment
compensation.

(b) Taxability of retrenchment compensation:


Though it is a capital receipt, in view of specific provisions of the Act, it is taxable in the
hands of employee (subject to the exemption available U/s 10 (10B)).

(c) Format for computation of taxable retrenchment compensation


1 Amount of retrenchment compensation received *****
2 Exemption under section 10 (10B) (***)
3 Taxable portion of retrenchment compensation (1-2) *****

(d) Computation of exemption U/s 10 (10B):


Least of the following is exempted
1 Amount of retrenchment compensation received
2 Amount calculated under S. 25F (b) of the Industrial disputes Act [i.e. (Average salary of
last 3 months * 15/ 26) *completed years of service and part thereof in excess of 6
months].
3 Rs. 500000

Note: The entire amount of retrenchment compensation is exempted if it is paid in


accordance with the scheme approved by CG.

MT Educare –CA Inter Income Tax Classes – R.SOUMYANARAYANAN.FCA. GRAD CWA. Page-77
Chapter-5: Salary

11. Compensation received on voluntary retirement:

(a) Meaning of voluntary retirement compensation & its taxability:

1 In order to achieve overall reduction in the existing strength of employees, the employers may
introduce golden hand shake schemes under which, the employees may opt for voluntary
retirement.
2 On account of voluntary retirement, the employees will be paid a compensation, which
though a capital receipt, in view of specific provisions of the Act, will be assessed to tax in the
hands of employees U/H ‘Salaries’ (Subject to exemption U/S. 10 (10C).

(b) Format for computation of taxable voluntary retirement compensation:

SN Particulars Amount
1 Compensation received or receivable under VRS *****
2 Exemption U/S 10(10C) (***)
3 Taxable voluntary retirement compensation (1-2) *****

(c) Exemption U/s 10 (10C):

Conditions for availing exemption:

1 The employee should be an eligible employee.


2 The employee should not have availed exemption under this section in the past. That is,
exemption under this section is available to an assessee only once in his life time.
3 The employee should not have availed relied U/s 89 (1) in the past in respect of
voluntary retirement compensation.
4 The voluntary retirement scheme framed by the employer has to conform to the
guidelines prescribed in R. 2BA of the Income tax Rules.

Note: Exemption U/s 10 (10C) and relief U/s 89 (1) are mutually exclusive. Assessee has to
choose any one among them.
Who are eligible employees?
Following are eligible employees for the purpose of S. 10 (10C):
1. Employees of Public Sector Company
2. Employees of any other Company
3. Employees of an authority established under a Central / State Act
4. Employees of Local Authority
5. Employees of Co-operative Society
6. Employees of University
7. Employees of IIT
8. Employees of SG
9. Employees of CG
10. Employees of any institution having importance thro’ out India or in any State which
the CG may notify in this regard.
11. Employees of any Institute of Management as CG may notify

MT Educare –CA Inter Income Tax Classes – R.SOUMYANARAYANAN.FCA. GRAD CWA. Page-78
Chapter-5: Salary

Guidelines prescribed U/R 2BA:

1 The scheme should intend overall reduction of existing strength of the employees.
2 The resulting vacancies shall not be filled up.
3 The retiring employees shall not be employed in another company or concern belonging
to the same management.
4 The scheme should apply only to employees who have completed 40 years of age or
completed 10 years of service. However, there is no such requirement in respect of
employees of Public sector companies.
5 The scheme should not apply to the directors of the company or society (employers).
6 The compensation payable shall not exceed the higher of the following: (a) Last drawn
salary*3 Months* No. of completed years of service or (b) Last drawn salary * residual
period represented in months.

Quantum of exemption

Least of the following is exempted:


1 Amount of compensation received or receivable
2 Higher of [(Last drawn salary * 3 months * No. of completed years of service) or (Last
drawn salary * residual period represented in months)
3 Rs. 500000
Note: The exemption under this section is available, even if the compensation is paid in
installments.

Salary for this purpose:


Basic Salary + DA (if considered for retirement benefits) + Fixed % commission on turnover
achieved by employees.

MT Educare –CA Inter Income Tax Classes – R.SOUMYANARAYANAN.FCA. GRAD CWA. Page-79
Chapter-5: Salary

Guidelines U/R 2BA-an overview:

Does
Scheme of voluntary
retirement / Separation -
intends overall reduction in the
NO Compensation is taxable
existing strength of employees
and resulting vacancy not to be
filled up
?

Yes

Does
Retiring employee
employed in another
Yes Compensation is taxable
company - belonging to the
same management
?

No

Is
Person retiring director of
Yes Compensation is taxable
the Company
?

No

Is person retiring employee


Yes
/ Worker of Public sector
Co.
?

No Does
Completed age of
Compensation
Does the employee / worker >= No
is taxable
Employee/worker 40
No
completed 10 years of ?
service
?
Yes
Yes
Try on your own:
Does the
employee / worker
availed the exemption
Yes Compensation is taxable
already u/s 10(10C) in
earlier years
?

No

Compensation to the extent


exempted not taxable

MT Educare –CA Inter Income Tax Classes – R.SOUMYANARAYANAN.FCA. GRAD CWA. Page-80
Chapter-5: Salary

12. Provident fund

(a) Meaning of provident fund scheme:

It is one of the welfare schemes for the benefit of employees which is intended to
encourage employees to make systematic savings for their future.

(b) How does the provident fund scheme operate?

1 Provident fund is nothing but a Fund/Trust constituted by the employer or under the EPF
& MP Act 1952 or under PF Act 1925.
2 It is administered by the trustees or by the PF commissioner, as the case may be.
3 Every month, under this scheme, the employer deducts a certain sum of money (specified
by the employee) from the salary payable to the employee.
4 Then, the employer contributes such amount to the provident fund account of the
employee.
5 In addition to this, he also contributes a certain percentage of salary of the employee to
the provident fund account of the employee.
6 These contributions are deposited or invested in gilt-edged securities (i.e. risk-free
securities like Government securities).
7 Out of the income earned from these investments, after meeting the expenses of the PF,
some interest is credited to the PF account of the employees at such rate which varies
from time to time.
8 Thus, the balance in the PF account of the employee keeps accumulating year after year.
9 Upon retirement or resignation, the accumulated amount is paid to the employee.
10 There are 4 elements in the lump-sum amount paid to the employee at the time of
retirement or resignation: (a) employer’s contribution; (b) employees contribution; (c)
interest on employer’s contribution; (d) interest on employee’s contribution.

Points requiring attention:

1 Employer’s contribution- The contribution made by the employer is over and above the
income salary of the employee. Therefore, it is income deemed to be
received by the employee though it is not immediately made
available to him. [S. 7]
2 Interest credited to PF Interest credited to PF account of the employee is also income
account-income deemed to be received by the employee. [S. 7].

(c) Kinds of Provident Funds:

1 Statutory provident fund (SPF) This fund is set up under the provident fund Act 1925.
The scheme under this Act is mainly meant for Government
employees/Semi-Government employees, employees of
University/ EI affiliated to University established by statute.

MT Educare –CA Inter Income Tax Classes – R.SOUMYANARAYANAN.FCA. GRAD CWA. Page-81
Chapter-5: Salary

2 Recognised provident fund (RPF)


(a) Meaning of RPF Scheme The Recognised provident fund scheme is a scheme to
which the employee’s provident funds and miscellaneous
provisions Act, 1952 applies.
(b) To whom RPF scheme According to this Act, any person who employs 20 or more
obligatory? employees is having an obligation to register himself under
the PF Act 1952 and start a PF scheme for the employees in
his organization.
(c) Voluntary application of RPF Even a person who employs less than 20 employees can
scheme register himself voluntarily and start a PF scheme for his
employees.
(d) Options to the employer He may opt for the Government scheme set up by the PF
regarding PF scheme commissioner under the PF Act 1952. This scheme is called
EPF Scheme 1952 (framed U/s 5 of the EPF & MP Act 1952).
Alternatively, he may start a PF scheme of his own in the
accordance with the provisions of the Act and rules framed
thereunder and get the approval of PF commissioner. (Own
scheme).
(d) Approval of Commissioner of EPF Scheme 1952 is already recognised by CIT. But for the
Income-tax (CIT) own scheme, approval of CIT is to be obtained. If approval
of CIT is obtained, it becomes a RPF.
3 Unrecognized provident fund If the approval of CIT could not be obtained, it becomes an
(URPF) unrecognized provident fund.
4 Public provident fund This is a scheme which is covered under the Public
Provident Fund Act, 1968.
Any member of a public whether in employment or not,
may contribute to this fund. He has to open PPF account
with SBI or Banks or Post office.
This is a scheme where there is assessee’s contribution
only.
The minimum contribution to this fund is Rs. 500 and the
maximum is Rs. 150000 per year.
The contributions made to the fund along with interests are
payable after 15 years.
The interest rate, under the scheme, varies from time to
time.

(d) Tax treatment of provident fund:


Items SPF RPF URPF PPF
Employee’s Deduction U/S. Deduction U/S. Deduction U/S. 80C is not Deduction U/S.
contribution 80C is available 80C is available available. 80C is available
Employer’s Fully exempt Exempt up to It is not taxable in the year Not applicable.
contribution 12% of salary. of contribution. But it is
Excess amount taxable when lump sum
is taxable. payment is received upon
termination of service.

MT Educare –CA Inter Income Tax Classes – R.SOUMYANARAYANAN.FCA. GRAD CWA. Page-82
Chapter-5: Salary

Interest credited Fully exempt Exempt from It is not taxable in the year Fully Exempt
to provident from tax tax up to 9.5%. of credit of interest. But it is from tax
Fund Excess interest taxable when lump sum
is included in payment is received upon
gross salary. termination of service.
Lump Sum Fully Exempt Exempt (S. 10 Accumulated employee’s Fully Exempt
payment at the from tax (S. 10 (12)). However contribution is not taxable. from tax
time of (11)) taxable in Accumulated employer’s (S. 10 (11))
retirement or certain cases. contribution + interest
termination of thereon till date is
service includible in gross salary.
Interest on employee’s
contribution is taxable U/H
‘Other Sources’.

Salary for this purpose:

Basic salary + Dearness allowance (if the terms of employment so provide) + Fixed
percentage commission on turnover achieved by the employee.

Circumstances under which the lump sum amount received from RPF are not taxable:

Circumstance-1 The employee has rendered continuous service with his employer for a
period of 5 years or more.
Circumstance-2 He could not render such continuous service of 5 years, since, his service
got terminated due to (a) his ill-health (b) contraction or discontinuance
of business by his employer (c) any other reason beyond his control.
Circumstance-3 Upon termination of his employment before the aforesaid period of 5
years, he obtains employment with any other (new) employer. The
accumulated balance due and becoming payable to him is transferred to
his individual account in any RPF maintained by the new employer.

Note: For computing the period of 5 years, in circumstance 1 & 2, the period(s) for which
such employee rendered continuous service under his former employer (s) shall also be
included provided they were maintaining RPF and the accumulated balance due and payable
to him by them got transferred to his individual account in a RPF maintained by the
subsequent employer.
Tax treatment of lump sum amount received from RPF under other circumstances:
In other circumstances, no exemption is available in respect of lump sum received from RPF.

MT Educare –CA Inter Income Tax Classes – R.SOUMYANARAYANAN.FCA. GRAD CWA. Page-83
Chapter-5: Salary

13. Employer’s contribution to the account of employee under a pension scheme referred
to in S. 80CCD:

(a) National Pension Scheme is a scheme approved by the CG for Indian Citizen aged between
18-60 years.
(b) Subscriber of the NPS account contributes some amount in their account.
(c) In case of any employee, being a subscriber of the NPS account, employer may also
contribute into the employee’s account.
(d) However, while computing TI of the employee-assessee, a deduction U/s 80CCD is allowed
to the assessee in respect of employer’s as well as employee’s contribution.

14. Profit in lieu of salary - S. 17 (3):

1 The amount of any compensation due to or received by an assessee from his employer or former
employer at or in connection with the termination of his employment.
2 The amount of any compensation due to or received by an assessee from his employer or former
employer at or in connection with the modification of the terms and conditions of employment.
3 Any payment due to or received by an assessee from his employer or former employer except
the following:
a) Payment of gratuity exempted U/s 10 (10);
b) Payment of house rent allowance exempted U/s 10 (13A);
c) Payment of commuted pension exempted U/s 10 (10A);
d) Payment of retrenchment compensation exempted U/s 10 (10B);
e) Payment from an approved Superannuation Fund U/s 10 (13);
f) Payment from statutory provident fund or public provident fund;
g) Payment from recognised provident fund to the extent it is exempt U/s 10 (12)
4 Any payment from unrecognized provident fund or such other fund to the extent to which it does
not consist of contributions by the assessee or interest on such contributions.
5 Any sum received under a Keyman insurance policy including the sum allocated by ways of bonus
on such policy.
6 Any amount received (in lump sum or otherwise) prior to employment or after cessation of
employment.

15. Key man insurance policy – Tax implications:

1 Meaning of Keyman Businessman pays premium to the Insurance company to effect


insurance policy insurance on the life of keyman. This is called keyman insurance policy.
2 Meaning of keyman Keyman may be an employee or any other person connected in any
manner with the business carried on by the businessman (say partner,
agent or any other resource person).
3 Purpose of effecting Keyman insurance policy is effected to protect the business against the
keyman insurance financial loss which may occur in the likely eventuality of pre-matured
policy death of key-man.
4 Tax implications in the Premium paid is an expense wholly and exclusively incurred for the
hands of businessman purpose of business. It is eligible for deduction U/s 37 (1) while
computing the business income in the hands of businessman. [B.N

MT Educare –CA Inter Income Tax Classes – R.SOUMYANARAYANAN.FCA. GRAD CWA. Page-84
Chapter-5: Salary

Exports (Bom) + CBDT Circular 762].


The businessman will receive the proceeds of the policy upon maturity
of the policy or death of the keyman. Such proceeds are income in view
of S. 2 (24) (xi). It is taxable U/H PGBP. [S. 28 (vi)].
5 Tax implications in the Businessman may assign the policy in favour of keyman. In such case,
hands of keyman the keyman may receive the proceeds of the policy from the insurance
company upon surrender or maturity of the policy. Such proceeds are
income in view of S. 2 (24) (xi). It is taxable in the hands of keyman U/H
salary if the keyman is employee. (S. 17 (3) (ii). If keyman is not an
employee, it shall be taxed in the hands of keyman U/H IFOS. [S. 56 (2)
(iv)].

16. Perquisites:
(a) Meaning of perquisite:
(i) In addition to salary or wages, any benefit or amenity provided by the employer to the
employee either free of cost or at a concessional rate is called perquisite.
(ii) Such benefit or amenity, to be called as perquisite, should result in some personal
advantage to the employee and should not merely facilitate proper discharge of his
duties.
(iii) It may be in cash or in kind.
(iv) It may be recurring or non-recurring.
(v) To tax the benefit or amenity as perquisite, it is not necessary that it should have been
received under an enforceable right. To the contrary, the benefit or amenity provided
by the employer unilaterally without the aid of any agreement with the employee is
also taxable as perquisites.
(vi) But any unauthorized benefit or amenity enjoyed by the employee without the
employer’s authority is not taxable as perquisite. To tax any benefit or amenity as
perquisite, it should have a legal origin. Here, the employee is having a legal obligation
to restore back such advantage.
(vii) S. 17 (2) of the Act defines the term ‘perquisites’. S. 17 (2) has 8 sub-clauses each of
which is discussed detailed below along with the relevant rules.

(b) Rent-free accommodation – S. 17 (2) (i) + R. 3 (1):

Perquisites include value of rent-free accommodation provided to the assessee by his


employer. It shall be determined in a manner laid down in R. 3 (1).

Accommodation can be of two types: (a) Unfurnished; (b) furnished. Where the employer
not only provides accommodation to the employee but also household appliances, the
employee is said to be provided a furnished accommodation.

MT Educare –CA Inter Income Tax Classes – R.SOUMYANARAYANAN.FCA. GRAD CWA. Page-85
Chapter-5: Salary

Determination of value of rent-free unfurnished accommodation:

SN Situations Taxable value of perquisite


1 Accommodation is provided by the License fee determined by the Government for the
Government to its employees period of occupation.
2 Owned accommodation is provided 7.5% of salary for the period of occupation (if the
by an employer (other than accommodation is in a city/town having
Government) to his employees population not exceeding 10 Lakhs as per 2001
census).
10% of salary for the period of occupation (if the
accommodation is in a city/town having
population exceeding 10 Lakhs but not exceeding
25 Lakhs as per 2001 census).
15% of salary for the period of occupation (if the
accommodation is in a city or town having
population exceeding 25 Lakhs).
3 Leased accommodation is provided by 15% of salary for the period of occupation or lease
an employer (other than Government) rent paid by the employer whichever is less.
to his employees

Determination of value of rent-free unfurnished accommodation:

If the employer provides a furnished accommodation to the employee free of rent, the
perquisites in this regard shall be valued as follows:

Perquisite value of rent-free unfurnished accommodation *****


Add: Value of furniture
If owned by the employer, then 10% per annum of original cost of such ****
furniture
If hired by the employer from third party, then actual hire charges. ****
Perquisite value of rent-free furnished accommodation *****

Note: Furniture includes TV sets + Radio + Refrigerator +AC + other house hold appliances.

Points requiring attention:

1 For this purpose, salary is to be understood as follows:


Salary includes Salary excludes
Basic salary Employers contribution to PF
DA (if it is considered for retirement DA (if it is not considered for retirement
benefits) benefits)
All taxable allowances Exempted allowances
Bonus or commission or fees Perquisites
Any other monetary payment
2 Salary for this purpose shall be determined on ‘Due basis’. Salary pertaining to the next
PY received in advance in the current PY shall be ignored.

MT Educare –CA Inter Income Tax Classes – R.SOUMYANARAYANAN.FCA. GRAD CWA. Page-86
Chapter-5: Salary

3 Salary from two or more employers in respect of the period during which the
accommodation is provided shall be taken into consideration.
4 Perquisite U/s 17 (2) (i) is taxable for all categories of employees in the year in which the
accommodation is provided.
5 Where the accommodation is provided by the CG or SG to an employee who is serving
on deputation with anybody under the control of such government, such body shall be
deemed to be the employer of the employee and the normal valuation rules shall apply.
Under such circumstances the license fees mechanism shall not apply.
6 Only those perquisites which the employee actually enjoys have to be valued and taxed
in his hands. Suppose a company offers a housing accommodation rent-free to an
employee but the latter declines to accept it, then the value of such accommodation
obviously cannot be evaluated and taxed in the hands of the employees.

Cases where value of rent-free accommodation is exempt from tax:

1 Rent-free official residence provided to a Judge of a High Court or of the Supreme Court
is exempt from tax.
2 Rent-free furnished residence provided to an Official of Parliament, A Union Minister or
the Leader of Opposition in Parliament is exempt from tax.
3 Rent-free accommodation provided to an employee working at mining site or on shore
oil exploration site or off shore site or project execution site or dam site or power
generation site is not taxable if the following conditions are satisfied:
Condition-1 The accommodation should be of a temporary nature.
Condition-2 The plinth area of that accommodation shall not exceed 800 square
feet.
Condition-3 The accommodation shall be located atleast 8 km away from the local
limits of Municipality or cantonment in a remote area (an area
located atleast 40 km away from a town having population not
exceeding 20000).

Valuation of accommodation on transfer from one place to another:

For the first 90 days Where accommodation is provided to the employee both at the
from the date of existing and new place of work, the value of accommodation which
transfer has the lower value, shall be taxed.
After 90 days Value of both accommodations shall be taxed.

(c) Concessional accommodation – S. 17 (2) (ii) + R. 3 (1):


Perquisite includes value of any concession in the matter of rent respecting any
accommodation provided to the assessee by the employer.
It is taxable in the hands of all categories of employees. Taxable value of concession shall be
determined in a manner laid down below:
1 Perquisite value of rent-free accommodation *****
2 Amount recovered from the employee (***)
3 Taxable value of perquisite with respect to accommodation provided at *****
concessional rate (1-2)

MT Educare –CA Inter Income Tax Classes – R.SOUMYANARAYANAN.FCA. GRAD CWA. Page-87
Chapter-5: Salary

Accommodation provided in hotel:

1 Employer may provide accommodation to the employee in a hotel.


2 The taxable value of this perquisite is lower of (a) 24% of salary; (b) actual charges paid
or payable to the hotel.
3 If any amount is recovered from the employee in this regard, it shall be reduced while
computing the value of perquisite.
4 However, where the employee is provided such accommodation for a period not
exceeding in aggregate 15 days on his transfer from one place to another, there would
be no perquisite.

(d) Benefits or amenities provided free of cost or at concessional rate to specified


employees: S. 17 (2) (iii):

1 Perquisite includes benefits amenities provided free of cost or at concessional rate to


specified employees.
2 However, the use of any vehicle provided by the employer for journey by the assessee
from his residence to his office or other place of work, or from such office or place to his
residence shall not be regarded as perquisite. [Explanation to S. 17 (2) (iii)].

Specified employee:
Following employees are regarded as specified employees:
1 Director-employee An employee of a company who is a director is a specified employee. It is
immaterial whether he is a full-time director or part-time director. It does
not matter whether he is a nominee of the management, workers, financial
institutions or the Government. It is also not material whether or not he is a
director throughout the PY.
2 An employee who An employee of a company who has substantial interest in that company is
has substantial a specified employee. A person has a substantial interest in a company if he
interest in the is a beneficial owner of equity shares carrying not less than 20% voting
company power in the company. [Note-1].
3 Employee drawing An employee other than an employee described in (1) & (2) above, whose
in excess of Rs. income chargeable U/H Salaries exceeds Rs. 50000 is a specified employee.
50000 The above salary is to be considered exclusive of value of all benefits or
amenities not provided by way of monetary payments. [Note-2 & 3].

Note:
1 In order to determine whether a person has substantial interest in a company, it is the
beneficial ownership of equity shares carrying 20% or more voting power that is relevant rather
than the legal ownership.
2 For computing the limit of Rs. 50000, the following items are to be excluded or deducted:
(a) All non-monetary benefits;
(b) Monetary benefits which are exempt U/s 10.
(c) Deductions U/s 16
3 If an employee is employed with more than one employer, the aggregate of the salary received
from all employers is to be taken into account in determining the above ceiling of Rs. 50000.

MT Educare –CA Inter Income Tax Classes – R.SOUMYANARAYANAN.FCA. GRAD CWA. Page-88
Chapter-5: Salary

Question
Mr. A, karta of a HUF, is a registered shareholder of Bright Ltd. The amount for purchasing
the shares is financed by the HUF. The dividend is also received by the HUF. Mr. A is also an
employee of Bright Ltd. Determine as to whether he is a specified employee.
Answer:
1 Mr. A has no beneficial ownership in these shares. It vests with HUF.
2 It is only for the purpose of satisfying the statutory requirements that the shares are registered
in the name of Mr. A.
3 All benefits arising from the shareholding goes to the HUF.
4 Therefore, he is not a specified employee.
5 Thus, an employee who is not a registered shareholder will not be considered specified
employee even if he has beneficial interest in 20% or more of the equity shares in the company.

(e) Payments made by employer to meet employee’s obligation – S. 17 (2) (iv):


(i) Perquisite includes any sum paid by the employer in respect of any obligation which
but for such payment, would have been payable by the employee.
(ii) Example: Income-tax or professional tax paid by the employer on behalf of the
employee.
(iii) This is taxable in the hands of all employees.

Valuation of perquisites in respect of domestic servant – R. 3 (3):

Domestic Servant

Personal
Sweeper Watch Man Gardener
Attendant

Domestic Servant

Engaged by the employee and salary Engaged by the employer and


is paid/reimbursed by the employer salary is paid/reimbursed by the
employer

Taxable - in the hands of all Taxable - in the hands of Specified


employees employees

Taxable Perks = Taxable Perks =


Actual amount incurred Actual amount incurred
by the employer XXX by the employer XXX
Less: Amount recovered Less: Amount recovered
from employee XXX from employee XXX

MT Educare –CA Inter Income Tax Classes – R.SOUMYANARAYANAN.FCA. GRAD CWA. Page-89
Chapter-5: Salary

Valuation of perquisite in respect of gas, electricity/water supply provided for free of cost:
(R. 3(4)):

Gas/Electricity/Water
supply connection

Taken in the name of employee Taken in the name of employer


& &
bills for these supplies are paid/reimbursed he bears the expenditure there of
by the employer

Taxable as perks in the hands


Taxable as perks in the hands of all of specified employees
employees

Quantum of perquisite:

Gas/Electricity/Water

Purchased by the employer from Supplied by the employer


an outside agency out of own sources

Taxable perks = Taxable perks =


Amount paid to the outside agency Manufacturing cost per unit XXX
Less: Recovery from employee incurred by the employer
Less: Recovery from employee XXX

Value of perquisite in respect of free or concessional transport facility provided by a


transport undertaking to its employees – R. 3 (6):

1 Employer is engaged in the business of carriage of goods or passengers.


2 He provides free or concessional tickets for private journeys of employees or of
members of his household.
3 Taxable value of perquisite = Value at which the benefits are provided to general public
Less amount recovered from the employees.
4 Taxable only in the hands of specified employees.
5 Employees of railways and airlines are not liable to tax.
Free or concessional transport facility to
Employees

Employees of Railways/Airlines Employees of any other transport undertaking

Taxable Perks =
Taxable perks = NIL
Value at which such benefit
is offered by the employer to public XXX
Less: Recovery from the employee XXX
Note: This is taxable only in
case of specified employees

MT Educare –CA Inter Income Tax Classes – R.SOUMYANARAYANAN.FCA. GRAD CWA. Page-90
Chapter-5: Salary

Valuation of perquisite in respect of education facility provided to member of employees’


household: (R. 3(5)):

Educational facility:

Different Forms

Training of Payment of school Education facility in


Fixed Education Reimbursement of employer’s own
Employees fees of employees
allowance school fees of institue
children
employees children

Amount spent on Fixed Allowance School fees of family Reimbursement of


providing free- ➢ Rs. 100 p.m per child members of employee expenses incurred
education facilities to (restricted to 2 children) paid by the employer for education of
and training of exempt from tax directly to the school is family member is
employee is not ➢ Hostel Expense taxable (in the hands taxable as a perk
taxable Rs. 300/p.m per child of the specified as well in case of all
(restricted to 2 children) as non-specified employees
exempt from tax, employees)
(Balance taxable)

Education facility is
provided for Education facility is
employees children provided to member of
his household

Cost of Education Cost of Education Taxable Perks =


less than or equial greater than 1000 Cost of Education
to 1000 p.m per p.m per child in a similar institution XXX
child (no restriction Less:
on number of Amount recovered
children) from the employee XXX

Taxable Perks =
Cost of Education
in a similar institution XXX
Nothing is taxable Less:
Rs. 1000 p.m per child XXX
Amount recovered
from the employee XXX

Points requiring attention:


1 Member of household includes (a) Spouse; (b) Children and their spouses; (c) parents;
(d) servants & dependants.
2 Grand children – are not children of the employee but are considered as any other
member of employee’s house hold.
3 Rs. 1000 is not a standard deduction. It is a threshold limit. R. 3 (5) shall not apply only
when educational facility is provided to the children of the employee free of cost and the
cost of such education in a similar educational institution does not exceed Rs. 1000 p.m
per child. [Delhi Public School (P&H)].
4 No restriction on number of children contemplated in R. 3 (5).
5 Where education is provided to the family members of the employees at the institution
owned and maintained by the employer, the resulting perquisite is taxable in the hands
of specified employees only.

MT Educare –CA Inter Income Tax Classes – R.SOUMYANARAYANAN.FCA. GRAD CWA. Page-91
Chapter-5: Salary

Valuation of perquisite in respect of motor car-R. 3:

Valuation of perquisites when employee owns the motor car:

Situation Narration of the situation Taxable value of perquisite


1 Expenses are met by the employee No perks.
2 Expenses are met by the employer - car No perks – subject to conditions. (Refer note-1)
used wholly for official purposes (OP)
3 Expenses are met by the employer - car Expenses incurred by employer Less amount
used wholly for private purposes (PP) recovered from employee. (Refer note-1A)
4 Expenses are met by the employer - car Expenses incurred by employer Less Rs. 1800 pm
used partly for PP and partly for OP- Less Rs. 900 pm if driver is provided Less amount
Cubic capacity of car <= 1.6 Litres recovered from employee. (Refer note-1A & 2)
5 Expenses are met by employer - car Expenses incurred by employer Less Rs. 2400 pm
used partly for PP and partly for OP - Less Rs. 900 pm if driver is provided Less amount
Cubic capacity of car > 1.6 Litres. recovered from employee. (Refer note – 1A & 2)

Motor car owned by


Employee

Maintenance and running


Maintenance and running
expenses are met/
expenses are met by the
reimbursed by the
employee
employer

Not a perk - hence not


If car is used If car is partly used for taxable
If car is used wholly
wholly for private official purpose and
for official purpose
purpose partly for private purpose

Nothing is taxable if the Actual exp. Actual exp.


following conditions are by employer XXX by employer XXX
satisfied
Less Less
· Employer maintains Amount used
complete details of recovered from
the employee XXX for official pur. XXX
journey undertaken
for official purpose - Less
such as date of Taxable perks XXX
recovered from
journey, the employee XXX
destination,
mileage, amount of (For all Taxable perks XXX
expenditure employees)
· The Employer gives
a certificate to the
Amount used for official purpose
effect that the
Cubic capacity of engine
expenditure was
incurred wholly and · =< 1.6 litres Rs. 1800pm
exclusively for · > 1.6 litres Rs. 2400pm
official purpose · if driver is provided Rs. 900
pm (in addition)

MT Educare –CA Inter Income Tax Classes – R.SOUMYANARAYANAN.FCA. GRAD CWA. Page-92
Chapter-5: Salary

Note-1: The conditions to be satisfied are as follows:


Condition-1 The employer has maintained complete details of journey undertaken for
official purpose which may include date of journey, designation, mileage
and the amount of expenditure incurred thereon.
Condition-2 The employer gives a certificate to the effect that the expenditure was
incurred for the performance of official duties.

Note-1A: Taxable both in the hands of specified as well as non-specified employees.

Note-2: A higher sum for official purposes can be reduced if it could be proved from the
records maintained by the employer.

Valuation of perquisites when employer owns or hires the motor car:

Situation Narration of the situation Taxable value of perquisite


1 Maintenance & running expenses No perks – subject to condition. (Refer note-1)
are met by the employer – car
used wholly for OP.
2 Maintenance and running Actual expenditure incurred by the employer (including
expenses are met reimbursed by driver’s salary and normal wear and tear computed at
the employer – car used wholly the rate of 10% on the actual cost or hire charges if car
for PP. was taken on hire) Less amount recovered from the
employee. (Taxable only in the hands of specified
employees)
3 Maintenance and running Rs. 1800 pm + Rs. 900 pm if driver is provided. (Taxable
expenses are met by the only in the hands of specified employees). Nothing is
employer – car used partly for PP deductible, even if any amount is recovered from the
and partly for OP – Cubic capacity employee.
of car <= 1.6 Litres.
4 Maintenance and running Rs. 2400 pm + Rs. 900 pm if driver is provided. (Taxable
expenses are met by the only in the hands of specified employees). Nothing is
employer – car used partly for PP deductible, even if any amount is recovered from the
and partly for OP – Cubic capacity employee.
of car > 1.6 Litres.
5 Maintenance and running No perks.
expenses are met by the
employee – car used wholly for
OP.
6 Maintenance and running Driver’s salary paid by the employer and normal wear
expenses are met by the and tear computed at the rate of 10% on the actual cost
employee – car used wholly for or hire charges borne by the employer if car was taken
PP. on hire Less: amount recovered from the employee.
(Taxable only in the hands of specified employees)
7 Maintenance and running Rs. 600 pm + Rs. 900 pm if driver is provided. (Taxable
expenses are met by employee – only in the hands of specified employees). Nothing is
car used partly for PP and partly deductible, even if any amount is recovered from the
for OP – CC of car <= 1.6 Litres. employee.

MT Educare –CA Inter Income Tax Classes – R.SOUMYANARAYANAN.FCA. GRAD CWA. Page-93
Chapter-5: Salary

8 Maintenance and running Rs. 900 pm + Rs. 900 pm if driver is provided. (Taxable
expenses are met by employee – only in the hands of specified employees). Nothing is
car used partly for PP and partly deductible, even if any amount is recovered from the
for OP – CC of car > 1.6 Litres. employee.

Motor car owned or hired by


Employer

Maintenance & running


expenses are met or
reimbursed by employer

If car is If car is If car is used partly


wholly used wholly used used for official
for official for private purpose and partly
purpose purpose for private purpose

Nothing is taxable if the Actual expenses on Taxable perks for


specified Maintenance & running
following condition are running & maintenance
employees expenses are met or
satisfied by employer XXX
reimbursed by employee
Add: Driver’s Salary XXX
* Employer maintains Add: Normal wear & tear Rs.1800 p.m. (if the
complete details of journey (10% of actual cost of the car) cubic capacity<=
undertaken for official (if owned by the employer) XXX 1.6 litres)
purpose such as date of Add: Hire charges
journey, destination, (if car is taken on hire) XXX Rs.2400 p.m. (if the
mileage, amount of Less: Amt. recovered cubic capacity> 1.6
expenses from employee (XXX) litres)
* the employer gives a -------
certificate to the effect Taxable Perks for specified XXXX +
that the expenses was employees ------- Rs.900 p.m. (if
incurred wholly & Driver is provided)
exclusively for official
purpose
If car is If car is If car is used partly
wholly used wholly used used for official
for official for private purpose and partly
purpose purpose for private purpose

Taxable perks
Not a perk. Taxable perks for
for specified
Hence, not specified employees
employees
taxable. Rs.600 p.m. (if
Normal wear & tear
the cubic
(10% of actual cost)
capacity<= 1.6
(if car is owned by
litres)
employer)
Rs.900 p.m. (if
OR
the cubic
Hire charges borne
capacity> 1.6
by employer (if car
litres)
was taken on rent)
+
+
Rs.900 p.m. (if
Salary of Driver (if
Driver is
provided by
provided)
employer)
Less: Amount
recovered from
employee

MT Educare –CA Inter Income Tax Classes – R.SOUMYANARAYANAN.FCA. GRAD CWA. Page-94
Chapter-5: Salary

Employee owns any automotive conveyance (other than car) and running and
maintenance charges are met or reimbursed by the employer:

Situation Narration of the situation Taxable value of perquisite


1 Wholly used for official No perks – subject to the conditions (Refer note – 1)
purposes
2 Used partly for official and Actual expenditure incurred by the employer Less Rs.
partly for private purposes 900 per month or a higher sum for official purposes as
per the records of the employer Less amount
recovered from the employee. (Taxable in the hands
of specified as well as non-specified employees).

Automotive conveyance used

Partly for official purpose and


Wholly for official purposes
partly for private purposes

No value provided following ➢ Step 1: Find out actual


conditions are satisfied expenditure incurred by
the employer
➢ The employer has ➢ Step 2: Less: Amount
maintained complete used for official
details of journey purposes (i.e. a sum
undertaken for official calculated at the rate of
purpose which may Rs. 900 per month or a
include date of journey, higher sum for official
destination, mileage, purposes as per records
and the amount of of the employer and as
expenditure incurred certified by the
thereon. employer.
➢ The employer gives a ➢ Step 3: Less: Amount
certificate to the effect recovered from
that the expenditure employee.
was incurred wholly and
exclusively for the Balancing amount (if it
performance of official is positive) is taxable
duties. value of the perquisite.

MT Educare –CA Inter Income Tax Classes – R.SOUMYANARAYANAN.FCA. GRAD CWA. Page-95
Chapter-5: Salary

Points requiring attention:

1 Month Only completed month should be considered. A part of the month is left out of
consideration.
2 Car facility The use of motor car by an employee for the purpose of going from his
between office & residence to the place where the duties of employment are to be performed or
residence from such place back to his residence – is not chargeable to tax.
3 Conveyance Conveyance facility provided to the HC Judges / SC judges is not chargeable to
facility to Judges tax.
4 More than one car Where the employer gives more than one motor car to the employee, the
is provided by the perquisite in respect of those motor cars shall be valued as follows:
employer (a) In respect of one of such cars (as selected by the employee), the value of
perquisite shall be the amount calculated on the assumption that it is used
partly for official purposes and partly for private purposes.
(b) In respect of the remaining cars, the value of perquisite shall be computed as
if they are used exclusively for private purposes.

Value of perquisite in respect of medical facilities provided to employees:

Perquisite in respect of free or concessional medical facilities provided by the employer to


his employees or their family members is taxable only in the hands of specified employees.

Medical facilities exempted from tax:

However, in the following cases, no perquisite arises in the hands of employees and
consequently, nothing is taxable in the hands of employees:

1 Treatment in hospital Where medical treatment is provided to the employees or their family
maintained by the members in a hospital maintained by the employer, nothing is taxable by
employer way of perquisite.
2 Treatment in Where medical treatment is provided to the employees or their family
Government hospital members in a hospital maintained by the Government or local authority
or in a hospital approved by the Government for the purpose of medical
treatment to its employees.
3 Treatment for The employee or his family members are suffering from diseases or
specified diseases or ailments referred to in R. 3A (2).
ailments in approved Treatment is provided in any hospital approved by CIT or CCIT.
hospitals Medical treatment expenditure is borne by the employer.
Nothing is taxable by way of perquisites in the hands of employees. But
this is subject to the condition that the employee obtains a certificate
from the hospital specifying the disease or ailment for which medical
treatment was required and the receipt for the amount paid to the
hospital.
4 Effecting of Employer pays premium to effect insurance on the health of employee
insurance policy on or his family members.

MT Educare –CA Inter Income Tax Classes – R.SOUMYANARAYANAN.FCA. GRAD CWA. Page-96
Chapter-5: Salary

the health of The insurance is under a scheme framed by the GIC and approved by the
employee or his CG or under a scheme framed by any other insurer and approved by
family members. IRDA.
Nothing is taxable by way of perquisite in the hands of employees.

Tax treatment of medical reimbursement:

Where the employee incurs expenditure on treatment of any disease or ailment other than
those referred to in R. 3A (2) or in a hospital other than one maintained by the Government
or Local authority or one approved by CIT or CCIT and gets the amount reimbursed from the
employer, an amount up to Rs. 15000 is not chargeable to tax by way of perquisite in the
hands of employee. Amount in excess of Rs. 15000 is taxable in the hands of all employees.

Ailments or diseases prescribed in R. 3A (2):

1 Cancer
2 Tuberculosis
3 AIDS
4 Disease or ailment of the heart, blood lymph glands, bone marrow, respiratory system, central
nervous system, urinary system, liver, gall bladder, digestive system, endocrine glands or the
skin, requiring surgical operation;
5 Ailment or disease of the eye, ear, nose or throat, requiring surgical operation
6 Fracture in any part of the skeletal system or dislocation of vertebrate requiring surgical
operation or orthopedic treatment;
7 Gynecological or obstetric ailment or disease requiring surgical operation, caesarean operation
or laparoscopic intervention;
8 Ailment or disease of the organs mentioned at (4), requiring medical treatment in a hospital for
at least three continuous days;
9 Gynecological or obstetric ailment or disease requiring medical treatment in a hospital for at
least three continuous days;
10 Burn injuries requiring medical treatment in a hospital for at least three continuous days.
12 Mental disorder-neurotic or psychotic-requiring medical treatment in a hospital for at least
three continuous days;
13 Drug addiction requiring medical treatment in a hospital for at least seven continuous days;
14 Anaphylactic shocks including insulin shocks, drug reactions and other allergic manifestations
requiring medical treatment in a hospital for at least three continuous days.

Points requiring attention:


1 Family, for this purpose, means spouse, children, dependent parents, brothers & sisters.
2 Child may be son or daughter, natural or adopted, minor or major, married or
unmarried, dependent or independent.
3 Father-in-law and mother-in-law are not part of family.
4 Grandchildren are not part of family.
5 Grandparents are not part of family.
6 Parents, brothers and sisters are part of family only when they are dependent on the
assessee.

MT Educare –CA Inter Income Tax Classes – R.SOUMYANARAYANAN.FCA. GRAD CWA. Page-97
Chapter-5: Salary

Medical facilities outside India:

Any expenditure incurred by the employer (or reimbursement of expenditure incurred by


the employee) on medical treatment of the employee or any member of the family of such
employee outside India is exempt subject to the conditions given below:-

Perquisite not chargeable to tax Condition to be satisfied


Expenditure on medical treatment of employee Expenditure shall be excluded from
or any member of family of such employee perquisite only to the extent permitted by
outside India borne by the employer. the RBI.
Cost of stay abroad of the employee or any Expenditure shall be excluded from the
member of the family for medical treatment perquisite only to the extent permitted by
and cost of stay of one attendant who the RBI.
accompanies the patient in connection with
such treatment borne by the employer.
Cost on travel of the employee/any member of Expenditure shall be excluded from
his family and one attendant who accompanies perquisite only in the case of an
the patient in connection with treatment employee whose GTI, as computed
outside India borne by the employer. before including therein the expenditure
on traveling, does not exceed Rs. 200000.

Computation of exemption with respect to foreign travel expenditure:

Step-1: Compute GTI of the assessee (after considering the perquisite with respect to stay
and medical treatment but without considering foreign travel reimbursement).

Step-2:
Situations Quantum of exemption
GTI before including reimbursement of foreign travel Reimbursement of foreign travel
expenditure of the patient and the attendant <= Rs. expenditure is fully exempt.
200000
GTI before including reimbursement of foreign travel Reimbursement of foreign travel
expenditure of the patient and the attendant > Rs. expenditure is fully taxable.
200000

Step-3: If foreign travel reimbursement is taxable as per step 2, recompute the income U/H
salary after including foreign travel reimbursement and GTI must also be recomputed.

(f) Premium paid to effect a contract of assurance or annuity – S. 17 (2) (v):

Perquisites include (a) any sum payable by the employer to effect an assurance on the life of
the employee; (b) any sum payable by the employer to effect a contract of annuity for the
benefit of employee.

MT Educare –CA Inter Income Tax Classes – R.SOUMYANARAYANAN.FCA. GRAD CWA. Page-98
Chapter-5: Salary

Points requiring attention:

1 The perquisite referred to in S. 17 (2) (v) is taxable in the hands of all employees.
2 Though the sums referred to in S. 17 (2) (v) are included in computation of income
chargeable U/H salary, these are eligible for deduction U/s 80C.
3 However, the employer’s contribution to staff group insurance schemes and the
payment of annual premium by the employer on personal accident policy effected by
him on the life of the employee are not taxable as perquisites.

(g) ESOP – S. 17 (2) (vi):

1 Employee stock Employer may offer stock options to his employee. Stock option
option means the right (not obligation) to subscribe to specified number of
shares or securities of the employer-company at a specified price
(called strike price which is less than the FMV of the shares or
securities) on a future date.
2 Events in stock (a) Granting of options; (b) vesting of options; (c) exercise of options;
option (d) allotment of shares or securities.
3 Granting of Means conferring on the employees the right to subscribe to shares
options and securities subject to fulfillment of specified vesting conditions.
4 Vesting of Once the vesting conditions are fulfilled, the granted stock options
options will vest on the employee. That means, the employee gets the right
to exercise the options.
5 Exercise of Once the options get vested on the employee, then the employee
options may within the stipulated time period (which we call as exercise
period) can exercise the option. That is, he can subscribe to the
shares or securities using the option.
6 Allotment of If the employee subscribes to the shares or securities in exercise of
shares or the options that are vested on him, the employer-company allots
securities such shares or securities upon employee paying the strike price.
7 Grant or vesting Means the time gap between the date of grant of options and the
period date of vesting of option. That is, the period during which the
employee has to fulfill his vesting conditions for becoming entitled
to exercise the option.
8 Vesting (a) Service condition: Condition requiring the employee to complete
conditions a specified period of service.
(b) Performance condition: Condition requiring the employee to
meet the specified performance targets.
9 Purpose of ESOP (a) To improve the performance of the entity by securing consistent
and concentrated efforts of its dedicated employees during the
vesting period.
(b) To enable the employees to have ownership stake in the
employer-company, so that they get a sense of belongingness which
will stimulate growth.
(c) To reward employees for making available intellectual property
rights (through sweat equity shares).

MT Educare –CA Inter Income Tax Classes – R.SOUMYANARAYANAN.FCA. GRAD CWA. Page-99
Chapter-5: Salary

10 Perquisite U/s Perquisites include the value of any security or shares allotted by
17 (2) (vi) the employer, or former employer, free of cost or at concessional
rate to the assessee under ESOP.
11 PY of PY of allotment of shares or securities.
chargeability
12 Quantum of [FMV of share or security on the date of exercise of option – strike
perquisite price paid] * Number of shares or securities allotted.

Computation of FMV of equity shares allotted or transferred under ESOP – R. 3 (8):

SN Situation FMV on the date of Remarks


exercise of option
1 When shares Average of opening and If on the date of exercise of option, there
in the closing price of shares on is no trading in shares, the FMV shall be
company are the date of exercise of the closing price of the share on RSE on a
listed on a option date closest to the date of exercise of
single RSE option and immediately preceding such
date of exercise of option.
2 Where shares Average of opening and If on the date of exercise of option there
in the closing price of shares on is no trading in shares, the FMV shall be
company are the date of exercise of the closing price of the share on a RSE
listed on more option on a RSE which which records the highest volume of
than one RSE records the highest trading on the date closest to the date of
volume in trading in the exercise of option and immediately
shares preceding such date of exercise of
option.
3 Where shares Value on specified date as Specified date means: (a) The date of
in the determined by a category exercise of option or (b) any date earlier
company are I Merchant banker than the date of exercise of option not
not listed on a registered with SEBI being a date which is more than 180 days
RSE earlier than the date of exercise of
option.

Note: Where the quotes of buy prices and sell prices are available, we shall take only sell
price for computation.

Computation of FMV of securities other than equity shares allotted under ESOP – R. 3 (9):
The FMV of any specified security, not being an equity share in a company, on the date on
which the option is exercised by the employee, shall be such value as determined by a
merchant banker on the specified date.
Note: If the shares or securities which are allotted under ESOP are transferred, capital gains
shall be computed by taking the FMV as on the date of exercise of option as the cost of
acquisition of such shares are securities. [S. 49 (2AA)].

MT Educare –CA Inter Income Tax Classes – R.SOUMYANARAYANAN.FCA. GRAD CWA. Page-100
Chapter-5: Salary

(h) Employer’s contribution to approved superannuation fund – S. 17 (2) (vii):

Employer’s contribution to approved superannuation fund for the benefit of the employee,
to the extent it exceeds Rs. 150000 shall be regarded as perquisites in the hands of the
employee.

Approved superannuation fund – Tax treatment:

1 What is approved Like PF, Superannuation fund is also another scheme of


superannuation fund? retirement benefits for the employee. Approved
superannuation fund means a superannuation fund which
is approved by the CCIT or CIT in accordance with the rules
contained in Part B of the Fourth Schedule of the Income
tax Act.
2 Purpose These are funds established under trusts by the employer
for providing annuities: (a) to the employees of the
undertaking on their retirement at or after specified age, or
on their becoming incapacitated prior to such retirement,
(b) for the widows, children or dependents of the
employees in case of the employee’s earlier death.
3 How does it operate? The contributions made by the employer and employee are
invested by the trust in the form and mode prescribed. The
income earned on these investments is credited to the
fund. The amount accumulated is used for providing
annuities to the employee upon retirement at or after
specified age or on their becoming incapacitated prior to
such retirement.
4 Employee’s contribution Eligible for deduction U/s 80C
5 Employer’s contribution Taxable as perquisites U/s 17 (2) (vii) to the extent it
exceeds Rs. 150000.
6 Interest on accumulated Fully exempt from tax.
balance
7 Payment from ASAF on the Fully exempt from tax. [S. 10 (13)].
death of beneficiary
8 Payment in commutation Fully exempt from tax. [S. 10 (13)].
of annuity upon
retirement or
incapacitation prior to
retirement
9 Transfer of any sum from Fully exempt from tax. [S. 10 (13)].
ASAF to the pension
account of the employee
referred to U/s 80CCD

MT Educare –CA Inter Income Tax Classes – R.SOUMYANARAYANAN.FCA. GRAD CWA. Page-101
Chapter-5: Salary

(i) Other benefits or amenities as may be prescribed – S. 17 (2) (viii):

Perquisites include any other benefit or amenity as may be prescribed. In terms of


provisions contained in S. 17 (2) (viii), the following other fringe benefits or amenities are
provided in R. 3 (7). All these perquisites are taxable both in the hands of specified as well as
non-specified employees.

1 Interest free or concessional loan to employees R. 3 (7) (i)


2 Benefit arising on account of employer meeting the traveling, touring, R. 3 (7) (ii)
accommodation and any other expenses incurred by the employee while availing
any holiday
3 Free or concessional food and beverages R. 3 (7) (iii)
4 Gift or voucher or token R. 3 (7) (iv)
5 Benefits arising on account of employer meeting the credit card expenditure of R. 3 (7) (v)
employee
6 Benefits arising on account of employer meeting the club expenditure of R. 3 (7) (vi)
employee
7 Benefit to the employee resulting from the use by the employee of any R. 3 (7) (vii)
moveable asset owned or hired by the employer
8 Benefit to the employee resulting from the sale by the employer of any R. 3 (7)
moveable asset owned by him at a concessional price (viii)
9 Any other benefit or amenity, service, right, privilege provided by the employer R. 3 (7) (ix)

(1) Value of perquisites in respect of interest-free loan or loan at concessional rate of


interest- R. 3 (7) (i):

If an interest free loan or a loan at concessional rate of interest is given by an employer to


the employee (or any member of his household), it is a perquisite chargeable to tax in the
hands of all categories of employees. The value of perquisite shall be determined as under:
Find out the “maximum outstanding monthly balance” (i.e. the aggregate
Step 1
outstanding balance for each loan as on the last day of each month).
Find out rate of interest charged by the State Bank of India (SBI) as on the first day
Step 2
of the relevant PY in respect of loan for the same purpose advanced by it
Calculate interest for each month of the PY on the outstanding amount mentioned
Step 3
in Step 1 at the rate of interest given step 2
From the total interest calculated for the entire PY under step 3, deduct interest
Step 4
actually recovered, if any, from the employee during the PY.
Step 5 The balance amount (I.e. Step 3 minus Step 4) is taxable value of the perquisite.

When perquisite is not chargeable to tax?


If a loan is made available for medical treatment in respect of diseases
specified in R. 3A (the exemption is, however, not applicable to so much of
Exemption 1
the loan as has been reimbursed to the employee under any medical
insurance scheme).
Where the amount of original loan (or Loans) does not exceed in the
Exemption 2
aggregate Rs. 20000.

MT Educare –CA Inter Income Tax Classes – R.SOUMYANARAYANAN.FCA. GRAD CWA. Page-102
Chapter-5: Salary

Example 1: X takes a loan of Rs. 200000 from his employer on May 1, 2018 for medical
treatment of Mrs. X (medical Treatment is specified in R. 3A). Hospital bill is Rs. 200000.
The perquisite is not chargeable to tax. Suppose in this case insurance claim of Rs. 50000 is
received on October 15, 2018, which is retained by X. In such a situation, interest on Rs.
50000 will be chargeable in hands of X with effect from October 15, 2018.

Example 2: X takes a loan of Rs. 15000 from his employer on May 28, 2018 (no other loan is
taken so far). As the amount of loan does not exceed Rs. 20000, nothing is chargeable to
tax. Suppose in this case another loan of Rs. 5500 is taken from the employer on July 17,
2018. Now the aggregate amount of loan exceeds Rs. 20000. Consequently, interest on Rs.
20500 (i.e., Rs. 15000 + Rs. 5500) will be chargeable to tax with effect from July 17, 2018.

(2) Value of perquisite in respect of travelling, touring, accommodation – R. 3 (7) (ii):

The basis of valuation is as follows:


Perquisite in respect of travelling, touring, accommodation and any
Mode of valuation
other expenses paid by employer for any holiday availed by
as per R. 3 (7) (ii)
employee for any member of household) other than LTC.
Where such facility is available Where such facility is not available
uniformly to all employees uniformly to all employees

Step 1: Find out cost Expenditure incurred by the Value at which such facilities are
to the employer employer offered by other agencies to the
public

Step 2: Less: Amount Recovery from the employee Recovery from the employee
recovered from the
employee

Taxable value of the Balancing amount (if it is Balancing amount (if it is positive)
perquisite (step 1 – positive)
Step 2)

Note:
1 It is taxable in the hands of specified as well as non-specified employees.
2 Where the employee is on official tour and the expenses are incurred in respect of any
member of his household accompanying him, the amount of expenditure so incurred
shall also be a fringe benefit or amenity.
3 Where any official tour is extended as a vacation, the value of such fringe benefit will be
limited to the expenses incurred in relation to such extended period of stay or vacation.

MT Educare –CA Inter Income Tax Classes – R.SOUMYANARAYANAN.FCA. GRAD CWA. Page-103
Chapter-5: Salary

Valuation of perquisite in respect of leave travel concession in India:


LTC is a perquisite taxable in the hands of specified employees. However, exemption is
available subject to S. 10 (5) which is summarized as follows:
1. Applicability The exemption in respect of value of LTC is available if
the journey is to a place within India.
2. Journey by whom? Employee and his family
3. Family Spouse and children, Dependent parents, brothers and
sisters.
Children

Born before Born after


01.10.1998 01.10.1998

Exemption available for Exemption available only


all the surviving children for 2 surviving children

Note: Children born out of multiple births after the first


child will be treated as one child only.
4. Timing of journey Journey may be performed while in service or after
retirement. It is not necessary that the family member
shall perform the journey only along with the employee.
5. Quantum of exemption – R.2B See the chart below
6. No. of journeys exempt 2 journeys in a block of 4 calendar years (The first block
of 4 calendar year= 1986-89 & 9th block is 2018-21).
7. Carry over concession If an assessee has not availed travel concession or
assistance during any of the specified four-year block
periods on one of the two permitted occasions (or on
both occasions), exemption can be claimed in the first
calendar year of the next block (but in respect of only
one journey). This is in addition to 2 journeys in the
subsequent block.
8. Exemption – based on actual The quantum of exemption is limited to actual
expenditure expenditure incurred on the journey.
9. Expenses qualifying for The exemption is strictly restricted only to expenses on
exemption (a) Air fare (b) Rail fare (c) Bus fare. Porterage expenses,
lodging & boarding expenses – not qualified for
exemption.
10. Exemption - available in Where journey is performed by a circuitous route, the
respect of shortest route exemption is limited to what is admissible by the
shortest route.

MT Educare –CA Inter Income Tax Classes – R.SOUMYANARAYANAN.FCA. GRAD CWA. Page-104
Chapter-5: Salary

Quantum of
Exemption

Journey Journey performed by


performed by Air any other mode of
transport

Amount of
Place of origin & Place of origin &
Economy class
destination destination not
air fare of
connected by connected by
National carrier
Rail Rail
by the shortest
route or the
amount spent Where Public
Travel by Rail Travel by any other
whichever is less transport not
mode of transport
exists
Where Public
transport exists Air-conditioned
Amount of Air- first class rail
conditioned first class fare by the
First class or
rail fare by the shortest shortest route
route or amount spent Deluxe class fare
by the shortest (as if the
whichever is less journey had
route or amount
spent whichever been performed
is less by rail) or
amount spent
whichever is less

(3) Value of perquisite in respect of lunch/refreshments, etc-R. 3 (7) (iii):

The value of free food and non-Alcoholic beverages provided by the employer to an
employee is taxable as perquisite in the hands of employee.

The taxable value of perquisite is the amount of expenditure incurred by the employer. If
any amount is recovered from the employee, in computing the taxable value of perquisite, it
shall be deducted. The aforesaid perquisite is taxable both in the hands of specified as well
as non-specified employees. However, in the following cases, nothing is taxable in the hands
of employee:

1 Free food and non-alcoholic beverages are provided by the employer during working
hours in a remote area or an off shore installation.
2 Tea or snacks provided to the employees during working hours.
3 The value of food and non-alcoholic beverages provided by the employer during working
hours in office or other business premises does not exceed `50 per meal.
4 The value of food and non-alcoholic beverages provided by the employer through paid
vouchers which are not transferable and usable only at eating joints does not exceed Rs.
50 per meal.

Note: CBDT has clarified that in case 3 and 4, if the cost of the meal exceeds Rs. 50, the
excess shall only be taxed as perquisite.

MT Educare –CA Inter Income Tax Classes – R.SOUMYANARAYANAN.FCA. GRAD CWA. Page-105
Chapter-5: Salary

Lunch/Refreshment

Provided in working hours


Provided in working hours
in remote area or in an off
at any other place
shore installation

Not Taxable as Perks Food/Non-acholic beverage in office


Tea/Snacks in premises or through non-transferable
working hours paid vouchers usable only at eating
hours

Note:
➢ Working hours: Include extended Not Taxable as perks Taxable Perks =
office hours like working on holidays,
Cost to the employer
overtime, etc)
in excess of Rs. 50 per meal XXX
➢ taxable in the hands of specified
Less:
employees as well as non-specified
Amount recovered
employees
from employee XXX

(4) Value of perquisite in respect of gift, Voucher or token-R. 3 (7) (iv):

1 Employee covered Specified as well as non-specified employees.


2 Perquisite covered Gift, voucher or token (in lieu of which gift may be received by
the employee).
3 Taxable value of Aggregate value of gift received during the PY – Rs. 5000.
perquisite
4 Monetary gift Not exempted – entire amount is taxed.

Valuation of perquisite in respect of gift, voucher or taken

Gift

Cash Kind
Employer Employee (specified/
unspecified)
Taxability Member of employees
household
Does
Aggregate
value of Gifts
Nothing is taxable
during the previous
year < 5,000 Yes
?

No

Taxable perks =
Aggregate value of
Gift (-) 5,000
(Circular 15/2001)

MT Educare –CA Inter Income Tax Classes – R.SOUMYANARAYANAN.FCA. GRAD CWA. Page-106
Chapter-5: Salary

(5) Value of perquisite in respect of Credit cards provided by the employer- R. 3 (7) (v):

1 Employee Specified as well as non-specified employees.


covered
2 Perquisite Perquisite in respect of credit card provided by the employer.
covered
3 Taxable value of Expenditure incurred by the employer in respect of credit card
perquisite (including add on card) used by the employee or any member of his
house hold (including membership fees and annual fees) Less
Expenditure for official purposes Less Amount recovered from the
employee.
4 Details to be Employer shall maintain complete details in respect of expenditure
maintained by incurred for official purposes – especially, the date of expenditure
the employer and the nature of expenditure.
5 Certificate from To deduct the expenses attributable for official purpose, the
the employer employee should obtain a certificate from the employer to the
effect that the same was incurred wholly and exclusively for the
performance of official duties.
(6) Value of perquisite in respect of club expenditure borne by the employee-R. 3 (7) (vi):

1 Employee covered Specified as well as non-specified employees.


2 Perquisite covered Perquisite in respect of club facility.
3 Taxable value of Expenditure incurred by the employer in respect of club
perquisite facility used by the employee or any member of his house
hold (including annual or periodical fees) Less Expenditure for
official purposes Less Amount recovered from the employee.
4 Details to be Employer shall maintain complete details in respect of
maintained by the expenditure incurred for official purposes – especially, the
employer date of expenditure and the nature of expenditure.
5 Certificate from the To deduct expenses attributable for official purpose, the
employer employee should obtain a certificate from the employer to
the effect that the same was incurred wholly and exclusively
for the performance of official duties.
6 Health club, sports Health club, sports facilities, etc provided uniformly to all
facilities – uniformly classes of employees by the employer at employer’s premises
available for all are exempt.
employees
7 Initial one time deposit Exempt – since the benefit does not remain with a particular
for corporate or employee after cessation of employment.
institutional
membership

MT Educare –CA Inter Income Tax Classes – R.SOUMYANARAYANAN.FCA. GRAD CWA. Page-107
Chapter-5: Salary

(7) Value of perquisite in respect use of movable asset-R. 3 (7) (vii):

Use of Movable
asset

Being computer/ Being any other


Laptops asset

Taxable perquisite Owned by Taken on hire


= NIL employer by employer

Taxable perquisite (for


Taxable perquisite (for
employees) =
employees) =
Amount of rent paid or
10% pa of actual cost -
payable - recovery from
recovery from employee
employee

Note: Taxable in the hands of both specified as well as non-specified employees.

(8) Value of perquisite in respect of movable assets sold by an employer to its employees
at a nominal price-R. 3 (7) (viii):

Sale of movable assets


to employees

Being Electronic items/


Being motor cars Being any other asset
computers

1) Actual cost to 1) Actual cost to 1) Actual cost to


the employer **** the employer **** the employer ****
2) 50% for each 2) 20% for each 2) 10% for each
completed year by completed year by completed year by
reducing balance reducing balance straight line
method **** method **** method ****
3) Consideration 3) Consideration 3) Consideration
recovered from the recovered from the recovered from the
employee **** employee **** employee ****
4) Taxable value of 4) Taxable value of 4) Taxable value of
perquisite (1-2-3) **** perquisite (1-2-3) **** perquisite (1-2-3) ****

Note:

1 Taxable for all employees.


2 Electronic items = Computer + Digital Diaries + Printers + Other data storage and
handling devices.
3 Electronic items ≠ Washing machines + Micro wave ovens + Mixers + Hot plates + other
household appliances.

(9) Value of any other benefit or amenity – R. 3 (7) (ix):


The value of any other benefit or amenity, service, right or privilege provided by the
employer shall be determined on the basis of cost to the employer under an arm’s length
transaction as reduced by the employee’s contribution, if any. Provided that nothing
contained in this clause shall apply to the expenses on telephones including a mobile phone
actually incurred on behalf of the employee by the employer.

MT Educare –CA Inter Income Tax Classes – R.SOUMYANARAYANAN.FCA. GRAD CWA. Page-108
Chapter-5: Salary

10. Professional tax u/s 16 (iii)

Professional tax is levied by the SG (under Article = 276 of the Constitution). This is allowed
as deduction u/s 16(iii) on “payment basis”

Professional tax

Paid by employee Paid by employer

Deductible Obligation of employee -


u/s 16(iii) discharged by employer

Step-1: Include this is a


perquisite in case of all
employees
Step-2 Allow deduction u/s
16(iii)

11. Relief U/s 89 (1):

Who is entitled to get relief U/S 89(1)?

(i) A salaried assessee who in a financial year receives salary for more than 12 months
(either due to receipt of arrears of salary or due to receipt of advance salary) or
(ii) A salaried assessee who is in receipt of profit in lieu of salary.

Section 89 of the Income-tax Act contains provisions for providing tax relief where
salary, etc. is paid in arrears or in advance. The existing provisions of section 140A,
section 143, section 234A, section 234B and section 234C contain provisions relating to
computation of tax liability after allowing credit for prepaid taxes and certain admissible
reliefs, credits etc. However, the relief under section 89 is not specifically mentioned in
these sections, which is resulting into genuine hardship in the case of taxpayers who are
eligible for this relief.

Amendment by Finance (No. 2) Act 2019: With a view to avoiding genuine hardship in
the case of a person who is eligible for relief under section 89, the provisions of
Sections 140A, 143, 234A, 234B & 234C have been amended (with retrospective
effect from the assessment year 2007-08) to provide that computation of tax liability
shall be made under these sections after allowing relief under section 89.

MT Educare –CA Inter Income Tax Classes – R.SOUMYANARAYANAN.FCA. GRAD CWA. Page-109
Chapter-5: Salary

Procedure for computation of relief in case of receipt of arrears of salary/advance salary:


Table-1: Relief computation:
Step Particulars Amount
1 Total income for the year (excluding arrears of salary) *****
2 Arrears of salary received relating to prior previous years *****
3 Total income for the year (including arrears of salary) *****
4 Tax on total income (item no 3) after EC & SHEC *****
5 Tax on total income (item no 1) after SC, EC & SHEC *****
6 Tax on arrears of salary (had it been taxed in the PY of receipt) [4-5] *****
7 Tax on arrears of salary (had it been taxed in the respective years) [Refer *****
column 7 of table-2 below]
8 Relief U/s 89 (1) [6-7] *****
9 Tax liability after relief [4-8]
Table-2: Computation of tax on arrears (had it been taxed in the respective years):

PY Total Arrears of Total income Tax on Tax on Difference


income earlier including income in income in between
years arrears column (4) column (2) column (5) &
after SC & EC after SC & EC (6)
1 2 3 4 5 6 7

Note: The above table is applicable for receipt of salary in advance also.

MT Educare –CA Inter Income Tax Classes – R.SOUMYANARAYANAN.FCA. GRAD CWA. Page-110
Chapter-6 : Capital gains

1. Charging section – S. 45 (1):


1 Any profits or gains arising from the transfer of a capital asset effected during the PY shall be
chargeable to income-tax U/H CG and shall be deemed to be the income of the PY in which the
transfer took place.
2 However, the profits or gains (supra) shall not be charged to tax to the extent to which it is
exempt U/s 54, 54B, 54D, 54EC, 54EE, 54F, 54G, 54GA, 54GB, 10 (37), 10 (37A) and S. 115F.

Points requiring attention:


1 Meaning of CA means property of any kind held by an assessee, whether or not connected
capital asset with his business or profession. [S. 2 (14)].
2 Exclusions from (a) Stock-in-trade (SIT); (b) Personal effects; (c) Rural agricultural land; (d)
‘capital asset’ Gold bonds issued under the Gold Deposit Scheme 1999; (e) Deposit
certificates issued under the Gold Monetisation Scheme, 2015 notified by the
CG (interest thereon is exempt U/s 10 (15)).
3 Inclusion in Any securities held by a Foreign institutional investor which has invested in
‘Capital asset’ such securities in accordance with the SEBI Regulations shall be regarded as
CA (even if such securities are held by it as SIT).
4 Assets ≠ (a) Jewellery; (b) Archaeological collections; (c) Drawings; (d) Paintings; (e)
personal effect Sculptures; (f) Any work of art.
5 CBDT Circular Where the assessee himself, irrespective of the POH of the listed shares and
6/2016 securities, opts to treat them as SIT, the income arising from transfer of such
shares or securities would be treated as its business income.
In respect of listed shares and securities held for a period of more than 12
months immediately preceding the DOT, if the assessee desires to treat the
income arising from transfer thereof as CG, the same shall not be put to
dispute by the AO. However, this stand, once taken by the assessee in a
particular AY, shall remain applicable in subsequent AYs also and the
taxpayers shall not be allowed to adopt a different stand in this regard in
subsequent years;
6 Letter F. No. Income arising from transfer of unlisted shares (for which no formal market
225/12/2016 exists for trading) would be considered U/H CG, irrespective of POH, with a
view to avoid disputes or litigation and to maintain uniform approach.
7 Meaning of (a) Sale; (b) Exchange; (c) Relinquishment; (d) Extinguishment of rights; (e)
transfer [S. 2 Conversion of CA into SIT; (f) Compulsory acquisition; (g) power of attorney
(47)]. transactions; (h) Transaction having the effect of transferring or enabling the
enjoyment of any immovable property; (i) Maturity or redemption of ZCB.
8 Meaning of ZCB ZCB are bonds issued by any infrastructure capital company or infrastructure
capital fund or a public sector company or a scheduled bank in respect of
which no payment and benefit are received before maturity or redemption
from such issuing entity and are notified by the Central Government in the
official gazette. [S. 2 (48)].
9 K. R. Srinath Sum received for relinquishing the right of specific performance in a contract
(Mad) shall be charged to tax U/H CG.
10 Karthikeya. V. Reduction of share capital results in extinguishment of rights of shareholders
Sarabhai (SC) in the shares held by them.
11 Anarkali Redemption of preference shares comes within the expression ‘Sale,
Sarabhai (SC) exchange or relinquishment of CA’.

MT Educare –CA Inter Income Tax Classes – R.SOUMYANARAYANAN.FCA. GRAD CWA. Page-111
Chapter-6 : Capital gains

12 When transfer Transfer of movable property becomes effective and complete upon delivery
becomes pursuant to contract of sale.
effective and Transfer of immovable property becomes effective after registration of
complete? conveyance deed but with effect from the date of execution of the
conveyance deed.
In case of power of attorney transactions, transfer becomes effective once
possession is handed over upon receipt of consideration.

2. Mode of Computation of capital gains – S. 48:


The manner of computation of capital gains is provided in S. 48. The format for computation
of capital gains is as follows:
1 Full value of consideration XXX
2 Expenditure incurred wholly and exclusively in connection with transfer (XXX)
3 Net consideration (1-2) XXX
4 Cost of acquisition (XXX)
5 Cost of improvement (XXX)
6 Capital gain (3-4-5) XXX

3. Full value of consideration:


1 It means what the transferor receives or is entitled to receive as consideration for the asset
transferred.
2 FMV of the CA transferred has no relevance in computation of capital gains. Only actual
consideration received or receivable is relevant for computing capital gains. However, exceptions
are contemplated in S. 50C, S. 50CA and S. 50D.

FVC in case of real estate transaction: (S. 50C):


SN Situations FVC
1 SDV ≤ 105% of Actual sale consideration (ASC) ASC
2 SDV > 105% of ASC SDV
3 SDV > 105% of ASC - the assessee challenged the SDV in appeal under ASC (See note below)
the Indian Stamp Act – SDV is revised – Revised SDV ≤ 105% of ASC
4 SDV > 105% of ASC - the assessee challenged the SDV in appeal under Revised SDV (See note
the Indian Stamp Act – SDV is revised – Revised SDV > 105% of ASC below)
5 SDV > 105% - the assessee does not challenge the SDV before the FMV determined by
court or any other authority – but he pleads before the AO that SDV DVO (See note below)
> FMV on the DOT - AO refers the matter to the DVO) - FMV
determined by DVO < SDV – FMV > 105% of ASC
6 SDV > 105% - the assessee does not challenge the SDV before the ASC
court or any other authority - but he pleads before the AO that SDV
> FMV on the DOT - AO refers the matter to the DVO - FMV
determined by DVO < SDV – FMV ≤ 105% of ASC
7 SDV > 105% of ASC - the assessee does not challenge the SDV before SDV
the court or any other authority - but he pleads before the AO that
SDV>FMV on the DOT - AO refers the matter to the DVO - FMV
determined by VO > SDV

MT Educare –CA Inter Income Tax Classes – R.SOUMYANARAYANAN.FCA. GRAD CWA. Page-112
Chapter-6 : Capital gains

Note:
1 The AO shall amend the order of assessment passed in respect of AY relevant to the PY in which
transfer of immovable property took place to re-compute the capital gains by taking the ASC or
SDV as revised by the appellate authority, as the case may be.
2 The amendment order shall be passed within 4 years from the end of the PY in which the appellate
authority has passed order revising the SDV. [S. 155 (15)].
3 The AO is bound to consider the report of the VO when it is on record. The CG shall be computed
in conformity with the value determined by the DVO. [Ravjibhai Nagjibhai Thesia (2016) (Guj)].
4 Where the amount of consideration, or a part thereof, has been paid by way of an APC or APBD or
use of electronic clearing system through a bank account or other electronic mode as may be
prescribed, [FA 2019 Amendment] on or before the date of the agreement for the transfer of such
immovable property, then the SDV on the date of agreement fixing the consideration is to be
taken into account.
5 Otherwise, the SDV on the date of registration shall be taken into account.
6 S. 43A plays exactly the same role U/H PGBP which S. 50C plays U/H CG.

Illustration-1:
SN Particulars Situation-1 Situation-2
1 Consideration Rs. 200L Rs. 200L
2 SDV Rs. 220L Rs. 208L
3 105% of the consideration Rs. 210L Rs. 210L
4 Observation 2>3 2<3
5 S. 50C applicable? Yes No
6 FVC Rs. 220L (SDV) Rs. 200L (Consideration)

Illustration-2:
SN Particulars Case-1 Case-2 Case-3 Case-4
1 Consideration Rs. 200L Rs. 200L Rs. 200 Rs. 200L
2 SDV Rs. 220L Rs. 220L Rs. 220L Rs. 220L
3 FMV determined by Rs. 220L Rs. 208L Rs. 190L Rs. 230L
the DVO upon receipt
of reference from the
AO U/s 55A
4 105% of consideration Rs. 210L Rs. 210L Rs. 210L Rs. 210L
5 What shall be FMV FMV FMV SDV
compared with 105%
of Consideration?
6 Observation FMV > FMV > FMV < SDV >
Consideration Consideration Consideration Consideration
FMV > 105% of FMV < 105% of SDV > 105% of
Consideration Consideration Consideration
7 S. 50C applicable? Yes No No Yes
8 FVC Rs. 220L (FMV) Rs. 200L Rs. 200L Rs. 220L (SDV)
(Consideration) (Consideration)

MT Educare –CA Inter Income Tax Classes – R.SOUMYANARAYANAN.FCA. GRAD CWA. Page-113
Chapter-6 : Capital gains

Illustration-3:
SN Particulars Case-1 Case-2 Case-3
1 PY of transfer 19-20 19-20 19-20
2 Relevant AY 20-21 20-21 20-21
3 Consideration Rs. 200L Rs. 200L Rs. 200L
4 SDV Rs. 220L Rs. 220L Rs. 200L
5 105% of Consideration Rs. 210L Rs. 210L Rs. 210L
6 Observation SDV > 105% of SDV > 105% of SDV > 105% of
Consideration Consideration Consideration
7 FVC taken by the AO for Rs. 220L (SDV) Rs. 220L (SDV) Rs. 220L (SDV)
computing CG (as per S. 50C)
8 Date of completion of 01.03.2022 01.03.2022 01.03.2022
assessment U/s 143 (3) for the
AY 20-21
9 Revised SDV determined in Rs. 215L Rs. 208L Rs. 190L
appeal made by the assessee
under the Indian Stamp Act
10 Date of order revising the SDV 15.06.2023 15.06.2023 15.06.2023
11 Does the revised SDV > 105% Yes No No
of consideration
12 Whether S. 50C is applicable? Yes No No
13 FVC to be taken by the AO for Rs. 215L Rs. 200L Rs. 200L
re-computation of CG in the (Revised SDV) (Consideration) (Consideration)
amendment order
14 Time limit for passing the 31.03.2028 31.03.2028 31.03.2028
amendment order in respect
of AY 2020-21 for re-
computing CG.

Illustration-4:
SN Particulars Case-1 Case-2 Case-3
1 Date of agreement 18.04.2019 18.04.2019 18.04.2019
fixing consideration
2 Consideration Rs. 200L Rs. 200L Rs. 200L
3 Date of registration 15.12.2019 15.12.2019 15.12.2019
4 Consideration
received on
(a) 18.04.2019 Rs. 1L (APC) Rs. 1L (APC) Rs. 1L (Cash)
(Advance)
(b) 14.12.2019 Rs. 199L (RTGS) Rs. 199L (RTGS) Rs. 199L (RTGS)
(Balance payment)
5 SDV on the date of Rs. 220L Rs. 208L Rs. 220L
agreement
6 SDV on the date of Rs. 230L Rs. 230L Rs. 230L
registration

MT Educare –CA Inter Income Tax Classes – R.SOUMYANARAYANAN.FCA. GRAD CWA. Page-114
Chapter-6 : Capital gains

7 Relevant SDV to be Rs. 220L (SDV on Rs. 208L (SDV on the Rs. 230L (SDV on
considered the date of date of agreement) the date of
agreement) registration)
8 105% of Rs. 210L Rs. 210L Rs. 210L
Consideration
9 Observation SDV > 105% of SDV < 105% of SDV > 105% of
Consideration Consideration Consideration
10 Whether S. 50C is Yes No Yes
applicable?
11 FVC Rs. 220L (SDV on Rs. 200L Rs. 230L (SDV on
the date of (Consideration) the date of
agreement) registration)

SN Particulars Case-4 Case-5 Case-6


1 Date of agreement 18.04.2019 18.04.2019 18.04.2019
fixing consideration
2 Consideration Rs. 200L Rs. 200L Rs. 200L
agreed
3 Date of registration 15.12.2019 15.12.2019 15.12.2019
4 Consideration
received on
(a) 18.04.2019 Rs. 1L (Cash) - -
(Advance)
(b) 25.04.2019 Rs. 1L (RTGS) Rs. 1L (RTGS)
(c) 14.12.2019 Rs. 199L (RTGS) Rs. 199L (RTGS) Rs. 199L (RTGS)
(Balance payment)
5 SDV on the date of Rs. 208L Rs. 220L Rs. 208L
agreement
6 SDV on the date of Rs. 230L Rs. 230L Rs. 230L
registration
7 Relevant SDV to be Rs. 230L (SDV on Rs. 230L (SDV on Rs. 230L (SDV on
considered the date of the date of the date of
registration) registration) registration)
8 105% of Rs. 210L Rs. 210L Rs. 210L
Consideration
9 Observation SDV > 105% of SDV > 105% of SDV > 105% of
Consideration Consideration Consideration
10 Whether S. 50C is Yes Yes Yes
applicable?
11 FVC Rs. 230L (SDV on Rs. 230L (SDV on Rs. 230L (SDV on
the date of the date of the date of
registration) registration) registration)

MT Educare –CA Inter Income Tax Classes – R.SOUMYANARAYANAN.FCA. GRAD CWA. Page-115
Chapter-6 : Capital gains

TDS on payments on transfer of certain immovable property -S. 194-IA:


1 Payment to be Consideration for transfer (not being compulsory acquisition) of
subjected to TDS any immovable property (other than agricultural land situated in
rural area).
2 Consideration for It includes all charges of nature of club membership fee, car parking
transfer of fee, electricity and water facility fees, maintenance fee, advance fee
immovable property or any other charges of similar nature, which are incidental to
transfer of the immovable property. [FA 2019 amendment effective
from 01.09.2019].
Payee Resident (Transferor).
3 May be dealer or investor
4 Payer May be a dealer or investor.
5 Threshold limit If the consideration is less than Rs. 50L, there is no TDS obligation.
6 TDS rate 1% [Subject to S. 206AA]
7 Timing of TDS At the time of credit to the account of payee in the books of the
payer or at the time of payment (whichever is earlier).
8 Time limit for 30th of the month succeeding the month of deduction of tax at
remittance of TDS source.
9 No TAN required Payer need not have TAN for deducting tax under this section.

Points requiring attention:


1 S. 194-IA does not apply if the payee is a non-resident. In such case, tax shall be
deducted at source U/s 195 using rates in force (20% + Surcharge if applicable + EC).
2 Persons jointly purchasing immovable property from a resident shall deduct tax at
source U/s 194-IA, if the consideration payable by them put together is Rs. 50L or more,
though individually is less than Rs. 50L.
3 Rs. 50L is to be compared with the actual consideration and not the SDV.
4 THL of Rs. 50L is to be applied in respect of each and every property purchased
separately.
5 Where the immovable property is purchased along with some movable assets, then the
consideration attributable to the immovable property alone is to be compared with the
threshold limit of Rs. 50L (for deciding the applicability of S. 194-IA).

Previous exam questions:


Question No 1: Mr. Hari, a property dealer, sold a building in the course of his business to
his friend Mr. Rajesh, who is a dealer in automobile spare parts, for R s . 90L on 1.1.2020,
when the stamp duty value was R s . 150L. The agreement was, however, entered into on
1.9.2019 when the SDV was R s . 140L. Mr. Hari had received a down payment of R s . 15L
by a crossed cheque from Mr. Rajesh on the date of agreement. Discuss the tax
implications in the hands of Mr. Hari and Mr. Rajesh, assuming that Mr. Hari has purchased
th
the building for R s . 75L on 12 July, 2018. Would your answer be different if Hari was a
share broker instead of a property dealer?

MT Educare –CA Inter Income Tax Classes – R.SOUMYANARAYANAN.FCA. GRAD CWA. Page-116
Chapter-6 : Capital gains

Answer:
Case 1: Tax implications if Mr. Hari is a property dealer:

In the hands of Mr. Hari In the hands of Mr. Rajesh


In the hands of Hari, the provisions of S. 43CA would Since Mr. Rajesh is a dealer in automobile
be attracted, since the building represents his stock- spare parts, the building purchased would be a
in-trade and he has transferred the same for a capital asset in his hands. The provisions of S. 56
consideration less than the SDV and the SDV exceeds (2) (x) would be attracted in the hands of Mr.
105% of consideration. U/s 43CA, the option to adopt Rajesh who has received immovable property,
the SDV on the date of agreement can be being a capital asset, for inadequate
exercised only if whole or part of the consideration consideration and the difference between the
has been received on or before the date of consideration and S D V exceeds R s . 4,50,000,
agreement by way of A P C or draft or by use of being the higher of R s . 50,000 and 5% of
ECS through a bank account on or before the date of consideration.
agreement. In this case, since the payment is made
by crossed cheque, the option cannot be exercised. Therefore, R s . 60L, being the difference
between the SDV of the property on the date of
Therefore, R s . 75L, being the difference between registration (i.e., R s . 150L) and the actual
the S D V on the date of registration (i.e., R s . consideration (i.e., R s . 90L) would be taxable
150L) and the purchase price (i.e., R s . 75L), would U / s 56 (2) (x) in the hands of Mr. Rajesh, since
be chargeable as business income in the hands of the payment is made by crossed cheque and not
Mr. Hari, since S D V exceeds 105% of the APC /draft or ECS.
consideration.
In the hands of Hari, the provisions of S. 43CA would Since Mr. Rajesh is a dealer in automobile
be attracted, since the building represents his stock- spare parts, the building purchased would be a
in-trade and he has transferred the same for a capital asset in his hands. The provisions of S. 56
consideration less than the SDV and the SDV (2) (x) would be attracted in the hands of Mr.
exceeds 105% of consideration. U/s 43CA, the Rajesh who has received immovable property,
option to adopt the SDV on the date of being a capital asset, for inadequate
agreement can be exercised only if whole or part consideration and the difference between the
of the consideration has been received on or consideration and S D V exceeds R s . 4,50,000,
before the date of agreement by way of A P C being the higher of R s . 50,000 and 5% of
or draft or by use of ECS through a bank account on consideration.
or before the date of agreement. In this case, since
the payment is made by crossed cheque, the option Therefore, R s . 60L, being the difference
cannot be exercised. between the SDV of the property on the date of
registration (i.e., R s . 150L) and the actual
Therefore, R s . 75L, being the difference between the consideration (i.e., R s . 90L) would be taxable
S D V on the date of registration (i.e., R s . 150L) U / s 56 (2) (x) in the hands of Mr. Rajesh, since
and the purchase price (i.e., R s . 75L), would be the payment is made by crossed cheque and not
chargeable as business income in the hands of Mr. APC /draft or ECS.
Hari, since S D V exceeds 105% of the consideration.

MT Educare –CA Inter Income Tax Classes – R.SOUMYANARAYANAN.FCA. GRAD CWA. Page-117
Chapter-6 : Capital gains

Case 2: Tax implications if Mr. Hari is a stock broker:

In the hands of Mr. Hari In the hands of Mr. Rajesh


In case Mr. Hari is a stock broker and not There would be no difference in the
a property dealer, the building would taxability in the hands of Mr. Rajesh,
represent his capital asset and not whether Mr. Hari is a property dealer or a
stock-in-trade. In such a case, the stock broker.
provisions of S . 50C would be attracted
in the hands of Mr. Hari since building Therefore, the provisions of S . 56 (2) (x)
is transferred for a would be attracted in the hands of Mr.
consideration less than the S D V and the Rajesh who has received immovable
S D V exceeds 105% of consideration. property, being a capital asset, for
Thus R s . 75L, being the difference inadequate consideration and the
between the S D V on the date of difference between the consideration and
registration (i.e., R s . 150L) and the S D V exceeds R . 4,50,000, being the
purchase price (i.e., R s . 75L) would be higher of R s . 50,000 and 5% of
chargeable as STCG. consideration.

It may be noted that U / s 50C, the option Therefore, R s . 60L, being the difference
to adopt the S D V on the date of between the SD V of the property on the
agreement can be exercised only if whole date of registration (i.e. Rs. 150L) and the
or part of the consideration has been actual consideration (i.e., R s . 90L) would
received on or before the date of be taxable U/s 56 (2) (x) in the hands of
agreement by way of APC or draft or by use Mr. Rajesh since the payment on the date
of ECS through a bank account on or of agreement is made by crossed cheque
before the date of agreement. In this and not APC /draft or ECS.
case, since the payment is made by
crossed cheque, the option cannot be
exercised.

Question No 2: Mr. X sold his house property in Bangalore as well as his rural agricultural land
for a consideration of Rs. 60L and Rs. 15L, respectively, to Mr. Y on 01.08.2019. He has
purchased the house property and the land in the year 2018 for Rs. 40L and Rs. 10L,
respectively. The SDV on the date of transfer, i.e., 01.08.2019, is Rs. 85L and Rs. 20L for the
house property and rural agricultural land, respectively. Determine the tax implications in the
hands of Mr. X and Mr. Y and the TDS implications, if any, in the hands of Mr. Y, assuming that
both Mr. X and Mr. Y are resident Indians.

Answer:
(i) Tax implications in the hands of Mr. X
As per S. 50C, the SDV of house property (i.e. Rs. 85L) would be deemed to be the FVC
arising on transfer of property. Therefore, Rs. 45L (i.e. Rs. 85L – Rs. 40L, being the purchase
price) would be taxable as STCG in the AY 2019-20.
Since rural agricultural land is not a capital asset, the gains arising on sale of such land is
not taxable in the hands of Mr. X.

MT Educare –CA Inter Income Tax Classes – R.SOUMYANARAYANAN.FCA. GRAD CWA. Page-118
Chapter-6 : Capital gains

(ii) Tax implications in the hands of Mr. Y


In case immovable property is received for inadequate consideration, the difference
between the SDV and actual consideration would be taxable U/s 56 (2) (x) (b) (B), if such
difference exceeds higher of (a) Rs. 50,000; and (b) 5% of Consideration. Therefore, in this
case Rs. 25L (Rs. 85L – Rs. 60L) would be taxable in the hands of Mr. Y U/s 56 (2) (x) (b) (B).

Since agricultural land is not a capital asset, the provisions of 56 (2) (x) (b) (B) are not
attracted in respect of receipt of agricultural land for inadequate consideration, since the
definition of “property” U/s 56 (2) (x) includes only capital assets specified thereunder.
(iii) TDS implications in the hands of Mr. Y
Since the sale consideration of house property exceeds Rs. 50L, Mr. Y is required to deduct
tax at source U/s 194-IA. The tax to be deducted U/s 194-IA would be Rs. 60,000, being 1%
of Rs. 60L.
TDS U/s 194-IA is not attracted in respect of transfer of rural agricultural land.

FVC in case of transfer of unquoted shares – S. 50CA:


(i) Applicability Capital asset being share in a company (not being quoted
share) is transferred for a consideration less than the FMV of
such share determined in accordance with R. 11UA.
(ii) Impact of S. 50CA FMV of such share shall be regarded as FVC. Actual
consideration is to be ignored.
(iii) Meaning of quoted Quoted share means share quoted on a RSE with regularity
share from time to time.
(iv) Determination of For the purposes of S. 50CA, the FMV of the share of a
FMV (R. 11UAA) company other than a quoted share, shall be determined in
the manner provided in R. 11UA (1) (c) as on the date of
transfer.

FVC in case of transaction where the consideration is not determinable – S. 50D:


Where the consideration involved in transfer of capital asset is not ascertainable, then for the
purpose of computing income chargeable to tax as capital gains, the FMV of the said asset on
the date of transfer shall be deemed to be the FVC.

4. Expenses on transfer:
(a) In computing capital gains, the expenditure incurred wholly and exclusively in connection
with transfer is allowed as deduction from the FVC.
(b) Examples: (i) Brokerage/commission paid for securing a buyer; (ii) Travelling expenses
incurred in connection with transfer; (iii) Advertisement expenses.
(c) However, STT paid in relation to transfer of equity shares/equity oriented units/units of BT
shall not be eligible for deduction while computing CG U/s 48. [Proviso-7 to S. 48].

MT Educare –CA Inter Income Tax Classes – R.SOUMYANARAYANAN.FCA. GRAD CWA. Page-119
Chapter-6 : Capital gains

Levy of STT:
SN Taxable securities transaction STT Payable
Rate by
1 Purchase of equity share in a company or unit of a business trust through RSE 0.1% Purchaser
2 Sale of equity share in a company or unit of a business trust through RSE 0.1% Seller
3 Sale of unit of an equity oriented unit through RSE 0.001% Seller
4 Sale of equity oriented unit to the Mutual fund 0.001% Seller
5 Sale of unlisted equity shares by any holder of such shares under an offer for 0.20% Seller
sale to the public included in an initial public offer and where such shares are
subsequently listed on a RSE.
6 Sale of unlisted units of a business trust by any holder of such units (which 0.20% Seller
were acquired in exchange for shares of SPV) under an offer for sale to the
public included in an initial offer and where such units are subsequently listed
on a RSE.

Note:
The provisions of STT shall not apply to taxable securities transactions entered into by:
1 Any person for, or on behalf of, the NPST; or
2 Any person on a RSE located in an IFSC where the consideration for such transaction is
paid or payable in foreign currency.

5. COA of CA:
1 Meaning of COA is the price which the assessee has paid, or the amount which the assessee
COA has incurred, for acquisition of the asset.
2 Incidental All expenses incidental to acquisition of the CA shall be added to arrive at the
expenses COA. Example: (a) Advertisement expenses; (b) Travelling expenses; (c)
Brokerage; (d) Stamp duty; (e) Registration charges; (f) legal fee.
3 STT However, STT shall not be added to arrive at the COA. [Proviso-7 to S. 48].
4 Option U/s 55 Where the CA transferred was acquired by the assessee before 01.04.2001, the
(2) (b) assessee has option to take the FMV as on 01.04.2001 as the COA. However, such
option is not available in respect of intangible CA.
5 Tax treatment Before the CA was transferred, where the assessee has made attempts to sell the
of forfeited CA on earlier occasions which failed on account of buyer not honouring his
advance commitment and as a result the advances received from such defaulting buyer
were forfeited on or after 01.04.2014, then the forfeited advances are regarded
as income in view of S. 2 (24) (xvii) and are to be taxed U/H IFOS in the PY of
forfeiture (in view of S. 56 (2) (ix).
However, if forfeiture has taken place before 01.04.2014, S. 2(24) (xvii) and S. 56
(2) (ix) do not apply. Forfeited advance is a capital receipt and hence, it can’t be
taxed in the PY of forfeiture. But in the PY of transfer of such CA, it shall be
adjusted against the COA of CA and the adjusted COA is only considered in
computing CG U/s 48. [S. 51].
Where forfeited advance is taxed U/s IFOS as per S. 2 (24) (xvii) and S. 56 (2) (ix),
then it shall not be adjusted against COA of CA. [Proviso to S. 51].

MT Educare –CA Inter Income Tax Classes – R.SOUMYANARAYANAN.FCA. GRAD CWA. Page-120
Chapter-6 : Capital gains

6 Provisions of Where the assessee has obtained the asset U/s 49 (1) mode (Say distribution by
S. 49 (1) HUF, will, inheritance, gift etc), the COA is COA to the previous owner who had
not obtained it U/s 49 (1) mode.
Where the cost for which the previous owner acquired the property cannot be
ascertained, the COA to the previous owner means the FMV of the asset on the
date on which the asset became the property of the previous owner. (S. 55(3)).
If the previous owner has acquired the CA before 01.04.2001, then the COA in the
hands of assessee is higher of the following: (a) COA in the hands of previous
owner; (b) FMV as on 01.04.2001. [S. 55 (2) (b)].
Advance received and forfeited by the previous owner shall not be adjusted
against the COA. S. 51 requires only the adjustment of advance received and
forfeited by the assessee and not that received and forfeited by the previous
owner.
7 COA of CA SDV on the DOG (in case the gifted asset = immovable property). [Where the
obtained by assessee had suffered tax U/s 56 (2) (x)]. [S. 49 (4)].
way of gift FMV on the DOG (in case the gifted asset = specified movable property referred
to in S. 56 (2) (x)). [Where the assessee had suffered tax U/s 56 (2) (x)]. [S. 49 (4)].
COA to the previous owner. [S. 49 (1)]. [Where the assessee had not suffered tax
U/s 56 (2) (x)]. [S. 49 (4)].
8 COA of CA SDV on the DOA (in case the asset acquired = immovable property). [Where the
acquired for assessee had suffered tax U/s 56 (2) (x)]. [S. 49 (4)].
inadequate FMV on the DOA (in case the asset acquired = specified movable property
consideration. referred to in S. 56 (2) (x)). [Where the assessee had suffered tax U/s 56 (2) (x)].
[S. 49 (4)].
Actual COA. [Where the assessee had not suffered tax U/s 56 (2) (x)].
Note: Distribution of CA by HUF to its members upon partition, gift, transmission of CA under
will or inheritance ≠ Transfer. [S. 47 (i) & (iii)].

Illustration-1:
Particulars Case-1 Case-2 Case-3 Case-4 Case-5
Consideration (A) Rs. 8L Rs. 8L Rs. 8L Rs. 12L Rs. 12L
SDV (B) Rs. 8.39L Rs. 8.46L Rs. 8.70L Rs. 12.59L Rs. 12.80L

Discuss the tax consequences in the hands of seller and buyer.

Determination of amount taxable U/s 56 (2) (x) (b) (B) and COA in the hands of buyer:
SN Particulars Case-1 Case-2 Case-3 Case-4 Case-5
1 A Rs. 8L Rs. 8L Rs. 8L Rs. 12L Rs. 12L
2 B Rs. 8.39L Rs. 8.46L Rs. 8.70L Rs. 12.59L Rs. 12.80L
3 B-A Rs. 39000 Rs. 46000 Rs. 70000 Rs. 59000 Rs. 80000
4 5% of A Rs. 40000 Rs. 40000 40000 Rs. 60000 60000
5 Higher of (a) 5% of A; (b) Rs. 50000 Rs. 50000 Rs. 50000 Rs. 50000 Rs. 60000 Rs. 60000
6 Is 3 > 5? No No Yes No Yes
7 Amount taxable U/H IFOS by - - Rs. 70000 - Rs. 80000
virtue of S. 56 (2) (x) (b) (B)

MT Educare –CA Inter Income Tax Classes – R.SOUMYANARAYANAN.FCA. GRAD CWA. Page-121
Chapter-6 : Capital gains

8 Whether S. 49 (4) applies for No No Yes No Yes


determination of COA of the CA?
9 COA of the CA (S. 49 (4)). Rs. 8L Rs. 8L Rs. 8.70L Rs. 12L Rs. 12.80L
(Actual (Actual (SDV) (Actual (SDV)
COA) COA) COA)

Determination of FVC for computing CG in the hands of the Seller:


SN Particulars Case-1 Case-2 Case-3 Case-4 Case-5
1 Actual consideration Rs. 8L Rs. 8L Rs. 8L Rs. 12L Rs. 12L
2 105% of the consideration Rs. 8.40L Rs. Rs. Rs. 12.60L Rs.
8.40L 8.40L 12.60L
3 SDV Rs. 8.39L Rs. Rs. Rs. 12.59L Rs.
8.46L 8.70L 12.80L
4 Observation 3<2 3>2 3>2 3<2 3>2
5 Whether S. 50C is applicable No Yes Yes No Yes
for computing the FVC?
6 FVC Rs. 8L (Actual Rs. Rs. Rs. 12L (Actual Rs.
consideration) 8.46L 8.70L consideration) 12.80L
(SDV) (SDV) (SDV)

Illustration-2:
X acquires an immovable property from Y during April 2019. Relevant data is given below:
SN Particulars Case-1 Case-2 Case-3 Case-4 Case-5
1 SDV on the DOA Rs. 8L Rs. 8L Rs. 9L Rs. 9L Rs. 10L
2 SDV on the DOR Rs. Rs. Rs. Rs. Rs.
8.70L 8.70L 9.80L 9.80L 10.705L
3 Consideration for acquisition of Rs. Rs. Rs. Rs. Rs.
property from Y 7.62L 7.62L 8.55L 8.55L 10.20L
4 Whether advance is paid to Y by an Yes No Yes No No
APC on or before the DOA?
Discuss the tax consequences in the hands of X and Y.

Determination of amount taxable U/s 56 (2) (x) (b) (B) and COA in the hands of X:
SN Particulars Case-1 Case-2 Case-3 Case-4 Case-5
1 Actual consideration (A) Rs. 7.62L Rs. 7.62L Rs. 8.55L Rs. 8.55L Rs. 10.20L
2 Whether advance is Yes No Yes No No
paid to Y by an APC on
or before the DOA?
3 Relevant SDV (B) Rs. 8L Rs. 8.70L Rs. 9L Rs. 9.80L Rs. 10.705L
(SDV on (SDV on (SDV on (SDV on the (SDV on the
the DOA) the DOR) the DOA) DOR) DOR)
4 B-A Rs. 38000 Rs. 108000 Rs. 45000 Rs. 125000 Rs. 50500
5 5% of A Rs. 38100 Rs. 38100 Rs. 42750 Rs. 42750 Rs. 51000
6 Higher of (a) 5% of A; (b) Rs. 50000 Rs. 50000 Rs. 50000 Rs. 60000 Rs. 51000
Rs. 50000

MT Educare –CA Inter Income Tax Classes – R.SOUMYANARAYANAN.FCA. GRAD CWA. Page-122
Chapter-6 : Capital gains

7 Is 4 > 6? No Yes No Yes No


8 Amount taxable U/H - Rs. 108000 - Rs. 125000 -
IFOS by virtue of S. 56 (2)
(x) (b) (B)
9 Whether S. 49 (4) No Yes No Yes No
applies for
determination of COA of
the CA?
10 COA of the CA (S. 49 (4)). Rs. 7.62L Rs. 8.70L Rs. 8.55L Rs. 9.80L Rs. 10.20L
(Actual (SDV on (Actual (SDV on the (Actual
COA) the DOR) COA) DOR) COA)

Determination of FVC for computing CG in the hands of Y:


SN Particulars Case-1 Case-2 Case-3 Case-4 Case-5
1 Actual consideration Rs. 7.62L Rs. 7.62L Rs. 8.55L Rs. 8.55L Rs. 10.20L
2 105% of Rs. 800100 Rs. 800100 Rs. 897750 Rs. 897750 Rs. 1071000
consideration
3 Whether advance is Yes No Yes No No
paid to Y by an APC
on or before DOA?
4 Relevant SDV Rs. 8L (SDV Rs. 8.70L Rs. 9L (SDV Rs. 9.80L Rs. 10.705L
on the (SDV on the on the (SDV on the (SDV on the
DOA) DOR) DOA) DOR) DOR)
5 Observation 4<2 4>2 4>2 4>2 4<2
6 Whether S. 50C is No Yes Yes Yes No
applicable for
computing the FVC?
7 FVC Rs. 7.62L Rs. 8.70L Rs. 9L (SDV Rs. 9.80L Rs. 10.20L
(Actual (SDV on the on the (SDV on the (Actual
considerati DOR) DOA) DOR) consideration)
on)

6. Cost of improvement:
1 Meaning of COI It means any capital expenditure incurred by an assessee in making any
additions or improvement to the CA. In other words, any expenditure incurred
to increase the value of CA is treated as COI.
2 Improvement Any expenditure deductible in computing the income chargeable U/ H IFHP,
cost not eligible PGBP or IFOS shall not be eligible for deduction in computing income U/H CG.
for deduction- S. COI incurred by the assessee or previous owner before 01.04.2001 shall be
55 (1) (b) ignored.
3 Miss. Piroja C. Expenditure incurred in securing vacant possession of plot (subjected to
Patel (Bom) tenancy) for the purpose of effecting sale amounts to COI.

MT Educare –CA Inter Income Tax Classes – R.SOUMYANARAYANAN.FCA. GRAD CWA. Page-123
Chapter-6 : Capital gains

7. Classification of CA based on its nature – S. 2 (29A) & S. 2 (42A):


Capital assets (Group-I) POH ≤ 12 months POH > 12 months
(1) Securities (not being units) listed on RSE in STCA LTCA
India
(2) EOU of Mutual fund
(3) ZCB
Capital assets (Group-II) POH ≤ 24 months POH > 24 months
(1) Shares of companies not listed on RSE in India STCA LTCA
(2) Immovable property being land or building or
both (located in India or in foreign country)
Capital assets (Group-III) POH ≤ 36 months POH > 36 months
Any other capital asset STCA LTCA

Points requiring attention:


1 Meaning of POH Usually, POH starts on the DOA and ends on a day preceding the DOT.
2 POH of CA acquired under S. It Starts on the DOA by the previous owner who had not obtained the
49 (1) mode of acquisition CA U/s 49 (1) mode. [S. 2 (42A) Explanation].
3 POH of gifted asset Starts on the DOG. [Where assessee had suffered tax U/s 56 (2) (x)].
Starts on the DOA by the previous owner. [Where the assessee had
not suffered tax U/s 56 (2) (x)].
4 Smt. Rama Rani Kalia (All) Where the assessee converts his lease hold interest in a property to
free hold interest and subsequently effects sale, the POH starts on the
date of taking the property on lease. Assessee need not be the owner
of the CA through the POH. Note the word ‘held’.

Purpose of classification of CG into long-term and short-term:


1 Tax rates Tax rates for LTCG and STCG are different.
2 Set off LTCL could be adjusted only against LTCG and not against STCG.
3 Chapter VI-A Where the GTI of the assessee includes LTCG referred to in S. 112/S. 112A or
deduction STCG referred to in S. 111A, that portion of the GTI is not eligible for deduction
under Chapter VI-A.
4 Exemption Exemption U/s 54, 54EC, 54EE, 54F, 54GB and S. 115F are available only in
respect of LTCG and not against STCG.
5 Indexation Benefit of indexing COA & COI is available only in computation of LTCG.

8. Benefit of indexation:
1 S. 48 Proviso-2 While computing CG arising on account of transfer of a LTCA, ICOA and ICOI
shall be allowed as deduction.
2 Numerator index Index pertaining to the PY of transfer.
3 Denominator Situation Denominator index (COA)
index (for COA) CA was not obtained U/s Base index (if DOA < 01.04.2001)
49 (1) mode Index relating the PY of acquisition (if DOA ≥
01.04.2001)
CA was obtained U/s 49 (1) Base index (if DOA (by the previous owner) <
mode (gift) 01.04.2001)

MT Educare –CA Inter Income Tax Classes – R.SOUMYANARAYANAN.FCA. GRAD CWA. Page-124
Chapter-6 : Capital gains

Index relating the PY of acquisition by the


previous owner (if DOA (by the previous
owner) ≥ 01.04.2001)
CA was obtained by way of Base index (if DOA (by the previous owner) <
gift + assessee did not 01.04.2001)
suffer tax U/s 56 (2) (x) Index relating the PY of acquisition by the
previous owner (if DOA (by the previous
owner) ≥ 01.04.2001). [Manjula Shah (Bom)]
CA was obtained by way of Index relating to the PY of gift.
gift + assessee suffered tax
U/s 56 (2) (x)
4 Denominator Index pertaining to the PY of incurrence.
index (for COI)
5 S. 48 proviso-4 For debentures or bonds, no indexation.
Exception: Indexation is available for (a) Sovereign Gold Bonds (SGB) issued
by RBI; (b) Capital indexed bonds issued by CG.
6 S. 48 Proviso-3 For equity shares, EOU and units of BT covered by S. 112A, no indexation.
7 S. 47 (viic) No CG in the hands of individuals upon redemption of SGB by RBI.

Cost inflation index:


CII notified for various PYs are as under:
2001-2002 100 2007-2008 129 2013-2014 220
2002-2003 105 2008-2009 137 2014-2015 240
2003-2004 109 2009-2010 148 2015-2016 254
2004-2005 113 2010-2011 167 2016-2017 264
2005-2006 117 2011-2012 184 2017-2018 272
2006-2007 122 2012-2013 200 2018-2019 280
2019-2020 289

Test your knowledge:


On 01.01.20, X gifts a house property to Y. The property was purchased by X for Rs. 30L on
01.01.09. On 01.03.20, Y sells the house property to Z for Rs. 80L. The SDV of the property on
various dates are as follows:
01.01.09 Rs. 20L 01.01.20 Rs. 50L 01.03.20 Rs. 95L
Required:
(a) Discuss the tax implications in the hands of X and Y. Assume X and Y are not related.
(b) Assume that X and Y are brothers. What would be your answer?

MT Educare –CA Inter Income Tax Classes – R.SOUMYANARAYANAN.FCA. GRAD CWA. Page-125
Chapter-6 : Capital gains

9. Computation of CG under special circumstances:


A. Computation of CG upon transfer of assets declared under the Income Declaration
Scheme, 2016: [S. 49 (5) + S. 2 (42A) Explanation-1 (i) + R. 8AA (3)]:
1 Income declaration scheme, 2016 As per Income Declaration Scheme, 2016, the assets
which were acquired out of black money can be
declared.
The FMV of the assets as on 01.06.2016 (Date of
commencement of scheme) shall be deemed to be
the undisclosed income and assessee shall pay:
Income tax 30% on such FMV
Add: Krishi Kalyan Cess 25% of tax
Add: Penalty 25% of tax
Total payable 45% of FMV
2 COA of assets declared under the scheme FMV as on 01.06.2016. [S. 49 (5)].
3 POH of assets (supra) – R. 8AA (3):
Asset declared ≠ immovable property POH starts on 01.06.2016
Asset declared = immovable property POH starts on the actual DOA.
(the date of acquisition of which is
evidenced by a deed registered with any
authority of a SG)
Asset declared = immovable property (in POH starts on 01.06.2016
other cases)

(B) Computation of CG upon receipt of insurance compensation on account of destruction


of CA: [S. 45 (1A)]:
1 When S. 45 (1A) could be CA gets destroyed or damaged.
invoked? Destruction or damage is due to: (a) Flood; (b) Typoon; (c) Hurricane;
(d) Cyclone; (e) Earth quake; (f) Other convulsion of nature; (g) Riot;
(h) Civil disturbance; (i) Accidental fire; (j) Accidental explosion; (k)
Action by enemy; (l) Action taken in combating enemy
On account of destruction or damage, the assessee receives
compensation from the insurer.
2 PY of transfer PY in which damage or destruction took place.
3 PY of chargeability PY in which money or other asset is received from the insurer.
4 POH DOA to the day preceding the date of damage/destruction.
5 FVC Money received from insurer or
FMV of asset (on the date of receipt) received from the insurer (where
the compensation is in kind).
6 Depreciable asset In case of depreciable asset, the computation of CG is subject to S. 50
7 COA of the new asset The FMV of such asset on the date on which it was received should be
(received as compensation taken as its COA.
from the insurer)

MT Educare –CA Inter Income Tax Classes – R.SOUMYANARAYANAN.FCA. GRAD CWA. Page-126
Chapter-6 : Capital gains

Previous examination question:


Question No 3: The assessee was a company carrying on business of manufacture and sale of
art-silk cloth. It purchased machinery worth Rs. 4L on 1.5.2008 and insured it with United
India Assurance Ltd against fire, flood, earthquake etc., The WDV of the asset as on
01.04.2019 was Rs. 2,08,800. The insurance policy contained a reinstatement clause requiring
the insurance company to pay the value of the machinery, as on the date of fire etc., in case
of destruction of loss. A fire broke out in August 2018 causing extensive damage to the
machinery of the assessee rendering them totally useless. The assessee company received a
sum of Rs. 6L from the insurance company on 15th March 2020. Discuss the issues arising on
account on the transactions and their tax treatment. (Cost inflation index for financial year
2008-09 and 2019-20 are 137 and 289 respectively). [May 2000].

Answer:
1 As per S. 45 (1A), where any person receives any money or other assets under an insurance from an
insurer on account of damage to or destruction of CA as a result of, accidental fire then, any profits
and gains arising from the receipt of such money or other assets, shall be chargeable to tax U/H CG
and shall be deemed to be the income of such person of the PY in which such money or asset was
received.
2 For the purpose of S. 48, the money received or the market value of the asset shall be deemed to
be the FVC accruing as a result of the transfer of such CA. Since the asset was destroyed and the
money from the insurance company was received in the PY, there will be a liability to CG in respect
of the insurance moneys received by the assessee.
3 U/s 45 (1A) any profits and gains arising from receipt of insurance moneys is chargeable U/H CG. For
the purpose of S. 48, the moneys received shall be deemed to be the FVC. U/s 50 the CG in respect
of depreciable assets had to be computed in the following manner (assuming it was the only asset
in the block):
1 FVC Rs. 600000
2 WDV as on 01.04.2019 Rs. 208800
3 STCG RS. 391200

(C) Computation of CG upon conversion of CA into SIT – S. 45 (2):


As per S. 2 (47), even conversion of CA into SIT amounts to transfer and hence, CG needs to
be computed. In this regard, the following matters require attention:
1 PY of transfer PY in which CA got converted into SIT.
2 PY of chargeability PY in which such SIT was sold.
3 POH DOA to the day preceding the date of conversion (DOC)
4 FVC FMV on the DOC
5 Numerator index Index relating to the PY in which CA got converted into SIT.
6 Computation of In the PY in which the SIT was sold, in addition to the CG, the difference
business income between the sale proceeds of SIT and the FMV on the DOC shall be
assessed to tax U/H PGBP.

MT Educare –CA Inter Income Tax Classes – R.SOUMYANARAYANAN.FCA. GRAD CWA. Page-127
Chapter-6 : Capital gains

Conversion of stock-in-trade into capital asset – tax implications:


1 S. 2 (24) (xiia) + S. PGBP = [FMV on the DOC – Carrying amount of inventory].
28 (via)
2 PY of PY in which entry is passed. [If the assessee follows mercantile system].
chargeability PY of realisation. [If assessee follows cash system of accounting].
3 FMV for this Nature of asset FMV
purpose. [R. Immovable property SDV on the DOC
11UB]. Jewellery, archaeological collections, Price that could be
drawings, paintings, sculptures, any work of fetched if sold in the open
art, shares or securities etc market on the DOC
4 Explanation-1A to Upon conversion of inventory into CA, if the CA is used for the business or
S. 43 (1) profession, depreciation shall be allowed on it. For this purpose, the actual
cost is the FMV referred to in R. 11UB.
5 COA [S. 49 (9)]. FMV referred to in R. 11UB.
6 POH POH starts on the DOC. [Clause (ba) of Explanation-1 to S. 2 (42A)].
7 Denominator Index relating to the PY of conversion.
index

Test your knowledge on S. 45 (2):


X converts his CA (acquired on June 10, 2004 for Rs. 60,000) into SIT on March 10, 2019. The
FMV on the DOC was Rs. 3.50L. He subsequently sells the SIT so converted for Rs. 4L on June
10, 2019. Examine the tax implication. CII - FY 2004-05: 113; FY 2018-19: 289.

Previous examination question:


Question No 4 Tani purchased a land at a cost of Rs. 35L in the FY 2004-05 and held the same
as her CA till 31st May, 2018. Tani started her real estate business on 1st June 2018 and
converted the said land into SIT of her business on the said date, when the FMV of the land
was Rs. 210L. She constructed 15 flats of equal size, quality and dimension. Cost of
construction of each flat is Rs. 10L. Construction was completed in January 2020. She sold 10
flats at Rs. 30L per flat between January 2020 and March 2020. The remaining 5 flats were
held in stock as on 31st March 2020. She invested Rs. 50L in bonds issued by NHAI on 31st
March 2020 and another Rs. 50L in bonds of REC Ltd in April 2020. Compute the amount of
chargeable CG and business income in the hands of Tani arising from the above transactions
for AY 2019-20. CII: FY 2004-05: 113; FY 2018-19: 280; FY 2019-20: 289. [May 2013].

Answer:
Computation of CG and business income of Tani for AY 2020-21:
Particulars Rs.
FMV of land on the DOC deemed as FVC for the purposes of S. 45(2) 2,10,00,000
Less: ICOA [Rs. 35,00,000 × 280/113] 86,72,566
1,23,27,434
Proportionate CG arising during AY 2020-21 [Rs. 1,23,27,434 × 2/3] 82,18,289
Less: Exemption U/s 54EC 50,00,000
CG chargeable to tax for AY 2020-21 32,18,289
Business Income

MT Educare –CA Inter Income Tax Classes – R.SOUMYANARAYANAN.FCA. GRAD CWA. Page-128
Chapter-6 : Capital gains

Sale price of flats [10 × Rs. 30L] 3,00,00,000


Less: Cost of flats
FMV of land on the DOC [Rs. 210L × 2/3] 1,40,00,000
Cost of construction of flats [10 × Rs. 10L] 1,00,00,000
Business income chargeable to tax for AY 2020-21 60,00,000

Notes:
1 The conversion of a CA into SIT is treated as a transfer U/s 2 (47). It would be treated as a transfer
in the year in which the CA is converted into SIT.
2 However, as per S. 45(2), the CG arising from the transfer by way of conversion of CA into SIT will
be chargeable to tax only in the year in which the SIT is sold.
3 The indexation benefit for computing ICOA would, however, be available only up to the year of
conversion of CA into SIT and not up to the year of sale of SIT.
4 For the purpose of computing CG in such cases, the FMV of the CA on the date on which it was
converted into SIT shall be deemed to be the FVC. In this case, since only 2/3rd of the SIT (10 flats
out of 15 flats) is sold in the PY 2019-20, only proportionate capital gains (i.e., 2/3rd) would be
chargeable in the AY 2020-21.
5 On sale of such SIT, business income would arise. The business income chargeable to tax would be
computed after deducting the FMV on the DOC of the CA into SIT and cost of construction of flats
from the price at which the SIT is sold.
6 In case of conversion of CA into SIT and subsequent sale of SIT, the period of 6 months is to be
reckoned from the date of sale of SIT for the purpose of exemption U/s 54EC [CBDT Circular No.791
dated 2.6.2000]. In this case, since the investment in bonds of NHAI has been made within 6
months of sale of flats, the same qualifies for exemption U/s 54EC.
7 With respect to LTCG arising in any FY, the maximum deduction U/s 54EC would be Rs. 50L,
whether the investment in bonds of NHAI or RECL are made in the same FY or next FY or partly in
the same FY and partly in the next FY.
8 Therefore, even though investment of Rs. 50L has been made in bonds of NHAI during the PY 2019-
20 and investment of Rs. 50L has been made in bonds of RECL during the PY 2020-21, both within
the stipulated 6 month period, the maximum deduction allowable for AY 2020-21, in respect of
LTCG arising on sale of LTCA during the PY 2019-20, is only Rs. 50L.

(D) Computation of CG on transfer of dematted securities - S. 45 (2A):


1 How to find POH and FIFO basis
COA in respect of
dematted securities?
2 FIFO method in case of FIFO method shall be applied account-wise. [CBDT Circular- no. 768].
multiple accounts
3 FIFO method on Where in an existing account, old physical securities are
dematerialization of old dematerialized and entered at a later date, under the FIFO method,
physical securities in the basis for determining the movement out of the accounts is the
existing account date of entry into the account. [CBDT Circular- no. 768].

MT Educare –CA Inter Income Tax Classes – R.SOUMYANARAYANAN.FCA. GRAD CWA. Page-129
Chapter-6 : Capital gains

Provisions illustrated:
Demat account 0562
Date of Particulars Quantity COA
credit
01.06.2018 Purchased directly in dematerialized form on 25.05.2018 2000 20000
th
05.06.2018 Dematerialized shares originally purchased on 20 5000 100000
November, 2004
10.06.2018 Purchased directly in dematerialized form on 10.06.2018 4000 60000
15.06.2019 Dematerialized shares originally purchased on May, 1986 3000 15000

Details regarding quantity sold:


Date of sale Quantity sold
01.04.2018 2500 (Lot 1)
30.06.2018 3000 (Lot 2)
28.02.2019 3500 (Lot 3)

COA and POH:


Details of shares DOA DOT POH COA
Lot 1 2000 shares 25.05.2018 01.04.2019 25.05.2018 to 20000
31.03.2019
500 shares 20.11.2004 01.04.2019 20.11.2004 to 10000
31.03.2019 (100000*500/5000)
Lot 2 3000 shares 20.11.2004 30.06.2019 20.11.2004 to 60000
29.06.2019 (100000*3000/5000)
Lot 3 1500 shares 20.11.2004 28.02.2020 20.11.2004 to 30000
27.02.2020 (100000*1500/5000)
2000 shares 10.06.2018 28.02.2020 10.06.2018 to 30000
27.02.2020 (60000*2000/4000)

(E) Computation of CG upon transfer of CA by partner/member to a firm/AOP by way of


capital contribution or otherwise - S. 45 (3):
1 CG in the hands of CG is chargeable to tax in the hands of the partner/member.
partner/member
2 FVC Amount for which the transferred asset is recorded in the books of the
firm/AOP.
3 S. 45 (3) > S. 50C & S. Where the CA transferred by the partner/member to the firm/AOP = Non-
50CA quoted shares or immovable property, S. 50CA or S. 50C shall not be
invoked in determining FVC for the purpose of computing CG in the hands
of partner or member.
4 COA of such CA in the Amount at which the CA is recorded in the BOA of the firm/AOP. [Rajdoot
hands of firm/AOP Hotel enterprises (MP)].

MT Educare –CA Inter Income Tax Classes – R.SOUMYANARAYANAN.FCA. GRAD CWA. Page-130
Chapter-6 : Capital gains

(F) CG upon distribution of CA by firm/AOP on dissolution or otherwise: S. 45 (4):


1 Chargeability of CG CG is chargeable to tax in the hands of the firm/AOP.
2 PY of chargeability PY of distribution (not the PY of dissolution) [Vijayalakshmi metal
industries (Mad)]
3 FVC FMV on the date of distribution.
4 A. N. Naik Associates S. 45 (4) can be invoked even if the firm distributes CA to the partner at
(2004) (Bom). the time of his retirement to settle his dues. Note the word ‘otherwise’
in S. 45 (4).
5 COA of the CA in the Value agreed for such asset as per the dissolution deed.
hands of the partner
6 Lingamallu Raghu When assets are distributed to the retiring partner upon retirement,
Kumar (SC) what transpires is the realisation of his pre-existing right to get a share in
the net assets of the firm. Therefore, the question of computing CG in
the hands of retiring partner on the ground that the retiring partner has
assigned his interest in the partnership does not arise.
7 J.M. Mehta and As per S. 17 (1) (b) of the Registration Act, common immovable
Brothers (Bom) properties can’t be divided, possessed and enjoyed severally without a
duly executed and registered document. Therefore, partners of a firm
cannot take out immovable property from the ownership of firm by
mutual agreement and by passing entries in the BOA of the firm.
8 Sivalingam Nadar (SC) In case of dissolution of firm, there is no need for a separate document.
The dissolution deed itself stands as a proof of title to the partners.

Valuation of inventory on the date of dissolution of a firm:


1 For the purpose of accounting and taxation, the closing stock is to be valued at cost or NRV,
whichever is less.
2 This is provided so in AS-2 as well as in ICDS-II.
Even newly re-casted S. 145A (i) provides for valuation of inventory, other than certain securities,
at lower of actual cost or NRV which is to be computed as per ICDS-II.
3 However, where a firm is dissolved and the business is discontinued, the closing stock is to be
valued at market price to determine the correct profit taxable in the case of the firm. This was the
view expressed by the SC in ALA firm 189 ITR 285.
4 This view accords with the accounting practice on the issue, which provides for the valuation of all
assets and liabilities at market value on liquidation basis, when the going concern assumption is
not applicable.
5 However, the SC in its later decision in Sakthi Trading Company 250 ITR 871 (SC) has distinguished
its earlier decision by facts and ruled that where a firm is dissolved and business is not
discontinued, that is, business is continued by remaining partners, then closing inventory should
be valued as per the established rule of commercial practice and accountancy, that is, cost or
market price, whichever is less.
6 In essence, where the business of the dissolved firm is continued the stock in trade is to be valued
at cost or market value; however, the stock in trade has to be valued at market value if the business
is discontinued.
7 In order to neutralize the decision of the SC in Sakthi Trading Company, Paragraph 24 of ICDS II
on Valuation of Inventories requires the inventory on the date of dissolution to be valued at the
NRV, notwithstanding whether business is discontinued or not.

MT Educare –CA Inter Income Tax Classes – R.SOUMYANARAYANAN.FCA. GRAD CWA. Page-131
Chapter-6 : Capital gains

8 However, the Delhi HC in Chamber of Tax consultants [2017] 252 Taxman 77 (Delhi) held that the
above provision of ICDS is an attempt to over reach the binding judicial precedents by the device
of notifications issued by the CG. It is an exercise of excessive delegation of legislative power
which is impermissible in law.
9 Even the newly re-casted S. 145A (i) does not make any specific provision for valuation of
inventory on dissolution.
10 In view of the above, it appears that the provision relating to valuation in case of dissolution in
ICDS II (that is, at NRV) is in conflict with the provisions of the Act and accordingly the provisions
of the Act would prevail and not ICDS II.

(G) Computation of CG upon compulsory acquisition of CA - S. 45 (5):

(i) Computation of CG upon receipt of original compensation – S. 45 (5) (a):


S. 2 (47) defines transfer to interalia include compulsory acquisition of CA. Where a CA is
compulsorily acquired, CG shall be computed in accordance with the provisions of S. 45 (5).
In this regard, the following matters require attention:
1 PY of transfer PY in which CA is compulsorily acquired.
2 PY of chargeability PY in which the compensation (or part thereof) is first received. .
3 POH DOA to the date preceding the date of compulsory acquisition
4 FVC Compensation determined under the corresponding law
5 Numerator index Index relating to the PY in which CA was compulsorily acquired.
6 Denominator index Index relating to the PY in which the asset was first held by the assessee.

(ii) Tax treatment of enhanced compensation – S. 45 (5) (b):


Where the compensation is enhanced by the court or tribunal or any other authority, CG shall
be computed and in this regard, the following points require attention:
1 Year of chargeability PY in which the additional compensation is received.
2 COA Nil
3 COI Nil
4 Taxability of additional When the transferor dies before the receipt of additional
compensation upon death of compensation, the additional compensation shall be assessed to
the transferor tax in the hands of the recipient U/H “CG”.
5 Deductibility of litigation Litigation expenses incurred in getting compensation enhanced
expenses shall be allowed as a deduction in computing CG.
6 Nature of CG The CG computed upon receipt of additional compensation take
the character of CG computed in the first instance.

(iii) Tax treatment of additional compensation received pursuant to interim order of a


Court or Tribunal or other authority – Proviso to S. 45 (5) (b):
Where the assessee receives additional compensation pursuant to interim order of a Court or
Tribunal or other authority, it shall not be charged to tax U/H CG in the PY of receipt, but in
the PY in which final order of such Court or Tribunal or other authority is made.

MT Educare –CA Inter Income Tax Classes – R.SOUMYANARAYANAN.FCA. GRAD CWA. Page-132
Chapter-6 : Capital gains

(iv) Recomputation of CG upon subsequent reduction of compensation/consideration by


the court/ tribunal/any other authority – S. 45 (5) (c):
Where in the assessment for any year, the CG arising from the transfer of a CA is computed
by taking the compensation or, as the case may be, enhanced compensation, and
subsequently such compensation is reduced by any Court, Tribunal or other authority, such
assessed CG of that year shall be recomputed by taking the compensation as so reduced by
such Court, Tribunal or other authority to be the FVC.

Time limit for Re-computation -S. 155 (16):


The AO can re-compute the CG within 4 years from the end of the PY in which the order
reducing the compensation was passed by the Court or tribunal or any other authority.

CBDT Circular 36/2016:


The compensation received U/s 96 of Right to Fair Compensation and Transparency in Land
Acquisition, Rehabilitation and Resettlement Act, 2013 (RFCTLARR Act) even in respect of
compulsory acquisition of non-agricultural land is exempt even if there is no specific provision
for providing exemption under the IT Act.

Latest from Judiciary:


Receipt of higher compensation on account of negotiations does not transform the character
of compulsory acquisition into a voluntary sale, so as to deny exemption U/s 10 (37) (iii).
[Balakrishnan (2017) 391 ITR 178 (SC)]

Connected sections:
(i) Exemption of CG arising on a/c of compulsory acquisition of urban agricultural land - S.
10 (37):
1 Person eligible Individual or HUF
for exemption
2 CA to be Urban agricultural land.
transferred It may be a LTCA or STCA.
It should have been used for agricultural purpose either by the
assessee or by his parents during the period of 2 years immediately
preceding the DOT.
It should have been compulsorily acquired or the consideration for
its transfer should have been determined or approved by the CG or
RBI.
3 Condition for The compensation or consideration should have been received on
exemption. or after 01.04.2004.

(ii) TDS on payment of compensation on compulsory acquisition – S. 194LA:


1 Payment subjected to Sum being compensation or enhanced compensation on
TDS account of compulsory acquisition of immovable property (not
being agricultural land (rural or urban)).
2 Payee Resident
3 Threshold limit Sum paid during a FY > Rs. 2.50L, then there is TDS obligation.
Otherwise, there is no TDS obligation.

MT Educare –CA Inter Income Tax Classes – R.SOUMYANARAYANAN.FCA. GRAD CWA. Page-133
Chapter-6 : Capital gains

4 TDS rate 10% (Subject to S. 206AA)


5 Timing of TDS At the time of payment.

Note However, no tax shall be deducted from the compensation awarded U/s 96 of
RFCTLARR Act.

(iii) Tax treatment of interest on compensation or enhanced compensation – S. 145B (1)


+_S. 56 (2) (viii) + S. 57 (iv):
(i) Income by way of interest received on compensation or on enhanced compensation
shall be taxable U/H IFOS in the PY of receipt. [S. 145B (1) + S. 56 (2) (viii)].
(ii) A deduction of sum equal to 50% of such income shall be allowed. No other deduction
shall be allowed against it. [S. 57 (iv)].
(iii) This interest shall be subjected to TDS U/s 194A @ 10%. [Baldeep Singh (P&H)].

Latest from Judiciary:


1 Interest U/s 28 of the Land Acquisition Act, 1894 (S. 72 of RFCTLARR Act) which
represents enhanced value of land and thus, partakes the character of compensation
and not interest. Hence, the interest U/s 28 is liable to be taxed U/H CG and not U/H
IFOS.
2 On the other hand, interest U/s 34 of the Land Acquisition Act, 1894 (S. 80 of RFCTLARR
Act) is for the delay in making payment after the compensation amount is determined.
Such amount is liable to be taxed U/H IFOS. [Movaliya Bhikhubhai Balabhai (2016) 388
ITR 343 (Guj)].

Test your knowledge:


Q-1: On 01.04.2003, X acquired a land for Rs. 10L which was compulsorily acquired on
01.01.2013. Land acquisition officer fixed a compensation of Rs. 80L which is paid to X as
under:
Date of payment Amount paid
01.01.2018 Rs. 40L
01.01.2019 Rs. 40L

Aggrieved X filed an appeal before the District Court challenging the quantum of
compensation. On 01.12.2021, the District Court enhanced the compensation to Rs. 170L.
Additional compensation of Rs. 90L was received on 01.05.2022. Legal expenses incurred by
X in this regard amounted to Rs. 10L.

The SG, aggrieved by the decision of the District Court enhancing the compensation, filed an
appeal before the HC challenging the order of the District Court. The HC, through its order
dated 01.01.2028, reduced the compensation to Rs. 140L. Legal expenses incurred by X in
defending his case amounted to Rs. 5L. Rs. 30L is refunded to the Government on 01.05.2028.
Discuss the tax implications in the hands of Mr. X for the various AYs.

MT Educare –CA Inter Income Tax Classes – R.SOUMYANARAYANAN.FCA. GRAD CWA. Page-134
Chapter-6 : Capital gains

Q-2: Assessee’s land (being a LTCA) was acquired on 01.01.2017 by the Government. Original
compensation was received in the PY 17-18. Assessee filed a case for additional compensation
in the HC and the HC passed an interim order on 10.07.2018 awarding an interim enhanced
compensation of Rs. 50L which was received by the assessee on 31.07.2018. The High Court
passed the final order on 31.12.2019 awarding enhanced compensation of Rs. 75L. Discuss
the tax implications under the following situations:
Case-1 Rs. 25L was received by the assessee on 31.01.2020.
Case-2 Rs. 25L was received by the assessee on 30.04.2020.

Previous examination question:


Question No 5: Mr. Krishnan owned vast area of agricultural land in TN. The SG acquired the
property for development of a techno park. Mr. Krishnan was awarded compensation of Rs.
15L. Aggrieved by the amount, he initiated negotiations with the Collector, further to which
compensation was fixed at Rs. 35L. Mr. Krishnan claimed exemption from CG U/s 10 (37) since
the transfer of agricultural land was on account of compulsory acquisition. The AO contended
that exemption U/s 10 (37) would not be available in this case, as it was not a compulsory
acquisition but a voluntary sale, since he had received higher compensation consequent to
negotiation. Examine whether the claim for exemption U/s 10 (37) is tenable in law in this
case.
Solution:
1 The issue under consideration is whether receipt of higher compensation on account of
negotiation would transform the character of compulsory acquisition into a voluntary sale, so as
to deny exemption U/s 10 (37).
2 This issue came up before the SC in Balakrishnan v. UOI. The SC observed that the acquisition
process was initiated under the Land Acquisition Act, 1894. The assessee entered into negotiations
only for securing the market value of the land without having to go to the Court. Merely because
the compensation amount is agreed upon, the character of acquisition will not change from
compulsory acquisition to a voluntary sale. The SC also drew attention to a recently enacted
legislation titled, RTFCTLARR Act, 2013, which empowers the Collector to pass an award with the
consent of the parties. Despite the provision for consent, the acquisition would continue to be
compulsory.
3 Accordingly, the SC held that when proceedings were initiated under the Land Acquisition Act,
1894, even if the compensation is negotiated and fixed, it would continue to remain as compulsory
acquisition.
4 Applying the rationale of the SC ruling to the case on hand, the claim of exemption from capital
gains U/s 10 (37) in this case by Mr. Krishnan is tenable in law.

(H) Special provisions for computation of CG in case of JDA – S. 45 (5A) + S. 49 (7):


1 S.2 (47) defines the term ‘transfer of capital asset’
2 The definition of ‘transfer’ interalia includes any arrangement or transaction where any
rights are handed over in execution of part performance of contract, even though the legal
title has not been transferred.
3 In such a scenario, execution of JDA between the owner of immovable property and the
developer triggers CG tax liability in the hands of owner in the year in which the possession
of immovable property is handed over to the developer for development of project.

MT Educare –CA Inter Income Tax Classes – R.SOUMYANARAYANAN.FCA. GRAD CWA. Page-135
Chapter-6 : Capital gains

4 With a view to minimise the genuine hardship which the owner of land may face in paying
CG tax in the year of transfer, S. 45 (5A) is inserted with effect from AY 2018-19.
5 Notwithstanding anything contained in S. 45 (1), where the CG arises to an assessee being
an individual or a HUF from transfer of a CA, being land or building or both, under a specified
agreement, the CG shall be chargeable to tax as income of the PY in which the certificate of
completion for the whole or part of the project is issued by the competent authority (i.e.
authority empowered to approve the building plan by or under any law for the time being in
force). [S. 45 (5A)].
6 For this purpose, the SDV on the date of issue of the said certificate, of his share, being land
or building or both in the project, as increased by the consideration received in cash, if any,
shall be deemed to be the FVC.
7 However, the provisions of S. 45 (5A) shall not apply where the assessee transfers his share
in the project on or before the date of issue of the said certificate of completion, and the CG
shall be deemed to be the income of the PY in which such transfer takes place and the
provisions of the Act, other than the provisions of S. 45 (5A), shall apply for the purpose of
determination of FVC. [Proviso to S. 45 (5A)].
8 ‘Specified agreement’ means a registered agreement in which a person owning land or
building or both, agrees to allow another person to develop a real estate project on such
land or building or both, in consideration of a share, being land or building or both in such
project, whether with or without payment of part of the consideration in cash;
9 Where the CG arises from the transfer of a CA, being share in the project, in the form of land
or building, referred to in S. 45 (5A), not being the CA referred to in the proviso to S. 45 (5A),
the COA of such asset shall be the amount which is deemed as FVC in S. 45 (5A). [S. 49 (7)].

Illustration-1:
Mr. X purchased a residential plot on 01.01.1999 for Rs. 50L. FMV of the plot as on 01.04.2002
is Rs. 65L. ‘Alpha builders’ enters into a Development Agreement with Mr. X on 01.05.2019
on the following terms and conditions:
(a) Mr. X will hand over the possession of the plot to Alpha builders on 01.05.2019.
(b) Alpha builders will pay a cheque of Rs. 60L to Mr. X on 01.05.2019.
(c) Alpha builders will construct 10 residential units on the plot of land and will give 6 units to Mr. X.
(d) 10 units shall be completed by 30.06.2021 and on that date 6 units will be handed over to Mr. X.
Assume the SDV of the plot as on 01.05.2019 to be Rs. 200L. Also assume that the SDV of each
flat on 30.06.2021 to be Rs. 45L. The project completion certificate is issued by the Competent
authority on 30.06.2021. 6 units are handed over to Mr. X on 30.06.2021. Discuss the tax
implications.

Computation of CG in the hands of Mr. X on account of transfer of 40% of plot:


SN Particulars Case-1
(PY of transfer = PY 19-20); (PY of chargeability = PY 21-22)
Computation Amount
1 Period of holding 01.01.1999 to 30.04.2019 LTCA
2 FVC (Rs. 45L*6) + Rs. 60L Rs. 330L
3 ICOA [S. 55 (2) (b) + S. 48 Proviso-2] (Rs. 65L *40%)* 289/100 (Rs. 75.14L)
4 LTCG (2-3) Rs. 254.86L

MT Educare –CA Inter Income Tax Classes – R.SOUMYANARAYANAN.FCA. GRAD CWA. Page-136
Chapter-6 : Capital gains

Illustration-2:
Suppose, in Illustration-1, the residential plot was owned by X Ltd. Will your answer differ?
S. 45 (5A) is applicable only to an individual and HUF. If the plot is owned by X Ltd, then the
CG arise on 01.05.2019 being date of possession and CG on account of transfer of 40% of plot
shall be taxable in the PY 2019-20.

1 POH 01.01.1999 to 30.04.2019 LTCA


2 FVC As per S. 50D (See note Rs. 200L * 40% = Rs.
below) 80L
3 ICOA [S. 55 (2) (b) + S. 48 (Rs. 65L*40%) * 289/100 (Rs. 75.14L)
Proviso-2]
4 LTCG (2-3) Rs. 4.86L
Note:
FVC is the SDV of 6 flats plus Rs. 60L. Since the flats are not in existence and are yet to be
constructed, their SDV is not ascertainable. Hence, S, 50D is being invoked. SDV of the plot
transferred is deemed to be the FMV of the plot for the purpose of S. 50D and hence, it is
regarded as FVC.

Illustration-3:
Suppose, in Illusrtration-1, Mr. X sells two units on 01.01.2023 for Rs. 80L each. What will be
the tax implications?

Solution:
As per S. 49 (7), the COA of share of the assessee in the project in the form of land or building
or both shall be the amount which has been deemed as FVC U/s 45 (5A). The DOA of the share
of the assessee in the project shall be the date on which possession of the share of the
assessee in the project is handed over to the assessee.
1 COA/unit Rs. 45L
2 POH (for two units sold) 30.06.2021 – 31.12.2022
3 STCG/unit Rs. 80 – Rs. 45L = Rs. 35L

Illustration-4:
Suppose, in Illustration-1, Mr. X sells 2 flats before the issue of completion certificate on
30.12.2020 for Rs. 75L each. Discuss the tax implications.

Computation of CG arising on account of transfer of 40% of the plot for the AY 2020-21:
1 POH 01.01.1999 to 30.04.2019 LTCA
2 FVC As per S. 50D Rs. 200L * 40% = Rs. 80L
3 ICOA [S. 55 (2) (b) + S. 48 Proviso-2] (Rs. 65L*40%) * 289/100 (Rs. 75.14L)
4 LTCG (2-3) Rs. 4.86L
Note: Apart from this, CG is to be computed on account of transfer of 2 flats before the issue
of completion certificate.

MT Educare –CA Inter Income Tax Classes – R.SOUMYANARAYANAN.FCA. GRAD CWA. Page-137
Chapter-6 : Capital gains

TDS on payment under specified agreement – S. 194-IC:


1 Payment to be Any sum by way of consideration, not being consideration in
subjected to TDS kind, under the agreement referred to in S. 45 (5A).
2 Payee Resident
3 Payer Developer
4 TDS rate 10% (Subject to S.206AA)
5 Timing of TDS At the time of credit to the account of the payee in the books of
the payer or at the time of payment (whichever is earlier).
6 Threshold limit Not prescribed.

(I) Special exemption to specified people of Andhra Pradesh – S. 10 (37A):


Background
(i) After split of AP into Telangana and AP, the Government of AP had announced to have their new
State Capital at Amaravati. 33000 Acres of land will be used to create a world class capital city
Amaravati.
(ii) In this regard, the Government of AP had announced a scheme called Land pooling scheme.
(iii) The Land pooling Scheme is an arrangement made by the Government of AP for acquisition of land
from people for formation of new capital city of Amaravati to avoid land acquisition disputes and
to lessen the financial burden associated with payment of compensation.
(iv) Under this scheme, the people have to compulsorily transfer their land and or building to Specified
Authority.
(v) These people will be given land or plot in the new capital city of Amaravati, when it is constructed
and developed.
(vi) At the time of transfer of land and or building referred to in (iii), the people will be given Land
Pooling Certificates.
(vii) After the Amaravati City is built-up, people will be given reconstituted plot or land in lieu of Land
Pooling Certificates. Thus, in the land pooling scheme, the compensation in the form of
reconstituted plot or land is provided to land owners.

Provisions of S. 10 (37A)
1 Any income chargeable U/H CG in respect of transfer of specified capital asset arising to an assessee,
being an individual or a HUF, if he transfers that specified capital asset under the Land Pooling Scheme
of AP Government shall be exempt from tax. [S. 10 (37A)].
2 Explanation to S. 10 (37A) defines the term ‘specified capital asset’.
3 ‘Specified capital asset’ means
(a) Land or building or both owned by the assessee as on 02.06.2014 and which has been
transferred under the scheme; or
(b) Land Pooling Certificates issued under the scheme to the assessee in respect of land or
building or both referred to in (a); or
(c) the reconstituted plot or land, as the case may be, received by the assessee in lieu of land or
building or both referred to in (a) in accordance with the scheme, if such plot or land, as the
case may be, so received is transferred within 2 years from the end of the FY in which the
possession of such plot or land was handed over to him.

MT Educare –CA Inter Income Tax Classes – R.SOUMYANARAYANAN.FCA. GRAD CWA. Page-138
Chapter-6 : Capital gains

COA in case of specified capital asset referred to in Clause (c) of Explanation to S. 10 (37A)-
S. 49 (6):
Where the CG arise from the transfer of a specified capital asset referred to in clause (c) of
Explanation to S. 10 (37A), which has been transferred after the expiry of 2 years from the
end of the FY in which the possession of such asset was handed over to the assessee, the cost
of acquisition of such specified capital asset shall be deemed to be its SDV as on the last day
of the second FY after the end of the FY in which the possession of the said specified capital
asset was handed over to the assessee.

Illustration:
Mr. Rehman purchased land and building in Andhra Pradesh on 01.01.2010 for Rs. 10L. Under
the Land pooling Scheme of AP Government, Mr. Rehman on 01.01.2015 transferred land &
building to Specified Authority when SDV of Land & Building was Rs. 25L. On 01.01.2015, Mr.
Rehman is allotted Land Pooling Ownership Certificates and as per the Certificate, he shall be
allotted a Land in Amaravati on 01.01.2020 in lieu of Land Pooling Ownership Certificate.
Discuss the tax implications in the following cases:

Case-I He sells Land Pooling Certificate on 01.01.2019 for Rs. 30L.


Case-II On 01.01.2020, he is allotted and given possession of a plot in Amaravati in lieu
of Land Pooling Certificates and SDV of the Plot is Rs. 40L. He sells the residential
plot on 01.01.2021 for Rs. 45L.
Case-III On 01.01.2020, he is allotted and given possession of a plot in Amaravati in lieu
of Land Pooling Certificates and SDV of the Plot is Rs. 40L. He sells the residential
plot on 01.01.2023 for Rs. 70L. SDV of the plot on 31.03.2022 is Rs. 50L

Tax implications for the AY 2015-16:


As per S. 10 (37A) introduced retrospectively, the capital gains on transfer of Land and
Building in Land Pooling Scheme are exempt.

Case-I: Tax implications for the AY 2019-20:


As per S. 10 (37A), the capital gains on transfer of Land Pooling Certificate are exempt.

Case-II: Tax implications for the AY 2021-22:


As per S. 10 (37A), the capital gains on transfer of residential plot in Amaravati are exempt
since the plot has been transferred before 01.04.2022.

Case-III: Tax implications for the AY 2023-24:


CG on sale of residential plot in Amaravati is not exempt since sale takes place after
31.03.2022.
POH 01.01.2020 to 31.12.2022
Nature of CG LTCG
Sale price Rs. 70L
Cost as per S. 49 (6) Rs. 50L

MT Educare –CA Inter Income Tax Classes – R.SOUMYANARAYANAN.FCA. GRAD CWA. Page-139
Chapter-6 : Capital gains

Connected provision: S. 94 (8):


1 Meaning of bonus Bonus stripping is a process by which an investor or dealer gets bonus units by
stripping investing in units shortly before the record date and exiting shortly after the record
date at a lower price thereby incurring loss.
2 Motivation for bonus The assessee gets bonus units which compensates the loss incurred by him
stripping on account of sale of units. Further, the investor can set off such loss against
other income (if permitted) and can also carry forward the unabsorbed loss
for set off in future years.
3 Measure for curbing S. 94 (8) is inserted to curb the practice of bonus stripping.
bonus stripping

Applicability of S. 94 (8):
S. 94 (8) will apply if the following cumulative conditions are satisfied:
1 The assessee purchases units within a period of 3 months prior to the record date.
2 He is allotted additional units without any payment on the basis of holding of units on such date.
3 He sells all or any of the units referred to in (2) within a period of 9 months after the record date.
4 On the date of sale, he shall hold at least one of the additional bonus units allotted.
Effect of S. 94 (8):
Loss arising on account of such purchase and sale of units shall be ignored. However, such
loss will be considered to be the COA of the bonus units held on the date of sale.
Points requiring attention:
1 This provision applies only to units and not to shares.
2 It is applicable even in case where units are held as stock in trade.
Test your knowledge:
Q-1: A mutual fund declared 1:1 bonus units on its units on 30.04.19. The fund fixed the record
date for bonus entitlement to be 31.05.19. Mr. A purchased 1000 units on 20.05.19 @ Rs. 20
per unit. He sold 1000 original units on 11.11.19 for Rs. 9 per unit. Discuss the tax implications.

Q-2: Suppose in Q-1, Mr. A sold 1000 original units and 800 bonus units on 11.11.19 for Rs. 9
per unit. Discuss the tax implications.

Q-3: Suppose in Q-2, Mr. A sold 1000 original units on 11.11.19 @ Rs. 9 per unit and 800 bonus
units on 15.11.19 @ Rs. 9 per unit. Discuss the tax implications.

(M) Computation of CG on account of transfer of right shares and right entitlement:


Tax implications in the hands of renouncer on account of renouncing the right entitlement
in favour of any other person:
Right entitlement is a capital asset. Renouncing right entitlement amounts to relinquishment
of CA. CG needs to be computed in the hands of the renouncer and the following points
require attention:
1 COA of right entitlement Nil. (S. 55 (2) (aa) (ii)).
2 POH Date of offer of the right to the day preceding the date of transfer.
[Exp-1 to S. 2 (42A)].
3 Criterion for deciding the 36 months.
nature of CA
4 Nature of CG Invariably the gain is a STCG, since the offer cannot be kept open
for a long period.

MT Educare –CA Inter Income Tax Classes – R.SOUMYANARAYANAN.FCA. GRAD CWA. Page-140
Chapter-6 : Capital gains

Tax implications in respect of transfer of right shares subscribed using the right
entitlement:
In computing the CG arising on account of transfer of right shares the following points require
attention:
1 COA of right shares Amount actually paid for acquiring such shares. S. 55 (2) (aa) (iii).
2 POH Date of allotment of right shares to the day preceding the date
of transfer. Sub clause (d) of clause (i) of Exp-1 to S. 2 (42A).

Tax implication in the hands of renouncee in the event of sale of shares subscribed using
the right entitlement:
In computing the capital gains arising on account of transfer of shares subscribed by the
renouncee using the right entitlement, the following points require attention:
1 COA of Amount paid to the company for acquiring such shares + amount paid to the
shares renouncer for acquiring the right entitlement. [S. 55 (2) (aa) (iv)].
2 POH Date of allotment of shares by the company to the day preceding the date of transfer.
[Exp-1 to S. 2 (42A)].

(N) Computation of CG arising on account of shares or securities allotted under ESOP:


1 COA of shares or securities allotted under ESOP = FMV on the date of exercise of option. [S. 49 (2AA)].
2 POH of such shares or securities shall commence on the date of allotment. [S.2 (42A) Explanation-1].

Test your knowledge:


ABC Ltd allotted 1000 sweat equity shares to Shri. Chand in June 2019. The shares were
allotted at Rs. 200 per share as against the FMV of Rs. 300 per share on the date of exercise
of option by the allottee [determined as per R. 3 (8)].
(i) What is the perquisite value of equity shares allotted to Shri. Chand?
(ii) On 28.12.2019, these shares were sold for Rs. 450 per share. Determine the taxable
CG in this regard.

(O) COA of intangible CA – S. 55 (2) (a):


CA COA (if self- COA (if
generated) acquired)
Goodwill of a business Nil Actual COA
Trade mark associated with a business
Brand name associated with a business
Right to manufacture, produce or process an article or thing
Right to carry on a business or profession
Tenancy rights
Stage carriage permits (i.e. Route permits)

(P) COI of certain intangible CA – S. 55 (1) (b) (1):


CA Cost of improvement
Good will of a business Nil
Right to manufacture, produce or process an article or thing Nil
Right to carry on a business or profession Nil

MT Educare –CA Inter Income Tax Classes – R.SOUMYANARAYANAN.FCA. GRAD CWA. Page-141
Chapter-6 : Capital gains

(Q) Transfer of CA between holding company and its WOS:


S. 47 (iv) + S. 47 (v) + S.49 (1) + S. 2 (42A) Explanation-1:
1 Where a CA is transferred as a CA by holding company to its WOS (being an Indian company), this
shall not be regarded as transfer. [S. 47 (iv)]. Hence, the question of computing CG does not arise.
2 Where a CA is transferred as a CA by WOS to its holding company (being an Indian company), this
shall not be regarded as transfer. [S. 47 (v)]. Hence, the question of computing CG does not arise.
3 Where the CA is acquired by way of transfer referred to in S. 47 (iv) or S. 47 (v), the COA of such
CA is the COA to the previous owner. [S. 49 (1)].
4 The POH of such CA shall also include the period for which the CA was held by the previous owner.
[S. 2 (42A) Explanation-1].
Note: For this purpose, subsidiary means direct subsidiary and not chain subsidiary.

Withdrawal of exemption granted U/s 47 (iv) / (v) – S. 47A (1) & S. 155 (7B):
1 Circumstances in which the Subsidiary company ceases to be WOS within 8 years from the
exemption granted U/s 47 (iv) / DOT of CA. (or)
(v) could be withdrawn U/s 47A Transferee-company converts the transferred CA into SIT within
(1) 8 years from DOT.
2 Manner of withdrawal The order of assessment passed in respect of AY relevant to the
PY of transfer in respect of transferor shall be amended to re-
compute the TI on account of withdrawal of exemption granted
U/s 47 (iv) or S. 47 (v).
The order of amendment shall be passed within 4 years from the
end of the PY of violation. [S. 155 (7B)].
3 COA of CA in the hands of Actual COA. [S. 49 (3)]. Provisions of S. 49 (1) shall not apply.
transferee (post withdrawal of
exemption).
4 POH of CA in the hands of Starts on the actual DOA. Provisions of Explanation-1 to S. 2
transferee (post withdrawal of (42A) shall not apply.
exemption).

Illustration:
S Ltd is a WOS of A Ltd. Both are Indian companies. On 10.04.07, S Ltd transfers a CA to A Ltd
(acquired on 06.04.02 for Rs 50,000) for Rs 1.5L. Assessment of S Ltd for the AY 08-09 got over
U/s 143 (3) on 01.01.10. A Ltd has converted the CA into SIT on 10.05.11. The FMV of the CA
on 10.05.11 is Rs. 5L. On 10.05.19, A Ltd sells the converted asset for Rs. 10L. Determine the
assessable profits of A Ltd and S Ltd.

Tax implication in the hands of S Ltd at the time of transfer of CA:


1 Transferor = WOS
2 Transferee = Holding company
3 Transferee = Indian company
4 CA is transferred as CA
5 All conditions of S. 47 (v) are complied with.
6 Hence, there will be no CG in the hands of S Ltd

MT Educare –CA Inter Income Tax Classes – R.SOUMYANARAYANAN.FCA. GRAD CWA. Page-142
Chapter-6 : Capital gains

Tax implications in the hands of S Ltd upon violation of S. 47A:


1 DOT of CA by S Ltd to A Ltd 10.04.07
2 8 years from the DOT ends on 09.04.15
3 DOC of CA into SIT by A Ltd 10.05.11
4 Observation Provisions of S. 47A (1) are violated.
5 Consequence of violation CG exempted in the hands of S Ltd for the AY 08-09 shall
be charged to tax in the hands of S Ltd.
6 PY of chargeability PY 07-08 (not the PY 11-12 being the year of violation)

Computation of CG for the AY 08-09


1 PY of chargeability 07-08
2 POH 06.04.03 to 09.04.07
3 Observation POH > 36 months
4 Nature of CA LTCA
5 FVC 1,50,000
6 ICOA (50,000*129)/109 = 59174
7 LTCG Rs. 90826
Note: The order of assessment pertaining to AY 2008-09 shall be amended within 31.03.2016.
[S. 155 (7B)].

Tax implications in the hands of A Ltd:


Computation of CG in the hands of A Ltd for the AY 20-21
1 PY of acquisition 07-08
2 PY of transfer 11-12
3 PY of Chargeability 19-20
4 POH 10.04.07 to 10.05.11
5 Observation POH > 36 months
6 Nature of CA LTCA
7 FVC 5,00,000
8 ICOA (S. 49 (3)) (1,50,000 * 184)/129 = 213953
9 LTCG 286047

Computation of business income in the hands of A Ltd for the AY 20-21


1 Sale proceeds of SIT 10L
2 Cost of SIT (Bai Shirinbai K. Kooka 46 ITR 86 (SC)) 5L
3 Business income (1-2) 5L

(R) Amalgamation and capital gains:


1. Conditions to be fulfilled to be regarded as amalgamation under the Act – S. 2 (1B):
For mergers to be regarded as amalgamation, the following conditions are to be satisfied:
(a) All the properties of the amalgamating company before amalgamation shall become
properties of the amalgamated company.
(b) All the liabilities of the amalgamating company before amalgamation shall become
liabilities of the amalgamated company.
(c) Shareholders holding atleast 75% in the value of shares in amalgamating company
shall become shareholders in the amalgamated company by virtue of amalgamation.

MT Educare –CA Inter Income Tax Classes – R.SOUMYANARAYANAN.FCA. GRAD CWA. Page-143
Chapter-6 : Capital gains

2. Transfer of CA to the amalgamated company pursuant to the scheme of amalgamation


– S. 47 (vi) + S. 49 (1) + S. 2 (42A) Explanation-1:
1 Where a CA gets transferred from the amalgamating company to the amalgamated
company (being an India company) pursuant to the scheme of amalgamation, it shall
not be regarded as transfer. [S. 47 (vi)]. Hence, the question of computing CG does not
arise.
2 The COA of such CA in the hands of amalgamated company is the COA to the previous
owner (i.e. amalgamating company). [S. 49 (1)].
3 While computing the POH of such CA in the hands of amalgamated company the period
for which the CA was held by the amalgamating company shall also be included. [S. 2
(42A) Explanation-1].
4 Subsequently, if these assets are transferred by the amalgamated company, for
computing the CG, if the COA is to be indexed, the denominator index is the index
pertaining to the PY in which these assets were acquired by the amalgamating company.
[M. Sankar Trading Corporation and Consultancy (P) Ltd. ITA. No. 2103/Mds/2012]

3. Transfer of shares held in amalgamating company pursuant to amalgamation – S. 47


(vii):
(a) Where amalgamation takes place, the amalgamating company comes to an end. The shares in
the amalgamating company will become valueless. The rights of the shareholders in the
amalgamating company get extinguished.
(b) Extinguishment of rights is transfer as per S. 2 (47). If the amalgamated company allots its
shares to the shareholders of amalgamating company as a consideration for such
extinguishment of rights, no CG shall be computed in the hands of such shareholders. This is
because S. 47 (vii) provides that nothing contained in S. 45 shall apply to such extinguishment.
(c) However, this exemption is available only if the amalgamated company is an Indian company.
(d) S. 47(vii) excludes the requirement of issue of shares to the shareholder where such
shareholder itself is the amalgamated company. However, the amalgamated company will
continue to be required to issue shares to the other shareholders of the amalgamating
company.

4. Transfer of shares allotted by amalgamated company to the shareholders of


amalgamating company:
Where, pursuant to the scheme of amalgamation, shares are allotted by the amalgamated
company to the shareholder of an amalgamating company and such shares are transferred
by such shareholder, the CG shall be computed and charged to tax in his hands in the PY of
transfer. In this regard, the following points require attention:
1 POH of shares in amalgamated DOA of shares in the amalgamating company to the
company day preceding the DOT. [S. 2 (42A) Explanation 1].
2 COA of shares in the amalgamated Cost of acquisition of shares in the amalgamating
company = company. (S. 49 (2)).
3 Denominator index for indexing COA of Index of the PY in which the shares were allotted by
shares in amalgamated company for the amalgamating company. (Decision of Bombay HC
computing CG arising on account of in Manjula Shah case applied).
transfer of such shares =
Note: However, the aforesaid provisions are applicable only if the amalgamated company is
an Indian company.

MT Educare –CA Inter Income Tax Classes – R.SOUMYANARAYANAN.FCA. GRAD CWA. Page-144
Chapter-6 : Capital gains

5. Transfer of shares held in an Indian company by one foreign company to another


foreign company in a scheme of amalgamation – S. 47 (via):
If the following conditions are satisfied, nothing is chargeable to tax U/s 45:
(a) Atleast 25% of the shareholders of the amalgamating company shall become the
shareholders of the amalgamated company.
(b) This transfer shall not attract tax on capital gains in the country, in which the
amalgamating company is incorporated.

Note:
1 COA of shares in Indian company COA to the previous owner (i.e. amalgamating
in the hands of amalgamated foreign company). [S. 49 (1)].
foreign company =
2 POH of shares (supra) for Starts on the DOA by the previous owner (i.e.
amalgamated foreign company amalgamating FC). [S. 2 (42A) Explanation-1].
3 Denominator index for indexing Index pertaining to the PY in which the shares
COA of shares (supra) for (supra) were acquired by the amalgamating foreign
computing CG in the hands of company. (Decision of Bombay HC in Manjula
amalgamated foreign company Shah case applied).
arising on account of transfer of
shares (supra) =

(S) Demerger & Capital gains:


1. Meaning of Demerger, Demerged company & Resulting Company:
(a) The term ‘Demerger’ means transfer, by one company, of its one or more
undertakings to another company. Here, undertaking means a business activity. (S. 2
(19AA).
(b) The transfer of undertaking should be pursuant to a scheme of demerger referred to in S.
391 to S. 394 of the Companies Act 1956 (i.e. S. 230 to S. 232 of the Companies Act 2013).
(c) The company which transfers the undertaking(s) is called as demerged company. (S. 2
(19AAA).
(d) The company to which the undertaking of the demerged company is transferred is
called as resulting co. The resulting company may or may not be a WOS of the
demerged company. (S. 2 (41A)).

2. Conditions to be satisfied to be called as demerger – S. 2 (19AA):


(a) All the properties of the undertaking transferred by the demerged company shall become the
properties of the resulting company.
(b) All the liabilities which arise out of the activities or operations of the undertaking transferred,
shall become the liabilities of the resulting company.
(c) Assets & liabilities transferred to the resulting company shall be recorded it at such values appearing in
the books of a/c of the demerged company immediately before demerger (figures before any revaluation
shall be taken).
However, the requirement of recording property and liabilities at book value by the resulting company
shall not be applicable in a case where the property and liabilities of the undertaking received by it, are
recorded at a value different from the value appearing in the books of account of the demerged
company immediately before the demerger in compliance of the Indian Accounting Standards specified
in the Annexure to the Companies (Indian Accounting Standards) Rules, 2015. [FA 2019 Amendment].

MT Educare –CA Inter Income Tax Classes – R.SOUMYANARAYANAN.FCA. GRAD CWA. Page-145
Chapter-6 : Capital gains

(d) The resulting company shall issue shares to the shareholders of the demerged company on a
proportionate basis as a consideration for demerger.
However, S. 2 (19AA) excludes the requirement of issue of shares where resulting company itself
is a shareholder of the demerged company. But the requirement of issuing shares would still have
to be met by the resulting company in case of other shareholders of the demerged company.
(e) Shareholders holding atleast 75% in the value of shares in demerged company shall become
shareholders in the resulting company by virtue of demerger
(f) The demerged company shall transfer its undertaking as a going concern. That is the business
should be continuing at the time of demerger.
(g) U/s 72A (5), the CG has powers to notify in the official gazette such conditions as it considers
necessary to ensure that the demerger is for genuine business purposes. the demerger should
be in accordance with the conditions, if any, notified U/s 72A (5).

Note:
The reconstruction or splitting up of a company, which ceased to be a PSC as a result of
transfer of its shares by the CG, into separate companies, shall be deemed to be demerger, if
such reconstruction or splitting up has been made to give effect to any condition attached to
the said transfer of shares and also fulfills such other condition as may be notified by the CG
in the official gazette. [Explanation-5 to S. 2 (19AA)].

3. Transfer of CA by the demerged company to the resulting company pursuant to


demerger – S. 47 (vib):
1 Pursuant to demerger, all assets of the undertaking transferred become the assets of
the resulting company. Such assets may interalia include CA.
2 No CG shall be chargeable to tax in the hands of the demerged company on account of
this transfer. This is because S. 47 (vib) provides that nothing in S. 45 shall apply to such
transfer. However, this exemption is available only if the resulting company is an Indian
company. The demerged company may be an Indian company or otherwise.
3 The COA of such CA in the hands of resulting company is the COA to the previous owner
(i.e. demerged company). [S. 49 (1)].
4 While computing the POH of such CA in the hands of resulting company the period for
which the CA was held by the demerged company shall also be included. [S. 2 (42A)
Explanation-1].
5 Subsequently, if these assets are transferred by the resulting company, for computing
CG, if the COA needs to be indexed, the denominator index shall be the index pertaining
to the PY in which these assets were acquired by the demerged company. (Decision of
Bombay HC in Manjula Shah case applied).

4. Issue of shares in resulting company pursuant to the scheme of demerger – S. 47 (vid):


1 On account of demerger, the networth of the demerged company gets reduced. Hence,
it is followed by reduction of share capital of demerged company. This results in
extinguishment of rights in shares of demerged company which comes within the
transfer definition contemplated in S. 2 (47). The resulting company allots its shares to
the shareholders of demerged company as a consideration for such extinguishment.
2 However, there will be no CG computation since such extinguishment is not to be
regarded as transfer for the purpose of S. 45. [S. 47 (vid)].

MT Educare –CA Inter Income Tax Classes – R.SOUMYANARAYANAN.FCA. GRAD CWA. Page-146
Chapter-6 : Capital gains

5. Computation of CG arising from transfer of shares in demerged company & resulting


company – S. 49 (2C) + S. 49 (2D):
The following points require attention for computation of CG arising on account of transfer
of shares in the demerged company and in the resulting company:
SN Particulars Shares in demerged company Shares in resulting (Indian)
company
1 POH DOA of shares in the demerged DOA of shares in the demerged
company to the date of transfer. company to the date of transfer
2 COA See table given below. (S. 49 (2D)) See table given below. (S. 49 (2C))
3 Denominator Index of the PY in which the shares Index for the PY in which the shares
index were allotted by the demerged were allotted by the demerged
company. company.

COA of shares in the resulting company and the demerged company:


1 COA of original shares held by the assessee in the demerged company ****
2 Net book value of assets transferred pursuant to demerger ****
3 Net worth of the demerged company immediately before demerger ****
4 COA of shares in the resulting company (1*2/3) (as per S. 49 (2C)) ****
5 COA of shares in the demerged company (1-4) (as per S. 49 (2D)) ****

Net worth = paid up share capital + General Reserves (immediately before demerger).

6. Transfer of shares held in an Indian company by one foreign company to another


foreign company in a scheme of demerger – S. 47 (vic):
If the following conditions are satisfied, nothing is chargeable to tax U/s 45:
(a) Shareholders holding not less than 75% in value of the shares of the demerged foreign
company continue to remain shareholders of the resulting foreign company.
(b) This transfer shall not attract tax on CG in the country in which the demerged foreign
company is incorporated.

Note:
1 COA of shares in Indian company COA to the previous owner (i.e. Demerged foreign
in the hands of resulting foreign company). [S. 49 (1)].
company =
2 POH of shares (supra) for resulting Starts on the DOA by the previous owner (i.e.
foreign company Demerged foreign company). [S. 2 (42A)
Explanation-1].
3 Denominator index for indexing Index pertaining to the PY in which the shares
COA of shares (supra) for (supra) were acquired by the Demerged foreign
computing CG in the hands of company. (Decision of Bombay HC in Manjula
resulting foreign company arising Shah case applied).
on account of transfer of shares
(supra)

MT Educare –CA Inter Income Tax Classes – R.SOUMYANARAYANAN.FCA. GRAD CWA. Page-147
Chapter-6 : Capital gains

Previous examination question:


Question No 6 SS(P) Ltd., a domestic Indian company having two undertakings engaged in
manufacture of cement and steel, decided to hive off cement division to RV (P) Ltd., a
domestic Indian company, by way of demerger. The net worth of SS (P) Ltd. immediately
before demerger was Rs. 40 Crores. The net book value of assets transferred to RV (P) Ltd.
was Rs. 10 Crores. The demerger was made in January 2020. In the scheme of demerger, it
was fixed that for each equity share of Rs. 10 each (fully paid up) of SS(P) Ltd., two equity
shares of Rs. 10 each (fully paid up) were to be issued. One Mr. N.K. held 25,000 equity shares
in SS (P) Ltd. which were acquired in the FY 2004-05 for Rs. 6L. Mr. N.K. received 50,000 equity
shares from RV (P) Ltd. consequent to demerger in January 2020. He sold all the shares of RV
(P) Ltd. for Rs. 8L in March 2020. In this background you are requested to answer the
following:
(i) Does the transaction of demerger attract any income tax liability in the hands of SS (P) Ltd.
and RV (P) Ltd.?
(ii) Compute the CG that could arise in the hands of Mr. N.K. on receipt of shares of RV (P) Ltd.
(iii) Compute the CG that could arise in the hands of Mr. N.K. on sale of shares of RV (P) Ltd.
(iv) Will sale of shares by Mr. N.K. affect tax benefits availed by SS (P) Ltd. and/or RV (P) Ltd.?
(v) Is Mr. N.K. eligible to avail any tax exemption under any of the provisions of the Income-tax
Act, 1961 on the sale of shares of RV (P) Ltd.? If so, mention in brief.

Note:
FY CII
2003-04 113
2019-20 289

Answer:
(i) No, the transaction of demerger would not attract any tax in the hands of SS(P) or RV(P) Ltd.
As per S. 47 (vib), any transfer in a demerger, of a CA, by the demerged company to the
resulting company would not be regarded as “transfer” for levy of CG tax if the resulting
company is an Indian company. Hence, CG tax liability would not be attracted in the hands
of SS(P) Ltd., the demerged company, in this case, since RV(P) Ltd. is an Indian company.
(ii) There would be no CG liability in the hands of Mr. N.K. on receipt of shares of RV (P) Ltd.,
since as per S. 47 (vid), any issue of shares by the resulting company in a scheme of demerger
to the shareholders of the demerged company will not be regarded as “transfer” for levy of
CG tax, if the issue is made in consideration of demerger of the undertaking.
(iii) Yes, CG would arise in the hands of Mr. N.K. on sale of shares of RV (P) Ltd.
Sale consideration 8,00,000
Less: ICOA of shares of RV (P) Ltd.
COA of shares of RV(P) Ltd. as per S. 49 (2C):
Net book value of assets transferred in a demerger
Cost of acquisition of sharesof SS ( P ) Ltd 
Networth of the demerged company immediately before demerger
10 crores
60000  = 1,50,000
40 crores
ICOA of shares of RV (P) Ltd. [Rs. 1,50,000 × 289/113] Rs. 3,83,628
LTCG (since POH of shares in demerged company is also to be considered) Rs. 4,14,372.

MT Educare –CA Inter Income Tax Classes – R.SOUMYANARAYANAN.FCA. GRAD CWA. Page-148
Chapter-6 : Capital gains

(iv) No, sale of shares by Mr. N.K. would not affect the tax benefits availed by SS(P) Ltd. or RV (P)
Ltd.
One of the conditions to be satisfied is that the shareholders holding not less than three-
fourths in value of the shares in the demerged company become shareholders of the resulting
company by virtue of the demerger. It is presumed that the condition is satisfied in this case.
There is no stipulation that they continue to remain shareholders for any period of time
thereafter.
(v) Since the resultant CG on sale of shares of RV(P) Ltd. is a LTCG (on account of the POH of
shares in demerged company being considered by virtue of S. 2 (42A) (g)), Mr. N.K. can avail
exemption –
(a) U/s 54EE, by investing the LTCG units of specified fund, within a period of 6 months
from the DOT.
(b) U/s 54F by investing the entire net consideration in purchase (within 1 year before
and 2 years after the DOT) or construction (within 3 years after the DOT) of one
residential house in India. If part of the net consideration is invested, only
proportionate exemption would be available.

(T) Reduction of share capital – Tax implications:


1. Dividend U/s 2 (22) (d)
1 Where a company, upon reduction of its share capital, makes any distribution to its
shareholders, such distribution to the extent attributable to the accumulated profits of
the company on the date of distribution shall be regarded as dividend.
2 Accumulated profits, for this purpose, shall include capitalised profits also. That is, share
capital to the extent attributable to issue of bonus shares shall also be regarded as part
of AP.
New Explanation 2A is inserted in S. 2 (22) w.e.f 1st April 2018 to widen the scope of
the term 'accumulated profits' so as to provide that in the case of an amalgamated
company, its accumulated profits, whether capitalized or not, or losses as the case may
be, shall be increased by the accumulated profits of the amalgamating company,
whether capitalized or not, on the date of amalgamation.
3 If the company reducing share capital happens to be a domestic company, then it has
obligation to pay DDT U/s 115-O.
4 Dividend U/s 2 (22) (d) is exempt in the hands of shareholders U/s 10 (34). (Subject to
S. 115BBDA).
2. Exclusions from S. 2 (22) (d)
1 Any amount distributed to the preference shareholders upon reduction of preference
shares is not to be regarded as dividend U/s 2 (22) (d). However, in the hands of
preference shareholder, CG needs to be computed in view of decision of the SC in
Anarkali Sarabhai case.
2 Any amount distributed to the SH upon buy back of shares by the company shall not be
regarded as dividend U/s 2 (22) (d).
3. Computation of CG in the hands of shareholder
1 When the share capital is reduced by paying off a part of the capital by reducing the
face value of the share, the share remains but the right of the shareholder to dividends
on his share capital and the right to share in the distribution of the net assets upon

MT Educare –CA Inter Income Tax Classes – R.SOUMYANARAYANAN.FCA. GRAD CWA. Page-149
Chapter-6 : Capital gains

liquidation is extinguished proportionately to the extent of reduction in capital. Thus,


the reduction in share capital amounts to transfer. (Karthikeya. V. Sarabhai (SC)).
2 Accordingly, CG needs to be computed in the hands of shareholder.
3 FVC = Money distributed + FMV of assets distributed – Dividend U/s 2 (22) (d). [G.
Narasimhan (SC)].
4 COA = COA of share to the extent attributable to extinguished part.

(U) Buy back of shares – Tax implications:


1 S. 2 (22) (d) defines ‘Dividend’ to include any distribution by a company to its
shareholders upon reduction of its share capital to the extent of its accumulated profits.
2 However, S. 2 (22) (iv) specifically excludes from the ambit of the term ‘dividend’ any
distribution by a company to its shareholders upon buy back of its shares in accordance
with the provision of the Companies Act. Hence, if a company makes any distribution
pursuant to buy back of its own shares, it is not liable to DDT U/s 115-O.
3 However, the domestic company which buys backs its shares (unlisted or listed) [FA
2019 Amendment w.e.f 05.07.2019] has to pay an additional income tax (called buy-
back distribution tax (BBDT)) U/s 115QA within 14 days of distribution.
4 Quantum of BBDT = (Distributed income * 23.296%).
5 Distributed income = [A –B].
6 A = Consideration paid by the company to its shareholders upon buy back of its shares.
7 B = Amount received by the company for issue of such shares determined in a manner
prescribed U/R 40BB.
8 There will be no CG in the hands of shareholder U/s 46A. It is exempt U/s 10 (34A).

Note:
However, in order to provide relief to listed companies which have made a Public
announcement of the buy-back of shares before 05.07.2019 but actual buy-back of shares
happens on or after 05.07.2019, the Ordinance inserts a new Proviso to S. 115QA that BBDT
shall not be charged.

Determination of amount received by the company for issue of shares – R. 40BB:


Provision Situation Amount received by the company for issue
of shares bought back
S. 40BB Shares (bought back) have been Amount actually received by the company in
(2) issued by the company to any respect of such shares including amount
person by way of subscription. received by way of premium.
S. 40BB Company had at any time, prior Amount received in respect of such share (–
(3) to the buyback of share, ) amount so returned (if it was not charged
returned any sum out of the to DDT U/s 115-O).
amount received in respect of
such share.

MT Educare –CA Inter Income Tax Classes – R.SOUMYANARAYANAN.FCA. GRAD CWA. Page-150
Chapter-6 : Capital gains

S. 40BB (4) Shares (which are bought back) FMV determined U/R 3 (8) to the extent
were issued by the company credited to the share capital and share
under ESOP. premium by the company.
S. 40BB (5) Shares (which are bought back) Amount received by the amalgamating
were issued by amalgamated company in respect of shares in
company, under a scheme of amalgamating company in lieu of which
amalgamation, in lieu of shares in
shares were allotted by the amalgamated
amalgamating company. company.
S. 40BB (6) Shares (which are bought back) Amount received by the Demerged company
were issued by Resulting in respect of shares in Demerged company *
company, under a scheme of (Net book value of assets transferred
demerger, for extinguishment of pursuant to demerger/Net worth of
rights in shares in Demerged Demerged company immediately before
company. demerger).
S. 40BB (7) Shares of Demerged company Amount received by the Demerged company
(after demerger) are bought back in respect of original shares in Demerged
by the Demerged company company – [Amount received by the
Demerged company in respect of original
shares in Demerged company * (Net book
value of assets transferred pursuant to
demerger/Net worth of Demerged company
immediately before demerger)].
S. 40BB (8) Shares (which are bought back) X÷Y
have been issued or allotted by
the company as a part of
consideration for acquisition of
any asset or settlement of any
liability

X An amount being lower of the following


amounts:
(a) [FMV of the asset or liability, as
determined by a merchant banker] * Part of
consideration being paid by issue of shares/
total consideration;
(b) the amount of consideration for
acquisition of the asset or settlement of the
liability to be paid in the form of shares, to
the extent credited to the share capital and
share premium account by the company;
Y The number of shares issued by the company
as part of consideration;

MT Educare –CA Inter Income Tax Classes – R.SOUMYANARAYANAN.FCA. GRAD CWA. Page-151
Chapter-6 : Capital gains

S. 40BB (9) Firm or SPC gets converted into (X-Y)/Z


Company. Company has allotted
shares to the partners or sole
proprietor. Later, the company
buys back the shares from the
partner(S) or sole proprietor.
X Book value of the assets (not being fictitious
assets) in the B/S of the firm or SPC (ignoring
revaluation).
Y Book value of liabilities shown in the B/S of
the firm or SPC excluding (a) capital of the
proprietor or partners of the firm; (b) R&S; (c)
Contingent & unascertained liabilities.
Z Number of shares issued on conversion.
S. 40BB Bonus shares are bought back Nil
(10)
S. 40BB Shares (which are bought back) Amount received by the company in respect
(11) have been issued on conversion of such instrument as so converted.
of preference shares or bond or
debenture, debenture-stock or
deposit certificate in any form or
warrants or any other security
issued by the company
S. 40BB Share being bought back is held in The amount received for the issue of share
(12) demat form and the same cannot determined in accordance with this rule on
be distinctly identified the basis of FIFO method.
S. 40BB In any other case The face value of the share
(13)

Points requiring attention:


1 Obligation to pay BBDT is independent of whether the company has obligation to pay tax on its TI
computed in accordance with the provisions of this Act.
2 BBDT payment is a final payment in respect of which no credit is allowed to the company or the
shareholder.
3 BBDT is neither eligible for deduction in the hands of the company nor in the hands of shareholders.
4 If there is delay in payment of BBDT, interest is levied @ 1% p.m or part thereof for the period
beginning with 15th day from the date of distribution to the date of payment. [S. 115QB].
5 If there is a default in payment of BBDT, the company and its principal officer becomes assessee in
default. Collection and recovery proceedings could be initiated against them. [S. 115QC].

Illustration-1:
A company buys back 10000 unlisted equity shares of face value of Rs. 10 each at a price of
Rs. 100 per share on 01.01.2020. These shares are held by four shareholders equally (i.e. 2500
each). They have subscribed to these shares on 01.01.2013 for Rs. 10 each by paying a
premium of Rs. 20 per share.

MT Educare –CA Inter Income Tax Classes – R.SOUMYANARAYANAN.FCA. GRAD CWA. Page-152
Chapter-6 : Capital gains

SN Tax implications in the hands of company SN Tax implications in the hands of shareholders
1 Distribution by the company upon buy back 1 Distribution by the company upon buy back is
is not dividend (S. 2 (22) (iv)). The company not dividend (S. 2 (22) (iv)).
has no DDT obligation U/s 115-O.
2 Company as to pay BBDT U/s 115QA within 2 Since the company was liable to pay BBDT, the
14 days of distribution. BBDT = 10000 shares CG arising on account of buy back is exempt in
* [Rs. 100 – Rs. 30] * 23.296% the hands of shareholders U/s 10 (34A).

Illustration-2:
X Ltd issued equity shares of face value of Rs. 10 as under:
Shareholder Number of shares Issue price
A 10L Rs. 10
B 10L Rs. 10
C 10L Rs. 30
D 10L Rs. 30

Balance sheet of X Ltd


Liabilities Amount Assets Amount
Equity share capital Rs. 400L Non-current assets Rs. 1200L
40L shares @ 10 each
Share premium Rs. 400L Current assets Rs. 600L
Reserves & Surplus Rs. 1000L
Total Rs. 1800L Total Rs. 1800L
Suppose X Ltd reduced the face value of the shares from Rs. 10 to Rs. 8 and paid Rs. 5 per
share in cash to each shareholder. Thereafter, the company buys back 1L shares from each of
the shareholders by paying Rs. 50 per share.
SN Tax implications in the hands of X Tax implications in the hands of shareholders
Ltd
1 Amount distributed to the Amount distributed to the shareholders upon
shareholders upon reduction of reduction of share capital is to be regarded as dividend
share capital = Rs. 5 * 40L shares = U/s 2 (22) (d). Dividend in the hands of each
Rs. 200L. This is to be regarded as shareholder is Rs. 50L. This is exempt in their hands U/s
dividend U/s 2 (22) (d). Company is 10 (34).
liable to pay DDT U/s 115-O.
2 Company as to pay BBDT U/s Reduction of share capital results in extinguishment of
115QA within 14 days of rights which amounts to transfer in view of S. 2 (47). In
distribution. BBDT = {([Rs. 50 -10] * the hands of shareholders, CG need to be computed.
1L shares) + ([Rs. 50 -10] * 1L [Karthikeya Sarabhai (SC)].
shares) + ([Rs. 50 -30] * 1L shares) Since the company was liable to pay BBDT, the CG
+ ([Rs. 50 -30] * 1L shares)} * arising on account of buy back is exempt in the hands
23.296%. of shareholders U/s 10 (34A).

MT Educare –CA Inter Income Tax Classes – R.SOUMYANARAYANAN.FCA. GRAD CWA. Page-153
Chapter-6 : Capital gains

Illustration-3:
Import the facts from illustration-2. Assume X Ltd distributed Rs. 2 per share on account of
reduction of face value from Rs. 10 to Rs. 8. Also assume that X Ltd had nil reserves & surplus
at the time of reduction of share capital. Subsequently long after, X Ltd bought back 1L shares
from each shareholder for a price of Rs. 50 each.
SN Tax implications in the hands of X Ltd Tax implications in the hands of shareholders
1 Amount distributed to the shareholders No dividend U/s 2 (22) (d).
upon reduction of share capital = Rs. 2 *
40L shares = Rs. 80L. This is not to be
regarded as dividend U/s 2 (22) (d), since
at the time of distribution, X Ltd had Nil
reserves and surplus. Therefore, there will
be no DDT obligation U/s 115-O.
2 Company as to pay BBDT U/s 115QA Reduction of share capital results in
within 14 days of distribution. extinguishment of rights which amounts to
transfer in view of S. 2 (47). In the hands of
BBDT = {([Rs. 50 – (Rs. 10-Rs. 2)] * 1L shareholders, capital gains need to be computed.
shares) + ([Rs. 50 – (Rs. 10 – Rs. 2)] * 1L [Karthikeya Sarabhai (SC)].
shares) + ([Rs. 50 – (Rs. 30 – Rs. 2)] * 1L Since the company was liable to pay BBDT, the CG
shares) + ([Rs. 50 – (Rs. 30 – Rs. 2)] * 1L arising on account of buy back is exempt in the
shares)} * 23.296%. hands of shareholders U/s 10 (34A).

Illustration-4:
A Company acquires an asset from Mr. X and allots Rs. 10L shares of face value of Rs. 10 each
at a premium of Rs. 15 per share to Mr. X. The Company also pays a cheque of Rs. 30L for the
asset. FMV of asset determined by Merchant Banker is Rs. 150L.
X= An amount being lower of the following amounts:
(a) (Rs. 150L * Rs. 250L)/ Rs. 280L = Rs. 133.92L
(b) Rs. 250L
Y= 1000000 shares
Amount received = X÷Y 133.92L ÷ 10L = Rs. 13.39.

(V) Distribution of assets by company upon liquidation – Tax implications:


1 Immunity from CG tax for Where a company, upon liquidation, distributes CA to its
the company in liquidation shareholders, it shall not be regarded as transfer for the purpose of
S. 45 and accordingly, there is no CG tax liability in the hands of
company. [S. 46 (1)].
However, if the company sells CAs and distributes the sale proceeds
among the shareholder, the immunity from S. 45 shall not apply. In
other words, the company shall be liable to CG tax. [Shri Kannan Rice
Mills Ltd (Mad)].
2 DDT obligation U/s 115-O Dividend includes any distribution made to the shareholders of a
for the company company on its liquidation, to the extent to which the distribution is
attributable to the accumulated profits of the company immediately
before its liquidation. [S. 2 (22) (c)].
For this purpose, accumulated profits shall include even capitalised
profits.

MT Educare –CA Inter Income Tax Classes – R.SOUMYANARAYANAN.FCA. GRAD CWA. Page-154
Chapter-6 : Capital gains

If the company under liquidation is a domestic co, then it has


obligation to pay DDT U/s 115-O.
Dividend U/s 2 (22) (c) is exempt in the hands of shareholders U/s 10
(34) [Subject to S. 115BBDA].
3 CG in the hands of Where a shareholder on the liquidation of a company receives any
shareholders. money or other assets from the company, he shall be chargeable to
tax U/H CG in respect of the money so received or the MV of the
other assets on the date of distribution, as reduced by the amount
assessed as dividend within the meaning of S. 2 (22) (c) and the sum
so arrived at shall be deemed to be the FVC for the purposes of S. 48.
[S. 46 (2)].
4 Manner of computation of CG U/s 46 (2) (on a/c of extinguishment of rights in shares in
company in liquidation).
(a) PY of transfer PY of liquidation
(b) PY of chargeability PY of distribution
(c) POH DOA of shares in the company to the date on which company goes
into liquidation.
(d) Criterion for deciding the 12 months or 24 months (depending upon whether shares are listed
nature of CA. or unlisted).
(e) FVC Money + FMV of assets distributed – Dividend U/s 2 (22) (c).

(f) Logic behind deducting To the extent of accumulated profits, the assets distributed are to be
dividend U/s 2 (22) (c) regarded as dividend U/s 2 (22) (c) and is traceable to the head IFOS.
Accordingly, the assets distributed to the extent not regarded as
dividend U/s 2 (22) (c) shall alone qualify as FVC. Though dividend
U/s 2 (22) (c) is exempt U/s 10 (34), it gets taxed in the hands of the
company U/s 115-O.
(g) COA COA of shares in the company.
(h) Numerator index Index related to the PY in which company goes into liquidation.
(i) Denominator index Index related to the PY in which shares were acquired.
7 Manner of computing CG on account of transfer of CA distributed by the company in
liquidation.
(a) COA of acquisition of distributed capital asset in the FMV on the date of distribution by the
hands of shareholder company. [S. 55 (2) (b) (iii)].
(b) POH of such CA in the hands of shareholder Starts on the date of distribution by the
company.

(W) Reverse mortgage – tax implications:


1 The reverse mortgage scheme is for the benefit of senior citizens, who own a residential house
property.
2 In order to supplement their existing income, they can mortgage their house property with a
scheduled bank or housing finance company, in return for a lump-sum amount or for a regular
monthly or quarterly or annual income.
3 The senior citizen can continue to live in the house and receive regular income, without
botheration of having to pay back the loan.
4 The loan will be given up to, say, 60% of the value of the residential house property mortgaged. Also, the
bank or housing finance company would undertake a revaluation of the property once every 5 years.

MT Educare –CA Inter Income Tax Classes – R.SOUMYANARAYANAN.FCA. GRAD CWA. Page-155
Chapter-6 : Capital gains

5 The borrower can use the loan amount for renovation and extension of residential property, family’s
medical and emergency expenditure etc, amongst others. However, he cannot use the amount for
speculative or trading purpose.
6 The Reverse mortgage scheme, 2008 includes within its scope, disbursement of loan by an
approved lending institution, in part or in full, to the annuity sourcing institution, for the purposes
of periodic payments by way of annuity sourcing institution, for the purposes of periodic
payments by way of annuity to the reverse mortgagor.
7 This would be an additional mode of disbursement i.e. in addition to direct disbursements by the
approved lending institution to the reverse mortgagor by way of periodic payments or lumpsum
payment in one or more tranches.
8 Annuity sourcing institution = LIC or any other insurer registered with IRDA.
9 Maximum period of reverse mortgage loan
Mode of disbursement Maximum period of loan
(i) Where the loan is disbursed directly to reverse 20 years from the date of signing the
mortgagor agreement by the reverse mortgagor
and the approved lending institution.
(ii) Where the loan is disbursed, in part or in full, to the The residual life time of the
annuity sourcing institution for the purposes of borrower.
periodic payments by way of annuity to the Reverse
mortgagor
10 The bank will recover the loan along with the accumulated interest by selling the house after the
death of the borrower.
11 The excess amount will be given to the legal heirs.
12 However, before resorting to sale of the house, preference will be given to the legal heirs to repay
the loan and interest and get the mortgaged property released.
13 S. 47 (xvi) clarifies that any transfer of a CA in a transaction of reverse mortgage under a scheme
notified by the CG would not amount to a transfer for the purpose of CG.
14 S. 10 (43) provides that the amount received by the senior citizen as a loan, either in lumpsum or
in installments, in a transaction of reverse mortgage would be exempt from income-tax.
15 CG tax liability would be attracted only at the stage of alienation of the mortgaged property by
the bank or HFC for the purpose of recovering the loan.

Points requiring attention:


RM. Arunachalam (SC)
When the previous owner had mortgaged the property, then after his death the legal heir inherits only
the mortgagor’s interest in the property. By discharging the mortgage debt his heir who has inherited
the property acquires the interest of mortgagee in the property. As a result of such payment made for
clearing off the mortgage, the interest of the mortgagee is acquired by the heir. The said payment is,
therefore, regarded as COA.
Therefore, the COA to the legal heir is the aggregate of the cost to the previous owner and the amount
paid to clear the mortgage.
V. S. M. R. Jagadishchandran (SC)
Where the mortgage is created by the assessee himself, there will be no tax treatment of discharge of
mortgage debt and the same cannot be said to be COA or COI of the property.
Conclusion
There is distinction between the obligation to discharge mortgage debt created by previous owner and
the obligation to discharge the mortgage created by the assessee himself.

MT Educare –CA Inter Income Tax Classes – R.SOUMYANARAYANAN.FCA. GRAD CWA. Page-156
Chapter-6 : Capital gains

Where the property acquired by the assessee is subject to the mortgage created by the previous owner,
the assessee acquires absolute interest in that property only after the interest created in the property in
favour of the mortgage is transferred to the assessee, that is, after the discharge of mortgage debt. In
such case, the expenditure incurred by the assessee to discharge the mortgage debt created by the
previous owner to acquire absolute interest in the property is treated as cost of acquisition.
However, the expenditure incurred by the assessee to remove the encumbrance created by the assessee
himself on the property which was acquired by the assessee without any encumbrance is not allowable
as deduction U/s 48.
Aditya Kumar Jajodia (2018) (Cal)
The assessee obtained a leasehold property under a will which gave some interest to a trust deed and,
thus, the assessee’s acquisition of the perpetual lease was subject to rights of the trust as flowing from
the will. The testator of the trust had also entered into an agreement to sell with a third party. The
assessee had to, thus, perfect the ownership title before he transferred the property. For this purpose,
he made payment to DDA for conversion of leasehold rights to freehold rights. He also made payments
to the trust and to the third party to give up his right under the agreement.
These amounts are incurred by the assessee towards perfecting title of property acquired through will,
for making further sale and are to be included in the COA for computing CG.
(X) Depreciation & Capital gains:
(i) Classification of depreciable assets:
1 Meaning of Depreciable asset means asset in respect of which depreciation allowance
depreciable asset is quantified for claiming deduction U/s 32 (1).
2 Classification of Depreciable assets are classified into (a) tangible; (b) intangible.
depreciable assets
3 Tangible (a) Building; (b) furniture (assets for convenience or decoration); (c) Plant
depreciable assets & machinery.
4 Intangible (a) Patent; (b) know-how; (c) trade-mark; (d) copy-right; (e) licence; (f)
depreciable assets franchise; (g) any other business or commercial rights of similar nature.
5 Building includes (a) Roads laid within factory premises; [Gwalior Rayon (SC)]; (b)
bridges/culverts; (c) wells; (d) tube wells.
6 Furniture It means assets meant for convenience and decoration.
It includes electrical fittings such as (a) Switches; (b) wiring; (c) sockets; (d)
fans; (e) other electrical fittings.
7 Machinery It means assets used in manufacturing or production or processing of
goods or articles.
8 Plant It means any tool that is necessary for carrying on B/P.
It includes (a) Ships; (b) aircrafts; (c) vehicles; (d) books; (e) surgical
equipments; (f) scientific apparatus. [S. 43 (3)].
9 Plant excludes Livestock. [S. 43 (3)].
Building, furniture &fittings. [S.43 (3)]
10 Treatment of loss Loss on sale of animals used for the purpose of business or profession upon
on sale of animals their death or they becoming permanently useless is allowed as deduction
used for business. U/s 36 (1) (vi) while computing income U/H PGBP.

MT Educare –CA Inter Income Tax Classes – R.SOUMYANARAYANAN.FCA. GRAD CWA. Page-157
Chapter-6 : Capital gains

(ii) Judicial ruling in respect of depreciable assets:


1 Payments Intangible assets are invaluable assets which are required for carrying on the
made for business acquired by the assessee without interruption.
Goodwill In their absence, the assessee would have to commence the business from scratch
and go through the gestation period, whereas by acquiring these intangible assets
along with tangible assets, the assessee got a running business.
Thus, these intangible assets are, therefore, comparable to a license to carry on
the existing business of the transferor.
Therefore, payments made for securing goodwill are eligible for depreciation U/s
32 (1) (ii). [B. Raveendran Pillai (2011) (Ker) + Areva T and D India Ltd (2012) (Del)
+ Smifs Securities Ltd (SC) + Mis Bharti Teletech (2015) (Del).].
2 Abkari licence It is issued by the Kerala Government under the Foreign liquor Rules to the person
carrying on liquor trade. It is renewed every year unless a general policy decision
is taken against it. It is transferrable. It is, undoubtedly, an intangible depreciable
asset which is eligible for depreciation U/s 32 (1) (ii). [S. Ambika (2011) (Ker)].
3 Non-compete This is payment made for warding off or eliminating competition for a
fees substantially long period. It results in enduring benefit in capital field. Therefore,
it is a capital expenditure. It is a payment made for securing the right to operate
in competition-free environment. Thus, it is a payment made for securing a
business or commercial right. Therefore, it is eligible for depreciation U/s 32 (1)
(ii). [Medicorp Technologies India (P) Ltd (Chennai ITAT)].
4 Membership It is a licence to trade in the floor of BSE. Therefore, it is also eligible for
card in BSE depreciation U/s 32 (1) (ii).
5 Expenditure on Assessee was engaged in the business of ‘Aquaculture’. It grew prawns in specially
specially designed ponds. Assessee claimed depreciation on the expenditure incurred on
designed pond, treating it as plant. The claim of the assessee was upheld by the SC in Victory
ponds Aqua Farm Ltd.

(iv) Conditions to be fulfilled for the purpose of availing deduction in respect of


depreciation allowance:
1 Ownership Depreciable asset shall be owned by the assessee. The ownership may be
complete or partial.
2 Usage Depreciable asset shall be used for the purpose of business or profession
carried on by the assessee.

(v) Issues relating to ownership:


1 Beneficial ownership or economic ownership is suffice
Beneficial ownership = ‘Ownership’ for the purpose of S. 32 should be understood in a
Sufficient. broader sense. Beneficial or economic ownership is sufficient.
[Mysore Minerals Ltd (SC) + Podar Cements (P) Ltd (SC) + Smt.
A. Sivakami (Mad).].
2 Significance of ‘Owned To become entitled to depreciation, it not necessary that the
wholly or partly’ assessee should be a complete owner. Depreciation can be
allowed even on assets partially owned by the assessee, to the
extent of his share in such assets.

MT Educare –CA Inter Income Tax Classes – R.SOUMYANARAYANAN.FCA. GRAD CWA. Page-158
Chapter-6 : Capital gains

3 Depreciation on assets Firm can enjoy depreciation even on assets which are received
received by the firm by it from its partners by way of capital contribution so long as
from partners by way of these assets are used for the purpose of business or profession
capital contribution carried on by it. [Amber Corporation (Raj) + Manjeet
Engineering Industries (Del)].
4 Ownership of land not In case of buildings, the assessee must own the superstructure
prerequisite and not necessarily the land on which the building is
constructed.
Assessee can claim depreciation on building which he has
constructed in a lease hold land, provided the building is used
for the purpose of business or profession carried on by him.
[Revathi C. P. Equipments Ltd (Mad) + Chandra Agro (P) Ltd
(All)].
Lease premium paid for securing lease hold interest in the land
on which building which is used for business is constructed
cannot be added to the cost of construction of building for the
purpose of claiming depreciation. [Indian Oil Corporation
(Bom)].
5 Concept of deemed If the assessee is occupying any building as a tenant for the
building [Explanation-1 purpose of carrying on his B/P, any capital expenditure
to S. 32 (1)]. incurred towards renovation, extension or improvement to
such building can be treated as value of building belonging to
him and depreciation can be claimed on such amount.
Note: This is an exception to the rule that the depreciation is allowed only with respect to
the assets owned by the assessee. This is called the concept of deemed building, since the
capital expenditure incurred in construction etc of a structure in a building taken on lease
is regarded as a separate building in itself and is eligible for depreciation.
6 Depreciation on assets In case of assets purchased on installment basis, ownership
purchased on passes on to the assessee immediately. Even if the assessee
installment basis. [CBDT commits default in paying the installments, the asset cannot be
Circular 9 dated repossessed. Only suit can be filed for recovery of arrears of
23.03.1943]. installments. Hence, depreciation can be allowed on the entire
amount agreed to be paid to be paid as price.
7 Depreciation on assets Where the terms of agreement provide that the asset shall
purchased on hire eventually become the property of hirer or confer on the hirer
purchase basis an option to purchase an asset, the transaction should be
regarded as one of hire purchase. In such case, depreciation
shall be allowed on the cash price (i.e. the amount for which
the hired asset would have been sold for cash at the date of
agreement.
The difference between the aggregate of periodical payments
and the cash price (referred to as hire charges) shall be allowed
as deduction equally over the period of agreement.
8 Depreciation on assets In case of lease transaction, irrespective of whether it is finance
taken on lease lease or operating lease, depreciation shall be allowed only to
lessor and not to lessee, since the ownership is only with lessor.
[CBDT circular 2/2001].

MT Educare –CA Inter Income Tax Classes – R.SOUMYANARAYANAN.FCA. GRAD CWA. Page-159
Chapter-6 : Capital gains

If leasing of asset constitutes business, depreciation is to be


allowed U/s 32. Even if it doesn’t constitute business, it shall be
allowed as deduction U/s 57 (ii) while computing income U/H
IFOS.
ICDS Ltd (SC)
Facts Assessee interalia was engaged in the business of leasing & hire purchase. Assessee
purchased vehicles & leased them to customers who took physical possession and got the
vehicles registered in their name as owners under the Motor Vehicles Act. Assessee
claimed depreciation on these vehicles U/s 32.
Contention AO denied depreciation on the following grounds:
of AO (a) Assessee’s use of vehicles was only by way of leasing out the vehicles to others. There
was no actual user of vehicles in the business of running on hire.
(b) Customers were registered as owners under the Motor Vehicles Act. Assessee merely
financed the purchase of vehicles.
Decision (i) S. 32 requires that the asset must be used in the course of business. It does not mandate
of SC actual usage by the assessee itself.
(ii) Registration as owners under the Motor Vehicles Act is only for the purpose of law in
general. (i.e. for the purpose of establishing identity in case of traffic violation). Further,
registration in the name of lessee during the period of lease is mandatory as per Motor
Vehicles Act).
(iii) Therefore, no inference could be drawn from the registration certificate as to the
ownership of vehicles.
(iv) Lease agreement provides: (a) Assessee is the exclusive owner of vehicles at all points
of time; (b) Assessee is empowered to re-possess vehicles on default by the lessee; (c) At
the end of lease period, lessee is obliged to return the vehicle to the assessee; (d) Assessee
had a right of inspection of vehicles at all times.
Hence, assessee is entitled to depreciation.

(vi) Issues relating to usage:


1 Depreciation on building used Usage of building as staff quarters amounts to usage for
as staff quarters and the house- the purpose of business. Accordingly, the building,
hold appliances provided as a furniture and other house-hold appliances provided as a
part thereof. part of accommodation are eligible for depreciation.
[CBDT Circular 10/14/66 dated 12.12.19966].
2 Depreciation on temple Allowable. [Atlas Cycle Industries Ltd (P&H) + Associated
building constructed with the Flour Mills (P) Ltd (Gau)].
factory compound.
3 Depreciation on decoders given Allowable. [Turner International (P) Ltd (Del)].
to cable operators on loan.
4 Even usage of machine for trial [Ashima Syntax Ltd (Guj) + Mentha and allied products (All) +
run amounts to usage for the Vindhyachal Distilleries Ltd (MP) + Chamundeshwari Sugar Ltd
purpose of S. 32. (Kar) + Escorts Tractors Ltd (Del) + Kosha Cubidor Containers
Ltd (Guj)].
5 Passive user – enough
(a) The user of the asset should be understood in a wider sense so as to embrace passive as well
as active user.

MT Educare –CA Inter Income Tax Classes – R.SOUMYANARAYANAN.FCA. GRAD CWA. Page-160
Chapter-6 : Capital gains

(b) Specific nature of business of certain assessees requires them to keep stand-by-equipments
or spares.
(c) For example, additional boilers are kept as stand-by in factories driven by steam power. Stand-by
generators are kept by electricity supply companies. Spare engines are kept in stores by transport
companies. Fire extinguishers, projectors, visualizers, mike and sound system etc are held by MT
educare.
(d) When an asset is devoted to the needs of business, it is actually used for the needs of
business, as it is required for efficient conduct of business. Thus, the same qualifies for
depreciation U/s 32. [Pepsu Road Transport Corporation (P&H) + Shahbad Co-operative
Sugar Mills Ltd (2011) (P&H)].
(e) Likewise, machinery spares which can be used only in connection with an item of tangible
fixed asset and their use is expected to be irregular, has to be capitalised.
6 Whether depreciation is Yes. [Chennai Petroleum Corporation Ltd (2013) (Mad)].
available on assets which were
not put to use throughout the
relevant PY due to paucity of
raw materials?
7 Some machines forming part of U/s 32 (1) (ii), depreciation is allowed on WDV basis in
block are under repairs respect of block of asset and not in respect of individual
throughout the PY. assets.
Are these eligible for Therefore, the test of user should be applied not in
depreciation? respect of individual assets forming part of a block but in
respect of block as a whole.
If the block as a whole is used for the purpose of business,
though some assets being part of it were not put to use, it
does not affect the claim of depreciation.
8 Are discarded assets forming Depreciation is allowable on the WDV of the entire block,
part of block eligible for even though the block includes some machinery which
depreciation U/s 32 (1) (ii)? has already been discarded and hence, can’t be put to use
during the relevant PY.
The expression ‘used’ in S. 32 in respect of discarded
machinery would mean the use in the business, not in the
relevant PY, but in earlier PYs. [Yamaha Motor India (P)
Ltd (2010) (Del)].
Even if the Department wants to disallow depreciation
attributable to discarded machines, there is no
computational mechanism available in this regard. Asset
added to the block loses its identity.

(vii) Restriction of the quantum of depreciation U/s 32 (1) Proviso-2:

1 To avail depreciation, the asset need not be put to use throughout the year.
2 Even if the asset is put to use or made available for use during a part of the PY, the
assessee would be entitled to claim depreciation for the whole year.
3 However, an exception is contemplated in the 2nd proviso to S. 32 (1).

MT Educare –CA Inter Income Tax Classes – R.SOUMYANARAYANAN.FCA. GRAD CWA. Page-161
Chapter-6 : Capital gains

4 If the asset is put to use for a period of less than 180 days in the PY in which it is acquired,
the deduction U/s 32 shall be restricted to 50% of the amount of depreciation.
5 The aforesaid restriction is applicable only in the year in which the asset is acquired and
not in subsequent years.
6 In other words, in the subsequent years, full deduction will be allowed with respect to
depreciation, even if the asset is put to use for a period of less than 180 days.
7 In the PY of transfer of depreciable asset, assessee is not eligible for depreciation.

Illustration:
Asset is purchased on 01.01.2020 for Rs. 1L. Depreciation is allowable @ 15%. Asset is
installed and put to use on (a) 01.01.2020; (b) 31.12.2020. Determine the quantum of
depreciation deductible for the AY 2020-21 and AY 2021-22.
AY Particulars Case-1 (DOU = 01.01.2020) Case-2 (DOU = 31.12.2020)
20-21 Depreciation Rs. 100000 * 15% * 50% = Rs. 7500 No deduction in respect of
[Restricted U/s 32 (1) proviso-2] depreciation
21-22 Depreciation Rs. 92500 * 15% Rs. 100000 * 15% = Rs. 15000 [S.
32 (1) Proviso-2 shall not apply]

(viii) Used partly for business purpose - (S. 38 (2)):


1 Where the building, machinery, plant or furniture is not exclusively used for the purpose of business
or profession (say, the assets are partly used for business or professional purpose and partly for
personal purpose), the assessee will not get deduction with respect to the full depreciation
allowance.
2 In view of S. 38 (2), the AO will restrict the depreciation allowance to a fair proportionate part having
regard to the user of the assets for the purpose of business or profession.

Illustration:
Mr. X purchased a motor car (eligible for depreciation @ 15%) for Rs. 100000 on 01.04.2019.
It is used both for professional purpose as well as personal purposes. 30% of usage of car is
for personal purposes. Determine the quantum of depreciation allowance deductible for the
AY 2020-21 and AY 2021-22.

AY 2020-21:
1 Actual cost of car Rs. 100000
2 Depreciation @ 15% Rs. 15000
3 Disallowed for personal usage U/s 38 (2) @ 30% Rs. 4500
4 Deductible depreciation allowance (2-3) Rs. 10500
5 WDV as on 01.04.2020 (1-4) Rs. 89500

AY 2021-22:
1 WDV as on 01.04.2020 Rs. 89500
2 Depreciation @ 15% Rs. 13425
3 Disallowed for personal usage U/s 38 (2) @ 30% Rs. 4028
4 Deductible depreciation allowance (2-3) Rs. 9398
5 WDV as on 01.04.2021 (1-4) Rs. 80103

MT Educare –CA Inter Income Tax Classes – R.SOUMYANARAYANAN.FCA. GRAD CWA. Page-162
Chapter-6 : Capital gains

Note:
WDV in case of assets acquired before the PY means the actual cost to the assessee less all
depreciation actually allowed to him under this Act. [S. 43 (6)].

Sayaji Iron and Engineering co (Guj):


1 Assessee-company provided use of it car for the personal purposes of its directors pursuant to the
terms of their appointment. It is provided as perquisites for the services rendered by them.
2 The car shall be regarded as used for the purpose of business or profession carried on by the
assessee. Accordingly, the fuel expenses, maintenance, repairs, insurance, driver’s salary and
depreciation shall be allowed as deduction.
3 Provisions of S. 38 (2) can’t be invoked to disallow the depreciation. Similarly, the provisions of S.
37 (1) can’t be invoked to disallow these expenses U/s 37 (1) as personal expenses.
(ix) Computation of depreciation:
Classification of depreciable assets for the purpose of computation of depreciation:
For the purpose of computation of depreciation, depreciable assets are divided into two:
(i) Depreciable assets Tangible DA acquired on or after 01.04.1997 which are used by
covered by S. 32 (1) (i). power generating units and in respect of which option U/R 5 (1A) has
not been exercised to depreciate them U/s 32 (1) (ii) are covered by
S. 32 (1) (i).
(ii) Depreciable assets All depreciable assets (which are not covered by S. 32 (1) (i)) are
covered by S. 32 (1) (ii). covered by S. 32 (1) (ii).

Points requiring attention:


1 Option referred to U/R 5 (1A) shall be exercised on or before the due date for filing return
for the AY relevant to the PY in which power generation commences.
2 Once the option is exercised, it shall be final and shall apply to all the subsequent years.

Purpose of classification of depreciable assets as those covered by S. 32 (1) (i) & (ii):
SN DA covered by S. 32 (1) (i) DA covered by S. 32 (1) (ii)
1 These assets are depreciated on an asset to These assets are depreciated on block basis.
asset basis.
2 Straight line method is used for computation of WDV method is used for computation of depreciation.
depreciation.
3 For these assets depreciation rates are given in For these assets depreciation rates are given in
Appendix-IA of IT Rules. Appendix-I of IT Rules.
4 Gain or loss on transfer of these assets are Gain or loss on transfer of these assets are treated U/s
treated U/s 32 (1) (ii), S. 41 (2) and S. 50A. 50.
5 These assets are not eligible for additional These assets may be eligible for additional depreciation
depreciation U/s 32 (1) (iia) U/s 32 (1) (iia) subject to conditions stipulated therein.

Computation of depreciation in respect of assets covered by S. 32 (1) (i):


SN Particulars Amount
1 Actual cost ****
2 Depreciation rate (as per Appendix-1A of IT Rules) **
3 Depreciation (1*2) ***

MT Educare –CA Inter Income Tax Classes – R.SOUMYANARAYANAN.FCA. GRAD CWA. Page-163
Chapter-6 : Capital gains

Tax implications on account of transfer of depreciable assets covered by S. 32 (1) (i):


SN Situation Treatment
1 Moneys payable > Original cost Moneys payable – Original cost = CG; Taxable in the PY of
transfer.
Original cost – Opening WDV = Balancing charge (which is
charged to tax U/H PGBP in the PY in which moneys payable
becomes due – Vide S. 41 (2)).
2 Moneys Moneys Moneys payable – Opening WDV = Balancing charge (which is
payable < payable > charged to tax U/H PGBP in the PY in which moneys payable
Original cost Opening WDV becomes due – Vide S. 41 (2)).
Question of computing CG does not arise.
3 Moneys Moneys Opening WDV – Moneys payable = Terminal depreciation. It
payable < payable < is allowed as deduction U/s 32 (1) (iii) while computing
Original cost Opening WDV income U/H PGBP in the PY of transfer.
Question of computing CG does not arise.

Note:
Balancing charge is to be taxed in the PY in which moneys payable becomes due U/H PGBP
even though the business does not exist in that PY.

Meaning of moneys payable:


SN Situation Moneys payable
1 Asset is sold Net sale proceeds
2 Asset gets destroyed or demolished Insurance compensation
3 Asset is discarded Realisable scrap value

Format for computing CG in case of transfer of assets covered by S. 32 (1) (i):


SN Particulars Amount
1 FVC ****
2 Expenses incurred in relation to transfer (***)
3 Net consideration (1-2) ****
4 COA (= Adjusted WDV) (S. 50A) ***
5 STCG (deemed) (3-4) ****

Meaning of Adjusted WDV:


SN Situation Adjusted WDV
1 Where terminal depreciation arises Opening WDV – Terminal depreciation
2 Where balancing charge arises Opening WDV + Balancing charge

Test your knowledge:


X Ltd is a power generating unit. On 20.12.2018, it purchases a plant for Rs. 20L which is
eligible for depreciation @ 12.77% on straight-line basis. The plant is sold for (a) Rs. 30,000 or
(b) Rs. 19L or (c) Rs. 21.50L on 20.05.2019. Compute terminal depreciation/balancing
charge/CG.

MT Educare –CA Inter Income Tax Classes – R.SOUMYANARAYANAN.FCA. GRAD CWA. Page-164
Chapter-6 : Capital gains

Depreciable assets covered by S. 32 (1) (ii):


In respect of DA covered by S. 32 (1) (ii), both depreciation as well as capital gains are
computed only in respect of block of asset and not in respect of individual assets forming part
of block.

Concept of block of asset [S. 2 (11)]:


(i) The term ‘block of assets’ is defined in S. 2 (11). It means group of assets falling under
a class of asset being building, plant, machinery, furniture, know-how, patent, trade
mark etc having same rate of depreciation.
(ii) Thus, a group of assets will be regarded as a block of assets if the following cumulative
conditions are satisfied:
1 The assets which form the group fall within a class of assets mentioned above.
2 All assets within this group have the same rate of depreciation.

Depreciation in respect of depreciable assets covered by S. 32 (1) (ii): S. 43 (6):


1 Opening WDV of the block ****
2 Actual cost of assets acquired during the PY and falling within this block ****
3 ‘Moneys payable’ in respect of any asset in the block which is sold, ****
discarded, demolished or destroyed, together with the scrap value, if any.
4 WDV for the purpose of depreciation (1+2-3) ****
5 Depreciation (4*prescribed rate of depreciation) ****
6 Closing WDV of the block (4-5) ****

Points requiring attention:


1 Netting off Where an asset belonging to the block is sold, in computing the monies
expenses in payable, we should not take the take the gross consideration received
connection from the assessee. All expenses incidental to sale should be reduced
with transfer from the gross consideration.
2 Negative If the WDV for the purpose of depreciation is a negative figure, it shall
WDV be taken as nil.

Situations under which no depreciation is admissible:


1 WDV of the block for the purpose of depreciation = 0
2 Block is empty (That is, none of the assets belonging to the block exists on the last day of the PY).

Depreciation rates as per Appendix-I of IT Rules:


Buildings
Block-1 Buildings which are used mainly for residential purpose 5%
Block-2 Buildings which are not used mainly for residential purposes and not covered 10%
by Block-1 and Block-3
Block-3 Buildings acquired on or after 01.09.2002 for installing machinery and plant 40%
forming part of water supply project or water treatment system and which is
put to use for the purpose of business of providing infrastructure facilities
Block-4 Purely temporary erections such as wooden structures 40%

MT Educare –CA Inter Income Tax Classes – R.SOUMYANARAYANAN.FCA. GRAD CWA. Page-165
Chapter-6 : Capital gains

Furniture and fittings including electrical fittings 10%

P&M
Block-1 Motor buses, motor lorries, motor taxis used in the business of running them on hire 45%
acquired during 23.08.2019 to 31.03.2020 and put to use on or before 31.03.2020.
Block-2 Motor buses, motor lorries, motor taxis used in the business of running them on hire 30%
(other than those mentioned above)
Block-3 Motor cars other than those used in the business of running them on hire acquired 30%
during the period from 23.08.2019 to 31.03.2020 and put to use on or before
31.03.2020.
Block-4 Motor cars other than those used in the business of running them on hire (other than 15%
those mentioned above).
Block-5 Aeroplanes & Aeroengines 40%
Block-6 Specified air, water pollution control equipments, solid waste control equipment and 40%
solid waste recycling and resource recovery systems
Block-7 Renewable energy savings devices
Windmills and specifically designed devices which run on windmills installed on or 40%
after 01.04.2014
Any devices including electric generators and pumps running on wind energy installed 40%
on or after 01.04.2014
Windmills and specifically designed devices which run on windmills installed on or 15%
before 31.03.2014
Any devices including electric generators and pumps running on wind energy installed 15%
on or before 31.03.2014
Block-8 Computers including computer software 40%
Block-9 Annual publications owned by assessees carrying on a profession 40%
Block-10 Books owned by assessees carrying on business in running lending libraries 40%
Block-11 Books, (annual publications or other than annual publications), owned by assessees 40%
carrying on a profession
Block-12 Life-saving medical equipments 40%
Block-13 Plant & Machinery (General rate) 15%
Block-14 Ocean-going Ships 20%
Block-15 Vessels ordinarily operating on inland waters 20%
Block-16 Speed boats operating on inland water 20%
Block-17 Moulds used in rubber and plastic goods factory 30%
Block-18 Plant and machinery used in semi-conductor industry covering all Integrated Circuits 30%
Block-19 Machinery and plant acquired and installed on or after 01.09.2002 in a of water supply 40%
project or water treatment system and which is put to use for the purpose of business
of providing infrastructure facilities
Block-20 Oil wells 15%

Intangible depreciable assets 25%

MT Educare –CA Inter Income Tax Classes – R.SOUMYANARAYANAN.FCA. GRAD CWA. Page-166
Chapter-6 : Capital gains

Note:
Any new machinery or plant installed to manufacture or produce any article or thing by using
any technology or other know-how developed in a laboratory owned or financed by the
Government or a laboratory owned by a public sector company or a University or an
institution recognised by the Secretary, Department of Scientific & Industrial Research, GOI,
shall be treated as a part of block of asset qualifying for depreciation @ 40%.

Illustration:
Details regarding Block (P&M-15%) of X Ltd for the AY 2020-21 are given as under:
SN Name of P&M DOA Date of installation COA
1 A-B-C Rs. 100L
D 01.01.2019 01.01.2020 Rs. 20L
2 E 01.07.2019 01.07.2019 Rs. 40L
3 F 01.08.2019 01.02.2020 Rs. 40L
4 G 01.09.2019 - Rs. 40L
Plant-A is sold for Rs. 41L on 01.12.2019. Expenses incurred in relation to transfer = Rs. 1L.
Determine the depreciation for the block for the AY 2020-21.

Determination of depreciation U/s 32 (1) (ii):


SN Particulars Amount
1 Opening WDV of the block Rs. 120L
2 Actual cost of Plant-E Rs. 40L
3 Actual cost of Plant-F Rs. 40L
4 Actual cost of Plant-G Rs. 40L
5 Total (1+2+3+4) Rs. 240L
6 Moneys payable on account of sale of Plant-A Rs. 40L
7 WDV as on 31.03.2020 (5-6) Rs. 200L
8 Depreciation U/s 32 (1) (ii) [See note below] Rs. 21L
9 WDV as on 01.04.2020 [7-8] Rs. 179L

Note:
SN Particulars Computation Depreciation
1 WDV attributable to E Rs. 40L *15% Rs. 6L
2 WDV attributable to F Rs. 40L * 15% * 50% Rs. 3L
3 WDV attributable to G Rs. 40L *0% -
4 Balance WDV Rs. 80L *15% Rs. 12L
5 Total depreciation Rs. 21L

Computation of CG on a/c of transfer of depreciable assets covered by S. 32 (1) (ii) – S. 50:


When S. 50 (1) can be invoked?
S. 50 (1) can be invoked if the following conditions are satisfied:
1 The capital asset transferred should be a depreciable asset.
2 It shall belong to a block to which depreciation has been allowed in earlier years.
3 The block is not empty after the sale of this capital asset. In other words, the block exists at the end of the PY.
4 Sale consideration of the asset transferred > [Opening WDV of the block + cost of additions + expenses wholly
and exclusively for the purpose of transfer].

MT Educare –CA Inter Income Tax Classes – R.SOUMYANARAYANAN.FCA. GRAD CWA. Page-167
Chapter-6 : Capital gains

Computation of CG U/s 50 (1):


1 FVC of the asset transferred ****
2 Expenses wholly and exclusively for the purpose of transfer ****
3 Opening WDV of the block ****
4 Cost of assets added to the block during the PY ****
5 STCG (1-2-3-4) ****

Computation of CG U/s 50 (2):


The CG shall be computed as under:
1 FVC of the assets transferred ****
2 Expenses wholly and exclusively for the purpose of transfer ****
3 Opening WDV of the block ****
4 Cost of assets added to the block during the PY ****
5 STCG/STCL (1-2-3-4) ****

Test your knowledge:


1 On April 1, 2019, depreciated value of a block of assets (rate of depreciation: 15%) is Rs. 80,000. It
consists of Plants A and B. The assessee purchases Plant C (rate of depreciation: 15%) during the
PY 2019-20 for Rs. 30,000 and sells Plant A on May 3, 2019 for Rs. 1,80,000. Discuss the tax
implications.
2 X Ltd owns two plants – Plant A and Plant B – on April 1, 2019 (rate of depreciation: 15%,
depreciated value on April 1, 2019: Rs. 2, 37,000). The company purchases Plant C on May 31, 2019
for Rs. 20,000 and sells Plant A (on April 10, 2019), Plant B (on December 12, 2019) and Plant C (on
March 1, 2020) for Rs. 10,000, Rs. 15,000 and Rs. 24,000, respectively. Discuss the tax implications.
3 X Ltd. owns two buildings A and B on April 1, 2019 (rate of depreciation: 10%, depreciated value:
Rs. 14, 15,700). It purchases on December 1, 2019 building C for Rs. 3, 10,000 (rate of depreciation:
10%) and sells building A during the PY 2019-20 (say on January 10, 2020) for Rs. 8,70,000. Discuss
the tax implications. Will your answer be different if it is sold for Rs. 15, 87,000?
4 You are given the following details regarding a block (P&M – 15%). Discuss the tax implications.
(i) Opening WDV of block as on 01.04.2019 (Assets = A, B, C, D & E) Rs. 2L
(ii) Asset-F acquired on 30.06.2019 Rs. 1L
(iii) Asset-A destroyed in fire on 31.12.2019 & Insurance compensation received is Rs. 1.5L
5 You are given the following details regarding a block (P&M – 15%). Discuss the tax implications.
(i) Opening WDV of block as on 01.04.2019 (Assets = A, B, C, D & E) Rs. 2L
(ii) Asset-F acquired on 30.06.2019 Rs. 1L
(iii) Asset-A destroyed in fire on 31.12.2019 & Insurance compensation Rs. 7L
received is
6 You are given the following details regarding a block (P&M – 15%). Discuss the tax implications.
(i) Opening WDV of block as on 01.04.2019 (Assets = A, B, C, D & E) Rs. 2L
(ii) Asset-F acquired on 30.06.2019 Rs. 1L
(iii) All assets are destroyed in fire on 01.01.2020 & Insurance compensation received Rs.
is 1.5L

MT Educare –CA Inter Income Tax Classes – R.SOUMYANARAYANAN.FCA. GRAD CWA. Page-168
Chapter-6 : Capital gains

(x) Additional depreciation – S. 32 (1) (iia):


Eligibility
(i) In addition to normal depreciation allowed U/s 32 (1) (ii), an assessee is entitled to get additional
depreciation u/s 32 (1) (iia) in the year in which the asset is first put to use. It is not allowed in
the subsequent years. In other words, it is a one-time benefit.
(ii) However, the additional depreciation is available only upon fulfillment of the following
conditions:
1 The assessee must be engaged in manufacture or production of an article or thing or
generation or transmission or distribution of power.
2 The assessee should purchase and install a new P&M.
3 Such plant and machinery should be an eligible P&M.

Quantum of additional depreciation


(i) The quantum of additional depreciation is 20% of the actual cost of the eligible P&M.
(ii) In the year of acquisition, if the P&M is put to use only for a period of less than 180 days, the
additional depreciation should be computed at the rate of 10%. This is because the 2nd proviso
to S. 32 (1) provides so.
(iii) The unallowed additional depreciation shall be allowed in the immediately succeeding PY.
[Proviso-3 to S. 32 (1)].
(iv) However, if the P&M is first put to use in a year subsequent to the year of acquisition, the 2 nd
proviso to S. 32 (1) shall not apply. That is, the additional depreciation is to be computed at the
rate of 20%.
P&M not eligible for additional depreciation
1 Ships
2 Aircrafts
3 Any machinery or plant which, before the installation by the assessee, was used either within
or outside the India by any other person
4 Any machinery or plant which is installed in any office premises or any residential
accommodation or accommodation in the nature of a guest house.
5 Any office appliances or road transport vehicles.
6 Any machinery or plant, the whole of the actual cost of which is allowed as a deduction (whether
by way of depreciation or otherwise) in computing the income chargeable U/H PGBP of any one
PY.

Points requiring attention:


1 No additional depreciation is available in respect of building or furniture even if the aforesaid
conditions are satisfied.
2 No additional depreciation is available in respect of old plant and machinery.
3 Additional depreciation is available on computers used for software development, since it is
regarded as manufacture or production of article or thing. [Statronics & Enterprises (P) Ltd (Guj)].
4 Even eligible P&M forming part of power generating unit, the power generated from which is
captively consumed for running the factory, is eligble for additional depreciation provided the
option U/R 5 (1A) was exercised to bring those assets U/s 32 (1) (ii). [Texmo Precision Castings (Mad)
+ VTM Ltd (Mad) + Hi-tech Arai Ltd (Mad)].

MT Educare –CA Inter Income Tax Classes – R.SOUMYANARAYANAN.FCA. GRAD CWA. Page-169
Chapter-6 : Capital gains

CBDT Circular 15/2016:


Whether or not an assessee engaged in printing or printing and publishing is eligible for
grant of additional depreciation U/s 32 (1) (iia)?
(I) Printing or printing and publishing amounts to manufacturing activity.
(ii) Assessee engaged in printing or printing and publishing is eligible for grant of
additional depreciation U/s 32 (1) (iia).

Points requiring attention:


1 Does fork-lift
Fork-lift-truck used inside the factory would not fall within the definition of
truck eligible for ‘vehicle’. Hence, it is eligible for additional depreciation U/s 32 (1) (iia).
additional As per S. 2 (28) of Motor Vehicles Act 1988, the definition of a vehicle excludes
depreciation? interalia a vehicle of special type adapted for use only in a factory or in any
enclosed premises. Therefore, fork-lift-truck used within factory ≠ Vehicle.
2 Computers Computers used in factory or in production system are eligible for additional
installed at depreciation U/s 32 (1) (iia). These are not to be viewed as additional
factory. depreciation. These are integral part of production system.
3 WDV of the block The sum total of normal depreciation and additional depreciation can’t exceed
can’t be negative. the WDV of the block and make it negative. In such a case, the additional
depreciation shall be restricted to such figure which makes WDV of the block
nil.

(xi) Enhanced additional depreciation – S. 32 (1) (iia) Proviso-1:


If the following conditions are fulfilled, additional depreciation shall be calculated @ 35% on
the actual cost:
(i) Assessee sets up an undertaking or enterprise for manufacture or production of an article or thing,
on or after 01.04.2015 in any backward area notified by the CG in Andhra Pradesh, Telangana,
Bihar or West Bengal.
(ii) Assessee acquires and installs any eligible machinery or plant for the purposes of said undertaking
or enterprise during 01.04.2015 to 31.03.2020 in the said backward area.

Points requiring attention:


(i) Assessee may be a corporate assessee or a non-corporate assessee.
(ii) If the assessee is owning an undertaking or enterprise which is engaged in manufacturing or
production in such backward area even before 01.04.2015, additional depreciation shall be
computed only at the rate of 20%.
(iii) If the assessee sets up an undertaking or enterprise in the aforesaid backward areas to carry on
the business of generation or transmission or distribution of power on or after 01.04.2015, still
additional depreciation is to be computed only at the rate of 20%.
(iv) If, in the PY of acquisition, the eligible plant or machinery was put to use for less than 180 days,
then the enhanced additional depreciation is to be calculated at the rate of 17.5%. [In view of S.
32 (1) Proviso-2].
(v) The unallowed additional depreciation shall be allowed in the immediately succeeding PY.
[Proviso-3 to S. 32 (1)].

MT Educare –CA Inter Income Tax Classes – R.SOUMYANARAYANAN.FCA. GRAD CWA. Page-170
Chapter-6 : Capital gains

(xii) Additional investment allowance – S. 32AD:


S. 32 Applicability Assessee (corporate or non-corporate) sets up an undertaking or
AD (1) enterprise for manufacture or production of an article or thing, on or after
01.04.2015 in any backward area notified by the CG in Andhra Pradesh,
Telangana, Bihar or West Bengal.
Assessee acquires and installs any eligible machinery or plant for the
purposes of said undertaking or enterprise during 01.04.2015 to
31.03.2020 in the said backward area.
Benefit provided While computing income U/H PGBP, an additional investment allowance is
under this section available as deduction.
Quantum of 15% of actual cost of eligible plant or machinery.
deduction
PY of deduction PY in which eligible plant or machinery is installed.
Additional benefit Benefit under this section is in addition to the benefit of additional
depreciation @ 35%.
Quantum unrestricted The quantum of additional investment allowance is not restricted U/s 32
(1) Proviso-2. In the PY of acquisition, even if the eligible plant or machinery
was put to use for less than 180 days, additional investment allowance is
not restricted to 50%.
Block value - not Additional investment allowance does not reduce the block value (unlike
affected additional depreciation).
Eligible plant or Same meaning as in additional depreciation.
machinery
Assessees in power Not entitled to benefit U/s 32AD.
sector
S. 32 Lock-in-period The eligible plant or machinery shall not be transferred within 5
AD (2) years from the date of its installation.
& (3) Violation of lock-in- If the lock-in-period condition is violated, then (a) the amount of
period condition deduction allowed U/s 32AD (1) in respect of eligible plant or
(not on account of machinery shall be deemed to be income of the assessee chargeable
business re- U/H PGBP of the PY in which the eligible plant or machinery is sold
organisation) or transferred; (b) STCG shall also be computed within the
parameters of S. 50.
Violation of lock-in- If the lock-in-period condition is violated on account of (a)
period on account of amalgamation; or (b) demerger; or (c) conversion of firm into
business re- company; or (d) conversion of sole proprietary concern into
organisation company; or (e) conversion of company into LLP, then the
amalgamated company, resulting company or company or LLP, as
the case may be shall not transfer the eligible plant or machinery
within the aforesaid 5 years.

If it is transferred, then the deduction allowed U/s 32AD (1) in


respect of eligible plant or machinery (in the hands of amalgamating
company or demerged company or firm or SPC or company) shall be
deemed to be income of the amalgamated company or resulting
company or company or LLP in the PY in which the eligible plant or
machinery is sold. In addition, STCG is computed within the
parameters of S. 50.

MT Educare –CA Inter Income Tax Classes – R.SOUMYANARAYANAN.FCA. GRAD CWA. Page-171
Chapter-6 : Capital gains

Notified backward areas:


SN State Notified backward areas
1 Telangana Adilabad, Nizamabad, Karimnagar, Warangal, Medak, Mahbubnagar, Rangareddy,
Nalgoda, Khammam.
2 West Bengal South 24 Parganas, Bankura, Birbhum, Dakshin Dinajpur, Uttra Dinajpur,
Jalpaiguri, Malda, East Medinipur, West Medinipur, Murshidabad, Purulia.
3 Bihar Patna, Nalanda, Bhojpur, Rohtas, Kaimur, Gaya, Jehanabad, Aurangbad, Nawada,
Vaishali, Samastipur, Darbhanga, Madhubani, Purnea, Katihar, Araria, Jamui,
Lakhisarai, Supaul, Muzaffarpur, SheoharArwal, Banka, Begusarai, Bhagalpur,
Buxar, Gopalganj, Khagaria, Kishanganj, Madhepura, Munger, West Champaran,
East Champaran, Saharsa, Saran, Sheikpura, Sitamarhi, Siwan.
4 AP Anantapur, Chittor, Cuddapah, Kurnool, Srikakulam, Vishakhapatnam,
Vizianagaram.

Previous examination question:


Question No 8: Mr. X sets up a manufacturing unit in Warangal in the state of Telangana on
01.06.2019. It invested Rs. 30 Crores in new P&M on 01.06.2019. Further, it invested Rs. 25
Crore in the plant and machinery on 01.11.2019, out of which Rs. 5 Crore was second hand
P&M. Compute the depreciation allowable U/s 32. Is Mr. X entitled for any other benefit in
respect of such investment? If so, what is the benefit available?

Answer
Computation of depreciation U/s 32 for X Ltd for AY 2020-21:
Particulars (Rs. in Crores)
P&M acquired on 01.06.2019 30.000
P&M acquired on 01.11.2019 25.000
WDV as on 31.03.2020 55.000
Less: Depreciation @ 15% on Rs. 30 crore = Rs. 4.500 Crores
Additional Depreciation @ 35% on Rs. 30 crore = Rs. 10.500 Crores
Additional Depreciation @ 17.5% (50% of 35%) on Rs. 20 crore = Rs. 3.500 Crores 20.375
WDV as on 01.04.2020 34.625

Computation of deduction U/s 32AD for X Ltd. for A.Y. 2020-21:


Particulars (Rs. in Crores)
Deduction U/s 32AD @ 15% on Rs. 50 crore 7.50
Total benefit 7.50
Notes:
1 As per the proviso-2 to S. 32 (1) (ii), where an asset acquired during the PY is put to use for less
than 180 days in that PY, the amount deduction allowable as normal depreciation and additional
depreciation would be restricted to 50% of amount computed in accordance with the prescribed %.
Therefore, normal depreciation on P&M acquired and put to use on 1.11.2019 is restricted to 7.5%
(being 50% of 15%) and additional depreciation is restricted to 17.5% (being 50% of 35%).
2 The balance additional depreciation of Rs. 3.5 crore, being 50% of Rs. 7 crore (35% of Rs. 20 crore)
would be allowed as deduction in the AY 2021-22.

MT Educare –CA Inter Income Tax Classes – R.SOUMYANARAYANAN.FCA. GRAD CWA. Page-172
Chapter-6 : Capital gains

3 As per S. 32 (1) (iia), additional depreciation is allowable in the case of any new machinery or
plant acquired and installed by an assessee engaged, inter alia, in the business of manufacture or
production of any article or thing. In this case, since new P&M acquired was installed by a
manufacturing unit set up in a notified backward area in the State of Telengana, the rate of additional
depreciation is 35% of actual cost of new P&M. Since P&M of Rs. 20 crore was put to use for less than
180 days, additional depreciation @ 17.5% (50% of 35%) is allowable as deduction. However,
additional depreciation shall not be allowed in respect of second hand P&M of Rs. 5 crore.
4 Likewise, the benefit available U/s 32AD would not be allowed in respect of second hand P&M.
5 Accordingly, additional depreciation and investment allowance U/s 32AD have not been provided on
Rs. 5 crore, being the actual cost of second hand P&M acquired and installed in the PY.

Question No 9 :Compute the quantum of depreciation available U/s 32 in respect of the


following items of P&M purchased by PQR Textile Ltd., by paying through APC, which is
engaged in the manufacture of textile fabrics, for the year ended 31-3-2020:
(Rs. In Crores)
New machinery installed on 1-5-2019 84
New Windmill purchased and installed on 18-6-2019. 22
Items purchased after 30th November 2019:
Lorries for transporting goods to sales depots 3
Fork-lift-trucks, used inside factory 4
Computers installed in office premises 1
Computers installed in factory 2
New imported machinery 12

Answer:

Particulars Normal Additional


Depreciation Depreciation [u/s
[u/s 32(1)(ii)] 32(1)(iia)]
(` in Crores)
(A) P&M (15% block) (Put to use for 180 days or more)
- New machinery installed on 01.05.2019 84.00 84.00
Normal Depreciation @ 15% & additional 12.60 16.80
deprecation @ 20%
(B) P&M (15% block) (Put to use for less than 180
days – hence, depreciation is restricted to 7.5%, being
50% of 15%)
- Lorries for transporting goods to depots 3.00 -
- Fork-lift trucks, used inside a factory 4.00 4.00
7.00 4.00
Normal Depreciation @ 7.5% & additional 0.53 0.40
depreciation @ 10%
(C) P&M (40% block) (Put to use for less than 180 days,
hence depreciation restricted to 20%, i.e., 50% of 40%)
- Computers installed in office premises 1.00 -
- Computers installed in factory 2.00 2.00

MT Educare –CA Inter Income Tax Classes – R.SOUMYANARAYANAN.FCA. GRAD CWA. Page-173
Chapter-6 : Capital gains

Normal depreciation @ 20% & additional


depreciation @ 10%
(D) P&M (40% block) (Put to use for 180 days or more) (See
Note 1)
- New windmill purchased and installed on 18.06.2019 22.00 22.00
Normal Depreciation @ 40% & additional
depreciation @ 20% 8.80 4.40
Total depreciation and additional depreciation
- Plant and Machinery (15% block) (A +B) 13.13 17.20
- Plant and Machinery (40% block) (C + D) 9.40 4.60
Depreciation available U/s 32 = Rs. 44.33 Crores

Computation of WDV as on 01.04.2020


Particulars P&M
15% 40%
(Rs. in Crores)
WDV as on 01.04.2019 Nil Nil
Add: P&M acquired during the year
New Machinery installed on 01.05.2019 84.00
Lorries for transporting goods to sales depots 3.00
Fork-lift trucks, used inside factory 4.00
New imported machinery 12.00 103.00 -
New Windmill purchased and installed on 18.6.2019 - 22.00
Computers installed in office premises - 1.00
Computers installed in factory - 2.00
103.00 25.00
Less: Asset sold during the year Nil Nil
WDV as on 31.3.2020 (before charging depreciation) 103.00 25.00
Less: Depreciation for the PY 2019-20
- Normal depreciation 13.13 9.4
- Additional depreciation 17.20 4.60
WDV as on 1.4.2020 72.67 11.00

Notes:
1 Windmills and any specially designed devices which run on windmills installed on or after 1.4.2014
would be eligible for depreciation @ 40%.
2 New imported machinery was not installed during the PY 2019-20. Hence, it would not be eligible
for additional depreciation for AY 2020-21. It would also not be eligible for normal depreciation for
AY 2020-21, since it was not put to use in the PY 2019-20 being the year of acquisition.
3 It may be noted that investment in the following P&M would not be eligible for additional
depreciation U/s 32 (1) (iia): (a) Lorries for transporting goods to sales depots, being vehicles/road
transport vehicles; and (b) Computers installed in office premises.
4 As per S. 2(28) of the Motor Vehicles Act, 1988, the definition of a “vehicle” excludes, inter alia, a
vehicle of special type adopted for use only in a factory or in any enclosed premises. Therefore,
fork-lift trucks used inside the factory would not fall within the definition of “vehicle”. Hence, it is
eligible for additional deprecation U/s 32 (1) (iia).

MT Educare –CA Inter Income Tax Classes – R.SOUMYANARAYANAN.FCA. GRAD CWA. Page-174
Chapter-6 : Capital gains

(xiii) Depreciation in case of succession in business:


(i) On account of amalgamation, demerger, conversion of firm into company, conversion of SPC
into company or conversion of company into LLP, there will be succession in business.
(ii) Income from such business up to the date of succession shall be charged to tax in the hands of
predecessor (i.e. amalgamating company, demerged company, firm or SPC or company).
Income thereafter shall be charged to tax in the hands of successor (i.e. amalgamated company,
resulting company, company or LLP). [S. 170 (1)].
(iii) Depreciation on block of assets shall be computed as if there is no succession and it shall be
apportioned between the predecessor and successor based on the number of days of usage of
assets.

Test your knowledge:


Mr. Gopi carrying on business as proprietor converted the same into a limited company by
name Gopi Pipes (P) Ltd from 01.07.2019. The details of the assets are given below:
Block-1 (Plant and machinery (15%)) WDV as on 01.04.2019 Rs. 12L
Block-2 (Building (10%)) WDV as on 01.04.2019 Rs. 25L
The company acquired P&M in December 2019 for Rs. 10L. It has been doing the business
from 01.07.2019. Compute the quantum of depreciation to be claimed by Mr. Gopi and the
successor company for the AY 2020-21. Ignore additional depreciation. [Answer: Rs. 107205
& Rs. 397795]

(xiv) Actual cost – S. 43 (1):


1 ‘Actual cost’ means actual cost of the asset to the assessee as reduced by that portion of the cost
thereof, if any, as has been met directly or indirectly by any other person or authority. [S. 43 (1)].
2 However, where an assessee incurs any expenditure for acquisition of any asset in respect of which
a payment or aggregate payments made to a person in a day, otherwise than by an APC drawn on
a bank or APBD or use of electronic clearing system through a bank account or other electronic
mode prescribed, exceeds Rs. 10000, such expenditure shall not form part of actual cost of such
asset. [Proviso to S. 43 (1)].

Components of actual cost:


All cost incurred in bringing the asset to be present location and working condition shall be
capitalised. An illustrative list of items of costs which shall form part of actual cost is as under:
1 Purchase price ****
2 Taxes & duties (for which credit is not availed) [Explanation-9 to S. 43 (1)] ****
3 Cost of placing order ****
4 Travelling cost [L. G. Balakrishnan & Bros (P) Ltd (Mad) + J. M. A. Industries Ltd (Del)] ****
5 Loading charges ****
6 Transportation charges ****
7 Transit insurance ****
8 Unloading charges ****
9 Inspection charges ****
10 Interest on borrowing up to the date on which the asset is first put to use [Proviso to S. ****
36 (1) + Explanation-8 to S. 43 (1)].
11 Erection-commissioning-installation expenses ****

MT Educare –CA Inter Income Tax Classes – R.SOUMYANARAYANAN.FCA. GRAD CWA. Page-175
Chapter-6 : Capital gains

12 Trail run expenses (net of realisation from trial run) [Food specialities Ltd (Del) + ICDS V] ****
13 Subsequent price adjustments ****
14 Adjustment on account of change in duties or taxes ****
15 Trade discounts (****)
16 Asset specific subsidy [S. 43 (1) Explanation-10 and the proviso thereunder + ICDS VII] (****)
17 Refund of asset specific subsidy [ICDS VII]. ****
18 Receipts incidental to acquisition of asset (****)
19 Adjustment U/s 43A ****
20 Portion of cost of the asset met by any other person (****)
21 Actual cost of the asset ******

Points requiring attention:


1 Tax paid on fee for technical services availed in connection with installation of machinery which is
paid pursuant to tax insulation clause in the agreement, is also a consideration for the engineering
services availed in connection with installation of machinery. It becomes part and parcel of the
installation cost which needs to be capitalised. [Standard polygraph machines (P) Ltd 243 ITR 788
(Mad)].
2 Expenses incurred in training employees for installation of machinery forms part of actual cost of
the asset. [Sunil Synchem Ltd 163 ITR 0467 (Raj).].
3 Expenditure incurred on account of the foundation stone laying ceremony is to be added to the cost
of the factory building as the foundation is a part of the construction and forms part of the actual
cost of the assets for the grant of depreciation allowance. [Nirlon synthetic fibres & chemicals Ltd
137 ITR 1 (Bom)].
4 A residential building could be used for commercial purpose only after converting into a commercial
building. In this regard, if commercialization charges are paid, it shall form part of actual cost.
[Hindustan Times Ltd 231 ITR 741 (SC].
5 Guarantee commission paid to bank in connection with import of machinery on deferred payment
basis = Revenue expenditure. Deductible U/s 37 (1). It cannot be adjusted in computing the actual
cost of the machinery. [Sivakami mills Ltd (SC) 227 ITR 465 + Akkamamba Textiles Ltd 227 ITR 464
(SC)].
6 ICDS V provides that administration and other general overhead expenses are to be excluded from
the cost of tangible fixed assets, if they do not relate to a specific tangible asset. However, the
expenses which are specifically attributable to construction of a project or to the acquisition of a
tangible fixed asset or bringing it to its working condition, shall be included as a part of the cost of
the project or as a part of the cost of tangible fixed asset.
7 Expenditure incurred in infusing new life into an existing discarded asset = Capital expenditure. It is
to be capitalised and is eligible for depreciation. [Bharat Gears Ltd (Del)].
8 Waiver of loan taken for purchase of machinery shall not be adjusted against the block value. It is a
capital receipt.
However, if the loan waived was taken from the Government, then the amount waived is deemed
to be income U/s 2 (24) (xviii). Accordingly, it will be taxed.

MT Educare –CA Inter Income Tax Classes – R.SOUMYANARAYANAN.FCA. GRAD CWA. Page-176
Chapter-6 : Capital gains

Special provisions consequential to changes in rate of exchange of currency – S. 43A:


1 S. 43A applies only in respect of assets acquired for the purpose of business or profession from a
country outside India through a loan in foreign currency or foreign supplier’s credit.
2 In consequence of a change in exchange rate during the PY after the acquisition of such asset there
is an increase or decrease in the liability of the assessee as expressed in Indian currency (as compared
to the liability existing at the time of acquisition of the asset) at the time of making payment:
(a) towards the whole or part of the cost of the asset.
(b) towards the repayment of the whole or part of the moneys borrowed by him from any person
directly or indirectly, in any foreign currency specifically for the purpose of acquiring the asset along
with interest, if any.
3 The amount by which the liability as aforesaid is so increased or reduced during the PY and which is
taken into account at the time of payment shall be added to the actual cost of the asset or reduced
from the actual cost of the asset, as the case may be.

Points requiring attention:


1 The adjustments referred to in S. 43A shall be made only in the PY in which actual payment is made
to the foreign supplier or repay the foreign currency loan.
2 The adjustments referred to in S. 43A shall not be made if the foreign currency loan or supplier’s
credit is re-instated at the rate of exchange prevailing on Balance sheet date each year and no actual
payment is made.
3 Adjustment U/s 43A is possible even after the sale of the relevant asset provided the block exists.
4 However, adjustment U/s 43A is not possible after the disposal of block itself.

Explanation-3 to S.43A:
Where the assessee has entered into a contract with an authorised dealer for providing him
with a specified sum in foreign currency on or after a stipulated future date at the rate of
exchange specified in the contract to enable him to meet the whole or any part of the liability
aforesaid, the amount, if any, to be added to or deducted from, the actual cost of the asset
U/s 43A shall, in respect of so much of the sum specified in the contract is available for
discharging the liability aforesaid, be computed with reference to the rate of exchange
specified therein.

Illustration:
X Ltd is an Indian company engaged in manufacturing business. On 01.04.2019, it imported
second hand machinery from Y Inc (US) for a price of 1L USD. On that exchange rate is 1 USD
= Rs. 60. The machinery was installed on the same date. The acquisition of asset was funded
by a loan of 1L USD from Citi-Bank taken on 01.04.2019. The loan is repayable in 4 installments
as follows:
Due date Amount
30.06.2020 25000 USD
30.06.2021 25000 USD
30.06.2022 25000 USD
30.06.2023 25000 USD

To hedge against the foreign exchange fluctuation risk exposure, X Ltd entered into 4 forward
contracts with Citi-Bank.

MT Educare –CA Inter Income Tax Classes – R.SOUMYANARAYANAN.FCA. GRAD CWA. Page-177
Chapter-6 : Capital gains

Contract Amount Forward rate Maturity date


1 25000 USD Rs. 62 30.06.2020
2 25000 USD Rs. 64 30.06.2021
3 25000 USD Rs. 66 30.06.2022
4 25000 USD Rs. 68 30.06.2023

Computation & treatment of exchange gain/loan in accordance with Explanation-3:


PY Exchange gain or loss Treatment
2020-21 25000 USD * (Rs. 62 – Rs. 60) = Rs. 50000 (Loss) To be added to the block
2021-22 25000 USD * (Rs. 64 – Rs. 60) = Rs. 100000 (Loss) To be added to the block
2022-23 25000 USD * (Rs. 66 – Rs. 60) = Rs. 150000 (Loss) To be added to the block
2023-24 25000 USD * (Rs. 68 – Rs. 60) = Rs. 200000 (Loss) To be added to the block
Note:
For determination of exchange loss or gain on the date of payment of installments, we ignore
the spot rate on that date. Instead we take the forward rate in view of Explanation-3 to S.
43A.
Deemed actual cost under certain circumstances:
Explanation-2 Where an asset is acquired by the assessee by way of gift or inheritance, the actual
to S. 43 (1) cost of the asset to the assessee shall be the WDV in the hands of the previous owner
at the time of transfer of asset computed by assuming that the asset was the only
asset in the block of assets.
Explanation-3 Where before the DOA by the assessee, the assets were at any time used by any
to S. 43 (1) other person for the purpose of his B/P and the AO is satisfied that the main purpose
of the transfer of such assets, directly or indirectly to the assessee, was the reduction
of income tax liability (by claiming depreciation with reference to enhanced cost),
the actual cost to the assessee shall be such an amount as the AO may determine
with the previous approval of JCIT.
Explanation-4 Where any asset which had once belonged to the assessee and had been used by
to S. 43 (1) him for the purposes of his B/P and thereafter ceased to be his property by reason
of transfer or otherwise, is re-acquired by him, then the actual cost of the asset to
the assessee shall be the lower of the following:
(a) WDV of asset at the time of transfer by the assessee computed by assuming
that the asset was the only asset in the block of assets.
(b) the actual price for which the asset is re-acquired by him
Explanation-4A Where before the DOA by the assessee (referred to as first person), the assets were
to S. 43 (1) at any time used by any other person (referred to as second person) for the purposes
of his B/P and depreciation has been claimed in respect of such assets by the 2nd
person and such 2nd person acquires on lease, hire or otherwise, assets from the
first mentioned person, then notwithstanding anything contained in Explanation-3,
the actual cost of the transferred assets, in case of first person, shall be the WDV of
the said assets at the time of transfer thereof by the second person.
Explanation-5 Where a building previously the property of the assessee is brought into use for the
to S. 43 (1) purpose of B/P of the assessee, the actual cost to the assessee shall be the actual
cost of the building to the assessee as reduced by an amount equal to the
depreciation calculated at the rate in force on that date that would have been

MT Educare –CA Inter Income Tax Classes – R.SOUMYANARAYANAN.FCA. GRAD CWA. Page-178
Chapter-6 : Capital gains

allowable had the building been used for the purpose of business or profession since
the date of its acquisition.
Explanation-11 Where an asset which was acquired outside India by an assessee, being a non-
to S. 43 (1) resident, is brought by him to India and used for the purposes of his B/P, the actual
cost of the asset to the assessee shall be the actual cost to the assessee, as reduced
by an amount equal to the amount of depreciation calculated at the rate in force
that would have been allowable had the asset been used in India for the said
purposes since the date of its acquisition by the assessee.
Explanation-2 Where in any PY, any block of assets is transferred by a holding co to its subsidiary
to S. 43 (6) or by a subsidiary to its holding co and the conditions of S. 47 (iv) or S. 47 (v) are
satisfied, then the actual cost of the block in the case of the transferee-co shall be
the WDV of the block as in the case of the transferor-co for the immediately
preceding PY as reduced by the amount of depreciation actually allowed in relation
to the said preceding PY.
Explanation-6 When any CA is transferred by a holding co to its subsidiary or by a subsidiary to its
to S. 43 (1) holding co, then, if the conditions of S. 47 (iv) or (v) are satisfied, the actual cost of
the transferred CA to the transferee-co shall be taken to be the same as it would
have been if the transferor-co had continued to hold the CA for the purposes of its
business.
Explanation-2 Where any block of assets is transferred by the amalgamating company to
to S. 43 (6) amalgamated company and the amalgamated company is an Indian company, then
the actual cost of the block of assets in case of amalgamated company shall be the
WDV of the block of assets in the hands of the amalgamating company for the
immediately preceding PY as reduced by the amount of depreciation actually
allowed in relation to the said preceding PY.
Explanation-7 Where, in a scheme of amalgamation, any capital asset is transferred by the
to S. 43 (1) amalgamating company to the amalgamated company and the latter is an Indian
company, then the actual cost of the capital asset to the amalgamated company shall
be taken to be the same as it would have been if the amalgamating company had
continued to hold the capital asset for the purposes of its business.
Explanation-2B Where any asset forming part of a block of assets is transferred by a demerged
to S. 43 (6) company to the resulting company, the WDV of the block of assets in the case of
resulting company shall be the WDV of the transferred assets of the demerged
company immediately before demerger.
Explanation-2A Where in any PY, any asset forming part of a block of assets is transferred by a
to S. 43 (6) demerged company to the resulting company, then the WDV of the block of assets
of the demerged company at the beginning of the PY shall be reduced by the WDV
of the assets transferred to the resulting company pursuant to demerger.
Explanation-7 Where the income of an assessee is derived, in part from agriculture and in part from
to S. 43 (6) business chargeable to income-tax U/H PGBP, for computing the WDV of assets
acquired before the PY, the total amount of depreciation shall be computed as if the
entire income is derived from the business of the assessee U/H PGBP and the
depreciation so computed shall be deemed to be the depreciation actually allowed
under this Act.

MT Educare –CA Inter Income Tax Classes – R.SOUMYANARAYANAN.FCA. GRAD CWA. Page-179
Chapter-6 : Capital gains

Illustration on Explanation-3 to S. 43 (1):


Mr. X submits the following information:
Opening WDV as on 01.04.2019 of Block (15%) Rs. 10L
PGBP before depreciation Rs. 4L
B/F loss U/H CG Rs. 120L
Mr. X sells an asset out of the block to Mr. Y on 30.06.2019 for Rs. 130L. Mr. X files his ROI as
under:
PGBP Rs. 4L
STCG Rs. 120L
Less: B/F capital loss Rs. 120L Nil
Total income Rs. 4L
WDV of the block = Nil.
The relevant information of Mr. Y is as under:
PGBP before depreciation Rs. 50L
Opening WDV of block of assets (15%) as on 01.04.2019 Rs. 20L

Mr. Y files ROI as under:


PGBP before depreciation Rs. 50L
Less: Depreciation @ 15% on [Rs. 20L + Rs. 130L] Rs. 22.50L
Total income Rs. 27.50L
Now, the AO invokes Explanation-3 to S. 43 (1) and determines that the FMV of the asset
transferred for Rs. 130L is Rs. 15L. The AO with the permission of JCIT shall take the actual
cost of asset to Mr. Y as Rs. 15L and compute the income of Mr. Y as under:
PGBP before depreciation Rs. 50L
Less: Depreciation @ 15% on [Rs. 20L + Rs. 15L] Rs. 5.25L
Assessed income Rs. 44.75L
There will be no change in income of Mr. X since Explanation-3 to s. 43 (1) determines the
actual cost of the asset only.

Illustration on Explanation-4 to S. 43 (1):


Mr. A acquired an asset on 01.01.2014 for Rs. 1L and the asset was taken to the block of asset
on which depreciation rate is 15%. Mr. A sold the asset to Mr. B on 01.01.2016 for Rs. 150000
and Mr. B sold the asset to Mr. C on 01.01.2017 for Rs. 2L. Mr. C sells the asset to Mr. A on
01.01.2020 for Rs. 450000.

Explanation-4 to s. 43 (1) shall be attracted in hands of Mr. A in AY 2020-21. Actual cost to


Mr. A for AY 2020-21 shall be the lower of the following:
Actual price for which the asset is reacquired by him Rs.
450000
WDV of asset at the time of transfer which shall be computed as under: Rs. 78625
Actual cost to Mr. A when he first acquired the asset is Rs. 100000
Less: Depreciation for the PY 2013-14 @ 7.5% Rs. 7500
Less: Depreciation for the PY 2014-15 @ 15% Rs. 13875
Actual Cost Rs. 78625

MT Educare –CA Inter Income Tax Classes – R.SOUMYANARAYANAN.FCA. GRAD CWA. Page-180
Chapter-6 : Capital gains

Illustration on Explanation-4A to S. 43 (1):


(i) Triveni engineering Ltd is a loss making company.
(ii) Triveni Engineering Ltd purchased air pollution control equipment for Rs. 100L and
claimed depreciation @ 100% on Rs.100L. WDV of the block is nil.
(iii) Reliance Industries Ltd is a profit making company. Triveni engineering Ltd sells the
air pollution control equipment to Reliance Industries Ltd for Rs. 100L on 01.04.2019.
(iv) Reliance Industries Ltd leases back to Triveni Ltd the said equipment on 01.04.2019.
(v) Triveni engineering Ltd earns a STCG of Rs. 100L which can be set off against current
year business losses.
(vi) Reliance Industries Ltd is the owner of the asset and will claim depreciation. However,
as per Explanation-4A the actual cost of the asset to Reliance Industries Ltd shall be
taken as Nil.

Illustration on Explanation-5 to S. 43 (1):


Mr. X purchased a building on 01.01.2016 for his personal residence for Rs. 10L. The building
is converted into factory building on 01.01.2020 when its FMV is Rs. 8L. Assume that
depreciation rate on the building was 7.5% upto AY 2017-18 and depreciation rate on building
is 10% from AY 2018-19. Determine the depreciation allowable for the AY 2020-21.

Determination of depreciation on the building for the AY 2020-21:


SN Particulars Amount
1 Actual cost of acquisition Rs. 1000000
2 Depreciation for the AY 2016-17 @ 5% Rs. 50000
3 Depreciation for the AY 2017-18 @ 10% Rs. 95000
4 Depreciation for the AY 2018-19 @ 10% Rs. 85500
5 Depreciation for the AY 2019-20 @ 10% Rs. 76950
6 Actual cost for the AY 2020-21 (1-2-3-4-5) Rs. 692550
7 Depreciation for the AY 2020-21 Rs. 69255

Illustration on Explantion-2 to S. 43 (6) & Explanation-6 to S. 43 (1):


H Ltd has a block of assets the WDV of which as on 01.04.19 is Rs. 3L. On 30.06.19, H Ltd
purchases an asset P which is traceable to this block for Rs. 5L. Subsequently, on 31.07.19, H
Ltd transfers the entire block to its WOS for Rs. 10L. Both are Indian companies. Discuss the
tax implications.

The tax implications are summarized as follows:


1 Exemption is available to H Ltd U/s 47 (iv). Hence, no CG shall be charged to tax in the
hands of H Ltd.
2 In the hands of WOS, the actual cost of the assets acquired during the PY should be
taken as Rs. 8L (Rs. 3L + 5L).
3 As per Explanation 6 to S. 43 (1), the actual cost of asset P shall be taken to be Rs. 5L. As
per Explanation 2 to S. 43 (6) the actual cost of the assets contained in the opening block
of assets shall be taken to be Rs. 3L.

MT Educare –CA Inter Income Tax Classes – R.SOUMYANARAYANAN.FCA. GRAD CWA. Page-181
Chapter-6 : Capital gains

Illustration on Explanation-2 to S. 43 (6) & Explanation-7 to S. 43 (1):


A Company ‘P’ amalgamates with company S on 30.08.2019. Opening WDV of the block of
assets (carrying depreciation @ 15%) in the hands of P as on 01.04.2019 is Rs. 48L. P acquires
a second hand asset in this block on 01.06.2019 for Rs. 2L. Opening WDV of the block of assets
(carrying depreciation @ 15%) in the hands of S as on 01.04.2019 is Rs. 30L. S acquires a
second hand asset on 31.12.2019 for Rs. 5L. Under the scheme of amalgamation, P transfers
the block of assets to S on 30.08.2019 for Rs. 80L. Compute the depreciation in the hands of
P and S.

Computation of depreciation on the assumption that amalgamation had not taken place:
Depreciation to be apportioned between amalgamating company and amalgamated
company = 15% on (4800000 + 200000) = Rs. 750000.
Apportionment of depreciation based on 5th proviso to S. 32 (1):
P 750000 * 151/365 310274
S 750000 * 214/365 439726

Tax implications in the hands of amalgamating company:


1 No CG shall arise to the amalgamating company by virtue of S. 47 (vi).
2 Depreciation allowable to amalgamating company = 310274.

Tax implications in the hands of amalgamated company:


Opening WDV as on 01.04.2019 3000000
Add: Actual cost of the assets acquired during the PY 500000
Add: Actual cost of the assets acquired from the amalgamating company as per 200000
Explanation 7 to S. 43 (1)
Add: Actual cost of the assets acquired from the amalgamating company as per 4800000
Explanation 2 to S. 43 (6)
WDV for the AY 2020–21 8500000
Less: Depreciation on Rs. 3000000*15% 450000
Less: Depreciation on Rs. 500000* 7.5% (as per 2nd proviso to S. 32 (1)) 37500
Less: Apportioned depreciation on Rs. 5000000 439726
WDV as on 01.04.2020 7572774

Illustration on Explanation-2A & Explanation-2B to S. 43 (6):


Opening WDV of block of assets (A, B, C, D and E) of Demerged company, as on 01.04.2019 Rs. 3500000
Asset-F purchased by Demerged company on 01.12.2019 for Rs. 10L
Asset-A was transferred pursuant to scheme of demerger to the resulting company at the Rs. 2400000
book value of
WDV of Asset-A as on 01.04.2019 Rs. 1400000
Opening WDV of the block of assets of resulting company as on 01.04.2019 is Rs. 400000
Asset-Z was purchased by the resulting company on 01.01.2020 for Rs. 300000
Demerger takes place on 30.08.2019
Depreciation rate 15%

MT Educare –CA Inter Income Tax Classes – R.SOUMYANARAYANAN.FCA. GRAD CWA. Page-182
Chapter-6 : Capital gains

Apportionment of depreciation:
1 Depreciation for AY 2020-21 to be apportioned 15% of Rs. 1400000 = Rs. 210000
2 Depreciation allowable to demerger company for AY Rs. 210000 * 151/365 = Rs. 86877
2020-21
3 Depreciation allowable to the resulting company for AY Rs. 210000 * 214/365 = Rs. 123123.
2020-21

Demerged company – AY 2020-21:


Opening WDV Rs. 3500000
Add Actual cost of additions made during the PY 2019-20 Rs. 1000000
Less WDV of asset transferred in demerger (Rs. 1400000)
Less Depreciation on Rs. 210000 @ 15% (Rs. 315000)
Less Depreciation for Rs. 1000000 @ 7.5% (Rs. 75000)
Less Apportioned depreciation (Rs. 86877)
WDV as on 01.04.2020 Rs. 2623123
Resulting company – AY 2020-21:
Opening WDV Rs. 400000
Add WDV of assets received on demerger Rs. 1400000
Add Actual cost of assets acquired during the PY 2019-20 Rs. 300000
Less Depreciation on Rs. 400000 @ 15% (Rs. 60000)
Less Depreciation on Rs. 300000 @ 7.5% (Rs. 22500)
Less Apportioned depreciation (Rs. 123123)
WDV as on 01.04.2020 Rs. 1894377

Illustration on Explanation-7 to S. 43 (6):


During the PY 18-19, X Ltd, upon incorporation, started the business of growing and
manufacturing tea in India. In computing its composite business income, it claimed a
depreciation of Rs. 15L on its plant-15% block. The depreciation on this block for the PY 19-
20 shall be computed as follows:
1 WDV of the block as on 01.04.19 [Rs. 100L – 15L] 85L
2 Additions during the PY 19-20 6L
3 Total (1+2) 91L
4 Deduction from the block Nil
5 WDV for the purpose of depreciation (3-4) 91L
6 Depreciation @ 15% 13.65L

(xv) Leased commercial vehicles used by lorry operators for running them on hire –
Whether eligible for higher depreciation:
1 Even when commercial vehicles are leased by owner thereof and the lessee uses the same for
running them on hire, the owner will be eligible for depreciation @ higher rate of 30%.
2 The expression ‘Motor buses, lorries and taxis used in business of running on hire’ would not
only cover the case of use by the owner for the business of running on hire, but also use by lessee
for business of running on hire. [Bansal Credits Ltd 259 ITR 69 (Del) + Madan & Co 254 ITR 445
(Mad) + Agarwal Finance Co. (P) Ltd 332 ITR 549 (Cal)].
3 Higher depreciation is with reference to the vehicle and not with reference to the nature of
business.

MT Educare –CA Inter Income Tax Classes – R.SOUMYANARAYANAN.FCA. GRAD CWA. Page-183
Chapter-6 : Capital gains

(xvi) Some additional issues relating to depreciation rates:


1 Relevant Depreciation rate prevalent on the first day of the AY is relevant. [Isthmian
depreciation rate steamship lines 20 ITR 572 (SC) + Hong Kong Oceans Shipping 238 ITR 955 (Mad)
+ IDL Chemicals 275 ITR 218 (AP) + Rama Shanker 277 ITR 69 (All)].
2 ‘Router’ and ‘Router’ and ‘switches’ can be classified as computer hardware when they are
‘switches’ [Data used along with a computer and when their functions are integrated with a
craft India Ltd computer. In such a situation, routers and switches are to be included in block of
(Mum-SB)]. ‘computer’ for purpose of determining rate of depreciation applicable to them.
3 EPABX and mobile EPABX and mobile phones are communication equipments. Therefore, they
phones cannot be treated as computers to be entitled to higher depreciation at 40%. They
are eligible for depreciation at normal rate of 15% applicable to plant and
machinery. [Federal Bank Ltd Vs ACIT [2011] 332 ITR 319 (Ker)].
4 Computer Computer accessories and peripherals (such as printers, scanners, servers etc)
accessories and form an integral part of computer system and they can’t be used without
peripherals computer. Therefore, these are also eligible for higher rate of depreciation of
40%. [BSES Yamuna Powers Ltd (2013) (Del)].
5 UPS UPS is also eligible for higher rate of depreciation @ 40% and not @ general rate
of 15%. [Same reasoning]. [Orient Ceramics and Industries Ltd (Del)].

(xvii) Depreciation allowance – mandatory – S. 32 (1) Explanation-5:


Depreciation shall be allowed to the assessee in computing his total income whether or not
he has claimed it. In other words, depreciation allowance is mandatory.

(xviii) Tax treatment of unabsorbed depreciation – S. 32 (2):


(i) Where, in any PY, the profits or gains chargeable are not sufficient to give full effect to the
depreciation allowance, the unabsorbed depreciation shall be added to the depreciation
allowance for the following PY and shall be deemed to be part of that allowance.
(ii) If no depreciation allowance is available for that PY, the unabsorbed depreciation of the earlier
PY shall become the depreciation allowance of that year.
(iii) The effect of this provision is that the unabsorbed depreciation shall be carried forward
indefinitely till it is fully set off.
(iv) However, in the order of set off of losses under different heads of income, effect shall first be
given to the business losses and then to unabsorbed depreciation.

Essence:
(i) Since the unabsorbed depreciation forms part of the current year’s depreciation, it
can be set off against any other head of income except ‘salaries’.
(ii) The unabsorbed depreciation can be carried forward for indefinite number of PYs.
(iii) Set off will be allowed even if the same business to which it relates is no longer in
existence in the year in which the set off takes place.

MT Educare –CA Inter Income Tax Classes – R.SOUMYANARAYANAN.FCA. GRAD CWA. Page-184
Chapter-6 : Capital gains

(Y) Slump sale – Tax implications:


(1) Meaning of slump sale – S. 2 (42C):
(i) “Slump sale” means the transfer of one or more undertakings as a result of the sale
for a lump sum consideration without values being assigned to the individual assets
and liabilities in such sales:
(ii) Undertaking means a business activity.
(iii) The determination of the value of an asset or liability for the sole purpose of payment
of stamp duty, registration fees or other similar taxes or fees shall not be regarded as
assignment of values to individual assets or liabilities;

(2) Computation of capital gains in case of slump sale – S. 50B:


(i) Any profits or gains arising from the slump sale effected in the PY shall be chargeable to income-
tax as capital gains arising from the transfer of LTCA and shall be deemed to be the income of the
PY in which the transfer took place. [S. 50B (1)].
(ii) However, any profits or gains arising from the transfer under the slump sale of any capital asset
being undertaking(s) owned & held by an assessee for not more than 36 months immediately
preceding the date of its transfer shall be deemed to be the capital gains arising from the transfer
of STCA. [Proviso to S. 50B (1)].
(iii) In relation to capital assets being an undertaking or division transferred by way of such sale, the
"net worth" of the undertaking shall be deemed to be the COA and the COI for the purposes of S.
48 and no regard shall be given to the provisions contained in the 2 nd proviso to S. 48 (i.e.
Indexation benefit). [S. 50B (2)].
(iv) ‘‘Net worth’’ shall be the aggregate value of total assets of the undertaking or division as reduced
by the value of liabilities of such undertaking or division as appearing in its books of account.
[Explanation-1 to S. 50B].
(v) Any change in the value of assets on account of revaluation of assets shall be ignored for the
purposes of computing the net worth. [Proviso to Explanation-1 to S. 50B].
(vi) For computing the net worth, the aggregate value of total assets shall be, — (a) in the case of
depreciable assets, the WDV of the block of assets determined in accordance with the provisions
contained in S. 43 (6) (c) (i) (C); and (b) in the case of other assets, the book value of such assets.
[Explanation-2 to S. 50B].
(vii) Every assessee, in the case of slump sale, shall furnish along with the ROI a report of a Chartered
accountant (Form 3CEA) indicating the computation of the net worth of the undertaking or
division, as the case may be, and certifying that the net worth of the undertaking or division, as
the case may be, has been correctly arrived at in accordance with the provisions of this section.
[S. 50B (3)].

(3) COA & COI:


1 COA & COI Net worth of the undertaking
2 Net worth of the undertaking Value of assets of the undertaking transferred-
Value of liabilities of the undertaking transferred.
3 Value of assets of the undertaking Value of depreciable assets + Value of non-
transferred depreciable assets.
4 Value of non-depreciable assets Sum total of book value of non-depreciable assets.
5 Value of depreciable assets Value computed as per S. 43 (6) (c) (i) (c)

MT Educare –CA Inter Income Tax Classes – R.SOUMYANARAYANAN.FCA. GRAD CWA. Page-185
Chapter-6 : Capital gains

(4) Value of depreciable assets as per S. 43 (6) (c) (i) (c):


1 Actual cost of assets (transferred by way of slump sale) falling within the block ****
2 Depreciation up to the PY preceding the PY of slump sale (computed as if these ****
assets were the only assets falling within the block)
3 Value of depreciable assets determined as per S. 43 (6) (C) (i) (c). [1-2] ****

(5) WDV of the block for the purpose of depreciation (in the year of slump sale):
1 Opening WDV of the block ****
2 Actual cost of assets acquired during the PY and falling within this block ****
3 ‘Moneys payable’ in respect of any asset in the block which is sold, discarded, ****
demolished or destroyed, together with the scrap value, if any.
4 Value determined as per S. 43 (6) (c) (i) (c) ****
5 WDV for the purpose of depreciation (1+2-3-4) ****

Points requiring attention:


1 Contingent Ignore contingent liabilities
liabilities
2 Revaluation of Revaluation of assets shall not be considered while computing NW, irrespective
assets of the fact that revaluation is done in the current year or in past years.

(6) Carry forward of unabsorbed losses and depreciation in case of slump sale:
1 In case of slump sale, the unabsorbed losses and depreciation of the undertaking shall not
be available to the transferee for C/F. There is no provision in the Act in this regard.
2 But, the transferor can C/F such losses and depreciation since, for C/F, S. 72 does not require
the continuity of the same business.

Illustration on slump sale:


X Ltd. is engaged in manufacture of chemical (since 1965) and paper (since 2015). The
following data is noted from the balance sheet of X Ltd. as on March 31, 2019.
Particulars of shareholder’s funds (Rs in 000’s)
Equity share capital 6000
Preference share capital 1000
General reserve 4000
Revaluation Reserve 600
Share premium 800
Total 12400

Particulars of assets Chemical division Paper division Total


Land 3000 2000 5000
Plant and Machinery 1600 3600 5200
Stock 500 900 1400
Debtors & other current assets 400 1100 1500
Less: Creditors (400) (300) (700)
Total 5100 7300 12400

MT Educare –CA Inter Income Tax Classes – R.SOUMYANARAYANAN.FCA. GRAD CWA. Page-186
Chapter-6 : Capital gains

Revaluation reserve was created by making upward revision of land belonging to chemical
division (Rs. 1L) and paper division (Rs. 5L). The company wants to transfer paper division on
01.04.19 by way of slump sale for a consideration of Rs 88L (transfer expenses being Rs
28,000). By taking into consideration, the following additional information, find out the
amount of CG and other tax consequences.
1 Transfer agreement does not specify the value of individual assets and liabilities. However, the
value of land of paper division for the purpose of stamp duty is Rs. 46L. The same amount is
adopted by the stamp valuation authority of the MP Government.
2 The rate of depreciation on P&M owned by X Ltd is 15%. The depreciated value of the block
(consisting of chemical division & paper division) on 01.04.19 is Rs. 70L for income tax purpose.
3 Apart from transferring P&M of paper division, the company purchases an old plant P for Rs. 1L
and sells Plant Q for Rs. 50L in September 2019. Plant P and Q belongs to chemical division.
4 P&M of the paper division was purchased in May 2015 for Rs 95L. It was second-hand imported
machinery. The division started the commercial production in June 2015. However, one of the
plant (Cost Rs. 10L) was put to use in March 2016.
5 No other asset for paper division is purchased/sold between May 2015 and March 2019.

Computation of CG arising from slump sale for the AY 20-21:


SN Particulars Amount
1 FVC 88,00,000
2 Expenses incurred in connection with slump sale (28,000)
3 Net consideration (1-2) 87,72,000
4 COA and COI (Wn-1) (53,00,000)
5 LTCG (3-4) 34,72,000

Wn-1: Computation of COA and COI:


SN Particulars Amount
1 Aggregate value of all assets of undertaking transferred (Wn-2) 56,00,000
2 Aggregate value of liabilities of undertaking transferred (3,00,000)
3 Cost of acquisition and improvement (1-2) 53,00,000

Wn-2: Aggregate value of all assets of undertaking transferred:


SN Particulars Amount
1 Aggregate value of non-depreciable assets (Wn-3) 35,00,000
2 Aggregate value of depreciable assets 21,00,000
3 Aggregate value of all assets of the undertaking transferred 56,00,000

Wn-3: Aggregate value of non-depreciable assets:


SN Particulars Amount Amount
1 Stock 9,00,000
2 Debtors and other current assets 11,00,000
3 Land 20,00,000
4 Upward revaluation (5,00,000)
5 Land book value (3-4) 15,00,000
6 Aggregate value of non-depreciable assets 35,00,000

MT Educare –CA Inter Income Tax Classes – R.SOUMYANARAYANAN.FCA. GRAD CWA. Page-187
Chapter-6 : Capital gains

Wn-4: Aggregate value of depreciable assets:


A. WDV of the block from which assets are transferred:
SN Particulars Situation-2
1 Opening WDV 70,00,000
2 Additions to the block during the PY (Plant-P) 1,00,000
3 Total (1+2) 71,00,000
4 Moneys payable in respect of Plant-Q (50,00,000)
5 WDV before reducing the value of assets transferred under slump sale 21,00,000
6 Value of assets transferred under slump sale(See B) (21,00,000)
7 WDV for the purpose of depreciation (5-6) 0
8 Depreciation in respect of the block for the PY 0
9 Closing WDV (7-8) 0

B. Value of assets transferred under slump sale:


SN Particulars Computation Amount
1 Cost of assets purchased during PY 15-16 95,00,000
2 Depreciation for the PY 15-16 (85L*15%) + (10L*15%*50%) (13,50,000)
3 Opening WDV for the PY 16-17 81,50,000
4 Depreciation for the PY 16-17 81,50,000*15% (12,22,500)
5 Opening WDV for the PY 17-18 69,27,500
6 Depreciation for the PY 17-18 69,27,500*15% (10,39,125)
7 Opening WDV for the PY 18-19 58,88,375
8 Depreciation for the PY 18-19 58,88,375*15% (8,83,256)
9 Opening WDV for the PY 19-20 50,05,119

(Z) Succession of firm by company – Tax implications:


1. Immunity to the firm from CG tax – S. 47 (xiii):
Where a company succeeds a firm in the business carried on by the latter and as a result there
is a transfer of CA by the firm to the company, nothing contained in S. 45 shall apply to such
transfer. That is, the firm will not be liable to CG tax. However, this immunity from S. 45 is
available only when the following conditions given in Proviso to S. 47 (xiii) are fulfilled:
(a) All assets and liabilities of the firm relating to the business immediately before
succession shall be taken over by the company.
(b) All partners of the firm immediately before succession shall become the shareholders
of the company.
(c) The partners shall receive consideration only by way of shares (equity or preference)
allotted in the company.
(d) The partners shall become the shareholders in the company in the same proportion in
which their capital accounts stood in the books of the firm on the date of succession.
(e) The partners shall, in aggregate, have atleast 50% voting power in the company, for a
period of 5 years from the date of succession.

MT Educare –CA Inter Income Tax Classes – R.SOUMYANARAYANAN.FCA. GRAD CWA. Page-188
Chapter-6 : Capital gains

2. COA and POH of CA received from firm in the hands of company:


1 COA of CA received from firm (in = COA to the previous owner (i.e. firm). [S. 49 (1)].
the hands of company)
2 POH [S. 2 (42A) Explanation]. Starts on the DOA by the previous owner (i.e. firm).
3 Denominator index for indexing = Index pertaining to the PY in which the CA was acquired
COA in the hands of the company by the previous owner (i.e. firm). [Manjula Shah (Bom)].

3. Computation of depreciation in case of succession – Proviso-5 to S. 32 (1):


Where a firm is succeeded by a company and the requirements of S. 47 (xiii) are satisfied,
then in respect of assets transferred pursuant to succession, both the firm and company are
eligible for depreciation. For computing the depreciation allowable in the hands of the
predecessor (firm) and the successor (company), the following steps are to be followed:
1 Find out the depreciation on the assets as if succession had not taken place.
2 Then, apportion the depreciation between the predecessor (firm) and successor (company), in the
ratio of number of days for which the assets are used by them during the PY in which succession
took place.

4. Continuance of deduction U/s 35DDA in the hands of the company – S. 35DDA (4):
(a) VRC is allowed as a deduction U/s 35DDA in 5 equal annual installments beginning with the PY of
payment.
(b) S. 35DDA (4) provides that the deduction in respect of VRC paid by the firm shall continue in the
hands of the succeeding company for the unexpired residual period starting from the PY of
succession.

5. C/F of losses and depreciation of the firm by the company – S. 72A (6):
1 Where a firm is succeeded by a company fulfilling the conditions laid down in S. 47 (xiii), then,
notwithstanding anything contained in any other provision of this Act, the accumulated loss and the
unabsorbed depreciation of the predecessor firm shall be deemed to be the loss or allowance for
depreciation of the successor company for the purpose of PY in which succession took place and other
provisions of this Act relating to set off and C/F of loss and allowance for depreciation shall apply accordingly.
2 "Accumulated loss" means so much of the loss of the predecessor firm U/H PGBP (not being a loss
sustained in a speculation business) which such predecessor firm would have been entitled to carry
forward and set off U/s 72 if the succession of business had not taken place;
3 "Unabsorbed depreciation" means so much of the allowance for depreciation of the predecessor
firm which remains to be allowed and which would have been allowed to the predecessor firm under
the provisions of this Act, if succession of business had not taken place.

6. Consequences of violation of the conditions stipulated in Proviso to S. 47 (xiii):


1 Where any of the conditions laid down in the proviso to S. 47 (xiii) are not complied with, the amount of
profits or gains arising from the transfer of such capital asset not charged U/s 45 by virtue of conditions laid
down in the proviso to S. 47 (xiii) shall be deemed to be the profits and gains chargeable to tax of the
successor company for the PY in which the requirements of the proviso to S. 47 (xiii) are not complied with.
[S. 47A (3)]
2 Further, the set-off of loss or allowance of depreciation made in any PY in the hands of the successor
company, shall be deemed to be the income of the company chargeable to tax in the year in which such
conditions are not complied with. [Proviso to S. 72A (6)].

MT Educare –CA Inter Income Tax Classes – R.SOUMYANARAYANAN.FCA. GRAD CWA. Page-189
Chapter-6 : Capital gains

(ZA) Succession of SPC by a company – Tax implications:

1. Exemption to the SPC from CG tax – S. 47 (xiv) + Proviso:


Where a company succeeds a sole proprietary concern in the business carried on by the latter
and as a result, there is a transfer of CA by such concern to the company, nothing contained
in S. 45 shall apply to such transfer. That is, the proprietary concern will not be liable to CG
tax. However, the following conditions stipulated in the Proviso to S. 47 (xiv) are to be
satisfied:
(a) All the assets and liabilities of the sole proprietary concern in relation to the business
immediately before succession shall be taken over by the company.
(b) The sole proprietor shall receive consideration only in the form of shares in the
transferee company.
(c) The sole proprietor shall have atleast 50% VP in the co for a period of 5 years from the
date of succession.

2. COA and POH of CA received from SPC in the hands of company:


1 COA of CA received from SPC (in the = COA to the previous owner (i.e. SPC). [S. 49 (1)].
hands of company)
2 POH [S. 2 (42A) Explanation]. Starts on the DOA by the previous owner (i.e. SPC).
3 Denominator index for indexing = Index pertaining to the PY in which the CA was acquired by
COA in the hands of the company the previous owner (i.e. SPC). [Manjula Shah (Bom)].

3. Computation of depreciation in case of succession – Proviso-5 to S. 32 (1):


Where a SPC is succeeded by a company and the requirements of S. 47 (xiii) are satisfied, then
in respect of assets transferred pursuant to succession, both the SPC and company are eligible
for depreciation. For computing the depreciation allowable in the hands of the predecessor
(SPC) and the successor (company), the following steps are to be followed:
1 Find out the depreciation on the assets as if succession had not taken place.
2 Then, apportion the depreciation between the predecessor (SPC) and successor
(company), in the ratio of number of days for which the assets are used by them during
the PY in which succession took place.

4. Continuance of deduction U/s 35DDA in the hands of the company – S. 35DDA (4):
VRC is allowed as a deduction U/s 35DDA in 5 equal annual installments beginning with the
PY of payment. S. 35DDA (4) provides that the deduction in respect of VRC paid by the SPC
shall continue in the hands of the succeeding company for the unexpired residual period
starting from the PY of succession.

5. C/F of losses and depreciation of the SPC by the company – S. 72A (6):
1 Where a SPC is succeeded by a company fulfilling the conditions laid down in S. 47 (xiv), then,
notwithstanding anything contained in any other provision of this Act, the accumulated loss and
the unabsorbed depreciation of the predecessor SPC shall be deemed to be the loss or allowance
for depreciation of the successor company for the purpose of PY in which succession took place
and other provisions of this Act relating to set off and C/F of loss and allowance for depreciation
shall apply accordingly.

MT Educare –CA Inter Income Tax Classes – R.SOUMYANARAYANAN.FCA. GRAD CWA. Page-190
Chapter-6 : Capital gains

2 "Accumulated loss" means so much of the loss of the predecessor SPC U/H PGBP (not being a loss
sustained in a speculation business) which such predecessor SPC would have been entitled to C/F
and set off U/s 72 if the succession of business had not taken place;
3 "Unabsorbed depreciation" means so much of the allowance for depreciation of the predecessor
SPC which remains to be allowed and which would have been allowed to the predecessor SPC
under the provisions of this Act, if succession of business had not taken place.

6. Consequences of violation of the conditions stipulated in Proviso to S. 47 (xiii):


1 Where any of the conditions laid down in the proviso to S. 47 (xiv) are not complied with, the
amount of profits or gains arising from the transfer of such CA not charged U/s 45 by virtue of
conditions laid down in the proviso to S. 47 (xiv) shall be deemed to be the profits and gains
chargeable to tax of the successor company for the PY in which the requirements of the proviso
to S. 47 (xiv) are not complied with. [S. 47A (3)]
2 Further, the set-off of loss or allowance of depreciation made in any PY in the hands of the
successor company, shall be deemed to be the income of the company chargeable to tax in the
year in which such conditions are not complied with. [Proviso to S. 72A (6)].

(ZB) Conversion of company into LLP – Tax implications:


(1) Exemption of CG arising from transfer of CA pursuant to conversion of private
company/unlisted public company into LLP - S. 47 (xiiib):
Where there is transfer of a CA by a private company or unlisted public company to a LLP as
a result of conversion of the company into a LLP in accordance with the provisions of S. 56 or
57 of the LLP Act, then it shall not be regarded as transfer. S. 45 does not get invoked.
The question of computing CG does not arise. However the following conditions stipulated in
the Proviso to S. 47 (xiiib) are to be complied with:
(a) all the assets and liabilities of the company immediately before the conversion become the
assets and liabilities of the LLP;
(b) all the shareholders of the company immediately before the conversion become the partners
of the LLP and their capital contribution and profit sharing ratio in the LLP are in the same
proportion as their shareholding in the company on the date of conversion;
(c) the shareholders of the company do not receive any consideration or benefit, directly or
indirectly, in any form or manner, other than by way of share in profit and capital contribution
in the LLP;
(d) the aggregate of the profit sharing ratio of the shareholders of the company in the LLP shall
not be less than 50% at any time during the period of 5 years from the date of conversion;
(e) the total sales, turnover or gross receipts in business of the company in any of the 3 PYs
preceding the PY in which the conversion takes place does not exceed Rs. 60L;
(f) the total value of assets as appearing in the books of accounts of the company in any of the
three PYs preceding the PY in which conversion takes place should not exceed Rs. 5 Crores.
(g) no amount is paid, either directly or indirectly, to any partner out of balance of accumulated
profit standing in the accounts of the company on the date of conversion for a period of three
years from the date of conversion.

Points requiring attention:


S. 47 (xiiib) exempts also CG arising to the shareholders of the converting company due to
extinguishment of their rights in the shares of the company, provided the requirements of
Proviso to S. 47 (xiiib) are complied with.

MT Educare –CA Inter Income Tax Classes – R.SOUMYANARAYANAN.FCA. GRAD CWA. Page-191
Chapter-6 : Capital gains

(2) COA of transferred CA in the hands of LLP – S. 49 (1) (iii) (e):


Where CA becomes the property of LLP on account of conversion of company into LLP, the
COA of that CA in the hands of LLP shall be the COA to the previous owner (i.e. company).

(3) POH of the transferred asset in the hands of LLP – Explanation-1 to S. 2 (42A):
POH of the capital asset transferred from company to the LLP upon conversion shall start on
the date of acquisition by the previous owner (i.e. company).
Note:
The denominator index for indexing the COA in the hands of LLP is the index pertaining to the
PY in which the CA was acquired by the previous owner (i.e. company). [Decision of Bombay
HC in Manjula shah case applied].

(4) Actual cost of block of assets transferred to LLP – Explanation-2C to S. 43 (6):


Where in any PY, any block of assets is transferred by a private company or unlisted public
company to a LLP and the conditions specified in the proviso to S. 47 (xiiib) are satisfied, then,
notwithstanding anything contained in S. 43 (1), the actual cost of the block of assets in the
case of the LLP shall be the WDV of the block of assets as in the case of the said company on
the date of conversion of the company into the LLP.

(5) Computation of depreciation in the year of conversion – Proviso-5 to S. 32 (1):


Where a company gets converted into LLP and the requirements of S. 47 (xiiib) are satisfied,
then in respect of assets transferred pursuant to conversion, both the company and LLP are
eligible for depreciation. For computing the depreciation allowable in the hands of the
predecessor (Company) and the successor (LLP), the following steps are to be followed:
1 Find out the depreciation on the assets as if conversion had not taken place.
2 Then, apportion the depreciation between the predecessor (company) and successor
(LLP), in the ratio of number of days for which the assets are used by them during the
PY in which succession took place.

Illustration:
PQR (P) Ltd has converted into an LLP on 13.06.2019. The following are the particulars as on
31.03.2019:
1 WDV of plant and machinery (15%) Rs. 60L
2 WDV of building (10%) Rs. 90L
3 WDV of furniture (10%) Rs. 10L
Compute the depreciation allowable in the hands of the company and LLP for the AY 20-21.

Solution:
As per Proviso-5 to S. 32 (1), depreciation shall be apportioned between the company and the
LLP in proportion to the number of days the assets were used by them. In such a case, the
depreciation allowable in the hands of PQR Ltd and the LLP would be:

MT Educare –CA Inter Income Tax Classes – R.SOUMYANARAYANAN.FCA. GRAD CWA. Page-192
Chapter-6 : Capital gains

Particulars PQR Ltd (73 days) LLP (292 days)


Machinery Building Furniture Machinery Building Furniture
WDV as on 01.04.2019 Rs. 60L Rs. 90L Rs. 10L Rs. 58.2L Rs. 88.2L Rs. 9.8L
(Exp-2C to S. 43 (6))
Depreciation (Rs. 1.8L) (Rs. 1.8L) (Rs. (Rs. 7.2L) (Rs. 7.2L) (Rs. 0.80L)
apportioned in the ratio 0.20L)
of days of usage (73:292)
WDV as on the DOC Rs. 58.2L Rs. 88.2L Rs. 9.8L Rs. 51L Rs. 81L Rs. 9L

(6) Continuance of deduction U/s 35DDA in the hands of LLP – S. 35DDA (4A):
VRC is allowed as a deduction U/s 35DDA in 5 equal annual installments beginning with the
PY of payment. S. 35DDA (4A) provides that the deduction in respect of VRC paid by company
shall continue in the hands of the succeeding LLP for the unexpired residual period starting
from the PY of conversion.

(7) C/F and set off of losses and depreciation of the company by LLP – S. 72A (6A):
1 Where a company is converted into LLP fulfilling the conditions laid down in S. 47 (xiiib), then,
notwithstanding anything contained in any other provision of this Act, the accumulated loss and the
unabsorbed depreciation of the predecessor company shall be deemed to be the loss or allowance for
depreciation of the successor LLP for the purpose of PY in which conversion took place and other
provisions of this Act relating to set off and C/F of loss and allowance for depreciation shall apply
accordingly.
2 “Accumulated loss” means so much of the loss of the predecessor company U/H PGBP (not being a loss
sustained in a speculation business) which such predecessor company would have been entitled to
carry forward and set off U/s 72 if the succession of business had not taken place;
3 “Unabsorbed depreciation” means so much of the allowance for depreciation of the predecessor
company which remains to be allowed and which would have been allowed to the predecessor
company under the provisions of this Act, if succession of business had not taken place.

(8) Consequences of violation of the conditions prescribed under Proviso to S. 47 (xiiib) –


S. 47A (4) + Proviso to S. 72A (6A):
1 Where any of the conditions laid down in the proviso to S. 47 (xiiib) are not complied with, the amount
of profits or gains arising from the transfer of such capital asset not charged U/s 45 by virtue of
conditions laid down in the proviso to S. 47 (xiiib) shall be deemed to be the profits and gains
chargeable to tax of the successor LLP for the PY in which the requirements of the proviso to S. 47
(xiiib) are not complied with. [S. 47A (4)]
2 Further, the set-off of loss or allowance of depreciation made in any PY in the hands of the successor
LLP, shall be deemed to be the income of the company chargeable to tax in the year in which such
conditions are not complied with. [Proviso to S. 72A (6A)].
3 The capital gains which was exempt U/s 47 (xiiib) in the hands of the shareholders of the converting
company will be taxed in their hands in the PY in the conditions (s) stipulated in the Proviso to S. 47
(xiiib) are violated. [S. 47A (4)].

(9) COA of rights of partner in LLP – S. 49 (2AAA):


For computation of CG arising on account of assignment of interest in LLP which was obtained upon
conversion of company into LLP, the COA of shares in converting company which enabled the
shareholder to become a partner in LLP shall be regarded as COA of the interest in LLP. [S. 49 (2AA)].

MT Educare –CA Inter Income Tax Classes – R.SOUMYANARAYANAN.FCA. GRAD CWA. Page-193
Chapter-6 : Capital gains

(10) POH of rights of partner in LLP:


It starts with the date of becoming partner in LLP and ends with the date preceding the date
of transfer of interest in LLP. The criterion for deciding whether the gain is LT/ST is 36 months.

11. Other transactions which are not to be regarded as transfer:


S. 47 Any transfer of a CA, being a Government security carrying a periodic payment of interest,
(viib) made outside India through an intermediary dealing in settlement of securities, by a NR to
another NR ≠ Transfer.
S. 47 Any transfer of a CA, being any work of art, archaeological, scientific or art collection, book,
(ix) manuscript, drawing, painting, photograph or print, to the Government or a University or
the National museum, National Art Gallery, National Archives or any other public museum
or institution as may be notified by the CG in the Official Gazette to be of national
importance or to be of renown throughout any state or states ≠ Transfer.
S. 47 (xii)
S. 45 will not apply in case of the following transfer and accordingly, there is no CG tax liability for
the transferor:
1 CA Land
2 Transferor Sick industrial company managed by its worker’s cooperative.
3 Transfer Transfer is made pursuant to the scheme sanctioned by BIFR.
4 Timing of The transfer takes place during the period commencing on the first day of the
transfer PY in which the company has become sick and ending with the last day of the
PY in which the net worth of the company >= its accumulated losses.

Exemption in respect of transactions in IFSC (S. 47 (viiab)):


S. 47 (viiab) has been inserted w.e.f AY 2019-20 to provide that a transfer through IFSC will
be exempt. The conditions for exemption are as follows:
(a) There is a transfer of a CA by a NR through RSE;
(b) The capital asset is: (i) a bond referred to in S. 115AC (1); or (ii) a GDR referred to in S. 115AC
(1); or (iii) a RDB of an Indian company; or (iv) a derivative; (v) Such other securities as may be
notified by the CG in this behalf. [FA amendment 2019].
(c) The stock exchange is located in any IFSC;
(d) The consideration for such transaction is paid or payable in foreign currency.

10. Exemptions in respect of CG:


(A) Exemption of LTCG on transfer of property used for residence- U/s 54:
1 Person eligible for Individual & HUF
exemption
2 CA to be transferred for LTCA
availing exemption Residential house whose income is assessable U/H IFHP. (referred to
as original asset)
3 Condition to be satisfied The assessee should invest the CG arising from transfer of original asset
for availing exemption in one residential house located in India (referred to as new asset).
However, if the CG arising from transfer of original asset ≤ Rs. 2 Crores,
the assessee can invest in two residential houses located in India. But

MT Educare –CA Inter Income Tax Classes – R.SOUMYANARAYANAN.FCA. GRAD CWA. Page-194
Chapter-6 : Capital gains

this could be done only once in the life-time of the assessee. [Finance
Act 2019 Amendment].
4 Manner of investment The assessee can purchase a residential house.
Alternatively, he can also construct a residential house.
5 Within what time the new Within 1 year before the DOT of original asset or
asset is to be purchased? Within 2 years from the DOT of original asset.
6 Within what time the new Within 3 years from the DOT of original asset.
asset is to be
constructed?
7 Quantum of exemption LTCG arising from transfer of original asset or the amount invested in
the new asset (whichever is less).
8 Depositing in CG account Where it is not possible for the assessee to make investment in the new
scheme (CGDA Scheme) asset within the due date for filing ROI (specified in S. 139 (1)), still he
can avail exemption if he deposits the CG under CGDA scheme.
9 Utilization of amount The amount so deposited should be utilised for investment in the new
deposited asset within the aforesaid time limit.
10 Consequences of non- If the amount deposited is not fully utilised for investment in the new
utilization or partial asset within the aforesaid time limit, then the unutilized amount should
utilization of amount be treated as LTCG of the PY in which the period of 3 years from the
deposited in the aforesaid DOT of original asset expires.
account
11 Withdrawal of unutilized The assessee can withdraw the unutilised amount from the deposit
amount for any other account for any other purpose only after a period of 3 years from the
purpose DOT of original asset.
12 Withdrawal of exemption The exemption granted will be withdrawn if the new asset is
upon sale of the new transferred within 3 years from the DOA.
asset
14 PY in which the The exemption will be withdrawn in the PY in which the new asset is
exemption is withdrawn transferred.
15 Manner of withdrawal of See the table given below
exemption

Computation of CG upon transfer of new asset within 3 years from the DOA and
withdrawal of exemption:
1 FVC for new asset *****
Less: Expenses wholly and exclusively incurred in connection with
transfer of new asset *****
2 Net consideration *****
3 COA of new asset *****
Less: Exemption granted U/s 54 (to the extent remaining unwithdrawn) *****
4 Net COA of new asset *****
5 COI of new asset *****
6 STCG / LTCG (2-4-5) *****

MT Educare –CA Inter Income Tax Classes – R.SOUMYANARAYANAN.FCA. GRAD CWA. Page-195
Chapter-6 : Capital gains

Points requiring attention:


1 Cost of land The cost of the land is an integral part of the cost of the
residential house, whether purchased or constructed. In
computing the amount of CG invested, this shall also be
considered. (CBDT Circular 667).
2 Time limit in case of compulsory Where the original asset is compulsorily acquired, the time
acquisition (S. 54 H) limit U/s 54 shall be reckoned considering only the date of
receipt of compensation and not the date of compulsory
acquisition.
3 Investment not in the name of There is no bar in making investment in the name of close
the assessee but in the name of kin and kith. For enjoying exemption, the Act requires
his close kin and kith investment and not investment in the name of assessee.
[Gurnam Singh (P&H) + Ravinder Kumar Arora (Delhi) +
Mrs. Jennifer Bhide (Kar) + Kamal Wahal (Delhi) +
Natarajan (Mad)].
4 Can exemption U/s 54 be denied Investment of CG is a pre-requisite for exemption U/s 54
on the ground that the and not completion of construction. [Sambandam
construction of RHP was not Udaykumar (Kar)].
completed in all aspects within 3
years from the DOT of original
asset?
5 Whether exemption U/s 54 No. [Chandru L. Raheja (Bom) + C. Aryama Sundaram
could be denied merely on the (2018) (Mad)].
ground that the construction of
new house had begun before the
sale of original asset?
6 Builder handed over the If the payment is made to the builder within the time limit
possession of flat to the assessee specified, even though the builder might have handed over
beyond the time-limit specified the possession of flat to the assessee beyond the time limit
in S. 54. Whether CG exemption specified, exemption U/s 54 is available. The word
would be available? ‘purchase’ in S. 54 should not be understood in the sense of
legal transfer. [R. L. Sood (Del) + Balraj (Del)].
(B) Exemption w.r.t LTCG arising on account of transfer of a LTCA (not being a residential
house property) – S. 54 F:
1 Person eligible for Individual & HUF
exemption
2 CA to be transferred for LTCA
availing exemption Any CA other a residential house. (called original asset)
3 Condition to be satisfied The assessee should invest in one residential house in India (referred
for availing exemption to as new asset).
On the DOT of original asset, he should not own more than a
residential house other than the new asset.
4 Manner of investment The assessee can purchase a residential house.
Alternatively, he can construct a residential house.
5 Within what time the Within 1 year before the DOT or
new asset is to be Within 2 years from the DOT.
purchased?

MT Educare –CA Inter Income Tax Classes – R.SOUMYANARAYANAN.FCA. GRAD CWA. Page-196
Chapter-6 : Capital gains

6 Restriction on Assessee should not purchase a residential house other than the new
purchasing a residential asset within 2 year from the DOT of original asset.
house other than the
new asset
7 Within what time the Within 3 years from the DOT of original asset.
new asset is to be
constructed?
8 Restriction on Assessee should not complete the construction of a residential house
completion of other than the new asset within 3 years from the DOT.
construction of a RHP
other than the new
asset
9 Quantum of exemption (LTCG arising from transfer of the original asset * Amount invested in
new asset)/Net consideration
10 Net consideration FVC accruing from transfer of original asset (Less) Expenses wholly &
exclusively incurred for the purpose of transfer of original asset.
11 Depositing in CGDA Where it is not possible for the assessee to make investment in the
scheme new asset within the due date for filing ROI (U/s 139), still he can avail
exemption if he deposits the amount proposed to be invested in new
asset under CGDA Scheme.
12 Utilization of amount The amount so deposited should be utilised for investment in new
deposited asset within the TL (supra).
13 Consequences of non- If the amount deposited is not fully utilised for investment in the new
utilization or partial asset within the aforesaid time limit, then [(LTCG*Unutilized
utilization amount)/Net consideration] should be treated as LTCG of the PY in
which the period of 3 years from the DOT of original asset expires.
14 Withdrawal of The assessee can withdraw the unutilised amount from the deposit
unutilised amount account for any other purpose only after a period of 3 years from the
DOT of original asset.
15 Withdrawal of If the new asset is transferred within 3 years from the DOA, the LTCG
exemption upon sale of which arose on transfer of original asset and which got exempted U/s
the new asset. 54F, would be assessed to tax as LTCG in the PY in which the new asset
is sold.
In addition, in that year, LTCG or STCG arising on account of transfer
of the new asset will also be assessed to tax.
16 Consequences of If the assessee purchases a residential house, the income of which is
assessee purchasing a chargeable U/H IFHP, other than the new asset within 2 years from
residential house other the DOT of original asset, LCTG, exempted earlier, will be assessed to
than the new asset tax as LTCG of PY in which the residential house is purchased
within 2 years from DOT
of original asset
17 Consequences of If the assessee completes the construction of a residential house, the
assessee completing the income of which is chargeable U/H IFHP, other than the new asset
construction of a within 3 years from the DOT of original asset, LCTG, exempted earlier,
residential house other will be assessed to tax as LTCG of the PY in which the construction of
than the new asset residential house is completed.

MT Educare –CA Inter Income Tax Classes – R.SOUMYANARAYANAN.FCA. GRAD CWA. Page-197
Chapter-6 : Capital gains

within 3 years from DOT


of original asset

Points requiring attention:


1 Points discussed U/s 54 shall hold good even for S. 54F.
2 Where the SDV U/s 50C has been adopted as the FVC, the reinvestment made in acquiring a
residential property, which is in excess of the actual net sale consideration, shall be considered
for the purpose of computation of exemption U/s 54F, irrespective of the source of funds for
such reinvestment. [Gouli Mahadevappa (2013) 356 ITR 90 (Kar)].

(C) Exemption w.r.t CG arising on transfer of land used for agricultural purposes - S. 54B:
1 Person eligible for Individual & HUF.
exemption
2 CA to be transferred Urban agricultural land.
for availing exemption It may be a LTCA or STCA. (called original asset)
It should have been used for agricultural purposes either by the
assessee or by his parents or by HUF during the period of 2 years
immediately preceding the DOT.
3 Conditions to be The assessee should invest in an agricultural land (referred to as new
satisfied for availing asset).
exemption The agricultural land may be urban or rural.
4 Within what time the Within 2 years from the DOT of original asset.
new asset is to be
purchased?
5 Quantum of CG or Amount invested in new asset (whichever is less)
exemption
6 Depositing in CGDA Where it is not possible for the assessee to make investment in the
scheme new asset within the due date for filing ROI (U/s 139 (1)), still he can
avail exemption if he deposits the amount proposed to be invested in
new asset in CGDA scheme.
7 Utilization of amount The amount so deposited should be utilized for investment in new
deposited asset within the time limit (supra).

8 Consequences of non- If the amount deposited is not fully utilized for investment in the new
utilization or partial asset within the aforesaid time limit, then the unutilized amount
utilization should be treated as CG of the PY in which the period of 2 years from
the DOT of original asset expires.
9 Withdrawal of The assessee can withdraw the unutilized amount from the deposit a/c
unutilized amount for any other purpose only after a period of 2 years from the DOT of
original asset.
10 Withdrawal of The exemption granted will be withdrawn if the new asset is
exemption upon sale transferred within 3 years from the DOA.
of the new asset
13 PY in which the The exemption will be withdrawn in the PY in which the new asset is
exemption is transferred.
withdrawn

MT Educare –CA Inter Income Tax Classes – R.SOUMYANARAYANAN.FCA. GRAD CWA. Page-198
Chapter-6 : Capital gains

14 Manner of withdrawal Same as in S. 54.


of exemption

(D) Exemption of CG from compulsory acquisition of L&B forming part of an IU - S. 54 D:


1 Person eligible for Any assessee owning industrial undertakings.
exemption
2 CA to be Land or building. (referred to as original asset)
transferred for It may be LTCA or STCA.
availing It should have been used for purpose of business of the industrial undertaking
exemption for a period of atleast 2 years immediately preceding the DOT.
It should have been compulsorily acquired.
3 Conditions to be The assessee should invest in another land or building (referred to as new
fulfilled for asset) within the stipulated time.
availing Such land or building should be used for the purpose of re-establishing the
exemption aforesaid industrial undertaking or for the purpose of setting up of a new
industrial undertaking.
4 Manner of The assessee can purchase land or building or
investment He can construct building.
5 TL for investment The assessee can purchase the new asset within a period of 3 years from the
in new asset. date of receipt of compensation.
6 Quantum of Lower of (i) CG or (ii) Amount invested in new asset.
exemption
7 Depositing in Where it is not possible for the assessee to make investment in the new asset
CGSA scheme within the due date for filing ROI, still he can avail exemption if he deposits the
amount proposed to be invested in new asset under the CGDA scheme”.
8 Utilization of The amount so deposited should be utilized for investment in the new asset
amount deposited within the aforesaid time limit.
9 Consequences of If the amount deposited is not fully utilized for investment in the new asset
non-utilization or within the aforesaid time limit, then the unutilized amount should be treated
partial utilization as CG of the PY in which 3 years from the date of receipt of compensation
expires.
10 Withdrawal of The assessee can withdraw the unutilized amount from the deposit account for
unutilized amount any other purpose only after a period of 3 years from the date of receipt of
compensation.
11 Withdrawal of The exemption granted will be withdrawn if the new asset is transferred within
exemption upon 3 years from the DOA.
sale of the new
asset
12 PY in which the The exemption will be withdrawn in the PY in which the new asset is
exemption is transferred.
withdrawn
13 Manner of Same as in S. 54.
withdrawal of
exemption

MT Educare –CA Inter Income Tax Classes – R.SOUMYANARAYANAN.FCA. GRAD CWA. Page-199
Chapter-6 : Capital gains

(E) Exemption of LTCG from immovable property upon investment in certain bonds (S.
54EC):
1 Person eligible for exemption Any assessee.
2 CA to be transferred for Any LTCA being land or building or both. (called original asset)
availing exemption
3 Conditions to be fulfilled for The assessee should invest in specified assets within 6 months
availing exemption from the DOT of the asset.
4 Specified asset Bonds issued by NHAI or REC or any other bond notified by the
CG in the official gazette redeemable after 5 years.
5 Bonds notified by the CG Bonds issued by Power finance corporation.
Bonds issued by Indian railway finance corporation.
6 Quantum of exemption LTCG or Amount invested in specified asset (whichever is less).
7 Ceiling on investment in Investment in specified asset in a FY shall not be more than Rs.
specified asset 50L
Investment in specified asset in the FY of transfer and in the
succeeding FY, out of CG arising on account of transfer of LTCA,
shall not exceed Rs. 50L.
8 Withdrawal of exemption If the specified assets are transferred within 3 years from the
upon transfer of specified DOA, the LTCG arising from the transfer of original asset which
assets was not charged to tax, will be deemed to be the LTCG of the PY
in which specified assets are transferred.
In addition, the CG arising on account of transfer of the specified
assets shall also be assessed to tax.
9 Availing loan or advance on the If loan or advance is taken on the security of specified assets
security of specified asset within a period of 5 years of its acquisition, the exemption
within 3 years – impact. granted U/s 54EC shall be withdrawn.

Some important judicial pronouncements:


1 Exemption U/s 54EC cannot be denied on account of the bonds being issued after six months of
the date of transfer even though the payment for the bonds was made by the assessee within the
six month period. [Hindustan Unilever Ltd (2010) (Bom)].
2 An assessee cannot be deprived of claiming exemption U/s 54EC, if bonds of assessee’s choice are
not available or are available only for a broken period within the period of 6m after the DOT of CA
and the bonds are purchased shortly after it became available next time after the expiry of the said
6m. [Cello Plast (2012) (Bom)].
3 In a case where a depreciable asset held for more than 36 months is transferred, the benefit of
exemption U/s 54EC be claimed, if the capital gains on sale of such asset are reinvested in long-
term specified assets within the specified time. [V.S. Dempo Company Ltd (SC)].

(F) Exemption of CG arising from transfer of a LTCA upon investment in notified units of
specified fund (S. 54EE):
1 Person eligible for Any assessee.
exemption
2 CA to be transferred for Any LTCA. (referred to as original asset).
availing exemption

MT Educare –CA Inter Income Tax Classes – R.SOUMYANARAYANAN.FCA. GRAD CWA. Page-200
Chapter-6 : Capital gains

3 Conditions to be fulfilled for The assessee should invest in notified units of specified fund within
availing exemption 6 months from the DOT of the LTCA.
4 Quantum of exemption LTCG or Amount invested in notified units (whichever is less).
5 Ceiling on investment in Investment in notified units in a FY shall not be more than Rs. 50L
notified units Investment in notified units in the FY of transfer and in the
succeeding FY, out of CG arising on account of transfer of LTCA, shall
not exceed Rs. 50L.
6 Withdrawal of exemption If the notified units are transferred within 3 years from the date of
upon transfer of notified their acquisition, the LTCG arising from the transfer of original asset
units which was not charged to tax, will be deemed to be the LTCG of the
PY in which notified units are transferred.
In addition, the CG arising on account of transfer of the notified
units shall also be assessed to tax.
7 Availing loan or advance on If loan or advance is taken on the security of notified units within a
the security of notified period of 3 years of its acquisition, the exemption granted U/s 54EE
units within 3 years – shall be withdrawn.
impact.

11. Computation of tax on certain STCG – S. 111A:


S. 111A (1) STCG covered by this STCG arising from transfer of STCA being (a) equity shares;
section or (b) EOU; or (c) units of a business trust which is subjected
to STT.
Concessional tax rate STCG (supra) suffers tax @ 15% (flat).
Proviso-1 Benefit of If the BEL of a resident individual or HUF is not exhausted by
to S. 111A unexhausted BEL income other than STCG (referred to in this section), then
(1) the unexhausted BEL shall be adjusted against this STCG and
the balance STCG shall alone be subjected to tax @ 15%.
Proviso-2 Relaxation in respect Where the transaction resulting in aforesaid STCG is
to S. 111A of transaction effected effected in a RSE located in IFSC and the consideration is
in a RSE located in IFSC payable in foreign currency, then the concessional rate
stipulated in this section shall apply to such STCG even if the
transaction is not subjected to STT.
S. 111A (2) No chapter VI-A Where the GTI of the assessee includes STCG (covered by
deduction. this section), then that portion of GTI is not eligible for
deduction under Chapter VI-A.

Note:
1 Chapter VI-A deduction could be enjoyed against that portion of GTI which represents
STCG (not covered by S. 111A).
2 STCG (not covered by S. 111A) shall suffer tax at the rates given in the Finance Act.

MT Educare –CA Inter Income Tax Classes – R.SOUMYANARAYANAN.FCA. GRAD CWA. Page-201
Chapter-6 : Capital gains

12. Tax on LTCG in certain cases – S. 112A:


S. 112A Applicability If the following conditions are fulfilled, then the provisions of S. 112A
(1) shall apply:
C-1: The TI of the assessee includes income chargeable U/H CG.
C-2: The CG arises from the transfer of a LTCA.
C-3: LTCA = Equity share in a company or Equity oriented unit or unit of
a business trust.
C-4: In case of equity shares, STT has been paid on both acquisition as
well as transfer of shares.
C-5: In case of EOU or units of business trust, STT has been paid on
transfer.
S. 112A Relaxation from If the transfer is undertaken on a RSE located in any IFSC and the
(3) C-4 and C-5. consideration for such transfer is received or receivable in foreign
currency, then S. 112A shall apply even if C-4 and C-5 are not fulfilled.
S. 112A Relaxation from If the transfer is in respect of a type of acquisition of equity shares which
(4) C-4 is notified by the CG in the OG, then S. 112A shall apply even if C-4 is not
fulfilled.
S. 112A Computation of Tax on LTCG = [LTCG (supra) – Rs. 1L] * 10%
(2) tax on LTCG
referred to in S.
112A (1)
Proviso Benefit of In case of resident individual or HUF, benefit of unexhausted BEL is
to S. unexhausted BEL. available.
112A
(2)
S. 112A No Chapter VI-A No chapter VI-A deduction is available against that portion of GTI which
(5) deduction represents LTCG (supra).
S. 112A No rebate U/s Where the TI of the assessee includes LTCG (supra), then the rebate U/s
(6) 87A 87A shall be allowed from the income tax on the TI as reduced by tax
payable on such CG.
Note: Nothing contained in S. 48 proviso-1 and proviso-2 shall apply to the CG arising from
transfer of LTCA being equity shares or EOU or units of business trust referred to in S. 112A.

Notification of transactions in equity shares in respect of which the condition of


chargeability to STT at the time of acquisition for claiming concessional tax treatment U/s
112A shall not apply [Notification No.60/2018]
The Finance Act, 2018 has withdrawn exemption U/s 10 (38) and has inserted new S. 112A to
provide that LTCG arising from transfer of a CA, being an equity share in a company or a unit
of an equity oriented fund or a unit of a business trust, shall be taxed at 10% of such CG
exceeding Rs. 1L.

The said section, inter alia, provides that the provisions of the section shall apply to the CG
arising from a transfer of LTCA, being an equity share in a company, only if STT has been paid
on acquisition and transfer of such CA.
However, to provide for the applicability of the concessional tax regime U/s 112A to genuine
cases where the STT could not have been paid, it has also been provided in S. 112A (4) that
the CG may specify, by notification, the nature of acquisitions in respect of which the

MT Educare –CA Inter Income Tax Classes – R.SOUMYANARAYANAN.FCA. GRAD CWA. Page-202
Chapter-6 : Capital gains

requirement of payment of STT shall not apply in the case of acquisition of equity share in a
company.
In view of the above, the CG has, vide notification No. 60/2018, notified that the condition of
chargeability of STT shall not apply to the acquisition of equity shares entered into
(i) before 01.10.2004 or
(ii) on or after 01.10.2004 which are not chargeable to STT, other than the following
transactions.

In effect, only in respect of the following transactions mentioned in column (2), the
requirement of paying STT at the time of acquisition for availing the benefit of concessional
rate of tax U/s 112A would apply. It may be noted that the exceptions are listed in column (3)
against the transaction.
The requirement of payment of STT at the time of acquisition for availing benefit of
concessional tax rate U/s 112A will not apply to acquisition transactions mentioned in
column(3).

(1) (2) (3)


Transaction Non-applicability of condition of chargeability of STT
(a) Where acquisition of existing Where acquisition of listed equity share in a company–
listed equity share in a (i) has been approved by the SC, HC, National Company
company whose equity Law Tribunal, SEBI or RBI in this behalf;
shares are not frequently
traded in a RSE is made (ii) is by any NR in accordance with FDI guidelines issued by
through a preferential issue the Government of India;
(iii) is by an investment fund referred to in clause (a) of
Explanation 1 to S. 115UB or a VCF referred to in S. 10
(23FB) or a Qualified Institutional Buyer;

(iv) is through preferential issue to which the provisions of


chapter VII of the SEBI (Issue of Capital and Disclosure
Requirements) Regulations, 2009 does not apply.

(b) Where transaction for Following acquisitions of listed equity share in a company made
acquisition of existing listed in accordance with the provisions of the SCRA 1956:
equity share in a company is
not entered through a RSE in (i) acquisition through an issue of share by a company other
India than through preferential the issue referred to in (a);

(ii) acquisition by scheduled banks, reconstruction


or securitisation companies or public financial
institutions during their ordinary course of business;

(iii) acquisition by the SC, HC, National Company Law


Tribunal, SEBI or RBI in this behalf;
(iv) acquisition under employee stock option scheme or
employee stock purchase scheme framed under the SEBI
(Employee Stock Option Scheme and Employee Stock
Purchase Scheme) Guidelines,1999;

MT Educare –CA Inter Income Tax Classes – R.SOUMYANARAYANAN.FCA. GRAD CWA. Page-203
Chapter-6 : Capital gains

(v) acquisition by any non-resident in


accordance with FDI guidelines of the Government of
India;
(vi) acquisition in accordance with SEBI (Substantial
Acquisition of Shares and Takeovers) Regulation, 2011;

(vii) acquisition from the Government;


(viii) acquisition by an investment fund referred to in clause
(a) to Explanation 1 to S. 115UB or a VCF referred to in S.
10 (23FB) or a Qualified Institutional Buyer;
(ix) acquisition by mode of transfer referred to in S. 47 (e.g.,
transfer of CA under a gift, an irrevocable trust, transfer
of CA between holding company and its subsidiary,
transfer pursuant to amalgamation, demerger, etc.) or S.
50B (slump sale) or S. 45(3) (Introduction of CA as
capital contribution in firm/ AOPs/ BOIs) or S. 45(4)
(Distribution of CA on dissolution of firm/ AOPs/ BOIs), if
the previous owner or the transferor, as the case may be,
of such shares has not acquired them by any mode
referred to in (a), (b) or (c) listed in column (2) [other
than the exceptions listed in column(3)]
(c) acquisition of equity share of
a company during the period
beginning from the date on
which the company is
delisted from a RSE and
ending on the date
immediately preceding the
date on which the company is
again listed on a RSE in
accordance with the SCRA
1956 read with SEBI Act,1992
and the rules made there
under;

13. Computation of tax on LTCG – S. 112:


S. 112 Computation of tax on LTCG in On that portion of GTI which represents LTCG, tax shall be
(1) (a) case of a resident individual or computed @ 20% (flat).
HUF.
Proviso Benefit of adjustment of If the BEL of a resident individual or HUF is not exhausted
to S. unexhausted BEL. by income other than LTCG, then the unexhausted BEL
112 (1) shall be adjusted against LTCG and the balance LTCG shall
(a) alone be subjected to tax @ 20%.
S. 112 Computation of tax on LTCG in On that portion of GTI which represents LTCG, tax shall be
(1) (b) case of a domestic company. computed @ 20% (flat).

MT Educare –CA Inter Income Tax Classes – R.SOUMYANARAYANAN.FCA. GRAD CWA. Page-204
Chapter-6 : Capital gains

S. 112 Computation of tax on LTCG in Tax on LTCG arising from transfer of LTCA other than
(1) (c) case of a foreign company or a unlisted securities or shares in a CHC = 20% of LTCG. [S.
non-corporate non-resident. 112 (1) (c) (ii)].
Tax on LTCG arising from transfer of LTCA being unlisted
securities or shares in a CHC = 20% on LTCG computed
without giving effect to S. 48 Proviso-1 and Proviso-2. [S.
112 (1) (c) (iii)].
S. 112 Computation of tax on LTCG in On that portion of GTI which represents LTCG, tax shall be
(1) (d) case of any other resident. computed @ 20% (flat).
Proviso Relaxation in certain cases. Where the LTCA transferred happens to be (a) listed
to S. securities (not being units); or (b) ZCB, then the tax on
112 (1) LTCG computed U/s 112 (1) to the extent of excess over
10% of LTCG computed without giving effect to S. 48
Proviso-2 (indexation benefit) shall be ignored for the
purpose of computing the tax payable by the assessee.
S. 112 No chapter VI-A deduction Where the GTI of the assessee includes LTCG, then that
(2) portion of GTI is not eligible for deduction under Chapter
VI-A.

14. Reference to DVO – S. 55A:


This section provides that the AO may refer the valuation of CA to the DVO in the following
circumstances with a view to ascertaining the FMV of the CA for the purposes of CG:
(i) In a case where the value of the asset as claimed by the assessee is in accordance with
the estimate made by a registered valuer, if the AO is of the opinion that the value so
claimed is at variance with its FMV.
(ii) If the AO is of the opinion that the FMV of the asset exceeds the value of asset as
claimed by the assessee by more than 15% of the value of the asset as claimed or by
more than Rs. 25000 of the value of the asset as claimed by the assessee.
(iii) The AO is of the opinion that, having regard to the nature of asset and other relevant
circumstances, it is necessary to make the reference.

MT Educare –CA Inter Income Tax Classes – R.SOUMYANARAYANAN.FCA. GRAD CWA. Page-205
Chapter-7: IFOS

1. Charging section – S. 56:


S. 56 Any income which is not exempt under this Act and which is not chargeable to tax U/H (a)
(1) Salaries; or (b) IFHP; or (c) PGBP; or (d) CG, shall be charged to tax U/H IFOS.
S. 56 (i) Dividend [discussed in this chapter].
(2) (ii) Casual income (winnings from lotteries, crossword puzzles, races, card games and
other games, gambling, betting etc). [Discussed in Basic concepts].
(iii) Gifts or benefits arising on account of acquisition of immovable property or specified
movable property for inadequate consideration. [Discussed at the end of this chapter].
(iv) Interest received on compensation or enhanced compensation. (S. 145B (1)).
[Discussed in IFOS (Part-2)].
(v) Advance forfeited due to failure of negotiations of transfer of a capital asset.
[Discussed in Capital gains chapter].
(vi) Family pension. [Discussed in Salary chapter].
(vii) Interest on securities (if not chargeable to tax U/H PGBP).
(viib) Share premium in excess of fair market value
Where a company, not being a company in which the public are substantially interested,
receives, in any previous year, from any person being a resident, any consideration for
issue of shares that exceeds the face value of such shares, the aggregate consideration
received for such shares as exceeds the fair market value of the shares shall be treated
as income of the company.
The provision is not applicable where the consideration for issue of shares is received:
(i) by a venture capital undertaking from a venture capital company or a venture
capital fund or a specified fund; or
Specified fund means a fund established or incorporated in India in the form of a
trust or a company or a limited liability partnership or a body corporate which has
been granted a certificate of registration as a Category I or a Category II Alternative
Investment Fund and is regulated under the Securities and Exchange Board of India
(Alternative Investment Fund) Regulations, 2012 made under the Securities and
Exchange Board of India Act, 1992.
(ii) by a company from a class or classes of persons as may be notified by the
Central Government in this behalf.
(viii) Income from letting out on hire of machinery, plant or furniture (if not chargeable to
tax U/H PGBP).
(ix) Composite rent charged under inseparable agreement (if not chargeable to tax U/H
PGBP). [Discussed in IFHP chapter].
(x) Key man insurance policy proceeds (if not chargeable to tax U/H PGBP or U/H Salary).
[Discussed in Salary Chapter].
(xi) Any sum received by an employer from his employees as contribution to any
Provident fund, superannuation fund, or any other fund for the welfare of the
employees. [Discussed in detail in PGBP Chapter].
(xii) Income referred to in S. 56 (2) (viib). [Discussed in detail in this chapter].
(xiii) Any compensation or other payment, due to or received by any person, by
whatever name called, in connection with termination of his employment or the
modification of the terms and conditions relating thereto. [S. 56 (2) (xi)].
Note: Unless the Act provides otherwise, income U/H IFOS shall be computed in accordance
with the method of accounting regularly employed by the assessee. [S. 145 (1)]. Also, the

MT Educare –CA Inter Income Tax Classes – R.SOUMYANARAYANAN.FCA. GRAD CWA. Page-206
Chapter-7: IFOS

relevant ICDS notified by the Government shall be complied with in computing the income
chargeable U/H IFOS.

2. Deductions allowable U/H IFOS – S. 57:

S. 57 (i) Realisation charges (commission or remuneration paid to banker or agent for


realisation of dividend or interest) incurred in relation to dividend or interest
shall be allowed as deduction while computing the taxable dividend or interest.
S. 57 (ia) PF contributions of employee deposited into the employee’s PF account within
the due date specified in the PF regulations shall be allowed as deduction.
S. 57 (ii) Where income from letting out on hire of machinery, plant or furniture or
composite rent is chargeable U/H IFOS, the following could be allowed as
deduction: (a) Repairs; (b) insurance premium; (c) depreciation.
S. 57 (iia) Standard deduction in respect of family pension = Rs. 15000 or 1/3 of family
pension (whichever is less). [Discussed in Salary Chapter].
S. 57 (iii) Any other expenditure wholly and exclusively incurred for the purpose of
making or earning income chargeable U/H IFOS shall be allowed as deduction.
S. 57 (iv) 50% of interest on compensation or enhanced compensation shall be allowed as
deduction while computing income U/H IFOS.

3. Disallowable expenses – S. 58:

Following are disallowable while computing income chargeable U/H IFOS:

1 Any personal expenses of the assessee


2 Any interest chargeable to tax under the Act which is payable outside India on which tax
has not been paid or deducted at source.
3 Any payment taxable in India as salaries, if it payable outside India unless tax has been
paid thereon or deducted at source.
4 Any payment to a relative or associate person which is excessive or unreasonable.
5 Aggregate payments exceeding Rs. 10000 made to a person in a day otherwise than by
account payee cheque or draft or ECS through bank account.
6 30% of expenditure in respect of which sum is payable to a resident on which tax is
deductible at source, if such tax has not been deducted or after deduction has not been
paid on or before the due date of return specified in S. 139 (1).

4. Taxation of dividend:

(A) Meaning of dividend in common parlance:

Dividend means sum distributed by a company to its shareholders of its divisible profits in
the proportion of their shareholding.

MT Educare –CA Inter Income Tax Classes – R.SOUMYANARAYANAN.FCA. GRAD CWA. Page-207
Chapter-7: IFOS

(B) Classification of dividend:

1 Final Dividend declared by the company at its AGM upon recommendation of


dividend the BOD.
2 Interim Dividend made available to the shareholders out of current year’s profits
dividend before the end of the year.
3 Deemed There are certain distributions or payments which are not generally
dividend understood as dividend but are regarded as so in S. 2 (22) (a) to (e).

(C) Head under which dividend is taxable:

Dividend, if it is taxable, shall be taxed U/H IFOS. [S. 56 (2) (i)]. The position is same even if
the assessee (a dealer in shares) holds the shares as stock-in-trade.

(D) Basis of charge of dividend – S. 8:

Irrespective of the method of accounting employed by the assessee, the dividend shall be
charged to tax in a manner laid down in S. 8:

1 Final Final dividend, if taxable, shall be taxed in the PY in which it is declared


dividend in the AGM.
2 Interim Interim dividend, if taxable, shall be taxed in the PY in which it is
dividend unconditionally made available to the shareholders.
3 Deemed Deemed dividend, if taxable, shall be taxed in the PY in which
dividend distribution or payment is made.

(E) Taxability of dividend:

SN Dividend Tax implications in the hands Tax implications in the


of shareholders hands of company
1 Dividend from foreign Taxable in the hands of No obligation to pay DDT
company shareholders. No exemption U/s 115-O.
U/s 10 (34)
2 S. 2 (22) (e) dividend Exempt U/s 10 (34) Obligation to pay DDT U/s
from domestic company 115-O @ 34.994%.
3 Dividend (not being S. 2 Exempt U/s10 (34). [Subject Company has obligation to
(22) (e)) from domestic to S.115BBDA] pay DDT U/s 115-O @
company 20.555%.

Note: Deductions U/s 57 (i) or S. 57 (iii) shall be allowed only when the dividend is taxable. If
dividend is exempt, then the corresponding expenses are to be disallowed U/s 14A.

MT Educare –CA Inter Income Tax Classes – R.SOUMYANARAYANAN.FCA. GRAD CWA. Page-208
Chapter-7: IFOS

(F) Provisions of S. 115BBDA:

1 Dividend (not being dividend referred to in S. 2 (22) (e) from domestic companies
exceeding Rs. 10L received by a resident being specified assessee shall be charged to tax
@ 10% (flat).
2 No deduction in respect of any expenditure or allowance or set off shall be allowed.
3 Specified assessee means any person other than (a) a domestic company; (b) entities
referred to in S. 10 (23C) (iv)/(v)/(vi)/(via); (c) a trust or institution registered U/s 12AA.

(G) Deemed dividend U/s 2 (22):

1 S. 2 (22) (a) Dividend includes any distribution by a company of its accumulated profits, if such
distribution entails release by the company to its shareholders of all or any part of
the assets of the company.
Issue of equity shares as bonus shares to the equity shareholders ≠ dividend u/s 2
(22) (a). [Reason: Issue of bonus shares does not entail release of assets.]
2 S. 2 (22) (b) Dividend includes any distribution to its shareholders by a company of
debentures, debenture-stock, or deposit certificates in any form, whether with or
without interest, and any distribution to its preference shareholders of shares, by
way of bonus, to the extent to which the company possesses accumulated profits.
3 S. 2 (22) (c) Dividend includes any distribution made to shareholders of a company on its
liquidation, to the extent to which the distribution is attributable to the
accumulated profits of the company immediately before its liquidation.
[Discussed in Capital gains chapter].
4 S. 2 (22) (d) Dividend includes any distribution to its shareholders by a company on the
reduction of its capital to the extent to which the company possessed
accumulated profits.
5 S. 2 (22) (e) Dividend includes payment by way of loan or advance made by a closely held
Limb-1 company to its major shareholder, to the extent to which the company possesses
accumulated profits.
Major shareholder means shareholder being a person who beneficially owns
equity shares carrying not less than 10% voting power.
6 S. 2 (22) (e) Dividend includes payments made by a closely held company on behalf of major
Limb-2 shareholder, to the extent to which the company possessed accumulated profits.
7 S. 2 (22) (e) Dividend includes payments made by a closely held company for the individual
Limb-3 benefit of major shareholder, to the extent to which the company possessed
accumulated profits.
8 S. 2 (22) (e) Dividend includes payments by way of loan or advance made by a closely held
Limb-4 company to a concern in which the major shareholder is a member or partner and
has substantial interest, to the extent to which the company possessed
accumulated profits.
Substantial interest means (a) beneficial ownership in equity shares carrying not
less than 20% voting power (if the concern = company); (b) beneficial entitlement
to not less than 20% of the profits (if the concern ≠ company).

MT Educare –CA Inter Income Tax Classes – R.SOUMYANARAYANAN.FCA. GRAD CWA. Page-209
Chapter-7: IFOS

Points requiring attention:

1 S. 2 (22) (e) Limb-4 dividend is taxable in the hands of major shareholder and not in the
hands of concern.
2 If the loan is granted in the ordinary course of its business and lending of money is
substantial part of the company’s business, the loan or advance to the major
shareholder or the concern in which he is substantially interested shall not be regarded
as dividend u/s 2 (22) (e).
3 Where a loan has been treated as dividend and subsequently the company declares and
distributes dividend to all its shareholders including the borrowing shareholder, and the
dividend so paid is set off by the company against the previous borrowing, the adjusted
amount shall not again treated as a dividend.
4 Trade advances are not covered by S. 2 (22) (e). [CBDT Circular 19/2017].
5 Distribution made by a company on purchase of its own shares from a shareholder ≠
dividend U/s 2 (22) (d).

5. Casual income:

1 Casual income means income in the nature of winning from lotteries, crossword puzzles,
races including horse races, card games and other games of any sort, gambling, betting
etc.
2 Such winnings are chargeable to tax @ flat rate of 30%. [S. 115BB].
3 No deduction shall be allowed in respect of any expenditure or allowance.
4 Deduction under Chapter VI-A is not allowable from such income.
5 Adjustment of unexhausted basic exemption limit is also not permitted against such
income.

6. Interest on securities:
Interest on securities (if not chargeable to tax U/H PGBP) shall be charged to tax U/H IFOS.
However, the following interest income arising to certain persons would be exempt from tax
U/s 10 (15):

1 Interest on Post Office Savings Bank is exempt to the extent of (a) Rs. 3500 (in case of
an individual account); (b) Rs. 7000 (in case of a joint account).
2 Interest on tax free bonds issued by India Infrastructure company Ltd.
3 Interest on tax free, secured, redeemable, non-convertible bonds issued by Indian
Railway Finance Corporation Ltd.
4 Interest on tax free, secured, redeemable, non-convertible bonds issued by National
Highways Authority of India.
5 Interest on tax free, secured, redeemable, non-convertible bonds issued by Rural
Electrification Corporation Ltd.
6 Interest on tax free, secured, redeemable, non-convertible bonds issued by Housing
and urban development corporation Ltd.
7 Interest on tax free, secured, redeemable, non-convertible bonds issued by Power
finance corporation.

MT Educare –CA Inter Income Tax Classes – R.SOUMYANARAYANAN.FCA. GRAD CWA. Page-210
Chapter-7: IFOS

8 Interest on tax free, secured, redeemable, non-convertible bonds issued by Jawaharlal


Nehru Port Trust.
9 Interest on tax free, secured, redeemable, non-convertible bonds issued by Dredging
Corporation of India Ltd.
10 Interest on tax free, secured, redeemable, non-convertible bonds issued by Ennore Port
Ltd.
11 Interest on tax free, secured, redeemable, non-convertible bonds issued by Indian
Renewable energy development agency Ltd.
12 Interest on Gold Deposit bonds issued under the Gold deposit scheme 1999.
13 Interest on deposit certificates issued under the Gold monetization scheme 2015.
14 Interest on tax-free pooled finance development bonds issued under pooled finance
development fund scheme of Government of India.
15 Interest on income received by a non-resident or non-ordinarily resident from a deposit
made in an Off-shore banking unit.

Note: Interest on Non-SLR securities of Banks are chargeable to tax in their hands U/H
PGBP, since such investments are made by them in the ordinary course of business of
banking.

8. Taxation of gifts and benefits arising on account of acquisition of immovable properties


or specified movable properties for inadequate consideration – S. 56 (2) (x):

An overview of S. 56 (2) (x):

Sub-clause (a) Deals with taxation of sum of money which is received without
consideration.
Sub-clause (b) Deals with taxation of gift of immovable property.
Item-A
Sub-clause (b) Deals with taxation of benefit arising on account of acquisition of
Item-B immovable property for inadequate consideration.
Sub-clause (c) Deals with taxation of gift of specified movable properties.
Item-A
Sub-clause (c) Deals with taxation of benefit arising on account of acquisition of specified
Item-B movable properties for inadequate consideration.

S.56 (2) (x) (a): Taxation of monetary gifts:

1 Applicable to Any person


whom?
2 Income covered Sum of money received without consideration during the PY.
3 When taxable? If Ʃ sum of money received during the PY without consideration >
Rs. 50000.
4 Taxable quantum Ʃ sum of money received during the PY without consideration.
5 Exempted Receipts from relatives;
receipts Receipts on the occasion of marriage of individual;
Receipts under will or by way of inheritance;

MT Educare –CA Inter Income Tax Classes – R.SOUMYANARAYANAN.FCA. GRAD CWA. Page-211
Chapter-7: IFOS

Receipts in contemplation of death of the donor;


Receipts from local authorities;
Receipts from S. 10 (23C) institutions;
Receipts from trusts registered U/s 12AA;
Receipts by institutions referred to in S. 10 (23C) (iv)/ (v)/ (vi) / (via);
Receipts from an individual by a trust created solely for the benefit
of relative of the individual.

Points requiring attention:


1 Receipts from Government Are not within the ambit of S. 56 (2) (x) (a).
In view of the AP HC decision in CIT Vs
Dredging Corporation of India 174 ITR 682,
Government does not constitute a person.
2 Residential status of donee Is not relevant.
Donee may be resident or non-resident.
S. 56 (2) (x) (a) could be invoked if the receipt
comes within the scope of total income.
3 Place of receipt Is not relevant.
It may be in India or outside India.
S. 56 (2) (x) (a) could be invoked if the receipt
comes within the scope of total income.
4 Receipt - Indian currency or foreign Receipt of money may be in Indian currency
currency or foreign currency.
5 Receipts in cheque Receipt of sum of money through cheques is
also covered (provided cheques are not
dishonoured).
6 Meaning of ‘relative’ in case of Spouse;
individual brother or sister;
brother or sister of the spouse;
brother or sister of either of the parents;
any lineal ascendant or descendant;
any lineal ascendant or descendant of the
spouse;
Spouse of the person referred to above other
than spouse.
7 Meaning of ‘relative’ in case of HUF Any member thereof.
8 Lineal descendents of brothers and Are not covered by relative definition.
sisters + Lineal descendents of
brothers and sisters of spouse +
Brother and sisters of grandparents
9 Gifts on the occasion of marriage Not exempted.
anniversary
10 Donee ≠ person whose marriage is No exemption.
solemnized To avail exemption donee should be bride or
bridegroom. (Rajinder Mohan Lal [P&H]).
11 Gifts received by member from his Exempt

MT Educare –CA Inter Income Tax Classes – R.SOUMYANARAYANAN.FCA. GRAD CWA. Page-212
Chapter-7: IFOS

HUF HUF = cluster of relatives. (Vineet kumar


Raghavjibhai Bhalodia [2011] (Rajkot) +
Subhadra Devi Nevatia (Kol) [2011] + Nidhi
Goenka (Podder) (Kol) [2009]).
12 Meaning of ‘gift in contemplation of Gift in contemplation of death is defined in S.
death’ 191 of the Indian Succession Act. The
essential ingredients are as follows (a) Gift
must be of movable property; (b) It must be
made in contemplation of death; (c) the
donor must be ill and he expects to die
shortly of illness; (d) possession of the
property should be delivered to the donee;
However, the gift does not take effect if the
donor recovers from the illness or the donee
predeceases the donor.

S. 56 (2) (x) (b) (A): Taxation of gift of immovable properties:

1 Applicable to whom? Any person.


2 Income covered Receipt of immovable property (being a capital asset)
without consideration.
3 When taxable? If the SDV of the immovable property received without
consideration > Rs. 50000.
4 Taxable quantum SDV of the immovable property received without
consideration.
5 Exempted receipts Receipts from relatives;
Receipts on the occasion of marriage of individual;
Receipts under will or by way of inheritance;
Receipts from local authorities;
Receipts from S. 10 (23C) institutions;
Receipts from trusts registered U/s 12AA;
Receipts by institutions referred to in S. 10 (23C) (iv)/
(v)/ (vi) / (via);
Receipts from an individual by a trust created solely for
the benefit of relative of the individual.
Receipts by member upon partition of HUF.
Receipts by holding company from wholly-owned
subsidiary (Provided the holding company is an Indian
company);
Receipts by wholly-owned subsidiary from holding
company (Provided the wholly-owned subsidiary is an
Indian company).

MT Educare –CA Inter Income Tax Classes – R.SOUMYANARAYANAN.FCA. GRAD CWA. Page-213
Chapter-7: IFOS

Points requiring attention:

1 Significance of ‘immovable If a rural agricultural land is gifted, S. 56 (2) (x) (b) (A)
property being a capital cannot be invoked.
asset’ Rural agricultural land ≠ Capital asset.
2 Applicability of threshold The threshold of Rs. 50000 is in respect of SDV of each and
every immovable property received without consideration.
It is not in respect of aggregate of SDV of all immovable
properties received without consideration during the PY.

S. 56 (2) (x) (b) (B): Taxation of benefit arising on account of acquisition of immovable
property for inadequate consideration:

1 Applicable to whom? Any person.


2 Income covered Benefit arising on account of acquisition of immovable
property for an inadequate consideration.
3 When taxable? Consideration < SDV.
(SDV – Consideration) > [Higher of (a) Rs. 50000; (b) 5%
of consideration].
4 Taxable quantum (SDV – Consideration).
5 Exempted receipts Receipts from relatives;
Receipts from local authorities;
Receipts from S. 10 (23C) institutions;
Receipts from trusts registered U/s 12AA;
Receipts by institutions referred to in S. 10 (23C) (iv)/ (v)/
(vi) / (via);
Receipts by amalgamated Indian company from
amalgamating company pursuant to the scheme of
amalgamation.
Receipts by resulting company being Indian company on
demerger of demerged company.
Receipts by holding company from WOS (Provided the
holding company is an Indian company);
Receipts by wholly-owned subsidiary from holding
company (Provided the wholly-owned subsidiary is an
Indian company).

Points requiring attention:

(a) If the assessee feels that the SDV of the immovable property received without
consideration or received for an inadequate consideration is more than its FMV, then
he can challenge the SDV through an appeal under the Indian Stamp Act.
If the SDV is revised by an order passed by the appellate authority under the Indian
Stamp Act, then the AO shall amend the order of assessment passed in respect of AY
relevant to the PY in which the immovable property was received to re-compute the

MT Educare –CA Inter Income Tax Classes – R.SOUMYANARAYANAN.FCA. GRAD CWA. Page-214
Chapter-7: IFOS

income chargeable U/H IFOS.


This amendment order shall be passed within 4 years from the end of the PY in which
the SDV was revised by the appellate authority under the Indian Stamp Act. [3rd
Proviso to S. 56 (2) (x) (b) + S. 155 (15)].
(b) Alternatively, the assessee, instead of challenging the SDV in appeal under the Indian
Stamp Act, can approach the AO and request him to determine the FMV. [3rd Proviso
to S. 56 (2) (x) (b) + S. 50C (2)].
Thereafter, the AO has to refer the matter to the DVO who will determine the FMV.
If the FMV is less than the SDV, then the FMV shall be assessed to tax U/H IFOS.
However, if the FMV is more than the SDV, then the AO shall assess the SDV U/H IFOS.
(c) If the SDV on the date of agreement is different from the SDV on the date of
registration, for the purpose of S. 56 (2) (x) (b) (B), the SDV on the date of registration
shall be considered.
However, where the amount of consideration, or a part thereof, has been paid by way
of an account payee cheque or account payee bank draft or use of electronic clearing
system through a bank account, on or before the date of the agreement for the
transfer of such immovable property, then the SDV on the date of agreement fixing the
consideration is to be taken. [Proviso-1 and Proviso-2 to S. 56 (2) (x) (b)].
(d) In S. 56 (2) (x) (b) (B) also, the threshold of Rs. 50000 is in respect of each and every
immovable property received during the PY by the assessee for inadequate
consideration .

Note:-
Amendment in Section 56(2)(x) - Effective from 01/04/2019 [Finance (No. 2) Act, 2019]
a) Where the date of agreement fixing the amount of consideration for the
transfer of immovable property and the date of registration are not the
same, the stamp duty value on the date of agreement may be taken for the
purposes of this sub-clause.

Further, the above provisions shall apply only in a case where the amount
of consideration referred to therein, or a part thereof, has been paid by
way of an account payee cheque or an account payee bank draft or by use
of electronic clearing system through a bank account or through such
other electronic mode as may be prescribed on or before the date of
agreement for transfer of such immovable property.

b) Gift under this clause is not taxable if sum of money or any property
received from such class of persons and subject to such conditions, as
may be prescribed [Similar amendment is introduced in Section 50CA]

S. 56 (2) (x) (c) (A): Taxation of gift of specified movable properties:

1 Applicable to whom? Any person.


2 Income covered Receipt of specified movable property (being a capital asset)

MT Educare –CA Inter Income Tax Classes – R.SOUMYANARAYANAN.FCA. GRAD CWA. Page-215
Chapter-7: IFOS

without consideration.
3 Specified movable Following capital assets being:
property Shares and securities;
jewellery;
archaeological collections;
drawings;
paintings;
sculptures;
any work of art;
bullion;
4 When taxable? If ƩFMV of all specified movable properties received without
consideration during the PY > Rs. 50000.
5 Taxable quantum ƩFMV of all specified movable properties received without
consideration during the PY.
6 Exempted receipts Receipts from relatives;
Receipts on the occasion of marriage of individual;
Receipts under will or by way of inheritance;
Receipts in contemplation of death of the donor;
Receipts from local authorities;
Receipts from S. 10 (23C) institutions;
Receipts from trusts registered U/s 12AA;
Receipts by institutions referred to in S. 10 (23C)
(iv)/(v)/(vi)/(via);
Receipts from an individual by a trust created solely for the
benefit of relative of the individual.
Receipts by member upon partition of HUF.
Receipts by amalgamated Indian company from amalgamating
company pursuant to the scheme of amalgamation.
Receipts by resulting company being Indian company on
demerger of demerged company.
Receipt of capital asset being shares in Indian company by
foreign company on amalgamation of another foreign company.
Receipt of capital asset being shares in Indian company by the
resulting foreign company from the demerged foreign company
upon demerger.
Receipt of shares from resulting company on demerger of
undertaking by the shareholders of demerged company.
Receipt of shares from amalgamated company being an Indian
company by the shareholders of the amalgamating company.
Receipts by holding company from wholly-owned subsidiary
(Provided the holding company is an Indian company);
Receipts by wholly-owned subsidiary from holding company
(Provided the wholly-owned subsidiary is an Indian company).

S. 56 (2) (x) (c) (B): Taxation of benefit arising on account of acquisition of specified
movable properties for inadequate consideration:

MT Educare –CA Inter Income Tax Classes – R.SOUMYANARAYANAN.FCA. GRAD CWA. Page-216
Chapter-7: IFOS

1 Applicable to Any person.


whom?
2 Income covered Receipt of specified movable property (being a capital asset) for
consideration which is less than its FMV.
3 When taxable? If [ƩFMV of all specified movable properties received during the
PY – consideration] > Rs. 50000.
4 Taxable quantum [ƩFMV of all specified movable properties received during the PY
– consideration]
5 Exempted receipts Receipts from relatives;
Receipts from local authorities;
Receipts from S. 10 (23C) institutions;
Receipts from trusts registered U/s 12AA;

Receipts by institutions referred to in S. 10 (23C)


(iv)/(v)/(vi)/(via);
Receipts by holding company from wholly-owned subsidiary
(Provided the holding company is an Indian company);
Receipts by WOS from holding company (Provided the wholly-
owned subsidiary is an Indian company).

Note:

1 Where the assessee has acquired the shares and securities through RSE, the transaction
value itself shall be regarded as FMV and hence, the question of invoking S. 56 (2) (x) (c)
(B) does not arise. [R. 11UA].
2 Similarly, if the specified movable properties are acquired from a registered dealer, the
invoice value itself shall be regarded as FMV and hence, the question of invoking S. 56
(2) (x) (c) (b) does not arise. [R. 11UA].

Questions on S. 56 (2) (x):

9. Deemed income chargeable to tax – S. 59:

(i) The provisions of S. 41 (1) are made applicable, so far as may be, to the computation of
income under this head.
(ii) Accordingly, where a deduction has been made in respect of a loss, expenditure or
liability and subsequently, any amount is received or benefit is derived in respect of
such expenditure incurred or loss or trading liability allowed as deduction, then it shall
be deemed as income in the year in which the amount is received or the benefit is
accrued.

MT Educare –CA Inter Income Tax Classes – R.SOUMYANARAYANAN.FCA. GRAD CWA. Page-217
Chapter-8: Clubbing Provisions

1. Introduction:

(i) Under the Income tax Act, an assessee is generally taxed in respect of his own income.
However, there are certain cases where an assessee has to pay tax in respect of
income of another person.
(ii) The provisions for the same are contained in S. 60 to S. 65.
(iii) These provisions have been enacted to counteract the tendency on the part of the tax-
payers to dispose of their property or transfer their income in such a way that their
liability can be avoided or reduced.
(iv) For example, in the case of individuals, income-tax is levied on a slab system on the
total income. The tax system is progressive i.e. as the income increases, the applicable
rate of tax increases.
(v) Some tax payers in the higher income bracket have a tendency to divert some portion
of their income to their spouse, minor child etc to minimise the tax burden.
(vi) In order to prevent such tax avoidance, clubbing provisions have been incorporated in
the Act, under which income arising to certain persons (like spouse, minor child etc)
have to be included in the income of the person who has diverted his income for the
purpose of computing tax liability.

2. Transfer of income without transfer of asset – S. 60:

(i) If any person transfers the income from any asset without transferring the asset itself,
such income is to be included in the total income of the transferor.
(ii) It is immaterial whether the transfer is revocable or irrevocable.
(iii) For example, Mr. A confers the right to receive rent in respect of his property on his
wife, Mrs. A, without transferring the house itself to her. In this case, the rent received
by Mrs. A will be included with the income of Mr. A.

Q-1: Mr. Vatsan has transferred, through a duly registered document, the income arising
from a godown to his son, without transferring the godown. In whose hands will the rental
income from godown be charged?

Solution:

1 S. 60 expressly states that where there is transfer of income from an asset without
transfer of the asset itself, such income shall be included in the TI of the transferor.
2 Hence, the rental income derived from the godown shall be clubbed in the hands of Mr.
Vatsan.

3. Income arising from revocable transfer – S.61:

(i) Transfer of assets can be of two types: (a) Revocable; (b) Irrevocable.
(ii) S. 63 defines the term ‘revocable transfer’.
(iii) A transfer is deemed to be revocable if the instrument through which transfer is
effected (a) contains any provisions for retransfer, directly or indirectly, of the whole
or any part of the income or assets to the transferor, or
(b) gives, in any way to the transferor, a right to reassume power, directly or indirectly,

MT Educare –CA Inter Income Tax Classes – R.SOUMYANARAYANAN.FCA. GRAD CWA. Page-218
Chapter-8: Clubbing Provisions

over the whole or any part of the income or the assets.


(iv) S. 61 Provides for clubbing the income arising from assets transferred under revocable
transfer in the hands of the transferor.
(v) This clubbing provision will operate even if only part of income of the transferred asset
had been applied for the benefit of transferor. Once the transfer is revocable, the
entire income from the transferred asset is includible in the TI of the transferor.

4. Exceptions where clubbing provisions are not attracted even in case of revocable
transfer – S. 62:

(i) If there is a transfer of asset which is not revocable during the life time of the
transferee, the income from the transferred asset is not includible in the TI of the
transferor provided the transferor derives no direct or indirect benefit from such
income.
(ii) If the transferor receives direct or indirect benefit from such income, such income is to
be clubbed in his TI even though the transfer may not be revocable during the life time
of the transferee.
(iii) As and when the power to revoke the transfer arises, the income arising by virtue of
such transfer will be included in the TI of the transferor.

5. Clubbing of income arising to spouse:

A. Income by way of remuneration from a concern in which the individual has substantial
interest – S. 64 (1) (ii):

(i) In computing the total income of any individual, all such income which arises directly or
indirectly, to the spouse of such individual by way of salary, commission, fees or any
other form of remuneration, whether in cash or in kind, from a concern in which such
individual has a substantial interest shall be included.
(ii) However, this provision does not apply where the spouse of the said individual
possesses technical or professional qualifications and the income to the spouse is solely
attributable to the application of his or her technical or professional knowledge or
experience. In such an event, the income arising to the spouse is to be assessed in his
or her hands.

Points requiring attention:

(i) An individual is said to have substantial interest in a concern, being company, if he


singly or along with his relatives beneficially owns equity shares carrying not less than
20% voting power at any time during the PY.
(ii) An individual is said to have substantial interest in a concern, not being company, if he
singly or along with his relatives is beneficially entitled to not less than 20% of the
profits of the concern at any time during the PY.
(iii) Relative = Spouse + Brother + Sister + Lineal ascendant +Lineal descendant.

MT Educare –CA Inter Income Tax Classes – R.SOUMYANARAYANAN.FCA. GRAD CWA. Page-219
Chapter-8: Clubbing Provisions

Q-2: Mr. A holds shares carrying 25% voting power in X Ltd. Mrs. A is working as a computer
software programmer in X Ltd. at a salary of Rs. 30,000 p.m. She is, however, not qualified
for the job. The other income of Mr. A & Mrs. A are Rs. 7,00,000 & Rs. 4,00,000,
respectively. Compute the GTI of Mr. A and Mrs. A for the AY 2020-21.

Solution:

Mr. A holds shares carrying 25% voting power in X Ltd i.e. a substantial interest in the
company. His wife is working in the same company without any professional qualifications
for the same. Thus, by virtue of the clubbing provisions of the Act, the salary received by
Mrs. A from X Ltd. will be clubbed in the hands of Mr. A.

Computation of GTI of Mr. A:

Particulars Rs. Rs.


Salary received by Mrs. A (Rs. 30,000 × 12) 3,60,000
Less: Standard deduction U/s 16 (ia) 50,000 3,10,000
Other Income 7,00,000
Gross total income 10,10,000

The gross total income of Mrs. A is Rs. 4,00,000.

Q-3: Will your answer be different if Mrs. A was qualified for the job?

Solution:

1 If Mrs. A possesses professional qualifications for the job, then the clubbing provisions
shall not be applicable.
2 GTI of Mr. A = Rs. 7, 00,000 [other income].
3 GTI of Mrs. A = Salary received by Mrs. A [Rs. 30,000×12] less Rs. 50,000, being the
standard deduction U/s 16 (ia) plus other income [Rs. 4,00,000] = Rs. 7, 10,000.

MT Educare –CA Inter Income Tax Classes – R.SOUMYANARAYANAN.FCA. GRAD CWA. Page-220
Chapter-8: Clubbing Provisions

Pictorial representation of S. 64 (1) (ii):

Business / Spouse of Individual


Remuneration
Professional Concern (Employee of Concern)

Does the
Remuneration
Individual have
assessed in
substantial No
the hands of
interest in the
Individual’s
concern
spouse

Yes

Yes
Does the spouse have
any technical or
professional knowledge
and experience?

No

Remuneration
assessed in the
hands of Individual

Concern

Company Others

Individual
has no
Share of Individual and No
Does the Individual
relatives in profits > substantial
(either singly / together 20%? interest
with relatives) own
beneficially equity shares Yes Individual
carrying atleast 20% has
Yes
voting power? substantial
interest

No

MT Educare –CA Inter Income Tax Classes – R.SOUMYANARAYANAN.FCA. GRAD CWA. Page-221
Chapter-8: Clubbing Provisions

Husband Wife

Lineal
Relatives Brother
Descendant

Lineal Sister
Ascendant

Clubbing when both husband and wife are having substantial interest and both are
employed in the concern:

Situations Does husband Does wife possess Manner of clubbing


possess technical or technical or
professional professional
knowledge and knowledge and
experience? experience?
1 Yes No Club the salary of the wife from
the concern with the income of
the husband.
2 No Yes Club the salary of the husband
from the concern with the
income of the wife.
3 Yes Yes No need for clubbing. Assess
the respective salary income in
the hands of the respective
spouse.
4 No No Include the salary of the other
spouse in the total income of
the spouse whose total income
excluding such remuneration is
greater.

(B) Income from assets transferred to spouse - S. 64 (1) (iv):

Applicability of S. 64 (1) (iv):

If the following conditions are satisfied, S. 64 (1) (iv) shall apply:

1 The tax payer is an individual.


2 He/she transfers an asset other than a house property.
3 The asset is transferred to his/her spouse.
4 The transfer may be direct or indirect.
5 The asset is transferred otherwise than (a) for adequate consideration, or (b) in
connection with an agreement to live apart.

MT Educare –CA Inter Income Tax Classes – R.SOUMYANARAYANAN.FCA. GRAD CWA. Page-222
Chapter-8: Clubbing Provisions

Impact of S. 64 (1) (iv):

The income arising from the transferred asset shall be included in the income sheet of
transferor individual and not in the hands of transferee spouse.

Note: Where the subject matter of transfer is house property, then the provisions of S. 27 (i)
shall apply and the transferor individual is deemed to be the owner and the annual value
shall be assessed to tax in his hands. Provisions of S. 64 (1) (iv) has no relevance.

Pictorial representation of S. 64 (1) (iv):

Individual
transfers an
Asset

Clubbing not
Transferee = No possible U/s.
Spouse? 64(1)(iv)

Yes

Is the transfer in Yes


connection with an
agreement to live
apart?
Yes
No

Is it for adequate
consideration?

No

Asset transferred Individual =


= Yes Deemed
House property? Owner U/s.27

No
Income from such
asset clubbed with
the income of
transferor
(individual)

Direct
Transfer may be
Indirect

MT Educare –CA Inter Income Tax Classes – R.SOUMYANARAYANAN.FCA. GRAD CWA. Page-223
Chapter-8: Clubbing Provisions

Points requiring attention:

1 Spouse means legally wedded wife or husband. The term will not cover concubine.
Executors of the will of T.V. Krishna Iyer Vs CIT 38 ITR 144 (Ker).
2 Husband and wife relationship should subsist both on the date of transfer of the asset
and on the date of accrual of income. Then only the provisions of S. 64 (1) (iv) can be
invoked. Philip John Plasket Thomas Vs. CIT 49 ITR 97 (SC).
Situation 1 2 3 4
Whether husband & wife relationship exists on the date of Yes Yes No No
Transfer?
Whether it continues on the date of accrual of Income Yes No Yes No
Tax treatment Club Don’t club
3 Note the words “indirectly”. The effect of this is, even cross transfers are hit by S. 64 (1)
(iv). CIT Vs C.M. Kothari 49 ITR 107 (SC).
4 To avoid clubbing of the income of the transferee-spouse with that of the transferor-
spouse, what S. 64 (1) (iv) requires is “adequate consideration” and not “good
consideration”. Only such consideration which is capable of being measured in
monetary terms can fall within the ambit of the expression “adequate consideration”.
Therefore, natural love and affection of the transferee-spouse, though a good
consideration, is not an adequate consideration. Tulsidas Kilachand Vs. CIT 42 ITR 1
(SC).
5 Similarly, consent by the transferee-spouse to allow the transferor-spouse to adopt a
child cannot be regarded as adequate consideration. Potti Veerayya Sresty Vs. CIT 85
ITR 194 (AP).
6 Even if the form of the asset transferred by the transferor-spouse is changed by the
transferee-spouse, still, the income arising from such changed asset shall be clubbed in
the hands of the transferor-spouse. Mohini Thapar Vs CIT 83 ITR 208 (SC).
7 Bonafide loan by the husband to wife or vice versa will not invite the application of S.
64 (1) (iv). CIT Vs Hasina Begum 158 ITR 215 (Cal).
8 The Mumbai High Court in Patwardhan Vs CIT 76 ITR 279 held that the purpose of S. 64
(1) (iv) is to include the income of the transferred assets in computation of the total
income of the transferor-spouse only to the extent the consideration was found
inadequate. However, a contrary view has been expressed by the Kerala High Court in
CIT Vs Junus Haji Ummer Sait 173 ITR 3 (Ker).
9 Even the capital gains derived by the transferee-spouse upon sale of assets transferred
to her/him by the transferor-spouse had to be included in the total income of the
transferor-spouse-U/s 64 (1) (iv). Seventilal Maneklal Sheth Vs CIT-68 ITR 503 (SC).
Transfers
capital asset
Transfers
Another
Individual Spouse the same
without
capital asset Person
consideration

Results In
CG

Chargeable to tax in the hands of individual


and not in spouse

MT Educare –CA Inter Income Tax Classes – R.SOUMYANARAYANAN.FCA. GRAD CWA. Page-224
Chapter-8: Clubbing Provisions

10 Even the capital gains arising on account of transfer of an asset the form of which is
changed by the transferee-spouse shall be clubbed in the hands of the transferor-
spouse. CIT Vs Smt Pelleti Sridevamma 216 ITR 826 (SC).
11 Subsequent to the receipt of shares as gift from the transferor-spouse, if bonus shares
are allotted to the transferee-spouse and on which the transferee-spouse receives any
dividend, that is not bit by S. 64 (1) (iv). M.P. Birla 142 ITR 377 (Bom). Income arising
from accretions to the transferred asset shall not be clubbed invoking S. 64 (1) (iv).
12 Second generation income cannot be clubbed invoking the provisions of S. 64 (1) (iv).
For example, subsequent to the receipt of a house property as gift from the transferor-
spouse, if the transferee-spouse earns any income from fixed deposits placed with a
bank, made out of the rental income, such second generation income is not hit by S. 64
(1) (iv). CIT Vs M.S.S. Rajan 252 ITR 126 (Mad).
13 In CIT Vs Karamchand Premchand Ltd 40 ITR 106 (SC), the apex court held that income
includes loss. Therefore, U/s 64 (1) (iv) even loss could be clubbed.

Amount to be clubbed when transferred asset is invested in business:

Clubbing procedure [Explanation-3 to S. 64 (1)]:

Step 1 Find out the total investment of transferee-spouse in the business on the first day
of the PY.
Step 2 Find out the amount invested by the transferee spouse out of the assets
transferred to her without adequate consideration by her husband on the first day
of the PY in the said business.
Step 3 Find out the taxable income of the transferee spouse from the business.
If the transferees spouse becomes the partner of a firm by investing the aforesaid
asset, then only interest income from the firm is considered under step-3.
Share of profit from the firm is not considered under step-3 as it is exempt U/s 10
(2A).
Remuneration received from the firm cannot be considered for clubbing since it is
paid for the services rendered by the partner.
Step 4 The amount which shall be included in the hands of the transferor is determined
as follows: Step 3 * (Step 2/ Step 1)

Q-4: Mr. Vaibhav started a proprietary business on 01.04.2018 with a capital of Rs. 5,00,000.
He incurred a loss of Rs. 2,00,000 during the PY 2018-19.

To overcome the financial position, his wife Mrs. Vaishaly, a software Engineer, gave a gift of
Rs. 5,00,000 on 01.04.2018, which was immediately invested in the business by Mr. Vaibhav.
He earned a profit of Rs. 4,00,000 during the PY 2019-20.

Compute the amount to be clubbed in the hands of Mrs. Vaishaly for the AY 2020-21. If Mrs.
Vaishaly gave the said amount as loan, what would be the amount to be clubbed?

MT Educare –CA Inter Income Tax Classes – R.SOUMYANARAYANAN.FCA. GRAD CWA. Page-225
Chapter-8: Clubbing Provisions

Solution:

S. 64 (1) (iv) provides for the clubbing of income in the hands of the individual, if the income
earned is from the assets (other than house property) transferred directly or indirectly to
the spouse of the individual, otherwise than for adequate consideration or in connection
with an agreement to live apart.

In this case, Mr. Vaibhav received a gift of Rs. 5,00,000 on 1.4.2019 from his wife
Mrs. Vaishaly, which he invested in his business immediately. The income to be clubbed in
the hands of Mrs. Vaishaly for the AY 2020-21 is computed as under:

Particulars Mr. Vaibhav’s capital Capital contribution Total (Rs.)


contribution (Rs.) out of gift from Mrs.
Vaishaly (Rs.)
Capital as on 1.4.2019 3,00,000 5,00,000 8,00,000
(5,00,000 – 2,00,000)
Profit for PY 2019-20 to 1,50,000 2,50,000 4,00,000
be apportioned on the  3  5
 4,00,000×   4,00,000× 
basis of capital  8  8
employed on the first
day of the previous year
i.e. as on 1.4.2019 (3:5)

Therefore, the income to be clubbed in the hands of Mrs. Vaishaly for the AY 2020-21 is Rs.
2,50,000. In case Mrs. Vaishaly gave the said amount of Rs. 5,00,000 as a bona fide loan,
then, clubbing provisions would not be attracted.

Note: The provisions of S. 56 (2) (x) would not be attracted in the hands of Mr. Vaibhav,
since he has received a sum of money exceeding Rs. 50,000 without consideration from a
relative i.e., his wife.

6. Income from assets transferred to son’s wife S. 64 (1) (vi):

Applicability of S. 64 (1) (vii):

If the following conditions are satisfied, S. 64 (1) (vii) shall apply:

1 The tax payer is an individual.


2 He or She has transferred an asset.
3 The asset is transferred to his/her son’s wife.
4 The transfer may be direct or indirect.
5 The asset is transferred otherwise than for adequate consideration.

Impact of S. 64 (1) (vii):

The income arising from the transferred asset shall be included in the income sheet of
transferor individual and not in the hands of transferee daughter-in-law.

MT Educare –CA Inter Income Tax Classes – R.SOUMYANARAYANAN.FCA. GRAD CWA. Page-226
Chapter-8: Clubbing Provisions

Pictorial representation:

Individual transfers
an asset

Transferee
= No
Clubbing not possible
Son’s Wife u/s 64(1)(vi)
?

Yes

Transfer
Clubbing not possible takes
No
u/s 64(1)(vi) place after
31.5.1973
?

Yes Yes

Is
the transfer for
adequate
consideration
?

No
Income from assets
transferred - clubbed with
the income of Individual

Points requiring attention:

(i) The relationship of father-in-law or Mother-in-Law and Daughter-in-Law should subsist


both at the time of transfer and at the time of accrual of income.

Situation 1 2 3 4
Whether on the date of transfer, Father/Mother in law Yes Yes No No
and Daughter in law relationship exists?
Whether on date of accrual of Income, Father/Mother in Yes No Yes No
law and Daughter in law relationship exists?
Tax treatment Clubbed Not Clubbed
(ii) Even S. 64 (1) (vi) covers cross transfers - Om Dutt Vs CIT 277 ITR 63 (P&H).
(iii) The asset may be held by the transferee in the same form or in a different form.
(iv) Explanation-3 to S. 64 (1) is applicable here also. Where the assets transferred directly
or indirectly by an individual to his or her son’s wife are invested by the transferee in
the business, proportionate income arising from such investment is to be included in

MT Educare –CA Inter Income Tax Classes – R.SOUMYANARAYANAN.FCA. GRAD CWA. Page-227
Chapter-8: Clubbing Provisions

the TI of the transferor. If the investment is in the nature of contribution of capital, the
proportionate interest on capital will be clubbed with the income of the transferor.

Such proportion has to be computed by taking into account the value of the aforesaid
investment as on the 1st day of the PY to the total investment in the business or by way
of capital contribution in a firm as a partner, as the case may be, by the transferee as
on that day.

7. Income from assets transferred to a person for the benefit of spouse (S. 64 (1) (vii)):

Applicability of S. 64 (1) (vii):

S. 64 (1) (vii) is applicable if the following conditions are satisfied:

1 The taxpayer is an individual.


2 He/she has transferred an asset.
3 The transfer may be direct or indirect.
4 The asset is transferred to a person or association of person.
5 It is transferred for the immediate or deferred benefit of his/her spouse.
6 The transfer is for an inadequate consideration.

Impact of S. 64 (1) (vii):

Income from such asset to the extent of such benefit is taxable in the hands of the taxpayer
who has transferred the asset.
Provisions illustrated:

Mr. A transfers one of his house properties to his friend B with a direction that 50% of the
rental income is to be used for the benefit of his wife Mrs. A and 50% for others. Then, the
rental income to the extent of 50% shall be included in the total income of Mr. A.

8. Income from assets transferred to a person for the benefit of son’s wife (S. 64 (1) (viii):

Applicability of S. 64 (1) (viii):

S. 64 (1) (viii) is applicable if the following conditions are satisfied:

1 The tax payer is an individual.


2 He/she has transferred an asset.
3 The asset is transferred to any person or an association of person.
4 Transfer may be direct or indirect.
5 The asset is transferred for the immediate or deferred benefit of his or her son’s wife.
6 The asset is transferred otherwise than for adequate consideration.

MT Educare –CA Inter Income Tax Classes – R.SOUMYANARAYANAN.FCA. GRAD CWA. Page-228
Chapter-8: Clubbing Provisions

Impact of S. 64 (1) (vii):

Income from such asset to the extent of such benefit is taxable in the hands of the taxpayer
who has transferred the asset.

Provisions illustrated:

Mr. A transfers shares in a foreign company without consideration to an AOP subject to the
condition that, the dividend income will be utilised for the benefit of Mrs. X (his daughter in
law). Then, the dividend income shall be included in the hands of Mr. A.

Q-5: Mrs. Kasturi transferred her immovable property to ABC Ltd subject to a condition that
out of the rental income, a sum of Rs. 36000 p.a. shall be utilised for the benefit of her son’s
wife. Mrs. Kasturi claims that the amount of Rs. 36000 (utilised for her Son’s wife) should
not be included in her TI as she no longer owned the property. State with reasons whether
the contention of Mrs. Kasturi is valid in law.

Solution:

The clubbing provisions U/s 64 (1) (viii) are attracted in case of transfer of any asset,
directly or indirectly, otherwise than for adequate consideration, to any person to the
extent to which the income from such asset is for the immediate or deferred benefit of
son’s wife. Such income shall be included in computing the TI of the transferor-individual.
Therefore, income of Rs. 36,000 meant for the benefit of daughter-in-law is chargeable to
tax in the hands of transferor i.e., Mrs. Kasturi in this case. The contention of Mrs. Kasturi
is, hence, not valid in law.
Note: In order to attract the clubbing provisions U/s 64 (1) (viii), the transfer should be
otherwise than for adequate consideration. In this case, it is presumed that the transfer is
otherwise than for adequate consideration and therefore, the clubbing provisions are
attracted.
If it is presumed that the transfer was for adequate consideration, the provisions of S. 64
(1) (viii) would not be attracted.

9. Clubbing of income of minor (S. 64 (1A)):

Income accruing or arising to a minor child shall be clubbed in the hands of the parent of the
child. Provisions relating to clubbing of income of the minor child in the hands of parents are
given in S. 64 (1A). However, in respect of the income clubbed U/s 64 (1A), the parent is
eligible for exemption U/s 10 (32).

MT Educare –CA Inter Income Tax Classes – R.SOUMYANARAYANAN.FCA. GRAD CWA. Page-229
Chapter-8: Clubbing Provisions

Which income of the minor could be clubbed?

?
Child Income not clubbed
No
= with the parent
Minor

Yes

Minor Child

Suffering from disability


Other Minor Child
Specified u/s 80 U

Income of such Minor Income


child (from all sources) -
Not subject to Clubbing
provisions
On account of any On account of any
activity involving Other income
manual work
application of his
skill, talent or
specialized
knowledge and
experience
subject to
Not subject to Clubbing Provision
Clubbing Provision

Parent in whose hands the income of the minor is to be clubbed:

Marriage of Marriage of Parents


Parents subsists doesn’t subsist

Situation 1 Situation 2

Total income of Father (excluding Total income of Mother (excluding


the income includible u/s 64(1A) the income includible u/s 64(1A)
> >
Total income of Mother(excluding Total income of Father (excluding
the income includible u/s 64(1A) the income includible u/s 64(1A)

Clubbed with the Clubbed with the Income of Minor is clubbed


income of Father income of Mother with the income of that
parent who maintains the
Minor child in the relevant
Previous Year

Exemption U/s 10 (32):

Parent with whose income the income of minor is clubbed is entitled to an exemption to the
extent of Rs. 1500 per child or the income clubbed (whichever is less).

MT Educare –CA Inter Income Tax Classes – R.SOUMYANARAYANAN.FCA. GRAD CWA. Page-230
Chapter-8: Clubbing Provisions

Points requiring attention:

1 Meaning of “Child” (S. 2 Child in relation to a child includes a step child and an
(15B)) adopted child of that individual.
But it does not include an illegitimate child.
2 Minor married daughter Unlike S. 27, S. 64 (1A) does not exclude minor married
daughter.
Hence, U/s 64 (1A) even the income arising to a minor
married daughter would be clubbed with that of the
parent.
3 Attaining majority during the Where the minor child becomes major during the PY,
previous year then the income up to the date he remained minor in
that PY shall be clubbed in the hands of the parent.
4 Income from investments To be clubbed with that of the parent U/s 64 (1A).
made out of income earned
from manual work or
activities involving application
of skill, talent or specialized
knowledge and experience.

Comparative study of S. 27 Vis-à-vis S. 64:

SN Situation Applicability of S. 27 & S. 64


1 Mr. X transfers a house property to In this case, neither S. 27 nor S. 64 is applicable.
his wife for adequate consideration The income from property will be assessable in
or under an agreement to live the hands of Mrs. X.
apart.
2 Mr. X transfers debentures to his In this case, neither S. 27 nor S. 64 is applicable.
wife for adequate consideration or The income from debentures will be assessable
under an agreement to live apart. in the hands of Mrs. X.
3 Mr. X transfers a HP to his wife for
In this case, S. 27 is applicable. IFHP will be
inadequate consideration. assessable in the hands of Mr. X.
4 Mr. X transfers debentures to his In this case, S. 64 is applicable. The income from
wife for inadequate consideration. debentures will be clubbed in the hands of Mr. X.
5 Mr. X transfers a house property toIn this case, S. 27 is not applicable and S. 64 (1A)
his minor unmarried/married son is applicable. The income from the property will
for adequate consideration. be clubbed with the income of the parent whose
other incomes are higher. Exemption of Rs. 1500
U/s 10(32) is available to the parent in respect of
the income of minor child.
6 Mr. X transfers debentures to his In this case, S. 27 is not applicable and S. 64 (1A)
minor son for adequate is applicable.
consideration. The income from the debentures will be clubbed
with the income of the parent whose other
incomes are higher. Exemption of Rs. 1500 U/s
10(32) is available to the parent in respect of the
income of minor child.

MT Educare –CA Inter Income Tax Classes – R.SOUMYANARAYANAN.FCA. GRAD CWA. Page-231
Chapter-8: Clubbing Provisions

7 Mr. X transfers house property to In this case, S. 27 is applicable and S. 64 (1A) is


his minor unmarried/married son not applicable. Mr. X is deemed as the owner of
for inadequate consideration. the house property and income from property
will be included in his hands. Exemption of Rs.
1500 U/s 10 (32) is not available.
8 Mr. X transfers debentures to his S. 27 is not applicable and S. 64 (1A) is
minor unmarried/married son for in applicable. The income from the debentures will
adequate consideration. be clubbed with the income of the parent whose
other incomes are higher. Exemption of Rs. 1500
U/s 10(32) is available to the parent in respect of
the income of the minor child.
9 Mr. X transfers a house property to In this case, S. 27 is not applicable and S. 64 (1A)
his minor unmarried/married is applicable. The income from the property will
daughter for adequate be clubbed with the income of the parent whose
consideration. other incomes are higher. Exemption of Rs. 1500
U/s 10 (32) is available to the parent in respect
of the income of minor child.
10 Mr. X transfers debentures to his In this case, S. 27 is not applicable and S. 64 (1A)
minor unmarried/married daughter is applicable. The income from the debentures
for adequate consideration. will be clubbed with the income of the parent
whose other income are higher. Exemption of Rs.
1500 U/s 10(32) is available to the parent in
respect of the income of minor child.
11 Mr. X transfers a house property to In this case, S. 27 is applicable and S. 64 (1A) is
his minor unmarried daughter for not applicable. Mr. X is deemed as the owner of
inadequate consideration. the house property and income from property
will be included in his hands. Exemption U/s 10
(32) is not available.
12 Mr. X transfers a house property to In this case, S. 27 is not applicable and S. 64 (1A)
his minor married daughter for is applicable. The income from property will be
inadequate consideration. clubbed with the income of the parent whose
other incomes are higher. Exemption of Rs. 1500
U/s 10 (32) is available.
13 Mr. X transfers debentures to his In this case, S. 27 is not applicable and S. 64 (1A)
minor unmarried/married daughter is applicable. The income from the debentures
for inadequate consideration. will be clubbed with the income of the parent
whose other incomes are higher. Exemption U/s
10 (32) is available to the parent in respect of the
income of the minor child.

MT Educare –CA Inter Income Tax Classes – R.SOUMYANARAYANAN.FCA. GRAD CWA. Page-232
Chapter-8: Clubbing Provisions

Q-6: Mr. A has three minor children – two twin daughters and one son. Income of the twin
daughters is Rs. 2000 p.a each and that of the son is Rs. 1200 p.a.

Compute the income, in respect of minor children, to be clubbed in the hands of Mr. A.

Solution:

Taxable income, in respect of minor children, in the hands of Mr. A is:

Particulars Rs. Rs.


Twin minor daughters [Rs. 2,000 × 2] 4,000
Less: Exempt under section 10(32) [Rs. 1,500 × 2] 3,000 1,000
Minor son 1,200
Less: Exempt U/s 10 (32) 1,200 Nil
Income to be clubbed in the hands of Mr. A 1,000

Q-7: Compute the gross total income of Mr. A and Mrs. A from the following information:

(a) Salary income (computed) of Mrs. A Rs. 230000


(b) Income from profession of Mr. A Rs. 390000
(c) Income of minor son B from company deposit Rs. 15000
(d) Income of minor daughter C from special talent Rs. 32000
(e) Interest from bank received by C on deposit made out of her special talent Rs. 3000
(f) Gift received by C on 30.09.2019 from friend of Mrs. A Rs. 2500

Brief working is sufficient. Detailed computation under various heads of income is not
required.

Solution:

As per the provisions of S. 64 (1A) of the Income-tax Act, 1961, all the income of a minor
child has to be clubbed in the hands of that parent whose total income (excluding the
income of the minor) is greater. The income of Mr. A is Rs. 3,90,000 and income of Mrs. A is
Rs. 2,30,000. Since the income of Mr. A is greater than that of Mrs. A, the income of the
minor children have to be clubbed in the hands of Mr. A. It is assumed that this is the first
year when clubbing provisions are attracted.

Income derived by a minor child from any activity involving application of his/her skill,
talent, specialised knowledge and experience is not to be clubbed. Hence, the income of
minor child C from exercise of special talent will not be clubbed.

However, interest from bank deposit has to be clubbed even when deposit is made out of
income arising from application of special talent.

The GTI of Mrs. A is Rs. 2,30,000. The total income of Mr. A giving effect to the provisions of
S. 64 (1A) is as follows:

MT Educare –CA Inter Income Tax Classes – R.SOUMYANARAYANAN.FCA. GRAD CWA. Page-233
Chapter-8: Clubbing Provisions

Computation of gross total income of Mr. A for the AY 2019-20:

Particulars Rs. Rs.


Income from profession 3,90,000
Income of minor son B from company deposit 15,000
Less: Exemption U/s 10 (32) 1,500 13,500
Income of minor daughter C
From special talent – not to be clubbed -
Interest from bank 3,000
Gift of Rs. 2,500 received from a non-relative is not taxable U/s Nil
56 (2) (x) being less than the aggregate limit of Rs. 50,000
3,000
Less : Exemption U/s 10 (32) 1,500 1,500
Gross Total Income 4,05,000

Q-8: Mr. Vasudevan gifted a sum of Rs. 6L to his brother's wife on 14-6-2019. On 12-7-2019,
his brother gifted a sum of Rs. 5L to Mr. Vasudevan's wife. The gifted amounts were
invested as fixed deposits in banks by Mrs. Vasudevan and wife of Mr. Vasudevan's brother
on 01-8-2019 at 9% interest. Examine the consequences of the above under the provisions
of the Income-tax Act, 1961 in the hands of Mr. Vasudevan and his brother.

Solution:

In the given case, Mr. Vasudevan gifted a sum of Rs. 6L to his brother’s wife on 14.06.2019
and simultaneously, his brother gifted a sum of Rs. 5L to Mr. Vasudevan’s wife on
12.07.2019. The gifted amounts were invested as fixed deposits in banks by Mrs. Vasudevan
and his brother’s wife. These transfers are in the nature of cross transfers. Accordingly, the
income from the assets transferred would be assessed in the hands of the deemed
transferor because the transfers are so intimately connected to form part of a single
transaction and each transfer constitutes consideration for the other by being mutual or
otherwise.

If two transactions are inter-connected and are part of the same transaction in such a way
that it can be said that the circuitous method was adopted as a device to evade tax, the
implication of clubbing provisions would be attracted. It was so held by the Apex Court in
Keshavji Morarji (1967) 66 ITR 142.

Accordingly, the interest income arising to Mrs. Vasudevan in the form of interest on fixed
deposits would be included in the total income of Mr. Vasudevan and interest income
arising in the hands of his brother’s wife would be taxable in the hands of Mr. Vasudevan’s
brother as per S. 64 (1), to the extent of amount of cross transfers i.e., Rs. 5L.

This is because both Mr. Vasudevan and his brother are the indirect transferors of the
income to their respective spouses with an intention to reduce their burden of taxation.

MT Educare –CA Inter Income Tax Classes – R.SOUMYANARAYANAN.FCA. GRAD CWA. Page-234
Chapter-8: Clubbing Provisions

However, the interest income earned by his spouse on fixed deposit of Rs. 5L alone would
be included in the hands of Mr. Vasudevan’s brother and not the interest income on the
entire fixed deposit of Rs. 6L, since the cross transfer is only to the extent of Rs. 5L.

10. Conversion of self-acquired property into joint family property and subsequent
partition (S. 64 (2)):

Situations covered by S. 64 (2):

Situation An Individual converts his self-acquired property into property belonging to the
1 family. It is done by impressing such property with the character of joint family
property or throwing such property into common stock of the family.
Situation An individual transfers his self-acquired property, directly or indirectly, to the
2 family otherwise than for adequate consideration.

Clubbing of income before partition:

In the aforesaid situations, the income from such property is clubbed with the income of the
transferor.

Provisions illustrated:

Mr. A transfers his self-acquired property yielding an annual income of Rs. 60000 to his HUF,
consisting of Mr. A, Mrs. A, his major son P and minor son Q. Income of Rs. 60000 will be
included in the income of Mr. A (and not of the HUF) by virtue of this section.

Clubbing of income after partition:


If the property converted or transferred by an individual is subsequently transferred
amongst the members of the family, the income derived from such converted property, as is
received by the spouse of the transferor will be included in the income of the transferor.

Family partition takes place

Property distributed among


family members

Spouse of individual gets


the converted property

Income from such property


- clubbed in the hands of
the individual

Provisions illustrated:
Continuing with the same illustration, assume that the property is partitioned equally
among the family members. Income derived from the converted property by Mrs. A (1/4 th of
Rs. 60000=Rs. 15000) will be included in the income of Mr. A U/s 64 (2). Share of minor son
Rs. 15000 will be included in the income of Mr. A by virtue of S. 64 (1A) after claiming an
exemption of Rs. 1500.

MT Educare –CA Inter Income Tax Classes – R.SOUMYANARAYANAN.FCA. GRAD CWA. Page-235
Chapter-9: Set off & Carry forward of losses

1. Inter-source adjustment – S. 70:

1 Rule of inter-source Where the net result for any AY in respect of any source falling under
adjustment any head of income is a loss, the assessee shall be entitled to have the
amount of such loss set off against his income from any other source
under the same head. (See note-1).
2 Exceptions
(a) Loss from speculation Loss arising from speculation business could be adjusted only against
business. income from speculation business and not against any other income.
Thus, loss from speculation business shall not be adjusted against
income from non-speculation business. (See note-2).
(b) Long-term capital loss LTCL could be adjusted only against LTCG. It cannot be set off against
(LTCL). STCG. (See note-3).
(c) Loss from the activity The loss from the activity of owning and maintaining race horses can be
of owning and set off only against income from such activity. (See note-4).
maintaining race
horses.
(d) Loss cannot be set off No loss can be set off against income from lotteries, horse race, card
against winnings from games, cross word puzzles, betting, gambling etc. (See note-5).
lotteries, cross word
puzzles etc.
(e) Loss from lottery, card Loss from lottery, card games, betting, horse races etc. cannot be set
games, betting, horse off against any income. (See note-5).
races etc.
(f) Loss from exempted Loss from exempted source cannot be set off against income
source. chargeable to tax. (See note-6). For example, share of loss from a
partnership firm cannot be set-off against business income, since
share of income of the firm is exempt U/s 10 (2A).
(g) Loss from specified Any loss, computed in respect of any specified business referred to in S.
business referred to in 35AD shall not be set off except against profits, if any, of any other
S. 35AD specified business. (See note-7).
(h) Set off of losses No loss could be set off against income from undisclosed sources
against income from referred to in S. 68 to S. 69D. [S. 115BBE].
undisclosed sources –
not possible.
(i) Set off of losses No loss could be set off against income by way of dividend referred to in
against income by way S. 115BBDA. [S. 115BBDA (2)].
of dividend referred to
in S. 115BBDA.
(j) Dividend Discussed at the end of this Chapter.
stripping.[S.94(7)]
(k) Bonus stripping. [S. 94 Discussed at the end of this Chapter.
(8)].

MT Educare –CA Inter Income Tax Classes – R.SOUMYANARAYANAN.FCA. GRAD CWA. Page-236
Chapter-9: Set off & Carry forward of losses

Note 1:

SN Particulars of income/loss Amount


1 Income from let out property 100000
2 Loss from self-occupied property (80000)
Loss from self-occupied property shall be adjusted against income from let out
property.
The income from house property chargeable to tax is Rs. 20000 (100000-
80000).

Note 2:

Situation-1 Situation-2 Situation-3


Speculative Non- Speculative Non- Speculative Non-
speculative speculative speculative
Business 150000 150000 120000
1
Business (60000) (170000) (30000)
2
Business 190000 420000 210000
3
Business (70000) (110000) (220000)
4
Net 90000 120000 (20000) 310000 90000 (10000)
result

Analysis of the situations:

Situation 1 Speculative loss is fully absorbed by speculative profits. Non-speculative loss is


fully absorbed by non-speculative profits.
Situation 2 Speculative business loss cannot be set off against non-speculative business
income. Hence, the speculative business loss of Rs. 20000 shall be carry
forward to the next AY.
Situation 3 Non-speculative business loss shall be set off against speculative business
income. The net speculative business income chargeable to tax shall be Rs.
80000 (90000-10000).

Note 3:

Situation-1 Situation-2 Situation-3


ST LT ST LT ST LT
Capital asset-1 150000 150000 120000
Capital asset-2 (60000) (170000) (30000)
Capital asset-3 190000 420000 210000
Capital asset-4 (70000) (110000) (220000)
Net result 90000 120000 (20000) 310000 90000 (10000)

MT Educare –CA Inter Income Tax Classes – R.SOUMYANARAYANAN.FCA. GRAD CWA. Page-237
Chapter-9: Set off & Carry forward of losses

Analysis of the situations:

Situation 1 LTCL is fully absorbed by LTCG. STCL is fully absorbed by STCG.


Situation 2 STCL shall be adjusted against the LTCG. Hence, the taxable LTCG is Rs. 290000
(310000-20000).
Situation 3 LTCL cannot be adjusted against the STCG. Hence, it has to be carry forward to
the next AY.

Note 4:

SN Particulars of income/loss Situation-1 Situation-2 Situation-3


1 Dividend from a foreign company 100000 20000 100000
2 Interest on amount borrowed to invest in (40000) (40000) (40000)
shares of the aforesaid company
3 Net dividend (1-2) 60000 (20000) 60000
4 Income from owning and maintaining race 40000 40000 (40000)
horses

Analysis of the situations:

Situation 1 No need for set off since there is no loss from any source.
Situation 2 Net dividend of Rs (20000) shall be set off against the income from owning and
maintaining race horses.
Situation 3 Loss from owning and maintaining race horses cannot be set off against
dividend income.

Note 5:
SN Particulars of income/loss Situation 1 Situation 2
1 Dividend from a foreign company 20000 120000
2 Interest on amount borrowed to invest in shares of (40000) (20000)
the aforesaid company
3 Net dividend (1-2) (20000) 100000
4 Income from lotteries, horse race, card games, 120000 (40000)
cross word puzzles, bettings, gambling etc.
Conclusion Inter source adjustment not
possible - S. 58(4)

Note-6:
SN Particulars of income or loss Amount
1 LTCG on sale of gold 100000
2 LTCL on sale of listed equity shares through RSE (Transaction was (120000)
chargeable to STT)
Since LTCG on the aforesaid equity shares is not eligible for exemption U/s 10 (38) and is
chargeable to tax U/s 112A, the LTCL from such source is eligible for set off against LTCG on
sale of gold.

MT Educare –CA Inter Income Tax Classes – R.SOUMYANARAYANAN.FCA. GRAD CWA. Page-238
Chapter-9: Set off & Carry forward of losses

Note-7:

SN Particulars of income or Situation-1 Situation-2


loss
1 Profit from the business of Rs. 300000 (Rs. 200000)
production of fertilizer.
2 Profit from warehouse Rs. 100000 Rs. 100000
business
3 Profit from textile business (Rs. 200000) Rs. 200000
Conclusion Loss from non-specified Loss from specified
business could be adjusted business could be
against specified business. adjusted against income
from specified business
and not against income
from non-specified
business.

2. Inter-head adjustment – S. 71:


1 Rule of Inter-head Where the net result of computation under any head of
adjustment income is a loss, the assessee can set off such loss against his
income assessable for that AY under any other head. (See
note-1).
2 Exceptions

(a) Loss from speculation Speculative business loss cannot be set off against any other
business. income.
(b) Capital loss. Capital loss cannot be set off against income under any other
head of income.
(c) Loss from the activity Loss from the activity of owning and maintaining race horses
of owning and cannot be set off against any other income.
maintaining race
horses.
(d) Loss cannot be set off No loss can be set off against income from lotteries, horse
against winnings from race, card games, cross word puzzles, betting, gambling etc.
lotteries, cross word
puzzles etc.
(e) Loss from lottery, Loss from lottery, card games, betting, horse races etc. cannot
card games, betting, be set off against any income.
horse races etc.
(f) Loss from exempted Loss from exempted source cannot be set off against income
source. chargeable to tax.
(g) Business loss against Business loss cannot be adjusted against income U/H
salary income “Salaries”. (See note-2).
(h) Loss in a business Loss computed in respect of any specified business referred to
specified U/s 35AD in S. 35AD, cannot be set off against any other income. (See
note-3)

MT Educare –CA Inter Income Tax Classes – R.SOUMYANARAYANAN.FCA. GRAD CWA. Page-239
Chapter-9: Set off & Carry forward of losses

(i) Loss from house Where the net result of computation U/H IFHP is a loss and
property – set off the assessee has income assessable under any other head of
restriction [S. 71 income, the amount of such loss exceeding Rs. 2L would not
(3A)]. be allowable to be set off against income under the other
head.
(j) Set off of losses No loss could be set off against income from undisclosed
against income from sources referred to in S. 68 to S. 69D. [S. 115BBE].
undisclosed sources –
not possible.
(k) Set off of losses No loss could be set off against income by way of dividend
against income by referred to in S. 115BBDA. [S. 115BBDA (2)].
way of dividend
referred to in S.
115BBDA.
(l) Dividend stripping. [S. Discussed at the end of this Chapter.
94 (7)]
(m) Bonus stripping. [S. Discussed at the end of this Chapter.
94 (8)].

Note 1:

Income particulars of Mr. X are given below:

Business 1 (300000)
Business 2 70000
IFHP 530000

Loss from business 1 shall be adjusted against income from business 2 (as per S. 70). The
unabsorbed loss from business 1 (300000-70000=230000) shall be adjusted against IFHP (as
per S. 71).

Note-2:

SN Particulars of income/loss Amount


1 Income from salaries 470000
2 Loss from house property (Self-occupied) (70000)
3 Business loss (120000)
4 Bank interest 100000

Tax treatment The business loss cannot be set off against the salary income. It has to be
of loss adjusted against the bank interest.
The unabsorbed business loss of Rs. 20000 should be carried forward to
the future years. Loss from house property should be adjusted against the
salary income.

MT Educare –CA Inter Income Tax Classes – R.SOUMYANARAYANAN.FCA. GRAD CWA. Page-240
Chapter-9: Set off & Carry forward of losses

Note-3:

SN Particulars of income or loss Amount


1 Loss from the business of production of fertilizer. (Rs. 300000)
2 Profit from warehouse business Rs. 100000
3 IFHP Rs. 200000

Loss from specified business shall not be adjusted against IFHP.

Note-4:

1 Interest on debentures purchased out of borrowed funds 110000


2 Interest on aforesaid borrowings (257000)
3 Loss from other sources (1-2) (147000)
4 IFHP 180000
5 PGBP (160000)

Tax treatment of losses


1 Loss U/H IFOS cannot be carried forward to the future years. Therefore priority should
be given only to such loss. Adjust this against IFHP.
2 Business loss to the extent of Rs. 33000 (180000-147000) should be adjusted against
IFHP and the balance of Rs. 127000 should be carried forward to the next AY.

Points requiring attention:

1 Set off is compulsory and not optional.


2 Partial set off of losses is not possible. Losses need to be completely set off.

3. Carry forward and set off of losses from house property S. 71B:

(i) LFHP, to the extent not set off U/s 71, can be carried forward to the next AY and set off
only against IFHP of that AY.
(ii) The assessee can carry forward this loss for a period of 8 AYs succeeding the AY in
which the loss was first computed.

4. Carry forward and set off of capital losses – S. 74:

Where for any AY, the net result U/H ‘capital gains’ is STCL or LTCL, the loss shall be carried
forward to the following AY to be set off in the following manner:

(i) Where the loss so carried forward is a STCL, it shall be set off against any capital gains,
short term or long term, arising in that year.
(ii) Where the loss so carried forward is a LTCL, it shall be set off only against LTCG arising
in that year.
(iii) Net loss U/H capital gains cannot be set off against income under any other head.
(iv) Any unabsorbed loss shall be carried forward to the following AY up to a maximum of 8

MT Educare –CA Inter Income Tax Classes – R.SOUMYANARAYANAN.FCA. GRAD CWA. Page-241
Chapter-9: Set off & Carry forward of losses

AYs immediately succeeding the AY for which the loss was first computed.
(v) As per S. 80, the assessee must have filed a return of loss U/s 139 (3) in order to carry
forward and set off a capital loss. In other words, the non-filing of a return of loss
disentitles the assessee from carrying forward the loss sustained by him. Such a return
should be filed within the time allowed U/s 139 (1).

Q-2: During the PY 2019-20, Mr. C has the following income and the brought forward losses:

Particulars Amount
STCG on sale of shares Rs. 150000
LTCL of AY 2017-18 (Rs. 96000)
STCL of AY 2018-19 (Rs. 37000)
LTCG Rs. 75000

What is the capital gain taxable in the hands of Mr. C for the AY 2019-20?

Taxable capital gains of Mr. C for the AY 2020-21:

SN Particulars Amount Amount


1 STCG on sale of shares Rs. 150000
2 Less: Brought forward STCL of the AY 2019-20 (Rs. 37000)
3 STCG (1-2) Rs. 113000
4 LTCG Rs. 75000
5 Less: Brought forward LTCL of AY 2018-19 (See note below) (Rs. 75000)
6 LTCG (4-5) Nil
7 Taxable STCG Rs. 113000

Note:
LTCL cannot be set off against STCG. Hence, the unadjusted LTCL of AY 2018-19 of Rs. 21000
(i.e. Rs. 96000 – Rs. 75000) has to be carried forward to the next year to be set off against
LTCG of that year.

5. Carry forward and set off of loss from the activity of owning and maintaining race
horses: S. 74A:

1 Losses incurred by an assessee from the activity of owning and maintaining race horses
cannot be set off against the income from any other source other than the activity of
owning and maintaining race horses. [S. 74A (3)].
2 Such loss can be carried forward for a maximum period of 4 AYs for being set off against
the income from the activity of owning and maintaining race horses in the subsequent
years.
3 As per S. 80, the assessee must have filed a return of loss U/s 139 (3) in order to carry
forward and set off this loss. In other words, the non-filing of a return of loss disentitles
the assessee from carrying forward the loss sustained by him. Such a return should be
filed within the time allowed U/s 139 (1).

MT Educare –CA Inter Income Tax Classes – R.SOUMYANARAYANAN.FCA. GRAD CWA. Page-242
Chapter-9: Set off & Carry forward of losses

How to compute loss from the activity of owning and maintaining race horses?

1 Amount of stake money ****


2 Revenue expenditure incurred by the assessee wholly and exclusively for the ****
purposes of maintaining such horses
3 Loss from the activity of owning and maintaining race horses (1-2) (if it is ****
negative)

Note: Stake money means the gross amount of prize money received on a race horse by the
owner of the horse on account of the horse wining or being placed in the second or in the
lower position in the horse race.

Q-3: Mr. D has the following income for the PY 2019-20:


Particulars Amount
Income from the activity of owning and maintaining the race horses Rs. 75000
Income from textile business Rs. 85000
Brought forward textile business loss Rs. 50000
Brought forward loss from the activity of owning and maintaining the Rs. 96000
race horses (relating to AY 2016-17)

What is the total income in the hands of Mr. D for the AY 2020-21?

Total income of Mr. D for the AY 2020-21:

Particulars Amount
Income from the activity of owning and maintaining race horses Rs. 75000
Less: Brought forward loss from the activity of owning and maintaining race (Rs. 96000)
horses
Loss from the activity of owning and maintaining race horses to be carried (Rs. 21000)
forward to AY 2020-21

Income from textile business Rs. 85000


Less: Brought forward business loss from textile business (Rs. 50000)
Total income Rs. 35000

Note: Loss from the activity of owning and maintaining race horses cannot be set off against
any other source or head of income.

6. Carry forward & Set off of losses by specified businesses – S. 73A:

1 Any loss computed in respect of the specified business referred to in S. 35AD shall be set
off only against profits and gains, if any, of any other specified business.
2 The unabsorbed loss, if any, will be carried forward for set off against profits and gains of
any specified business in the following AY and so on.
3 There is no time limit specified for carry forward and set off and therefore, such loss can
be carried forward indefinitely for set off against income from specified business.

MT Educare –CA Inter Income Tax Classes – R.SOUMYANARAYANAN.FCA. GRAD CWA. Page-243
Chapter-9: Set off & Carry forward of losses

4 However, return of loss has to be filed on or before the due date of filing of return U/s
139 (1) for carry forward of loss from specified business.

7. Carry forward & Set off of losses from speculation business – S. 73:

1 If the losses sustained by the assessee in a speculation business cannot be set off in the
same year against any other speculation profit, they can be carried forward to
subsequent years and set off only against income from any speculation business carried
on by the assessee.
2 The loss in speculation business can be carried forward only for a maximum period of 4
years from the end of the relevant AY in respect of which the loss was computed.
3 However, return of loss has to be filed on or before the due date of filing of return U/s
139 (1) for carry forward of loss from speculation business.

Note:

1 If speculative transactions are carried out on a systematic and organized basis so as to


constitute business, then such business is called speculative business.
2 Speculative business shall be regarded as distinct and separate from any other
business. [S. 28 Explanation-2]. This is to facilitate the operation of S. 73.
3 Speculative transaction means the transaction in which the contract for purchase or
sale of any commodity (including shares and securities) is settled otherwise than by
way of actual delivery. [S. 43 (5)]. Example: Intra-day trading.
4 As per Proviso to S. 43 (5), the following are not to be regarded as speculative
transactions: (a) hedging transactions; (b) trading in derivatives carried out in a RSE; (c)
Trading in commodity derivatives carried out in a recognised association (which are
chargeable to commodities transaction tax).

5 Explanation to S. 73 provides that where any part of the business of a company


consists in purchase and sale of shares of other companies, such a company shall be
deemed to be carrying on speculation business to the extent to which the business
consists of purchase and sale of such shares.
6 However, this deeming provision does not apply to the following companies:
(a) A company whose GTI consists of mainly income chargeable U/H IFHP, CG and IFOS.
(b) A company, the principal business of which is (i) business of trading in shares; or (ii) the
business of banking; or (iii) the granting of loans and advances.

8. Carry forward and set off of business losses – S. 72:

S. 72 covers the C/F and set off of losses arising from business/profession. The assessee’s
right to carry forward losses U/s 72 is, however, subject to the following conditions:

1 The loss should have been incurred in business, profession or vocation.


2 The loss should not be in the nature of a loss in the business of speculation.
3 The loss may be carried forward and set off against the income from business or
profession though not necessarily against the profits and gains of the same business or

MT Educare –CA Inter Income Tax Classes – R.SOUMYANARAYANAN.FCA. GRAD CWA. Page-244
Chapter-9: Set off & Carry forward of losses

profession in which the loss was incurred.


4 However, a loss carried forward cannot be set off against income from any head other
than PGBP.
5 The loss can be carried forward and set off against the profits of the assessee who
incurred the loss. That is, only the person who has incurred the loss is entitled to carry
forward or set off the same. Consequently, the successor of a business cannot be carry
forward or set off the losses of his predecessor except in the case of succession by
inheritance.
6 A business loss can be carried forward for a maximum period of 8 AYs immediately
succeeding the AY in which the loss was incurred.
7 As per S. 80, the assessee must have filed a return of loss U/s 139 (3) in order to carry
forward and set off a loss.
In other words, the non-filing of a return of loss disentitles the assessee from carrying
forward the loss sustained by him. Such a return should be filed within the time allowed
U/s 139 (1).

9. Tax treatment of unabsorbed depreciation:

(i) Where, in any PY, the profits chargeable are not sufficient to give full effect to the
depreciation allowance, the unabsorbed depreciation shall be added to the
depreciation for the following PY and shall be deemed to be part of that allowance.
(ii) If no depreciation allowance is available for that PY, the unabsorbed depreciation of
the earlier PY shall become the depreciation allowance of that year.
(iii) The effect of this provision is that the unabsorbed depreciation shall be carried
forward indefinitely till it is fully set off.
(iv) However, in the order of set off of losses under different heads of income, effect shall
first be given to the business losses and then to unabsorbed depreciation.

Essence:

(i) Since the unabsorbed depreciation forms part of the current year’s depreciation, it can
be set off against any other head of income except ‘salaries’.
(ii) The unabsorbed depreciation can be carried forward for indefinite number of PYs.
(iii) Set off will be allowed even if the same business to which it relates is no longer in
existence in the year in which the set off takes place.

Note:

1 Unabsorbed capital expenditure on scientific research and unabsorbed expenditure on


promotion of family planning amongst employees shall be treated on par with
unabsorbed depreciation.
2 As per the provisions of S. 72 (2), brought forward business loss is to be set off before
setting off unabsorbed depreciation.
3 Therefore, the order in which set off will be effected is as follows: (a) Current year
depreciation; (b) Current year capital expenditure on scientific research and current year
expenditure on promotion of family planning; (c) brought forward loss from business or

MT Educare –CA Inter Income Tax Classes – R.SOUMYANARAYANAN.FCA. GRAD CWA. Page-245
Chapter-9: Set off & Carry forward of losses

profession; (d) unabsorbed depreciation; (e) unabsorbed capital expenditure on


scientific research; (f) unabsorbed expenditure on promotion of family planning.

10. Dividend stripping – S. 94 (7):

Introduction:

1 Meaning of Dividend stripping is a process by which an investor or dealer gets


dividend stripping tax free dividend by investing in shares or units shortly before the
record date and exiting after the record date at a lower price
thereby incurring loss.
2 Motivation for S. 10 (34) exempts dividend from domestic companies. Similarly S.
dividend stripping 10 (35) exempts income from units of mutual funds etc. This
exempted dividend or income compensates the loss suffered by
the assessee incurred on account of sale of such shares or units.

This loss can be set off against other income (if permitted) and if it
is still unabsorbed, it can be carried forward for set off in the
future years. Thus, dividend stripping helps the assessee in
creating artificial loss which can be used for reducing his income.
3 Measure for S. 94 (7) is inserted to curb the practice of dividend stripping.
curbing the practice
of dividend
stripping

Applicability of S. 94 (7):

S. 94 (7) will apply if the following cumulative conditions are satisfied:

In respect of shares In respect of units


1 The assessee shall purchase shares within 1 The assessee shall purchase units within
a period of 3 months prior to record date. a period of 3 months prior to record
date.
2 He shall sell such shares within a period 2 He shall sell such units within a period of
of 3 months after the record date. 9 months after the record date.
3 Dividend from such shares is exempt U/s 3 Income from such units is exempt U/s 10
10 (34). (35).

Effect of S. 94 (7):

Loss arising on account of such purchase and sale of shares/units shall be ignored to the
extent of the aforesaid of dividend or income.

MT Educare –CA Inter Income Tax Classes – R.SOUMYANARAYANAN.FCA. GRAD CWA. Page-246
Chapter-9: Set off & Carry forward of losses

Points requiring attention:

1 S. 94 (7) covers holding of shares/units both as capital asset and as stock in trade.
Hence, S. 94(7) would apply to both investors and traders of shares/units.
2 If shares or units are held as capital assets, the resultant loss will be a capital loss.
As per S. 71, such capital loss is not eligible for inter-head adjustment. Also carry forward
of such loss is not possible because of the provisions of S. 94 (7), which is otherwise
permitted U/s 74.
3 If shares or units are held as stock-in-trade, the resultant loss will be a business loss,
which is generally eligible for inter-source adjustment, inter-head adjustment and CF.
However, in view of S. 94 (7), none of these is possible. In short, such loss shall be
ignored.

Q-5: Mr. X purchases on 10.05.19, 1000 equity shares of Rs. 10 each in X Ltd @ Rs. 55.55. On
20.10.19, he transfers 800 equity shares @ Rs. 37 per share and remaining 200 shares are
transferred on 20.12.18 @ Rs. 20 per share. X Ltd declares 50% dividend (record date:
03.08.19). During the PY 2019-20, he has generated LTCG of Rs. 76000 on sale of gold.
Compute the total income of X for the PY 2019-20.

Computation of loss eligible for set off:

SN Particulars 800 shares 200 shares


1 Date of purchase 10.05.2019 10.05.2019
2 Record date 03.08.2019 03.08.2019
3 Date of transfer 20.10.2019 20.12.2019
4 Time gap between the date of Less than 3 months Less than 3 months
purchase & RD
5 Time gap between RD & date of < 3 months > 3 months
transfer
6 Whether S. 94 (7) is applicable? Yes No
7 Dividend exempted U/s 10 (34) Rs. 4000 (800*Rs. Rs. 1000 (200*Rs.
10*50%) 10*50%)
8 Sale consideration (800* 37) = Rs. 29600 (200*20) = Rs. 4000
9 Cost of acquisition (800*Rs. 55.55)= Rs. (200*Rs. 55.55)= Rs.
44440 11110
10 STCL (8-9) (Rs. 14840) (Rs. 7110)
11 Loss to be ignored as per S. 94 (7) Rs. 4000 Nil
12 Loss eligible for set off (10-11) Rs. 10840 Rs. 7110

Computation of total income:

1 LTCG on sale of gold Rs. 76000


2 STCL on sale of 800 shares (Rs. 10840)
3 STCL on sale of 200 shares (Rs. 7110)
4 Taxable LTCG (1-2-3) RS. 58050

MT Educare –CA Inter Income Tax Classes – R.SOUMYANARAYANAN.FCA. GRAD CWA. Page-247
Chapter-9: Set off & Carry forward of losses

11. Bonus Stripping – S. 94 (8):

Meaning of bonus stripping and the necessity of S. 94 (8):

1 Meaning of bonus Bonus stripping is a process by which an investor or dealer gets


stripping bonus units by investing in units shortly before the record date
and exiting shortly after the record date at a lower price thereby
incurring loss.
2 Motivation for The assessee gets bonus units which compensates the loss
bonus stripping incurred by him on account of sale of units. Further, the investor
can set off such loss against other income (if permitted) and can
also carry forward the unabsorbed loss for set off in future years.
3 Measure for S. 94 (8) is inserted to curb the practice of bonus stripping.
curbing the
practice of bonus
stripping

Applicability of S. 94 (8):

S. 94 (8) will apply if the following cumulative conditions are satisfied:


1 The assessee purchases units within a period of 3 months prior to the record date.
2 He is allotted additional units without any payment on the basis of holding of units on
such date.
3 He sells all or any of the units referred to in (2) within a period of 9 months after the
record date.
4 On the date of sale, he shall hold at least one of the additional bonus units allotted to
him.

Effect of S. 94 (8):

Loss arising on account of such purchase and sale of units shall be ignored. However, such
loss will be considered to be the COA of the bonus units held on the date of sale.

Points requiring attention:


1 This provision applies only to units and not to shares.
2 It is applicable even in case where units are held as stock in trade.

Q-6: A mutual fund declared 1:1 bonus units on its units on 30.04.19. The fund fixed the
record date for bonus entitlement to be 31.05.19. Mr. A purchased 1000 units on 20.05.19
@ Rs. 20 per unit. He sold 1000 original units on 11.11.19 for Rs. 9 per unit. Discuss the tax
implications.

Tax implications:
(a) Applicability of S. 94 (8): S. 94 (8) is applicable here since: Units were purchased within a
period of 3 months prior to record date. Bonus units were allotted on record date. Original
units were sold within a period of 9 months after the record date while continuing to hold
the bonus units.

MT Educare –CA Inter Income Tax Classes – R.SOUMYANARAYANAN.FCA. GRAD CWA. Page-248
Chapter-9: Set off & Carry forward of losses

(b) Computation of STCG on sale of original units:

Period of holding 20.05.18 to 10.11.18 Short term


Sale consideration 1000*Rs. 9 Rs. 9000
Cost of acquisition 1000*Rs. 20 (Rs. 20000)
STCL (Rs. 11000)

(c) This STCL of Rs. 11000 shall be ignored for the purpose of computing the total income. It
can neither be set off nor be carried forward.

(d) The cost of acquisition of 1000 bonus units shall be taken to be Rs. 11000.

Q-7: Suppose in Q-6, Mr. A sold 1000 original units and 800 bonus units on 11.11.16 for Rs. 9
per unit. Discuss the tax implications.

Tax implications:

(a) Applicability of S. 94 (8): S. 94 (8) is applicable here since units were purchased within a
period of 3 months prior to record date. Bonus units were allotted on record date. Original
units were sold within a period of 9 months after the record date while continuing to hold
200 bonus units.

(b) Computation of capital gains:

1000 Original units 800 Bonus units


Period of 20.05.19 to Short Period of 31.05.19 to Short
holding 10.11.19 term holding 10.11.19 term
Sale price 1000*Rs. 9 Rs. 9000 Sale price 800*Rs. 9 Rs. 7200
Cost of 1000*Rs. 20 (Rs. Cost of Nil as per S. 55 Nil
acquisition 20000) acquisition
STCL (Rs. STCG Rs. 7200
11000)

(c) As per S. 94 (8), STCL of Rs. 11000 shall be ignored and shall not be set off or carried
forward.

(d) The STCG of Rs. 7200 shall be taxable.

(e) The cost of acquisition of 200 bonus units shall be Rs. 11000.

Q-8: Suppose in Q-7, Mr. A sold 1000 original units on 11.11.19 @ Rs. 9 per unit and 800
bonus units on 15.11.19 @ Rs. 9 per unit. Discuss the tax implications.

MT Educare –CA Inter Income Tax Classes – R.SOUMYANARAYANAN.FCA. GRAD CWA. Page-249
Chapter-9: Set off & Carry forward of losses

Tax implications:

(a) Applicability of S. 94 (8): S. 94 (8) is applicable here since: Units were purchased within a
period of 3 months prior to record date. Bonus units were allotted on record date. Original
units were sold within a period of 9 months after the record date while continuing to hold
1000 bonus units.

(b) Computation of capital gains:

1000 Original units


Period of holding 20.05.19 to 10.11.19 Short term
Sale price 1000*Rs. 9 Rs. 9000
Cost of acquisition 1000*Rs. 20 (Rs. 20000)
STCL (Rs. 11000)

Note:

Now as per S. 94 (8), STCL of Rs. 11000 shall be ignored and shall not be set off or carried
forward.

Number of bonus units held on the date of transfer of original units is 1000 and as per S. 94
(8), the cost of acquisition of these 1000 bonus units shall be taken to be Rs. 11000.

800 Bonus units


Period of holding 31.05.18 to 14.11.18 Short term
Sale price 800*Rs. 9 Rs. 7200
Cost of acquisition (Rs. 11000/1000)*800 Rs. 8800
STCL (Rs. 1600)

(c) The short term capital loss of Rs. 1600 shall be allowed to be set off and carried forward.

(d) The cost of acquisition of 200 bonus units shall be Rs. 2200.

MT Educare –CA Inter Income Tax Classes – R.SOUMYANARAYANAN.FCA. GRAD CWA. Page-250
Chapter-10: Chapter-VIA deductions

Introduction:

1 Chapter VI-A (S. 80C – S. 80U) allows deductions against gross total income.
2 Aggregate amount of deductions under Chapter VI-A cannot exceed the gross total income.
3 If the gross total income includes casual income, income from undisclosed sources, LTCG or STCG
referred to in S. 111A, then Chapter VI-A deductions cannot be availed against that portion of
gross total income which represents the incomes aforesaid.
4 Deductions under Chapter VIA are of 3 types: (a) Deduction in respect of specified payments,
donations, subscriptions, investments etc (S. 80C to S. 80GGC); (b) Deduction in respect of
specified income (S. 80-IA to S. 80TTA); (c) Deduction on account of disability (S. 80U).

Sections forming part of syllabus:

1 S. 80C 10 S. 80GG
2 S. 80CCC 11 S. 80GGA
3 S. 80CCD 12 S. 80GGB
4 S. 80D 13 S. 80GGC
5 S. 80DD 14 S. 80JJAA
6 S. 80DDB 15 S. 80RRB
7 S. 80E 16 S. 80QQB
8 S. 80EE 17 S. 80TTA
(Already covered in IFHP chapter)
9 S. 80G 18 S. 80U

1. Deduction U/s 80GGB:

1 Assessee eligible for Indian company.


deduction
2 Nature of deduction Assessee is eligible for deduction in respect of contributions made to
registered political parties or electoral trust during the PY (otherwise
than by way of cash).
3 Extended meaning of Political contribution includes even the expenditure incurred on
political contribution advertisement in any publication (being a publication in the nature of a
souvenir, brochure, tract, pamphlet or the like) by or on behalf of a
political party or for its advantage.
4 Ceiling on deduction Nothing is specified in the section.

Note:

1 Any expenditure on advertisement in a political magazine incurred by an Indian company


is disallowed U/s 37 (2B) while computing income U/H PGBP.
2 However, it is allowed as deduction U/s 80GGB against the GTI.

MT Educare –CA Inter Income Tax Classes – R.SOUMYANARAYANAN.FCA. GRAD CWA. Page-251
Chapter-10: Chapter-VIA deductions

2. Deduction U/s 80GGB:

1 Assessee eligible Any person other than (a) Indian company; (b) local authority; (c)
for deduction AJP wholly or partly funded by the Government.
2 Nature of Assessee is eligible for deduction in respect of contributions made to
deduction registered political parties or electoral trust during the PY (otherwise
than by way of cash).
3 Ceiling on Nothing is specified in the section.
deduction

3. Deduction U/s 80E:

1 Assessee eligible for Individual


deduction
2 Nature of deduction Deduction is provided in respect of interest on loan paid
by the assessee during the PY out of his income
chargeable to tax.
3 Conditions for availing The loan must have been taken for the purpose of
deduction pursuing his higher education or for the purpose of
higher education of his relative.
The loan must have been taken from a financial
institution or approve d charitable institution.
3 Meaning of ‘relative’ Spouse + children + student for whom the individual is
the legal guardian.
4 Meaning of ‘higher education’ It means any course of study (including vocational
studies) pursued after passing the Senior Secondary
Examination or its equivalent from any school, board, or
university recognised by the CG or SG or local authority
or by any other authority authorised by the CG or SG or
local authority to do so. Precisely, higher education
means any course after Class XII.
5 Meaning of ‘financial (a) Banks; (b) any other financial institution notified by
institution’ the CG in official gazette.
6 Meaning of ‘approved Institution established for charitable purposes and
charitable institution’ approved by the prescribed authority (U/s 10 (23C)) or
an institution referred to in S. 80G (2) (a).

7 Period of deduction AY relevant to the PY in which assessee starts paying


interest on loan (called as ‘initial AY’) + 7 AYs
immediately succeeding the initial AY or until the
interest is paid in full by the assessee, whichever is
earlier.

MT Educare –CA Inter Income Tax Classes – R.SOUMYANARAYANAN.FCA. GRAD CWA. Page-252
Chapter-10: Chapter-VIA deductions

4. Deduction U/s 80C:

1 Persons eligible for Individual or HUF


deduction
2 Nature of deduction Deduction is allowed in respect of specified payments and
investments.

3 Specified payment Any sum paid or deposited in the PY by the assessee


and investments
(i) as life insurance policy to effect or keep in force insurance
on life (a) self, spouse and any child in case of individual
and (b) any member, in case of HUF. [See note-1].
(ii) to effect or keep in force a deferred annuity contract on
life of self, spouse and any child in case of individual.
[such contract should not contain a provision for cash
payment option in lieu of payment of annuity].
(iii) by way of deduction from salary payable by or on behalf
of the government to any individual for the purpose of
securing him a deferred annuity or making provision for
his spouse or children. [The sum deducted should not
exceed 1/5th of salary].
(iv) as contribution (not being repayment of loan) by an
individual to SPF.
(v) as contribution to PPF in the name of self, spouse and any
child in case of individual and any member in case of HUF.
(Maximum amount that could be deposited = Rs.
150000).
(vi) as contribution by an employee to RPF.
(vii) as contribution by an employee to ASAF.
(viii) under Sukanya Samriddhi account, in the name of the
individual or any girl child of that individual or any girl
child for whom such person is the legal guardian.
(ix) subscription to NSC.
(x) as a contribution to ULIP of UTI or LIC mutual fund
(Dhanraksha plan) in the name of self, spouse and any
child in case of individual and any member in case of HUF.
(See note-2).
(xi) to effect or to keep in force a contract for such annuity plan
of LIC (i.e. Jeevan Dhara, Jeevan Akshay and their
upgradations) or any other insurer as referred to in by the
CG. [Tata AIG Easy retire annuity plan of Tata AIG Life
Insurance company Ltd].
(xii) as subscription to any units of mutual fund. [Equity linked
savings scheme].
(xiii) as contribution by an individual to any pension fund set
up by any mutual fund or UTI. [UTI Retirement Benefit
pension fund].

MT Educare –CA Inter Income Tax Classes – R.SOUMYANARAYANAN.FCA. GRAD CWA. Page-253
Chapter-10: Chapter-VIA deductions

(xiv) as subscription to any such deposit scheme of National


housing bank [National Housing Bank (Tax savings) Term
Deposit Scheme, 2008] or as a contribution to any such
pension fund set up by NHB as notified by the CG.
(xv) as subscription to notified deposit schemes of (a) public
sector company providing long-term finance for purchase
or construction of residential houses in India; (b) any
authority constituted in India for the purposes of housing
or planning, development or improvement of cities,
towns and villages. [Public deposit scheme of HUDCO].
(xvi) as tuition fees (excluding any payment towards any
development fees or donation or payment of similar
nature), to any university, college, school or other
educational institution situated within India for the
purposes of full-time education of any two children of
individual.
(xvii) towards the cost of purchase or construction of a
residential house property. [The income from house
property should be chargeable to tax U/H IFHP or would
have been chargeable to tax U/H IFHP had it not been
used for assessee’s own residence.] (See note-3).
(xviii) as subscription to equity shares or debentures forming
part of any eligible issue of capital of public company or
PFI approved by the Board. (See note-5).
(xix) as term deposit for 5 years or more with Scheduled Bank
in accordance with a scheme framed and notified by the
CG. [Maximum limit for investment = Rs. 150000].
(xx) as subscription to any notified bonds of NABARD.
(xxi) in an account under the Senior Citizen Savings Scheme
Rules, 2004. (See note-4).
(xxii) as five-year term deposit in an account under the Post
office time deposit Rules, 1981. (See note-4).
(xxiii) Contribution by a Central Government employee to
additional account under NPS (specified account) referred
to in section 80CCD for a fixed period of not less than 3
years and which is in accordance with the scheme notified
by the Central Government for this purpose qualifies for
deduction under section 80C.

It may be noted that only the contribution to the


additional account under NPS will qualify for deduction
under section 80C.

There are two types of NPS account i.e., Tier I and Tier II,
to which an individual can contribute.

Section 80CCD provides deduction in respect of

MT Educare –CA Inter Income Tax Classes – R.SOUMYANARAYANAN.FCA. GRAD CWA. Page-254
Chapter-10: Chapter-VIA deductions

contribution to individual pension account [Tier I account]


under the NPS [referred to in section 20(2)(a) of the
Pension Fund Regulatory and Development Authority Act,
2013 (PFRDA)] whereas deduction under section 80C is
allowable in respect of contribution by Central
Government employee to additional account [Tier II
account] of NPS [referred to in section 20(3) of the
PFRDA], which does not qualify for deduction under
section 80CCD.

Thus, Tier II account is the additional account under NPS,


contribution to which would qualify for deduction under
section 80C only in the hands of a Central Government
employee.

Note-1: Points regarding (i):

1 Child may be son/daughter, own/adopted, minor/major, dependent/independent or


married/unmarried.
2 Ceiling on deduction:
Policy issued before 01.04.2012 20% of the actual
capital sum
assured.
Policy issued on or after 01.04.2012 10% of the actual
capital sum
assured.
Policy issued on or after 01.04.2013 [In case of persons with 15% of the actual
disability or person with severe disability as per S. 80U or capital sum
suffering from disease or ailment as specified in S. 80DDB] assured.

3 The actual capital sum assured shall mean the minimum amount assured under the
policy on happening of the insured event at any time during the term of the policy, not
taking into account (a) the value of any premium agreed to be returned; or (b) any
benefit by way of bonus or otherwise over and above the sum actually assured which is
to be or may be received under the policy by any person.
4 If an assessee terminates or discontinues a life insurance policy before the premiums for
two years have been paid, no deduction will be allowed in respect of premium paid in
the year of termination. The deduction allowed in the past year shall be deemed to be
the income of the assessee of the previous year in which the insurance policy is
terminated.
5 Any sum received under life insurance policy including sum allocated by way of bonus on
such policy is exempt U/s 10 (10D).

MT Educare –CA Inter Income Tax Classes – R.SOUMYANARAYANAN.FCA. GRAD CWA. Page-255
Chapter-10: Chapter-VIA deductions

6 However, the following are not exempt:


(a) Key man insurance policy proceeds.
(b) Life insurance policy proceeds, if the premium payable for any of the years during
the term of the policy exceeds 20% or 10% or 15%, as the case may be, of actual
capital sum assured. [However, if the sum is payable upon death of the insured,
S. 10 (10D) exemption is available].

Note-2: Points regarding (x):

1 Where a member participating in ULIP terminates his participation before making


contribution for 5 years, the amount contributed by him in the PY of termination shall
not be qualified for deduction U/s 80C.
2 The quantum of deduction already availed in the PY preceding the PY of termination,
would be deemed as income of the assessee of the PY of termination.

Note-3: Points regarding (xvii):

The approved types of payments are as follows:

1 Any instalment of the amount due under any self-financing or other scheme of any
development authority, housing board or other authority engaged in the construction
and sale of house property on ownership basis.
2 Any instalment of the amount due to any company or co-operative society of which the
assessee is a shareholder or member towards the cost of the house property allotted to
him.
3 Repayment of the amount borrowed by the assessee from
(i) the Central Government or any State Government or
(ii) any Bank, including a Co-operative Bank.
(iii) LIC
(iv) NHB
(v) Any public company formed and registered in India with the main object of
carrying on the business of providing long term finance for construction or
purchase of houses in India for residential purposes which is eligible for
deduction U/s 36 (1) (viii).
(vi) Any company in which the public are substantially interested or any co-
operative society, where such company or co-operative society is engaged in
the business of financing the construction of houses.
(vii) The assessee’s employer where such employer is a public company or a public
sector company or a university established by a law or a college affiliated to
such university or a local authority or a co-operative society.
(viii) The assessee’s employer where such employer is an authority or a board or a
corporation or any other body established or constituted under a Central or
state Act.
4 Stamp duty, registration fee and other expenses for the purpose of transfer of such
property to the assessee.

MT Educare –CA Inter Income Tax Classes – R.SOUMYANARAYANAN.FCA. GRAD CWA. Page-256
Chapter-10: Chapter-VIA deductions

However, the following amounts do not qualify for deduction:


(a) The admission fee, cost of share and initial deposit which a shareholder of a company or a
member of a co-operative society has to pay for becoming such shareholder or member.
(b) The cost of addition or alteration to, or renovation or repair of, the house property which is
carried out after the issue of the completion certificate in respect of the house property by the
authority competent to issue such certificate or after the house property or any part thereof
has either been occupied by the assessee or any other person on his behalf or been let out.
(c) Any expenditure in respect of which deduction is allowable U/s 24.

If the assessee transfers the house property, in respect of which deduction has been
claimed, before the expiry of 5 years from the end of the financial year in which possession
of such property was obtained by him, no deduction shall be allowable in the previous year
in which the house property is transferred. The aggregate deduction allowed in the past
years shall be deemed to be income of the assessee of the previous year in which the house
property is transferred.
Note-4: Points regarding (xxi) & (xxii):

1 Where any amount is withdrawn by the assessee from his account under the Senior
Citizens Savings Scheme or under the PO time deposit Rules before the expiry of a period
of 5 years from the date of its deposit, the amount so withdrawn shall be deemed to be
the income of the assessee of the PY in which the amount is withdrawn.
2 Accordingly, the amount so withdrawn would be chargeable to tax in the AY relevant to
such PY. The amount chargeable to tax would also include that part of amount
withdrawn which represents interest accrued on the deposit.
3 However, if any part of the amount relating to interest so received or withdrawn has
been subject to tax in any of the earlier years, such amount shall not be taxed again.
4 If any amount has been received by the nominee or legal heir of the assessee on the
death of such assessee, the amount would not be chargeable to tax. But if the amount
relating to interest on deposit was not included in the total income of the assessee in
any of the earlier years, then such interest would be chargeable to tax.

Note-5: Points regarding (xviii):

1 A lock in period of 3 years is provided in respect of such equity shares or debentures.


2 In case of any sale or transfer of shares or debentures within 3 years of the date of
acquisition, the aggregate amount of deductions allowed in respect of such equity shares
or debentures in the PY(s) preceding the PY in which such sale or transfer has taken
place shall be deemed to be income of the assessee of such PY and shall be liable to tax
in the AY relevant to such PY.

5. Deduction in respect of contribution to certain pension funds (S. 80CCC):

MT Educare –CA Inter Income Tax Classes – R.SOUMYANARAYANAN.FCA. GRAD CWA. Page-257
Chapter-10: Chapter-VIA deductions

1 Assessee eligible for Individual


deduction
2 Nature of deduction Under this section, the assessee is entitled to deduction in respect of
amount paid /deposited by him to effect a contract for annuity plan
of LIC or any other insurer for receiving pension from the fund set up
by LIC or any other insurer.
3 Conditions for availing He should have deposited a sum under an annuity plan of LIC or any
deduction other insurer for receiving pension.
Amount of bonus or interest credited to his accounts by the insurer
is not to be regarded as amount deposited.
Amount paid should be out of the taxable income of the current or
earlier years.
No deduction is allowed u/s 80C in respect of amount allowed as
deduction under this section
4 Quantum of deduction Amount deposited or Rs. 150000 whichever is less.
Note: Deduction U/s 80C, S. 80CCC and S. 80CCD (1) shall not exceed
Rs. 150000. [S. 80CCE].
5 Taxability of amount Where the assessee or his nominee, upon surrender of the annuity
received upon plan, receives any amount (including interest or bonus) standing to
surrender of annuity the credit of the assessee in respect of which deduction U/s 80CCC
was allowed, such amount shall be included in the total income of
the assessee or his nominee in the year of receipt.
6 Taxability of pension Where the assessee or his nominee receives pension from the
annuity plan, it is taxable in the hands of recipient in the year of
receipt.

6. Deduction U/s 80CCD:


1 Person who can avail Employees of CG who joined service on or after 01.01.2004.
the benefits of New Employees of non-Government enterprises (to whom the
Pension Scheme scheme is made available by the employer).
Self-employed persons who have subscribed to this scheme.
2 Modus operandi Every employee or subscriber will have an account opened
under this scheme to which he makes monthly contributions.
Employer also makes contributions to the a/c (supra).
The contributions are allocated to approved pension fund
managers who make investment in (a) government securities;
(b) debt securities; (c) equity; (d) money market instruments.
New Pension System Trust administers the funds and assets of
the scheme for the benefit of employees and subscribers.
The return on investment (net of expenses) is credited to the
pension account of the employee.
Upon closure of pension account or exit from the scheme, the
subscriber receives the accumulate balance lying in his pension
account (referred to as pension wealth).
The subscriber may, instead of receiving the pension wealth,
may require the NPST to purchase annuity out of pension wealth

MT Educare –CA Inter Income Tax Classes – R.SOUMYANARAYANAN.FCA. GRAD CWA. Page-258
Chapter-10: Chapter-VIA deductions

or part thereof.
Also, there may be partial withdrawal of pension wealth without
closure of pension account.
3 Tax implications in the hands of subscriber (being an employee) in respect of his
contribution and the contribution of employer:
(a) Employer’s It is salary in the hands of employee (as per S. 17 (1)).
contribution It is allowed as deduction U/s 80CCD (2) in the hands of
employee to the extent of 14% of (Salary + DA (if the terms of
employment so provides)).
(b) Employee’s In respect of this, deduction is allowed U/s 80CCD (1B) to the
contribution employee to the extent of Rs. 50000.
Also u/s 80CCD (1), deduction is allowed to the employee to the
extent of lower of the following: (a) Employer’s contribution in
excess of Rs. 50000; (b) 14% of (Salary + DA (if the terms of
employment so provides)).
4 Tax implications in the hands of subscriber (being self-employed person) in respect of his
contribution
Deduction in respect Contribution to pension account is allowed as deduction U/s
of own contribution 80CCD (1B) to the extent of Rs. 50000.
Also u/s 80CCD (1), deduction is allowed to the subscriber to the
extent of lower of the following: (a) Contribution in excess of Rs.
50000; (b) 20% of GTI.
5 Ceiling in S. 80CCE Ʃ Deductions U/s 80C + S. 80CCD (1) + S. 80CCC ≤ Rs. 150000.
6 Tax implications of payments out of pension wealth in the hands of the subscriber:
(a) Payment of pension Exempt in the hands of the nominee. [Proviso to S. 80CCD (3)].
wealth to the
nominee upon death
of the employee
(b) Partial withdrawal Any payment from the NPST to an employee under the pension
from NPS account scheme referred to in S. 80CCD, on partial withdrawal made out
of his account in accordance with the terms and conditions,
specified under the Pension Fund Regulatory and Development
Authority Act, 2013 and the regulations made thereunder, to
the extent it does not exceed 25% of the amount of
contributions made by him, shall be exempt from tax. [S. 10
(12B)].
(c) Subscriber closes his 60% of the pension wealth is exempt U/s 10 (12A).
account under NPS or Balance, if used for purchasing annuity, is exempt u/s 80CCD (5).
opts out of NPS Otherwise, it is taxable U/s 80CCD (3) (a).
(d) Pension paid by Taxable in the hands of the assessee. [S. 80CCD (3) (b)].
annuity service
provider

MT Educare –CA Inter Income Tax Classes – R.SOUMYANARAYANAN.FCA. GRAD CWA. Page-259
Chapter-10: Chapter-VIA deductions

7 Tax implications in the hands of the employer:


Deduction in respect of In respect of employer’s contribution to the pension account
his contribution to the of the employee, the employer gets deduction U/s 36 (1)
pension account of the (iva) while computing income U/H PGBP. Deduction is
employee restricted to 14% of (Salary + DA (if the terms of
employment so provides)).
8 Tax implications in the hands of NPST:
(a) Exemption U/s 10 (44) Any income received by NPST is exempt U/s 10 (44).
(b) Exemption from TDS Any payments made to NPST shall not be subjected to TDS.
[S. 197A (1E)].
(c) Exemption from STT Finance Act 2004 Chapter VII exempts NPST from STT
payable in respect of purchase and sale of equity and
derivatives.
9 Exemption from DDT - S. On dividend payable to NPST, the investee-company need
115-O (1A) not pay DDT.

‘Atal Pension Yojna’ notified U/s 80CCD (1) [Notification 7/2016 dated 19-02-16]

S. 80CCD (1) empowers the CG to notify a pension scheme, contribution to which would
qualify for deduction in the hands of an individual assessee. Accordingly, in exercise of the
powers conferred by S. 80CCD (1), the CG has notified the ‘Atal Pension Yojana (APY)' as
published in the Gazette of India, Extraordinary, Part I, Section 1, vide number F. No.
16/1/2015-PR dated 16th October, 2015 as a pension scheme, contribution to which would
qualify for deduction U/s 80CCD in the hands of the individual.

7. Deduction U/s 80D:


Payments Deduction in case of Deduction in case of
individual HUF
Insured or person Insured or person
treated treated
Family Parents Any member of HUF
A Mediclaim insurance premium Eligible Eligible Eligible
B Contribution to CGHS or Government notified Eligible
health schemes
C Preventive health check up payments Eligible Eligible
D Maximum deduction
General deduction (applicable in respect of A, B Rs. 25000 Rs. 25000 Rs. 25000
and C)
Additional deduction (applicable only in case of Rs. 5000 Rs. 5000 Rs. 5000
A when insured is a senior citizen)
E Medical expenditure on the health of a person Eligible Eligible Eligible
who is a super senior citizen (if mediclaim
insurance is not paid on the health of such
person)
F Maximum deduction in respect of E Rs. 30000 Rs. 30000 Rs. 30000
G Maximum deduction in respect of A, B, C and E Rs. 30000 Rs. 30000 Rs. 30000

MT Educare –CA Inter Income Tax Classes – R.SOUMYANARAYANAN.FCA. GRAD CWA. Page-260
Chapter-10: Chapter-VIA deductions

Points requiring attention:


1 Family = Individual + Spouse + Dependent children.
2 No restriction on number of children.
3 Father-in-law and mother-in-law are not part of family.
4 Grandparents, brothers and sisters are not part of family.
5 Parents may be dependent or independent.
6 Mediclaim insurance premium means premium paid to effect an insurance on the
health under a scheme framed by GIC and approved by the CG or by any other insurer
and approved by IRDA.
7 Deduction in respect of the aggregate payment on account of preventive health check
up of self, spouse, dependent children, father and mother cannot exceed Rs. 5000.
8 All payments (supra) other than preventive health check up payments should be made
through a mode other than cash.
9 However, payment on account of preventive health check up can be made by any mode
(including cash).
10 Senior-citizen = Resident individual who is atleast 60 years at any time during the PY.

8. Deduction U/s 80U:

1 Assessee eligible Disabled resident individual.


for deduction
2 Meaning of Person suffering from atleast 40% of
disabled person Disability

Blindness Leprosy - Locomotor


Mental illness
cured Disability

Low vision Hearing Mental Autism,


impairment retadation cerebral palsy

3 Meaning of Person suffering from atleast 80% of the aforesaid disabilities.


severely disabled
person
4 Quantum of If the assessee is ordinarily disabled, the deduction will be Rs.
deduction 75000. However, if the assessee is severely disabled, the deduction
will be Rs. 125000.
5 Certificate by The assessee shall have to furnish a copy of the certificate (in form
medical authority no. 10-IA) issued by the medical authority along with the return of
income. Where the condition of disability requires reassessment, a
fresh certificate from the medical authority shall have to be
obtained after the expiry of the period mentioned on the original
certificate in order to continue to claim the deduction.

9. Deduction U/s 80DD:

MT Educare –CA Inter Income Tax Classes – R.SOUMYANARAYANAN.FCA. GRAD CWA. Page-261
Chapter-10: Chapter-VIA deductions

1 Assessee eligible Individual/HUF (being resident in India).


for deduction
2 Nature of Deduction in respect of maintenance including medical treatment of
deduction disabled dependent.
3 Disabled person Person suffering from at least 40% of
Disability

Blindness Leprosy - Locomotor


Mental illness
cured Disability

Low vision Hearing Mental Autism,


impairment retardation cerebral palsy

4 Severely Person suffering from atleast 80% of the aforesaid disabilities.


disabled person
5 Dependent Dependent

In case of In case of
Individual HUF

Any member
Spouse Children Parents Brothers/
Sisters

Note: These persons should be dependant wholly or mainly on the


assessee (individual or HUF) for his support and maintenance.
6 Conditions for The assessee should have incurred an expenditure for medical treatment,
deduction training & rehabilitation of a disabled dependent or
He should have deposited an amount under a scheme framed by LIC or
other insurer or UTI and approved by CBDT, for the maintenance of
disabled dependent.
The aforesaid scheme should provide for payment of annuity or lump sum
amount to the disabled dependent in the event of death of assessee.
The disabled dependent should not claim deduction u/s. 80U.
The assessee should furnish a copy of certificate issued by the medical
authority in Form no. 10-IA along with his ROI.
Note: Where the condition of disability requires reassessment of its extent
after a period stipulated in the aforesaid certificate, no deduction under
this section shall be allowed for any assessment year relating to any
previous year beginning after the expiry of the previous year during which
the aforesaid certificate of disability had expired, unless a new certificate
is obtained from the medical authority in the form and manner prescribed
and a copy thereof is furnished along with the ROI. (R. 11A (2)).
7 Quantum of If the person maintained is ordinarily disabled, the deduction will be Rs.
deduction 75000. Whereas, the person maintained is severely disabled, the
deduction will be Rs. 125000.
Note: The quantum of deduction is fixed. No regard should be had to
actual expenditure.

MT Educare –CA Inter Income Tax Classes – R.SOUMYANARAYANAN.FCA. GRAD CWA. Page-262
Chapter-10: Chapter-VIA deductions

8 Death of If the disabled dependent predeceases the assessee, an amount equal to


disabled the amount paid or deposited under the scheme shall be deemed to be
dependent the income of the assessee of the PY in which such amount is received by
the assessee and shall accordingly be charged to tax as the income of that
PY. Exemption U/s 10 (10D) is not available.

10. Deduction U/s 80DDB:

1. Assessee eligible for deduction Individual/HUF (being resident in India)

2. Nature of deduction Deduction in respect of medical


treatment.
3. Conditions for availing deduction
The assessee should have actually paid (not merely incurred) towards medical treatment of
disease or ailment specified in R. 11DD.
The medical treatment should be for curing the specified ailment or disease of the assessee
(if the assessee is an individual) or the assessee’s dependants.
If the assessee is an individual, his spouse, children, parents, brothers or sisters dependant
wholly or mainly on him for his support and maintenance are regarded as dependants.
If the assessee is a HUF, any member of the family dependant wholly or mainly on the family
for his support and maintenance is regarded as dependant.

Medical
treatment

In case of In case of HUF,


Individual, shall shall relate to
relate to

Spouse Children Parents Brothers/


Any member
Sisters

The assessee should obtain the prescription for such medical treatment from a neurologist,
an oncologist, a urologist, a hematologist, an immunologist or such other specialist, as may
be prescribed.
The medical practitioner need not be employed in a Govt. hospital. He may be even a
visiting medical practitioner.
4. Quantum of deduction
If the person treated is a super senior citizen, the deduction = (Actual amount paid or Rs.
80000) – Insurance cover – Reimbursement from the employer.
If the person treated is a senior citizen, the deduction = (Actual amount paid or Rs. 60000) –
Insurance cover – Reimbursement from the employer.
If the person treated is a non-senior citizen, the deduction = (Actual amount paid or Rs.
40000) – Insurance cover – Reimbursement from the employer.

MT Educare –CA Inter Income Tax Classes – R.SOUMYANARAYANAN.FCA. GRAD CWA. Page-263
Chapter-10: Chapter-VIA deductions

11. New Section 80EEA - Deduction in respect of interest payable on loan taken for
acquisition of residential house property
Inserted by Finance (No. 2) Act 2019 – W.e.f. AY 20-21
Eligible assessee: An individual who has taken a loan for acquisition of residential
house property from any financial institution. Interest payable on such loan would
qualify for deduction under this section.
Conditions: The conditions to be satisfied for availing this deduction are as follows-
1) the loan has been sanctioned by the financial institution during the period
beginning on the 1st day of April, 2019 and ending on the 31st day of March,
2020;
2) the stamp duty value of residential house property does not exceed R S . 45 lakh;
3) the assessee does not own any residential house property on the date of
sanction of loan.
4) The individual is not eligible to claim deduction under Section 80EE

Period of benefit: The benefit of deduction under this section would be available
from A.Y. 2020-21 and subsequent assessment years till the repayment of loan
continues.

Quantum of deduction: The maximum deduction allowable is Rs .1,50,000.


The deduction of upto R s . 1,50,000 under section 80EEA is over and above the
deduction available under section 24(b) in respect of interest payable on loan borrowed
for acquisition of a residential house property.

In respect of self-occupied house property, interest deduction under section


24(b) is restricted to R s . 2,00,000. In case of let out or deemed to be let out property,
even though there is no limit under section 24(b), section 71(3A) restricts the amount
of loss from house property to be set-off against any other head of income to
Rs. 2,00,000.

Accordingly, if interest payable in respect of acquisition of eligible house property is


more than R s . 2,00,000, the excess can be claimed as deduction under section 80EEA,
subject to fulfilment of conditions.

No deduction under any other provision:


The interest allowed as deduction under section 80EEA will not be allowed as
deduction under any other provision of the Act for the same or any other assessment
year.

MT Educare –CA Inter Income Tax Classes – R.SOUMYANARAYANAN.FCA. GRAD CWA. Page-264
Chapter-10: Chapter-VIA deductions

Meaning of “Financial Institution”


A banking company to which the Banking Regulation Act, 1949 applies; or
Any bank or banking institution referred to in section 51 of the Banking Regulation Act,
1949; or
A housing finance company
A public company formed or registered in India with the main object of carrying on
the business of providing long-term finance for construction or purchase of houses in
India for residential purposes.

12. Deduction in respect of interest payable on loan taken for purchase of


electric vehicle [Section 80EEB] – NEW
Inserted by Finance (No. 2) Act 2019 – W.e.f. AY 20-21
Eligible Assessee: An Individual who has taken a loan for purchase of an electric
vehicle from any financial institution, Interest payable on such loan would qualify for
deduction under this section.
Conditions:
The assessee should be an individual. Loan should be taken for purchase of an electric
vehicle loan has been sanctioned by the financial institution during the period
beginning on the 1st day of April, 2019 and ending on the 31st day of March, 2023.
Period of benefit: The benefit of deduction under this section would be available
from A.Y. 2020-21 and subsequent assessment years till the repayment of loan
continues.
Quantum of deduction: Interest payable, subject to a maximum of R s . 1,50,000.
No deduction under any other provision:
The interest allowed as deduction under section 80EEB will not be allowed as
deduction under any other provision of the Act for the same or any other assessment
year.
Financial institution:
✓ A banking company to which the Banking Regulation Act, 1949 applies; or
✓ Any bank or banking institution referred to in section 51 of the Banking
Regulation Act, 1949; or
✓ Any deposit taking NBFC
✓ A systemically important non-deposit taking NBFC i.e., a NBFC which is not
accepting or holding public deposits and having total assets of not less than
Rs. 500 crore as per the last audited balance sheet and is registered with the
RBI.

Electric Vehicle:
A vehicle which is powered exclusively by an electric motor whose traction energy is
supplied exclusively by traction battery installed in the vehicle. The vehicle should have
electric regenerative braking system, which during braking provides for the conversion
of vehicle kinetic energy into electrical energy.

MT Educare –CA Inter Income Tax Classes – R.SOUMYANARAYANAN.FCA. GRAD CWA. Page-265
Chapter-10: Chapter-VIA deductions

13. Deduction U/s 80G:

(i) Eligible assessee Deduction under this section is available to all assessees, whether
corporate or non-corporate, whether having income U/H PGBP or
not.
(ii) Nature of Under this section, the assessee is eligible for deduction in respect
deduction of monetary donations made by him.
(iii) Eligible Deduction is allowed only with respect to donations made to
donations funds or institutions specified in this section.
(iv) Quantum of deduction
Donations U/s. 80G

Donations - to Other Donations


be restricted to
10% of GTI

Donations Donations Donations Donations


Qualifying for qualifying for Qualifying for qualifying for
50% deduction 100% deduction 50% deduction 100% deduction

See List - 1 See List - 2 See List - 3 See List - 4

List – 1

1. Donations made to any charitable trust/institution fulfilling the conditions of S. 80G (5).
2. Donations made to the Government or any local authority – where donations are to be utilized
by them for any charitable purpose other than for the purpose of promoting family planning:
3. Donations made to an authority constituted in India for the purpose of dealing with the need
for housing accommodation, planning and development of cities, towns etc.,
4. Donations made to any corporation established by the CG/SG, for promoting the interests of
the minority community.
5. Donations made to any temple, mosque, gurudwara, church or other place notified by the
Central Government to be of historic, archaeological or artistic importance or to be a place of
public worship of renown throughout any State or States – Where the donations to be made
for renovation or repair purposes.

List–2
1. Donations made to Government or any such local authority, institution or association as
may be approved by the CG. – where donations are to be utilized for promoting family
planning:
2. Donations made by companies to the Indian Olympic Association or to any other
association or institution for the development of infrastructure for sports and games; or
the sponsorship of sports and games.

MT Educare –CA Inter Income Tax Classes – R.SOUMYANARAYANAN.FCA. GRAD CWA. Page-266
Chapter-10: Chapter-VIA deductions

List – 3

1. Donations made to the Indira Gandhi Memorial Trust


2. Donations made to the Rajiv Gandhi Foundation
3. Donations made to the Jawaharlal Nehru Memorial Fund
4. Donations made to the Prime Minister’s Drought Relief Fund

List – 4

1. Donations made to the National Defence Fund set up by the CG.


2. Donations made to the Prime Minister’s National Relief Fund
3. Donations made to the Prime Minister’s Armenia Earthquake Relief Fund
4. Donations made to the Africa (Public Contributions – India) Fund
5. Donations made to the National Foundation for Communal Harmony
6. Donations made to a University or any educational institution of national eminence
approved by the Prescribed Authority
7. Donations made to the Maharashtra Chief Minister’s Relief Fund / the Chief Minister’s
Earthquake Relief Fund
8. Donations made to any fund set up by the Government of Gujarat for providing relief
to earthquake victims of Gujarat
9. Donations made to any Zila Saksharta Samiti constituted for improvement of primary
education in villages and towns
10. Donations made to the National Blood Transfusion Council or any State Blood
Transfusion Council
11. Donations made to any fund set up by SG to provide medical relief to the poor
12. Donations made to the Army Central Welfare Fund; the Indian Naval Benevolent Fund;
the Air Force Central Welfare Fund
13. Donations made to the Andhra Pradesh Chief Minister’s Cyclone Relief Fund
14. Donations made to the National Illness Assistance Fund
15. Donations made to the Chief Minister’s Relief Fund; the Lt. Governor’s Relief Fund
16. Donations made to the National Sports Fund set up by Central Government
17. Donations made to the National Cultural Fund set up by Central Government
18. Donations made to the Fund for Technology Development and Application
19. Donations made to the National Trust for Welfare of Persons with Autism etc.
20 Donations to Swachh Bharat Kosh, set up by the CG (other than the sum spent by the
assessee in pursuance of CSR U/s 135 (5) of the Companies Act 2013).
21 Donations to National children’s Fund.
22 Donations to Chief minister’s Relief Fund or Lieutenant Governor’s Relief Fund.
23 Donations to Clean Ganga Fund, set up by the CG, where such assessee is a resident
(other than the sum spent by the assessee in pursuance of CSR U/s 135 (5) of the
Companies Act 2013).
24 Donations to National Fund for Control of Drug Abuse constituted U/s 7A of the
Narcotic Drugs and Psychotroic Substances Act, 1985.

MT Educare –CA Inter Income Tax Classes – R.SOUMYANARAYANAN.FCA. GRAD CWA. Page-267
Chapter-10: Chapter-VIA deductions

Computation of GTI for S. 80G:

Gross Total Income before allowing Chapter VI A deductions XXX


Less: Deductions u/s. 80C to 80U XXX
Less: LTCG XXX
Less: Short-term capital gains arising out of transfer of certain equity XXX
shares/units which were subjected to STT (Sec.111A)
Gross Total Income for Sec. 80G(4) XXX

Mode of payment:

No deduction shall be allowed in respect of donation of any sum exceeding Rs. 2000 unless
such sum is paid by any mode other than cash.

No double deduction:

If the assessee has claimed and has been allowed deduction under this section in respect of
any amount of donation, the same amount does not again qualify for deduction under any
other provisions of the Act. For example, where an assessee claims a deduction U/s 80G in
respect of any donation to a scientific research association; he is not entitled to claim
deduction in respect of the same donation U/s 35. It is however, open for him to claim
deduction U/s 35 instead of S. 80G.

Subsequent withdrawal of approval to the institutions not to affect deduction:

Where the donation is made by an assessee to an approved institution and claimed


deduction, the subsequent withdrawal of the approval of such institution would not entitle
the department to reopen the case and disallow the deduction. (Jai Kumar Kankaria Vs CIT
251 ITR 707 (Cal)).

14. Deduction U/s 80GG:

1 Assessee eligible for Individual


deduction
2 Nature of deduction Rent paid by the assessee.
3 Conditions a) The assessee should be a self employed person. But if he is
employed, he should not get house rent allowance from his
employer.
b) He or his spouse or his minor child or the HUF in which he is a
member, should not own residential accommodation at the
place where he resides, performs the duties of his office or
carries on his business or profession.
c) In respect of a house property owned by him at any other place,
he should not avail self-occupied property benefit.
d) He should file a declaration in Form No. 10BA regarding the

MT Educare –CA Inter Income Tax Classes – R.SOUMYANARAYANAN.FCA. GRAD CWA. Page-268
Chapter-10: Chapter-VIA deductions

expenditure incurred by him towards payment of rent.


4 Quantum of deduction Least of the following:
(a) Rent paid in excess of 10% of total Income
(b) Rs. 5000 per month
(c) 25% of Total Income (Refer Note)

Meaning of total income for the purpose of S. 80GG:

Find out gross total income XXX


Less: LTCGs XXX
Less: STCG taxable U/s 111A XXX
Less: Amount deductible U/s 80C to 80U (except S. 80GG) XXX
Total income for the purpose of S. 80GG XXX

15. Deduction U/s 80GGA:

1. Assessee eligible for deduction


Assessee

Having Business Not having Business


Income Income

Not eligible for


deduction Eligible for deduction

2. Nature of deduction Deduction in respect of certain donations for scientific research or


rural development
3. Amount eligible for deduction
(a) (a) Any sum paid to a scientific research association, or to a University, college or other
institution, which have as their object the undertaking of scientific research;

(b) (b) Any sum paid to any approved University, college or other institution to be used for
research in social science or statistical research;

(c) (c) Any sum paid to an association or institution to be used for carrying out any programme
of rural development approved for the purposes of S. 35CCA;

(d) (d) Any sum paid to an association or institution which has the object of training persons for
implementing programmes of rural development;(S. 35CCA)

(e) (e) Any sum paid to a public sector company or a local authority or an association or
institution approved by the National Committee for carrying out any eligible project or
scheme provided that the assessee furnishes the certificate referred to in S.35AC

(f) (f) Any sum paid to National Fund for Rural Development set up and notified by the
Government.
(g) (g) Any sum paid to the notified National Urban Poverty Eradication Fund.

MT Educare –CA Inter Income Tax Classes – R.SOUMYANARAYANAN.FCA. GRAD CWA. Page-269
Chapter-10: Chapter-VIA deductions

Note: For purposes of (a) and (d) above, the assessee should furnish a certificate as
contemplated in S. 35CCA from such association or institution.
4. Eligible project or “Eligible project or scheme” means such project or scheme
scheme notified by the Central Government for promoting the social and
economic welfare of, or the uplift of, the public on the
recommendations of the National Committee.
5. No Deduction in any Where any deduction has been allowed U/s 80GGA for any AY, no
other assessment year deduction shall be allowed in respect of such amount under any
other provisions of this Act for the same or any other AY.

16. Deduction U/s 80JJAA:

1 Eligible assessee Person whose GTI includes profits and gains derived from
business and to whom S. 44AB applies.
2 Nature of deduction Deduction is allowed in respect of additional employee cost
incurred in the course of business in the relevant PY.
3 Quantum of 30% of additional employee cost incurred in the course of
deduction business in the relevant PY.
4 Period of deduction 3 AYs including the AY relevant to the PY in which the
employment is provided by the assessee.
5 Additional employee Total emoluments paid or payable to additional employees
cost employed during the PY.
6 Emoluments Any sum paid or payable to an employee in lieu of his
employment by whatever name called.
Note the words ‘paid or payable’. Accordingly, non-monetary
perquisites shall not be included in emoluments.
Even allowances which are wholly or partly exempted in the
hands of employees shall be taken into account for computing
emoluments.
7 Exclusions from (a) Any contribution paid or payable by the employer to any
emoluments pension fund or provident fund or any other fund for the benefit
of employees under any law for the time being in force
(b) any lump-sum paid or payable to an employee at the time of
termination of his service or superannuation or voluntary
retirement, such as gratuity, severance pay, leave encashment,
voluntary retirement benefits, commutation of pension etc.
8 Additional employee Employee who has been employed during the PY and whose
employment has the effect of increasing the total number of
employees employed by the employer as on the last day of the
preceding year.
9 Exclusions from Employee whose total emoluments are more than Rs. 25000
additional employee p.m.
Employee employed for a period of less than 240 days during the
PY.
Employee who does not participate in recognised provident
fund.

MT Educare –CA Inter Income Tax Classes – R.SOUMYANARAYANAN.FCA. GRAD CWA. Page-270
Chapter-10: Chapter-VIA deductions

10 Additional employee In the first year of new business, emoluments paid or payable to
cost for the first year employees employed during that PY shall be deemed to be the
in case of new additional employee cost.
business
11 Circumstances when In case of an existing business, the additional employee cost
additional employee shall be nil, if
cost in case of an (a) there is no increase in the number of employees from the
existing business is nil. total number of employees employed as on the last day of the
preceding PY.
(b) emoluments are paid otherwise than by an account payee
cheque or account payee bank draft or by use of electronic
clearing system through a bank account.
12 Report of CA For claiming deduction U/s 80JJAA, the assessee shall furnish,
along with ROI, a report of CA in form 10DA containing
prescribed particulars.

Points requiring attention:

1 The assessee may be an Individual or HUF or company or firm or any other person.
2 This section applies even to non-residents carrying on business in India.
3 If the GTI of the assessee includes profits and gains from profession and not business, S.
80JJAA shall not apply.
4 Business should not have been formed by splitting up or reconstruction of an existing
business.
5 Business should not have been acquired by way of transfer from any other person or as a
result of any business reorganisation
6 In case of an employee engaged in the business of manufacturing of apparel (which is
seasonal in nature), any employee employed for a period of less than 240 days but for 150
days or more in the PY shall, still, be considered as ‘additional employee’.
7 Deduction U/s 80JJAA is in addition to the deduction U/s 37 (1) in respect of emoluments.
8 In the first-year emoluments can be paid in cash. (i.e. other than through banking channels).

Question Mr. A commenced the business of manufacture of computers on 01.04.2019. He


employed 350 new employees during the PY 2018-19, the details of whom are as follows:
SN Number of Date of Regular / casual Total monthly emoluments
employees employment per employee
1 75 01.04.2019 Regular Rs. 24000
2 125 01.05.2019 Regular Rs. 26000
3 50 01.08.2019 Casual / Rs. 17000
Contractual
4 100 01.09.2019 Regular Rs. 24000

The regular employees participate in recognised provident fund while the casual employees
don’t. Further, out of 75, 125 and 100 regular employees employed on 01.04.2019,

MT Educare –CA Inter Income Tax Classes – R.SOUMYANARAYANAN.FCA. GRAD CWA. Page-271
Chapter-10: Chapter-VIA deductions

01.05.2019 and 01.09.2019, only 40, 30 and 60 qualify as a ‘workman’ under the Industrial
disputes Act, 1947.
Compute the deduction, if any, available to Mr. A for AY 2019-20, if the profits and gains
derived from manufacture of computers that year is Rs. 75L and his total turnover is Rs. 2.16
Crores.
Solution:

1 Total turnover from business for the PY 2018-19 > Rs. 1 Crore.
2 Therefore, the assessee is liable to tax audit U/s 44AB.
3 He has employed ‘additional employees’ during the PY 2019-20.
4 Therefore, he is eligible for deduction U/s 80JJAA.

Number additional employees:

1 Total number of employees employed during the PY 350


2 Less: Casual employees employed (50)
3 Less: Regular employees employed as on 01.05.2019, since their total monthly (125)
emoluments > Rs. 25000
4 Less: Regular employees employed on 01.09.2019 (since they have been (100)
employed for less than 240 days in the PY 2019-20
5 Number of additional employees 75

Deduction U/s 80JJAA:

1 Additional employee cost Rs. 24000 *12*75 = Rs. 216L


2 Deduction U/s 80JJAA 30% of Rs. 216L = Rs. 64.80L

Note: It is not necessary that the employee should qualify as a workman under the
Industrial Disputes Act, 1947 for the employer to avail the benefit U/s 80JJAA.

Question: X & Co is an LLP which commenced its business in the PY 2019-20. It owns and
operates retail outlets in different parts of North India.

During the PY 2019-20, it appoints the following persons:

Date of appointment Number of employees Designation Monthly salary per person


01.05.2019 8 Store keeper Rs. 18000
01.06.2019 12 Sales person Rs. 25000
01.07.2019 4 Supervisor Rs. 28000
01.10.2019 25 Helpers Rs. 11000

Salary to storekeepers is paid in cash up to 31.12.2019. In other cases, salary is transferred


by ECS through SBI, Noida.

Determine the amount of deduction available U/s 80JJAA for the AY 2020-21, assuming the
turnover of X & Co for the PY 2019-20 is Rs. 6 Crores.

MT Educare –CA Inter Income Tax Classes – R.SOUMYANARAYANAN.FCA. GRAD CWA. Page-272
Chapter-10: Chapter-VIA deductions

Solution:

SN Particulars Salary (deductible Deduction U/s 80JJAA


U/s 37 (1))
1 Storekeepers appointed on Rs.1584000 30% of Rs.1584000 = Rs.
01.05.2019 (Rs. 18000 * 8 * 11 475200 (See note-1)
months)
2 Sales person appointed on Rs. 3000000 Rs. 900000
01.06.2019 (Rs. 25000 *12* 10
months)
3 Supervisors appointed on Rs. 1008000 - (See note-2)
01.07.2019 (Rs. 28000 * 4 * 9
months)
4 Helpers appointed on 01.10.2019 Rs. 1650000 - (See note-3)
(Rs. 11000 * 25 * 6 months)
5 Total (1+2+3+4) Rs. 7242000 Rs. 1375200

Note:
1 If salary is not paid by an account payee cheque or account payee bank draft or by use of
electronic clearing system through a bank account, deduction U/s 80JJAA is not
available. However, this rule is applicable in case of an existing business and not in the
first year of a new business.
2 Deduction U/s 80JJAA is not available in respect of employees whose total emoluments
are more than Rs. 25000 p.m.
3 Helpers are employed for less than 240 days during the PY 2019-20. Hence, no deduction
is available U/s 80JJAA.

Question Suppose in previous Question, consider the following information pertaining to PY


2020-21.
Determine the amount of deduction U/s 80JJAA for the AY 2020-21, assuming the turnover
for the PY 2019-20 is Rs. 5 Crores.
Particulars Situation-1 Situation-2
Number of employees as on 31.03.2020 49 49
Sales persons resigned during April 2020 (6) -
Number of employees as on 30.04.2020 43 49
8 Deputy managers appointed on 01.05.2020 (Salary Rs. 24000 per 8 8
month; Increased to Rs. 28000 p.m w.e.f 01.02.2021)
Number of employees as on 31.05.2020 51 57
General managers appointed on 01.08.2020 (on part-time basis) 1 1
[Salary: Rs. 20000 p.m]
Number of employees as on 52 58

Deduction U/s 80JJAA:

SN Particulars Situation-1 Situation-2


1 Number of employees as on 31.03.2020 49 49

MT Educare –CA Inter Income Tax Classes – R.SOUMYANARAYANAN.FCA. GRAD CWA. Page-273
Chapter-10: Chapter-VIA deductions

2 Number of employees immediately after 51 57


appointment of 8 deputy managers
3 Increase in total number of employees as 2 8
compared to number of employees as on last
day of preceding PY
4 Number of deputy managers qualified for 2 8
deduction U/s 80JJAA
5 Salary payable to the deputy managers from Rs. 24000 *9 Rs. 24000 * 9
01.05.2020 to 01.02.2021 (from 01.02.2021, months * 2 = Rs. months * 8 = Rs.
salary is more than Rs. 25000, deduction U/s 432000 1728000
80JJAA is not available)
6 Salary to part-time general manager (Rs. 20000 * Rs. 160000 Rs. 160000
8 months)
7 Total Rs. 592000 Rs. 1888000
8 Deduction U/s 80JJAA (30% of total) Rs. 177600 Rs. 566400

17. Deduction in respect of royalty income of the authors-S. 80QQB:

1. Assessee eligible for Resident individual being an author.


deduction
2. Nature of deduction Under this section, assessee is eligible for deduction in respect of any lump
sum consideration received for the assignment or grant of any of his interests
in the copyright of any book being a work of literary, artistic or scientific
nature or royalty or copyright fees received (in lump sum or otherwise).
3. Items not regarded Books shall not include brochure, commentaries, diaries, guides, journals,
as books magazines, newspapers, pamphlets, textbooks for schools and other
publication of similar nature.
4. Quantum of Whole of such income or Rs. 300000 (whichever is less)
deduction
5. Restriction on royalty If royalty or copyright fee is not received in lump sum – amount in excess of
or copyright fees not 15% of value of such books sold during the previous year shall be ignored.
received in lump sum
6. Conditions for availing deduction
a) Any income earned from any source outside India shall be remitted within six months from the end
of the relevant previous year or such extended time by RBI.
b) A certificate in the prescribed Form 10CCD given by the person making payment shall be furnished
along with the Return of Income.
c) Where income is earned outside India, a certificate from the prescribed authority in the prescribed
form (Form No. 10H) shall be furnished along with return of income.
7. Double deduction not available
Where a deduction U/s 80QQB for any previous year has been claimed and allowed, no deduction in
respect of such income shall be allowed under any other provisions of the Act in any assessment year.
18. Deduction in respect of royalty on patents - S. 80RRB:

1. Assessee Resident individual being a patentee


eligible for

MT Educare –CA Inter Income Tax Classes – R.SOUMYANARAYANAN.FCA. GRAD CWA. Page-274
Chapter-10: Chapter-VIA deductions

deduction
2. Meaning of Patentee means the person (being the true and first inventor of the
patentee invention), whose name is entered on the patent register as the
patentee, in accordance with the patents Act, 1970.
3. Nature of Under this section, assessee is eligible for deduction in respect of any
deduction income earned by way of royalty in respect of a patent registered on or
after 1.4.2003 under the patents Act, 1970.
4. Quantum of Whole of such income or Rs. 300000 (whichever is less)
deduction
5. Conditions for availing deduction
a) Any income earned from any source outside India shall be remitted within six months
from the end of the relevant previous year or such extended time by RBI.

b) The assessee shall furnish a certificate in the prescribed Form 10CCE from the person
responsible for making such payment along with return of income.

c) For income earned outside India, a certificate from the prescribed authority in the
prescribed form (Form No. 10H) shall be filed along with return of income.
6. Double deduction not available
Where a deduction U/s 80RRB for any previous year has been claimed and allowed, no
deduction in respect of such income shall be allowed under any other provisions of the Act
in any AY.

17. Deduction in respect of interest on deposits in savings account – S. 80TTA:

1 Where the GTI of an assessee, being an individual or a HUF, includes any income by way
of interest on deposits (not being time deposits) in a savings account with-
(a) a banking company to which the Banking Regulation Act, 1949;
(b) a co-operative society engaged in carrying on the business of banking (including a co-
operative land mortgage bank or a co-operative land development bank); or
(c) a Post Office;
there shall be allowed, in computing the TI of the assessee a deduction as specified
hereunder, namely:—
(i) in a case where the amount of such income does not the whole of such
exceed in the aggregate Rs. 10000 amount;
(ii) in any other case Rs. 10000
2 For the purposes of this section, "time deposits" means the deposits repayable on expiry
of fixed periods.
Note: Post office savings bank interest is exempt up to Rs. 3,500 (in an individual account)
and Rs. 7,000 (in a joint account) U/s 10 ( 15) (i).

18. Deduction in respect of interest on deposits in savings account – S. 80TTB:

Eligible Assessee Resident senior citizen (i.e. an individual of the age of 60 years or
more at any time during the previous year)

MT Educare –CA Inter Income Tax Classes – R.SOUMYANARAYANAN.FCA. GRAD CWA. Page-275
Chapter-10: Chapter-VIA deductions

Eligible Income Interest on deposits with banking company, co-operative society


engaged in the business of banking or a post office.
Deduction Actual interest or Rs. 50,000, whichever is less.

MT Educare –CA Inter Income Tax Classes – R.SOUMYANARAYANAN.FCA. GRAD CWA. Page-276
Chapter-11: PGBP

A. Charging section-S. 28:

The following income shall be chargeable to tax U/H PGBP:

S. 28 (i) Profits and gains arising from business or profession which was carried on by
the assessee at any time during the PY.
S. 28 (ii) (a) Compensation to MD/WTD/manager of Indian company for loss of office or
modification of terms and conditions of appointment to their detriment.
S. 28 (ii) (b) Compensation to persons managing the affairs in India of any other company
for loss of office or modification of terms and conditions of appointment to
their detriment.
S. 28 (ii) (c) Compensation for termination of agency in India or for modification of terms
and conditions of agency to the detriment of the person holding agency.
S. 28 (ii) (d) Compensation for vesting in the Government or in any corporation owned or
controlled by the Government, of the management of any business under
any law for the time being in force.
S. 28 (iii) Income derived by a trade, professional or similar association from specific
services performed for its members.
S. 28 (iiia) to Export incentives.
(iiiae)
S. 28 (iv) Value of any benefit or perquisite, whether convertible into money or not,
arising from business or exercise of profession.
S. 28 (v) Any interest, salary, bonus, commission or remuneration, by whatever name
called, due to, or received by, a partner of a firm from such firm.
Proviso to S. However, interest and remuneration shall be taxed in the hands of the
28 (v) partners only to the extent to which it was allowed as deduction in the hands
of the firm.
S. 28 (va) Non-compete fee
Fee for exclusivity of rights
S. 28 (vi) Key man insurance policy proceeds.
s. 28 (vii) Discussed along with S. 35AD.

Points requiring attention:

1 Meaning of business. [S. 2 It means any trade, commerce or manufacture.


(13)]. It includes any adventure or concern in the nature of trade,
commerce or manufacture.

2 Meaning of profession. [S. It means an occupation in which earning of income is driven by


2 (36)]. skills, talents, knowledge and experience.
Profession includes vocation (i.e. any activity which is carried on for
livelihood).
A painter, a sculptor, an author, an auditor, a lawyer, a doctor, an
architect and, even an astrologer are persons who can be said to be
carrying on a profession but not business.
However, it is not material whether a person is carrying on a

MT Educare –CA Inter Income Tax Classes – R.SOUMYANARAYANAN.FCA. GRAD CWA. Page-277
Chapter-11: PGBP

`business` or `profession` or `vocation` since for purposes of


assessment, profits from all these sources are treated and taxed
alike.
3 Significance of ‘carried on’ For invoking the charging section, carrying on of business is enough
in S. 28 (i). though the assessee does not own the business.
4 Significance of ‘by the Promoters of a company started a business which shall be
assessee’ in S. 28 (i). continued by the company upon incorporation. The profits from
business attributable to the pre-incorporation period cannot be
taxed in the hands of the company but it shall be taxed in the hands
of promoters as an AOP.
This is because the company, since it is a creation of law, comes
into existence only after incorporation. In the pre-incorporation
period the business was not carried on by it. Nor we can that the
promoters of the company carried on the business as agents, on
behalf of the company. There cannot be an agent for unborn
principal.
Therefore, it shall be taxed only in the hands of promoters who
carried on the business, as an AOP.
5 Significance of ‘at any time To invoke the head PGBP, the business or profession should have
during the PY’ in S. 28 (i). been carried on at any time during the PY.
If the business or profession got discontinued even before the
commencement of the PY, the PGBP head cannot be invoked to tax
any income. [Subject to S. 41].
However, the difference between discontinuance of business and
suspension of business shall be appreciated.
There is no hurdle in invoking the head PGBP if the business is
suspended. This is because if the business is suspended it means
that the business still exists but in dormant mode.
4 Receipt of sum attributable Where the business or profession carried on by the assessee gets
to discontinued business discontinued, any sum received after the discontinuance shall be
or profession [S. 176 (3A) / deemed to be income of the recipient and charged to tax
S. 176 (4)]. accordingly, in the PY of receipt, if such sum would have been
included in the total income of the person who carried on business
or profession had it been received before such discontinuance.
However, the sum received shall not be taxed U/H PGBP but U/H
IFOS.
5 Speculative business – Where speculative transactions carried on by the assessee are of
distinct and separate. such a nature as to constitute business, such business shall be
[Explanation-2 to S. 28]. deemed to be distinct and separate from any other business.
This is to give effect to the provisions of S. 73.
6 Significance of ‘whether S. 28 (iv) captures only non-monetary benefit arising from business
convertible into money or or profession. Note the words ‘whether convertible into money or
not’ in S. 28 (iv). not’.
7 Method of accounting & Income U/H PGBP shall be computed in accordance with the
ICDS. [S. 145 (1) & S. 145 method of accounting regularly employed by the assessee unless
(2)]. the Act provides otherwise.
Also the relevant ICDS notified by the CG shall be considered in

MT Educare –CA Inter Income Tax Classes – R.SOUMYANARAYANAN.FCA. GRAD CWA. Page-278
Chapter-11: PGBP

computing income U/H PGBP (if the assessee follows mercantile


system of accounting).
8 Deemed profits chargeable If the assessee had claimed deduction in respect of any expenditure
to tax – S. 41 (1) (a). or loss in the earlier PY U/H PGBP and the assessee has obtained
any sum (in money or in kind) in respect of such loss or expenditure
in the PY under assessment, it shall be deemed to be profits of
business and shall be charged to tax in the hands of the assessee
U/H PGBP in the PY of receipt. [S. 41 (1) (a)].
If the assessee had claimed deduction in respect of any trading
liability in the earlier PY U/H PGBP and some benefit accrues to the
assessee in the PY of assessment on account of remission or
cessation of such trading liability, it shall be deemed to be profits of
business and shall be charged to tax in the hands of the assessee
U/H PGBP in the PY in which such benefit accrues. [S. 41 (1) (a)].
9 Catching the successor U/s Where there is succession in business, the provisions of S. 41 (1)
41 in case of succession in can be invoked against the successor to tax the sum obtained in
business. [S. 41 (1) (b)]. respect of expenses claimed as deduction by the predecessor or to
tax the benefit accrued on account of remission or cessation of
trading liability claimed as deduction by the predecessor.
10 Should business exist to No. Note the words ‘whether the business or profession…….is in
invoke S. 41 (1) (a)? existence in that year or not….’ in S. 41 (1) (a).

B. Format for computing income U/H PGBP:

SN Particulars Amount Amount


1 Net profits as per the P & L statement *****
2 Positive adjustments (list illustrative)
(a) Expenses attributable to income chargeable under other heads (debited ****
to the P&L account)
(b) Expenses attributable to exempt income (debited to the P&L account) ****
(c) Expenses to be disallowed U/s 40, S. 40A, S. 37 (2B) and S. 43B (debited ****
to the P&L account)
(d) Depreciation as per books (debited to the P&L account) ****
(e) Prior period expenses (debited to the P&L account) **** ****
3 Negative adjustments (list illustrative)
(a) Income chargeable under other heads (credited to the P&L account) ****
(b) Exempted income (credited to the P&L account) ****
(c) Additional deduction on account of weighted deduction ****
(d) Capital expenditure eligible for deduction (not debited to the P&L ****
account)
(e) Depreciation as per S. 32 (including additional depreciation, if applicable) ****
(f) Additional investment allowance (in applicable) ****
(g) Prior period income (Credited to the P&L account) **** ****
4 Income U/H PGBP (1+2-3) ****

C. Scheme of business deduction:

MT Educare –CA Inter Income Tax Classes – R.SOUMYANARAYANAN.FCA. GRAD CWA. Page-279
Chapter-11: PGBP

1 Deduction in respect of S. 32 (Discussed in detail in capital gains chapter).


depreciation allowance
(including additional
depreciation)
2 Deduction in respect of S. 32AD (Discussed in detail in capital gains chapter).
additional investment
allowance.
3 Deductions in respect of S. 30, S. 31, S. 35, S. 35AD, S. 35CCA, S. 35CCC, S. 35CCD,
business expenses. (to the S. 35D, S. 35DDA, S. 36 (1) (i), S. 36 (1) (ia), S. 36 (1) (ib), S.
extent covered in Inter 36 (1) (ii), S. 36 (1) (iii), S. 36 (1) (iiia), S. 36 (1) (iv), S. 36
Syllabus) (1) (iva), S. 36 (1) (v), S. 36 (1) (va), S. 36 (1) (vi), S. 36 (1)
(vii) & S. 36 (2), S. 36 (1) (ix), S. 36 (i) (xv), S. 36 (1) (xvi)
and S. 37.

1. Deduction U/s 30:

Following expenses which are incurred in relation to a building which is used for the
purpose of business or profession carried on by the assessee shall be allowed as deduction:

1 Rent paid for the premises used for the purpose of business or profession
2 Current repairs to the premises (not being capital expenditure) – [in case of person
occupying the building as owner.]
3 Repairs to the premises (not being capital expenditure) – [in case of person occupying
the building as tenant].
3 Land revenue/municipal taxes paid. [Subject to S. 43B].
4 Insurance premium paid in respect of insurance against risk of destruction of the
premises.

Note:

1 If the land revenue or municipal taxes are paid within the due date for filing ROI for the
PY under assessment, it shall be allowed as deduction while computing income U/H
PGBP. However, if it is paid after the due date (supra), it shall be allowed as deduction
only in the PY of payment.
2 If the expenses referred to above are incurred in relation to a building which is partly
used for business or profession and partly for other purposes, then that portion of
expenses which is attributable to the use for business or profession shall be allowed as
deduction. [S. 38 (1)].

MT Educare –CA Inter Income Tax Classes – R.SOUMYANARAYANAN.FCA. GRAD CWA. Page-280
Chapter-11: PGBP

2. Deduction U/s 31:

Following expenses which are incurred in relation to plant, machinery or furniture, which is
used for the purpose of business or profession carried on by the assessee shall be allowed
as deduction:

1 Current repairs in respect of plant, machinery and furniture (not being capital
expenditure)
2 Insurance premium paid in respect of insurance against risk of destruction of the plant,
machinery and furniture.

Note:

1 Hire charges paid in respect of plant, machinery or furniture are eligible for deduction
U/s 37 (1).
2 If the expenses referred to above are incurred in relation to plant, machinery or
furniture, which is partly used for business or profession and partly for other purposes,
then that portion of expenses which is attributable to the use for business or profession
shall be allowed as deduction. [S. 38 (1)].

3. Deduction U/s 35:

1 What is allowed as S. 35 allows deduction in respect of expenditure on


deduction U/s 35? scientific research related to the business carried on by
the assessee and also in respect of expenditure by way
of donations or contributions made by the assessee to
outside agencies carrying on scientific research.
2 Meaning of scientific It means activities for the extension of knowledge in the
research [S. 43 (4) (i)]. fields of natural or applied science including agriculture,
animal husbandry or fisheries.
3 Meaning of scientific Expenditure incurred on scientific research would
research expenditure [S. 43 include all expenditure incurred for the prosecution or
(4) (ii)]. the provision of facilities for the prosecution of scientific
research but does not include any expenditure incurred
in the acquisition of rights in or arising out of scientific
research.

4 When can we say that Scientific research related to a business would include
scientific research is related
to the business carried on (i) any scientific research which may lead to or
by the assessee? facilitate an extension of that business;
(ii) any scientific research of a medical nature which
has a special relation to the welfare of the
workers employed in that business.
5 Classification of expenditure (a) Revenue expenditure; (b) Capital expenditure.
on scientific research.
6 Classification of revenue Revenue expenditure on scientific research can be

MT Educare –CA Inter Income Tax Classes – R.SOUMYANARAYANAN.FCA. GRAD CWA. Page-281
Chapter-11: PGBP

expenditure on scientific classified into (a) post-commencement expenditure; (b)


research. pre-commencement expenditure.
7 Deductibility of post- Deductible in the PY in which it is incurred.
commencement revenue
expenditure on scientific
research. [S. 35 (1) (i)].
8 Deductibility of pre- Revenue expenditure on scientific research being (a) cost
commencement revenue of material used in scientific research; and (b) salary to
expenditure on scientific research personnel incurred during a period of 3 years
research. [Explanation to S. immediately preceding the date of commencement of
35 (1) (i)]. business, to the extent certified by the prescribed
authority, is allowed as a deduction in the PY in business
commences.
Expenditure incurred in providing perquisites to the
research personnel shall not qualify for deduction.
9 Prescribed authority DGIT (Exemptions) and Secretary, Department of
scientific research.
10 Classification of capital Capital expenditure on scientific research can be
expenditure on scientific classified into (a) post-commencement expenditure; (b)
research. pre-commencement expenditure.
11 Deductibility of post- Post commencement capital expenditure on scientific
commencement capital research (not being one on land) shall be allowed as
expenditure on scientific deduction in the PY in which it is incurred.
research. [S. 35 (1) (iv) + s. However, no depreciation U/s 32 shall be allowed on
35 (2)]. assets resulting from such expenditure.
12 Deductibility of pre- Capital expenditure on scientific research (not being one
commencement capital on land) incurred during a period of 3 years immediately
expenditure on scientific preceding the date of commencement of business is
research. allowed as deduction in the PY in which the business
commences.
However, the capital expenditure incurred before the
aforesaid 3 years is not eligible for deduction U/s 35.
But it is eligible for depreciation U/s 32.
13 Expenditure on construction Approach road to research lab is a necessary adjunct to
of approach road to it. Therefore, expenditure on construction of approach
research lab – deductible? road shall be treated on par with construction of
research lab and therefore, is eligible for deduction U/s
35 (1) (iv).

MT Educare –CA Inter Income Tax Classes – R.SOUMYANARAYANAN.FCA. GRAD CWA. Page-282
Chapter-11: PGBP

14 Buses purchased for providing Though it is not an expenditure for prosecution of scientific
pick-up and drop facility to the research, it is an expenditure incurred for provision of
research personnel – is it eligible facilities for provision of facilities for the prosecution of
for deduction? scientific research. Therefore it is eligible for deduction.
15 Whether expenditure on on-going Requirements of S. 35 are materially different from the
construction of research lab is requirements of S. 32.
eligible for deduction in the PY in To enjoy deduction in respect of depreciation allowance
which it is incurred or in the PY in U/s 32, the assessee should own the asset and put it to
which the construction of lab is business use.
completed? However, S. 35 allows deduction in respect of capital
expenditure on scientific research, the moment it is
incurred.
There is no further requirement that the expenditure
should have brought into existence an asset which is
complete in all respects.
Moreover, there is no requirement that the asset should be
used for research purpose for availing deduction U/s 35 in
the PY in which the expenditure is incurred.
Deduction is allowed U/s 35 even in respect of expenditure
on on-going construction of scientific research lab.
The expenditure incurred in respective years shall be
allowed as deduction in the respective years.
16 Tax treatment of unabsorbed It shall be treated on par with unabsorbed depreciation
capital expenditure on scientific which is governed by the provisions of S. 32 (2).
research. [S. 35 (4)].
17 Carry forward of unabsorbed The unabsorbed capital expenditure on scientific research
capital expenditure not to be of amalgamating company shall be allowed to be carried
affected on account of forward by the amalgamated company, being an Indian
amalgamation. [S. 35 (5)]. company.
18 Transfer of scientific research Asset acquired for scientific research is no longer required
asset without using it for any for research purpose. Therefore, it is transferred without
other purpose – tax implications. bringing it to business proper.
In such a case, the original cost of the asset or the sale
price of the asset, whichever is less, shall be taxed U/H
PGBP in the previous of transfer. [S. 41 (3)].
Capital gains shall be computed only when the asset is sold
for a price more than its original cost.
19 Bring the scientific research asset Asset acquired for scientific research is no longer required
to business proper after its use for research purpose. Therefore, it is brought to business
for research purpose ceases – tax proper.
implications. In such a case, it shall get added to the block of assets.
But the amount to be added to the block shall be nil. [S. 43
(1) Explanation-1].
Thereafter, if the scientific research asset gets transferred
out of the block, in the PY of transfer, STCG shall be
computed within the parameters of S. 50.

MT Educare –CA Inter Income Tax Classes – R.SOUMYANARAYANAN.FCA. GRAD CWA. Page-283
Chapter-11: PGBP

2 Weighted deduction U/s 35 (2AB) A company engaged in the business of bio-technology or in


in respect of expenditure on in- the business of manufacture or production of any article or
house research and development thing, not being an article or thing specified in the list of
facility. the Eleventh Schedule is eligible for deduction U/s 35
(2AB).
Beer, wine, alcoholic spirit for human consumption,
tobacco and its preparations, toilet cosmetics, tooth paste,
projector, photographic equipments etc are covered in the
Eleventh schedule.
S. 35 (2AB) allows weighted deduction of 150% on the
expenditure on in-house research and development facility
as approved by the prescribed authority.
However, the expenditure on land and building is not
eligible for deduction U/s 35 (2AB). But expenditure on
building is eligible for 100% deduction in S. 35 (1) (iv).
[Tube Investments of India Ltd (Mad)].
In respect of the expenditure for which deduction is
allowed U/s 35 (2AB), no deduction shall be allowed under
any other provisions of this Act for any AY.
Deduction U/s 35 (2AB) shall be allowed only if the
company has entered into an agreement with the
prescribed authority for co-operation in such research and
development facility and fulfills such conditions with regard
to the maintenance of accounts and audit thereof and
furnishing of reports in such manner as may be prescribed.
The prescribed authority shall submit its report in relation
to the approval of the said facility to the PCCIT or CCIT or
PDGIT or DGIT in such form and within such time as may be
prescribed.
21 Deduction in respect to Any sum contributed to scientific research association
contribution to approved approved by the CG is eligible for a weighted deduction of
scientific research association etc. 150%.
[S. 35 (1) (ii)]. Any sum contributed to a university, college or other
institution approved by the CG to be used for scientific
research is eligible for a weighted deduction of 150%.
The scientific research carried on by these entities need not
relate to the business carried on by the assessee.
Subsequent withdrawal of approval by the CG will not
affect the deduction available to the assessee.
For the amount contributed, if deduction was enjoyed U/s
35 (1) (ii), then for the same sum deduction is not available
under any other provisions of the Act.
For an assessee not carrying on business or profession, in
respect of the sums (supra) normal deduction is available
U/s 80GGA against the GTI.
However, if the sum contributed exceeds Rs. 10000, it
should not be made in cash. [S. 80GGA (2A)].

MT Educare –CA Inter Income Tax Classes – R.SOUMYANARAYANAN.FCA. GRAD CWA. Page-284
Chapter-11: PGBP

22 Deduction in respect of payment Any sum paid to a company to be used by it for scientific
to a company for scientific research is eligible for deduction U/s 35 (1) (iia).
research – S. 35 (1) (iia). However, such deduction would be available only if (a) the
company is registered in India; and (b) has as its main
object the scientific research and development.
Further, it should be approved by the prescribed authority
and should fulfill the other prescribed conditions.
The payee-company will not be entitled to claim weighted
deduction of 150% U/s 35 (2AB). However, it can continue
to claim deduction U/s 35 (1) (i) or (iv).
23 Deduction in respect of Any sum contributed to a research association approved by
contribution to entities carrying the CG, the object of which is to carry on social science
on social science research or research or statistical research, is eligible for deduction U/s
statistical research – S. 35 (1) (iii). 35 (1) (iii).
Any sum contributed to a university, college or other
institution approved by the CG to be used for social science
research or statistical research is eligible for deduction U/s
35 (1) (iii).
Subsequent withdrawal of approval by the CG will not
affect the deduction available to the assessee.
For the amount contributed, if deduction was enjoyed U/s
35 (1) (iii), then for the same sum deduction is not available
under any other provisions of the Act.
For an assessee not carrying on business or profession, in
respect of the sums (supra) normal deduction is available
U/s 80GGA against the GTI.
However, if the sum contributed exceeds Rs. 10000, it
should not be made in cash. [S. 80GGA (2A)].
24 Weighted deduction U/s 35 (2AA) Any sum paid by an assessee to a National Laboratory or
University or IIT or a specified person (approved by the
prescribed authority) for carrying out programmes of
scientific research approved by the prescribed authority
will be eligible for a weighted deduction of 150% of the
amount so paid.
The contribution for which weighted deduction was
enjoyed U/s 35 (2AA) is not eligible for deduction under
any other provisions of the Act.
Subsequent withdrawal of approval granted to the donor-
entities will not affect the deduction available to the
donor-assessee.
The prescribed authority before granting approval has to
satisfy itself about the feasibility of carrying out the
scientific research and shall submit its report in the
prescribed form to the jurisdictional PCCIT or CCIT or PDGIT
or DGIT.

MT Educare –CA Inter Income Tax Classes – R.SOUMYANARAYANAN.FCA. GRAD CWA. Page-285
Chapter-11: PGBP

Summary of deductions U/s 35:

Section Expenditure incurred/Contribution made to Deduction


(as a % of
contributio
n made)
35(1)(i) Revenue expenditure incurred on scientific research related to the 100%
assessee`s business
35(1)(ii) Research Association/ university/ collage/ other institution for scientific 150%
research
35(1 )(iia) Company for scientific research 100%
35(1 )(iii) Research association/ university/ collage/ other institution for research 100%
in social science or statistical research
35(1)(iv) Capital Expenditure (Other than expenditure on land) 100%
National Laboratory / University / IIT for scientific research undertaken
35(2AA) 150%
under a approved programme
35(2AB) By a company engaged in Bio-technology or any business of production 150%
or manufacturing of article or thing, not being listed in Eleventh
Schedule (other than cost of land & building)

Pictorial representation of deductions U/s 35:

MT Educare –CA Inter Income Tax Classes – R.SOUMYANARAYANAN.FCA. GRAD CWA. Page-286
Chapter-11: PGBP

4. Deduction U/s 35AD:

1 What is allowed as deduction S. 35AD allows deduction in respect of capital


U/s 35AD? expenditure incurred wholly and exclusively for the
purpose of businesses specified therein. Deduction is
allowed in the PY in which expenditure was incurred.
[S. 35AD (1)].
2 Non-eligible capital The following capital expenditures are not eligible for
expenditure. [S. 35AD (8) (f)]. deduction U/s 35AD:
(a) Expenditure incurred on acquisition of land.
(b) Expenditure incurred on acquisition of goodwill.
(c) Expenditure incurred on acquisition of financial
instrument.
(d) Expenditure in respect of which the payment or
aggregate payments made to a person in a day,
otherwise than by an APC or APBD or use of ECS
through a bank account, exceeds Rs. 10000.
3 Deductibility of pre- Any expenditure incurred wholly and exclusively for
commencement capital the purpose of business specified in S. 35AD before its
expenditure. [Proviso to S. commencement shall be allowed as deduction in the
35AD (1)] PY in which the business commences provided such
expenditure is capitalised in the books of accounts as
on the date of commencement of business.
4 Businesses specified in S. 35AD. 1. Setting up and operating cold-chain facility.
[S. 35AD (8) (c)] 2. Setting up and operating ware-house facility for
storage of agricultural produce.
3. Laying and operating cross-country pipe-line
network for storage & transportation of natural gas.
4. Laying and operating cross-country pipe-line
network for storage and transportation of petroleum
and crude oil.
5. Building and operating a hotel anywhere in India
(which is 2 star category or above).
6. Building and operating a hospital anywhere in India
(which has atleast 100 beds for patients).
7. Developing and building a housing project under a
scheme (framed by the CG or SG and notified by the
CBDT) for re-development or rehabilitation of slums.
8. Developing and building a housing project under a
scheme (framed by the CG or SG and notified by the
CBDT) for affordable housing.
9. Production of fertilizer in India in a new plant or in
an existing plant with newly installed capacity.
10. Setting up and operating inland container depot or
container freight station (approved and notified under
the customs Act).

MT Educare –CA Inter Income Tax Classes – R.SOUMYANARAYANAN.FCA. GRAD CWA. Page-287
Chapter-11: PGBP

11. Bee-keeping and production of honey and


beeswax.
12. Setting up and operating a warehousing facility for
storage of sugar.
13. Laying and operating a slurry pipeline for the
transportation of iron ore.
14. Setting up and operating a semiconductor wafer
fabrication manufacturing unit, if such unit is notified
by the Board in accordance with the prescribed
guidelines.
15. Developing or maintaining and operating or
developing, maintaining and operating a new
infrastructure facility.
5 Meaning of cold-chain facility A chain of facilities for storage or transportation of
agricultural and forest produce, meat and meat
products, poultry, marine and dairy products, products
of horticulture, floriculture and apiculture and
processed food items under scientifically controlled
conditions including refrigeration and other facilities
necessary for the preservation of such produce.
6 Meaning of inland container Business units in hinterland (i.e. an area far away from
depot or container freight ports) have difficulty in transporting goods to the
station. ports and waiting in queue to get customs formalities
complied with.
In order to avoid this traffic and congestion ICD or CFS
are set up across the country in the hinterland with
public authority status.
If these ICD or CFS are approved and notified under
the customs Act, they become customs stations where
formalities could be complied with and the goods
could be cleared thereafter.
7 Meaning of slurry pipeline. It is used in mining to transport mineral concentrates
from a mineral processing plant near a mine.
The concentrate of the ore is mixed with water and
then pumped over a long distance to a port where it
can be shipped for further processing.
At the end of the pipeline, the mineral is separated
from the slurry in a filter press to remove the water.
8 Meaning of semi-conductor It is the process used to create integrated circuits that
wafer fabrication. are present in every day electrical and electronic
devices.
9 Meaning of new infra-structure (i) A road including toll road, a bridge or a rail system.
facility (ii) A highway project including housing or other
activities being an integral part of the highway project.
(iii) A water supply project, water treatment system,
irrigation project, sanitation and sewerage system or
solid waste management system.

MT Educare –CA Inter Income Tax Classes – R.SOUMYANARAYANAN.FCA. GRAD CWA. Page-288
Chapter-11: PGBP

(iv) A port, airport, inland waterway, inland port or


navigational channel in the sea.
10 Date on or after which the The capital expenditure incurred in relation to
specified business should specified business becomes eligible for deduction U/s
commence to become eligible 35AD only if the business commences on or after
for deduction U/s 35AD. specified dates which are given as under:
(a) Specified business 1, 2 and 4 01.04.2009
(b) Specified business 3 01.04.2007
(c) Specified business 5, 6 and 7 01.04.2010
(d) Specified business 8 and 9 01.04.2011
(e) Specified business 10, 11 and 01.04.2012
12
(f) Specified business 13 and 14 01.04.2014
(g) Specified business 15 01.04.2017
11 Relaxation in respect of hotel Building and operating hotel anywhere in India is only
business. [S. 35AD (6A)]. of the business specified in S. 35AD.
It indicates that the owner of the hotel himself shall
operate hotel; otherwise, it will not be regarded as
specified business and deduction is not available U/s
35AD.
However, in service industry like hotel, a franchisee
business exists where the hotel owner may get the
hotel operated through an outsourcing arrangement.
In order to enable such hotel owners to get deduction
U/s 35AD, S. 35AD (6A) was inserted.
While continuing to own hotel, if the assessee
transfers the operations thereof to another person,
still the assessee shall be deemed to be carrying on
the specified business of building and operating hotel
and is eligible for deduction U/s 35AD.
12 Conditions to be fulfilled for Conditions are divided into two categories: (a)
enjoying deduction U/s 35AD. conditions applicable to all specified businesses; (b)
conditions applicable only in respect of certain
specified businesses.
13 Conditions applicable to all Condition-1: The specified business shall not be set up
specified businesses. by splitting up or reconstruction of existing business.
Condition-2: Second-hand machinery shall not be used
in setting up specified business.
Condition-3: Accounts of the assessee for the relevant
PY have been audited by a CA and the assessee
furnishes the audit report in the prescribed form, duly
signed and verified by such accountant along with his
ROI.

MT Educare –CA Inter Income Tax Classes – R.SOUMYANARAYANAN.FCA. GRAD CWA. Page-289
Chapter-11: PGBP

14 Relaxing the requirements of A second-hand machinery (SHM), if it was imported


condition-2 from abroad and was never used by any other person
in India before its installation by the assessee in India
and no deduction was allowed in respect thereof by
way of depreciation or otherwise under this Act, shall
be treated as brand new machinery and not as second
hand machinery.
If Ʃ Value of all SHM used for setting up specified
business ≤ 20% of Ʃ value of all machineries used for
setting up specified business, then condition-2 is
deemed to have been complied with.
15 Illustration for understanding Ʃ Value of all P&M used for setting up specified
the relaxation. business = Rs. 400L.
Ʃ Value of all SHM used for setting up specified
business (included above) = Rs. 90L (which includes Rs.
30L being the cost of imported machine-A).
Ʃ Value of SHM used for setting up specified business
does not cross 20% of Ʃ value of all machineries used
for setting up specified business.
16 Conditions which are specific to Condition-1: Specified business 3 & 4 shall be carried
specified business-3 and on by an Indian company or consortium of Indian
specified business-4. companies or by an authority or board or corporation
established under the Central or State Act.
Condition-2: The person carrying on these businesses
shall be approved by PNGRB.
Condition-3: Such person shall be notified by the CG in
OG.
Condition-4: A specified proportion of total pipe-line
capacity (1/3rd for natural gas pipeline network; 1/4th
for petroleum pipeline network) shall be made
available to others (not being associated persons) on
common carrier basis.
Conditon-5: Such other conditions as may be
prescribed shall be fulfilled.
17 Meaning of associated persons In relation to the assessee means a person—
(i) who participates directly or indirectly or through
one or more intermediaries in the management or
control or capital of the assessee;
(ii) who holds, directly or indirectly, shares carrying
not less than 26% of the voting power in the capital of
the assessee;
(iii) who appoints more than half of the Board of
directors or members of the governing board, or one
or more executive directors or executive members of
the governing board of the assessee;

MT Educare –CA Inter Income Tax Classes – R.SOUMYANARAYANAN.FCA. GRAD CWA. Page-290
Chapter-11: PGBP

(iv) who guarantees not less than 10% of the total


borrowings of the assessee.
18 Conditions specific to specified The business should be owned by a company
business-15. registered in India or by a consortium of such
companies or by an authority or a board or
corporation or any other body established or
constituted under any Central or State Act.
The entity should have entered into an agreement
with the CG or a SG or a local authority or any other
statutory body for developing or operating and
maintaining or developing, operating and maintaining,
a new infrastructure facility.
19 Transfer of goods and services Where any goods or services held for the purposes of
the specified business are transferred to any other
business carried on by the assessee, or vice versa, and
if the consideration for such transfer does not
correspond with the market value of the goods or
services then the profits and gains of the specified
business shall be computed as if the transfer was
made at market value.
Market value means the price such goods or services
would ordinarily fetch in the open market, subject to
statutory or regulatory restrictions, if any.
20 Close connection between Where due to the close connection between the
assessee and any other person assessee and the other person or for any other reason,
it appears to the AO that the profits of specified
business is increased to more than the ordinary
profits, the AO shall compute the amount of profits of
such eligible business on a reasonable basis for
allowing the deduction.
21 Consequences of availing No deduction (like depreciation, additional investment
deduction U/s 35AD. allowance etc) shall be allowed in respect of the same
expenditure under any other provisions for the same
AY or subsequent AYs.
Where deduction U/s 35AD is claimed and allowed in
respect of specified business for any AY, no deduction
shall be allowed under Chapter VI-A Part-C or U/s
10AA in relation to specified business for the same or
any other AY. [S. 35AD (3)].
Example: Assessee sets up a 4-star hotel in Agra. Hotel
starts functioning on 01.01.2013. Assessee can either
enjoy investment linked deduction U/s 35AD or profit
linked deduction U/s 80-ID (100% of profits of such
hotel can be claimed as deduction for 5 AYs beginning
with the AY relevant to the PY in which hotel starts
functioning).

MT Educare –CA Inter Income Tax Classes – R.SOUMYANARAYANAN.FCA. GRAD CWA. Page-291
Chapter-11: PGBP

22 Restriction of set-off and carry Loss from specified business can be set off only against
forward of losses from income from business specified in S. 35AD.
specified business. [S. 73A]. If it is remaining unabsorbed, it could be carried
forward to the next AY for set off against income from
business specified in S. 35AD and so on, for an
indefinite period.
However, the benefit of carry forward is available only
if a loss-return U/s 139 (3) for the PY in which loss was
incurred is filed within the time limit specified U/s 139
(1).
Losses specified businesses could be set off even
against the income from specified business which is
not eligible for deduction U/s 35AD.
Example: Loss from hotel set up on 01.04.2017 could
be set of against income from another hotel which is
in operation since 01.04.2000.
23 Asset in respect of which Any sum received or receivable (in cash or in kind) on
deduction U/s 35AD was account of any capital asset (in respect of which
enjoyed is sold, destroyed or deduction was allowed U/s 35AD) being demolished,
demolished – Tax implications. destroyed or transferred shall be charged to tax U/H
[S. 28 (vii)]. PGBP.
However, no capital gains will be computed.
24 Actual cost of asset in respect The actual cost of any capital asset on which
of which deduction was deduction is allowable to the assessee U/s 35AD shall
enjoyed u/s 35AD. [S. 43 (1) be treated as nil,
Explanation-13] In case of such assessee; and
In any other case if the capital asset is acquired or
received: (a) by way of gift or will or an irrevocable
trust; (b) on any distribution on liquidation of the
company; (c) by such mode of transfer as is referred to
in S. 47 (i)/(iv)/(v)/(vi)/(vib)/(xiii)/(xiv)/(xiiib).
25 Asset to be used for specified S. 35AD (7A) provides that any asset in respect of
business for 8 years. [S. 35AD which a deduction is claimed and allowed U/s 35AD
(7A)]. shall be used only for the specified business for a
period of 8 years beginning with the PY in which such
asset is acquired or constructed.
26 Asset used for any other If asset is used for any purpose other than the
business other than specified specified business, the total amount of deduction so
business [S. 35AD (7B)]. claimed and allowed in any PY in respect of such asset,
as reduced by the amount of depreciation allowable in
accordance with the provisions of S. 32 as if no
deduction had been allowed U/s 35AD, shall be
deemed to be income of the assessee chargeable U/H
PGBP of the PY in which the asset is so used.

MT Educare –CA Inter Income Tax Classes – R.SOUMYANARAYANAN.FCA. GRAD CWA. Page-292
Chapter-11: PGBP

27 Relaxation to sick companies. The provisions of S. 35AD (7B) shall not apply to sick
[S. 35AD (7C)]. companies.
There is no bar on sick companies using the assets
acquired for specified business for any other purpose
even during the aforesaid 8 years.
28 Determination of actual cost of However, where an asset, in respect of which
the asset consequent to deduction is claimed and allowed U/s 35AD is deemed
withdrawal of deduction U/s to be the income of the assessee in accordance with
35AD. [Proviso to Explanation- the provisions of S. 35AD (7B) (on account of being
13 to S. 43 (1)] used for a purpose other than specified business U/s
35AD), the actual cost of the asset to the assessee
shall be actual cost to assessee as reduced by the
amount of depreciation allowable had the asset been
used for the purpose of business, calculated at the
rate in force, since the date of its acquisition.

5. Deduction U/s 35CCA:

1 What is allowed as deduction Payment to an association or institution, having the


U/s 35CCA? objective of undertaking programmes of rural
development. Such payment must be used for
carrying out any programme of rural development
approved by the prescribed authority before
01.03.1983.
Payment to an association or institution (approved
by the prescribed authority before 01.03.1983)
having as its object the training of persons for
implementing rural development programme.
Payment to a rural development fund set up and
notified by the CG.
Payments made to "National Urban Poverty
Eradication Fund" (NUPEF) set up and notified by the
CG.
2 Meaning of rural development Programme of rural development includes any
programme. programme for promoting the social and economic
welfare, or the uplift of, the public in any rural area.
3 Bar on double deduction. Where any deduction in respect of contribution for
rural development is claimed by the assessee and
allowed to him for any AY in respect of any
expenditure incurred by way of payment of
contribution to the approved association or
institution, no deduction in respect of the same
expenditure can again be claimed by the assessee
under any other relevant provision for the same or
any other AY.

MT Educare –CA Inter Income Tax Classes – R.SOUMYANARAYANAN.FCA. GRAD CWA. Page-293
Chapter-11: PGBP

6. Deduction U/s 35CCC:

1 What is allowed as S. 35CCC provides a weighted deduction of a sum equal to 150%


deduction U/sof expenditure incurred by an assessee on agricultural extension
35CCC? project in accordance with the prescribed guidelines.
2 Project must be The agricultural extension project eligible for this weighted
notified. deduction shall be notified by the CBDT.
3 Conditions to be The project shall be undertaken by an assessee for training,
fulfilled for getting education and guidance of farmers.
the project notified. The project shall have prior approval of the Ministry of
Agriculture, Government of India; and
An expenditure (not being expenditure in the nature of cost of
any land or building) exceeding the amount of Rs. 25L is expected
to be incurred for the project.
4 Eligible expenditure All expenses (not being expenditure in the nature of cost of any
land or building), as reduced by the amount received from
beneficiary, if any, incurred wholly and exclusively for
undertaking an eligible agricultural extension project shall be
eligible for deduction U/s 35CCC.
However, expenditure incurred on the agricultural extension
project which is reimbursed or reimbursable to the assessee by
any person, whether directly or indirectly, shall not be eligible for
deduction U/s 35CCC.
5 Conditions for Assessee shall maintain separate books of account of such
claiming weighted agricultural extension project and shall get such books of account
deduction U/s audited by a CA.
35CCC. The assessee shall furnish the following on or before the due date
of furnishing the ROI to the CIT or DIT, as the case may be:
(a) the audited statement of accounts of the agricultural
extension project along with the audited report and amount of
deduction claimed under this section.
(b) a note on agricultural project undertaken and programme of
agricultural extension project to be undertaken during the
current year and financial allocation for such programme and
(c) a certificate from Ministry of Agriculture, GOI, regarding the
genuineness of such project.

7. Deduction U/s 35CCD:


1 What is allowed as As per the National Manufacturing Policy notified by the
deduction U/s 35CCD? Department of Industrial Policy & Promotion, the government
will provide weighted standard deduction of 150% of the
expenditure (other than land or building) incurred on Public
Private Partnership (PPP) project for skill development in the ITIs
in manufacturing sector. This is to encourage private sector to set
up their own institution in coordination with National Skill
Development Corporation.

MT Educare –CA Inter Income Tax Classes – R.SOUMYANARAYANAN.FCA. GRAD CWA. Page-294
Chapter-11: PGBP

In order to encourage companies to invest on skill development


projects in the manufacturing sector, S. 35CCD provides for a
weighted deduction of a sum equal to 150% of the expenditure
(not being expenditure in the nature of cost of any land or
building) on skill development project incurred by the company
in accordance with the prescribed guidelines.
2 Only expenditure on The skill development project eligible for this weighted deduction
notified projects is shall be notified by the CBDT.
eligible for deduction. A skill development project would be considered for notification
if is undertaken by an eligible company (a company engaged in
the business of manufacture or production of any article or thing,
not being beer, wine and other alcoholic spirits and tobacco and
tobacco preparations or engaged in providing specified services)
and the project is undertaken in separate facilities in a training
institute.
3 Conditions for availing The assessee shall maintain separate books of account of the skill
weighted deduction. development project and get such books of account audited by
an accountant.
The assessee shall furnish the audited statement of accounts of
the skill development project along with the audited report and
amount of deduction claimed under this section on or before the
due date of furnishing the ROI, to the CIT or DIT, as the case may
be.
4 Bar on double Where in respect of any expenditure deduction is allowed under
deduction. this section, then no deduction shall be allowed in respect of
such expenditure under any other provisions of the Act in the
same or any other AY.

8. Deduction U/s 35D:


1 What is allowed as S. 35D allows deduction in respect of specified preliminary
deduction U/s 35D? expenses incurred by the eligible assessee.
2 Eligible assessee Indian company
Non-corporate residents.
3 Timing of preliminary Preliminary expenses may be incurred before commencement of
expenses business in connection with setting up business.
Preliminary expenses may also be incurred after the
commencement of business in connection with extension of an
industrial undertaking or in connection with setting up of new
Unit.
4 List of specified Expenses on preparation of a feasibility report.
preliminary expenses. Expenses on preparation of a project report.
Expenses on conducting market survey or any other survey
necessary for business.
Expenses on engineering services relating to assessee’s business.
Legal charges for drafting any agreement relating to setting up or
extension of the business of the assessee.

MT Educare –CA Inter Income Tax Classes – R.SOUMYANARAYANAN.FCA. GRAD CWA. Page-295
Chapter-11: PGBP

Charges for drafting and printing of MOA & AOA.


Fees for registering the company.
Expenses in connection with issue, for public subscription, of
shares or debentures.
Underwriting commission.
Charges for drafting, typing, printing and advertisement of
prospectus.
Expenditure incurred in refunding the share application money to
the unsuccessful applicants.
Such other prescribed expenditure.
5 Work to be done by the The work in connection with the preparation of feasibility report
assessee or concern or the project report or the conducting of market survey or the
approved by the CBDT. engineering services referred above should be carried out by the
assessee himself or by a consultancy concern approved by the
Board.
6 Specified expenditure to The sum total of all the specified expenditure incurred by the
be subjected to ceiling assessee shall be restricted to a ceiling.
7 Ceiling in case of Indian 5% of project cost or 5% of capital employed (whichever is higher).
company
8 Ceiling in case of non- 5% of project cost.
corporate residents
7 Meaning of project cost The actual cost of the fixed assets, being land, buildings,
where preliminary leaseholds, plant, machinery, furniture, fittings, railway sidings
expenses are incurred (including expenditure on the development of land, buildings)
before commencement which are shown in the books of the assessee as on the last day of
of business (in the PY in which the business of the assessee commences.
connection with setting
up business)
8 Meaning of project cost The actual cost of the fixed assets being land, buildings,
where preliminary leaseholds, plant, machinery, furniture, fittings, and railway sidings
expenses are incurred (including expenditure on the development of land and buildings)
after the commencement which are shown in the books of the assessee as on the last day of
of business (in the PY in which the extension of the undertaking is completed, in
connection with so far as such fixed assets have been acquired or developed in
extension of the existing connection with the extension of the undertaking.
undertaking)
9 Meaning of project cost The actual cost of the fixed assets being land, buildings,
where preliminary leaseholds, plant, machinery, furniture, fittings, and railway sidings
expenses are incurred (including expenditure on the development of land and buildings)
after the commencement which are shown in the books of the assessee as on the last day of
of business (in the PY in which the new unit commences production or operation,
connection with setting in so far as such fixed assets have been acquired or developed in
up new unit). connection with setting up of the new unit.
10 Meaning of capital The aggregate of the issued share capital, debentures and long-
employed where term borrowings as on the last day of the previous year in which
preliminary expenses are the business of the company commences.
incurred before

MT Educare –CA Inter Income Tax Classes – R.SOUMYANARAYANAN.FCA. GRAD CWA. Page-296
Chapter-11: PGBP

commencement of
business (in connection
with setting up business).
11 Meaning of capital The aggregate of the issued share capital, debentures, and long-
employed where term borrowings as on the last day of the PY in which the
preliminary expenses are extension of the undertaking is completed, in so far as such
incurred after the capital, debentures and long-term borrowings have been issued or
commencement of obtained in connection with the extension of the undertaking.
business (in connection
with extension of the
existing undertaking)
12 Meaning of capital The aggregate of the issued share capital, debentures, and long-
employed where term borrowings as on the last day of the PY in which the unit
preliminary expenses are commences production or operation in so far as such capital,
incurred after the debentures and long-term borrowings have been issued or
commencement of obtained in connection with the setting up of the new
business (in connection undertaking.
with setting up new
unit).
13 Meaning of long-term Any moneys borrowed in India by the company from the
borrowing Government or the Industrial Finance Corporation of India or the
Industrial Credit and Investment Corporation of India or any other
financial institution eligible for deduction U/s 36 (1) (viii) or any
banking institution, or
Any moneys borrowed or debt incurred by it in a foreign country
in respect of the purchase outside India of plant and machinery
where the terms under which such moneys are borrowed or the
debt is incurred provide for the repayment thereof during a period
of not less than 7 years.
14 Quantum and period of Specified preliminary expenses which were restricted to ceiling are
deduction (in case allowed as a deduction in 5 equal annual instalments beginning
preliminary expenses with the PY in which the business commences.
were incurred before the
commencement of
business)
15 Quantum and period of Specified preliminary expenses which were restricted to ceiling are
deduction (in case allowed as a deduction in 5 equal annual instalments beginning
preliminary expenses with the PY in which the extension of the undertaking is
were incurred after the completed.
commencement of
business (in connection
with extension of existing
undertaking)).

MT Educare –CA Inter Income Tax Classes – R.SOUMYANARAYANAN.FCA. GRAD CWA. Page-297
Chapter-11: PGBP

16 Quantum and period of Specified preliminary expenses which were restricted to ceiling are
deduction (in case allowed as a deduction in 5 equal annual instalments beginning
preliminary expenses with the PY in which the new unit becomes operational.
were incurred after the
commencement of
business (in connection
with setting up new
unit)).
17 Audit requirement In cases where the assessee is a person other than a company or a
cooperative society, the deduction would be allowable only if the
accounts of the assessee for the year or years in which the
expenditure is incurred have been audited by a CA and the
assessee furnishes, along with his ROI for the first year in respect
of which the deduction is claimed, the report of such audit in the
prescribed form duly signed and verified by the auditor and setting
forth such other particulars as may be prescribed.
18 Bar on double deduction Where a deduction under this section is claimed and allowed for
any AY in respect of any item of expenditure, the expenditure in
respect of which deduction is so allowed shall not qualify for
deduction under any other provision of the Act for the same or
any other AY.
19 Special provisions for Where the assessee-company which has been enjoying
amalgamation. deduction U/s 35D, gets amalgamated with an Indian company,
then the deduction U/s 35D shall continue in the hands of
amalgamated company for the unexpired residual period
beginning with the PY in which amalgamation took place.
20 Special provision for Where the business undertaking of the assessee-company in
demerger. respect of the preliminary expenses of which deduction has been
enjoyed by the assessee-company gets transferred to another
Indian company pursuant to a scheme of demerger, then the
deduction U/s 35D shall continue in the hands of resulting
company for the unexpired residual period beginning with the PY
in which the demerger took place.

9. Deduction U/s 35DDA:

1 What is allowed as S. 35DDA allows deduction in respect of compensation paid to


deduction U/s employees who have opted to retire under a voluntary
35DDA? retirement scheme.
2 Deduction upon Irrespective of the method of accounting regularly employed by
payment the assessee, deduction shall be allowed under this section only
upon payment.
3 Quantum and period Voluntary retirement compensation paid shall be allowed as
of deduction deduction in 5 equal annual installments beginning with the PY
in which the compensation was paid.

MT Educare –CA Inter Income Tax Classes – R.SOUMYANARAYANAN.FCA. GRAD CWA. Page-298
Chapter-11: PGBP

4 No double Where the assessee avails deduction under this section with
deduction respect to the aforesaid expenditure, he is not entitled to get
deduction with respect to the same expenditure under any other
provisions of this Act.
5 Special provision in Where there is succession in business on account of
case of amalgamation, demerger, conversion of SPC into company,
amalgamation etc. conversion of firm into company or conversion of company into
LLP and as a result the predecessor could not enjoy deduction
under this section for a full period of 5 years, then the deduction
under this section shall continue in the hands of successor for the
unexpired residual period beginning with the PY of succession.

10. Deduction U/s 36:

S. 36 (1) (i)
S. 36 (1) (i) provides deduction in respect of premium paid to insure against the risk of damage or
destruction of stocks or stores of the business or profession.
S. 36 (1) (ia)
Assessee eligible for Federal milk co-operative society
deduction.
Expenditure eligible Premium paid to effect an insurance on the life of cattle owned
for deduction. by a member of a co-operative society being a primary society
engaged in supply of milk raised by its members to such Federal
Milk Co-operative Society.
S. 36 (1) (ib)
Expenditure eligible Provides deduction in respect of premium paid to effect an
for deduction insurance on the health of employees.
Conditions to be Premium should be paid otherwise than by way of cash.
fulfilled for enjoying Premium is paid under a scheme framed by GIC and approved by
deduction. the CG or under a scheme framed by any other insurer and
approved by IRDA.
S. 36 (1) (ii)
Expenditure eligible Bonus or commission paid to employees for the services rendered
for deduction. by them.
But this bonus or commission should not be one which would
have been otherwise distributed as dividend or profits.
PY in which deduction Deduction is allowed in the PY in which bonus or commission is
is allowed. [S. 43B]. incurred provided the payment is made either in the same PY or
within the due date for filing ROI for that PY.
However, if the payment is made beyond the due date for filing
ROI, it shall be allowed as deduction in the PY in which payment is
made.
S. 36 (1) (iii)
Expenditure eligible Amount of interest paid in respect of capital borrowed for the
for deduction. purpose of business or profession.
However, if the interest is paid on borrowing made for the

MT Educare –CA Inter Income Tax Classes – R.SOUMYANARAYANAN.FCA. GRAD CWA. Page-299
Chapter-11: PGBP

purpose of acquisition of asset used for the purpose of business


or profession, the interest attributable to the period up to the
date on which the asset first put to use shall not be allowed as
deduction U/s 36 (1) (iii). It shall be capitalised. [Proviso to S. 36
(1) (iii)].
Interest attributable to the period falling after the date on which
asset is first put to use shall not be capitalised but shall be
allowed as deduction U/s 36 (1) (iii). [Explanation-8 to S.43 (1)].
PY in which deduction Interest is allowed as deduction in the PY in which it is incurred.
is allowed. [S. 43B]. However, if interest is payable on any loan or advance taken from
a schedule bank or co-operative bank (other than a primary
agricultural credit society or primary co-operative agricultural and
rural development bank) or any PFI or any SFC or SIIC, deduction
is allowed in the PY in which it is incurred provided the payment is
made either in the same PY or within the due date for filing ROI
for that PY.
However, if the payment is made beyond the due date for filing
ROI, it shall be allowed as deduction in the PY in which payment is
made.

S. 36 (1) (iiia)
Expenditure eligible Discount on ZCB
for deduction
Discount Difference of the amount received or receivable by an
infrastructure capital company/infrastructure capital fund/public
sector company/ scheduled bank on issue of the bond and the
amount payable by such company or fund or bank on maturity or
redemption of the bond.
Manner of allowing On pro rata basis having regard to the period of life of the bond to
deduction be calculated in the manner prescribed.
Period of life of the The period commencing from the date of issue of the bond and
bond. ending on the date of the maturity or redemption shall be
regarded as life of the bond.
If the number of days in the month of issue of bond coming within
the life of the bond is less than 15 days, the month of issue shall
not be taken into account for computing the life of the bond.
Similarly, if the number of days in the month of redemption of
bond coming within the life of the bond is less than 15 days, the
month of redemption shall not be taken in to account for
computing the life of the bond.
S. 36 (1) (iv)
Expenditure eligible Contribution to recognised provident fund or approved
for deduction superannuation fund for the benefit of employees.
Satisfactory arrangement should have been made for deduction
of tax at source from the sum payable out of provident fund to
the employee at the time of termination of his service, in case it is

MT Educare –CA Inter Income Tax Classes – R.SOUMYANARAYANAN.FCA. GRAD CWA. Page-300
Chapter-11: PGBP

taxable in the hands of the employee.


Otherwise, the contribution made by the employer to these funds
shall not be allowed as deduction in his hands. [S. 40 (a) (iv)].
PY in which deduction Deduction is allowed in the PY in which contribution becomes due
is allowed. [S. 43B]. provided the contribution is made either in the same PY or within
the due date for filing ROI for that PY.
However, if the contribution is made beyond the due date for
filing ROI, it shall be allowed as deduction in the PY in which
contribution is made.
S. 36 (1) (v) + S. 40A (7)
Disallowance of Provision for gratuity is not to be allowed as deduction. [S. 40A (7)].
provision for gratuity.
Exception-1 Where the employee retires in the relevant PY and the gratuity
becomes payable to the employee, the provision created for payment
of such gratuity shall not be disallowed.
Where deduction was allowed in respect of such provision, deduction
can’t be allowed again in the PY in which payment was made.
Exception-2 Where the provision is created for making contribution to the
approved gratuity fund, such provision can’t be disallowed. [S. 40A
(7)].
It is eligible for deduction U/s 36 (1) (v).
Deduction is allowed in the PY in which contribution becomes due
provided the contribution is made either in the same PY or within the
due date for filing ROI for that PY.
However, if the contribution is made beyond the due date for filing
ROI, it shall be allowed as deduction in the PY in which contribution is
made.
Where deduction is enjoyed in respect contribution to the approved
gratuity fund, the deduction shall not be allowed again upon payment
of gratuity.
Satisfactory Satisfactory arrangement should have been made for deduction of tax
arrangement for at source from the taxable portion of gratuity payable to the
deduction of tax at employee at the time of termination of his service.
source Otherwise the contribution to the approved gratuity fund will not be
allowed as deduction in his hands. [S. 40 (a) (iv)].
S. 36 (1) (iva)
Expenditure eligible Employer`s contribution to the account of the employee under a
for deduction Pension Scheme referred to in S. 80CCD.
Ceiling on deduction 10% of [Basic salary + DA if the terms of employment so
provides].

MT Educare –CA Inter Income Tax Classes – R.SOUMYANARAYANAN.FCA. GRAD CWA. Page-301
Chapter-11: PGBP

S. 36 (1) (va)
Deemed income by Any sum retained by the employer out of the salary payable to
virtue of S. 2 (24) (x) the employee by way of contribution towards provident fund or
any other welfare fund shall be regarded as income in the hands
of the employer.
Deduction U/s 36 (1) However, if the sum retained is deposited into the account of
(va) employee under the relevant fund within the due date specified
in the Act governing such fund, then it shall be allowed as
deduction in the hands of employer.
S. 36 (1) (vi)
Where the animals used by the assessee in his business or profession have died or become
permanently useless, then the difference between the cost for which such animals were acquired
and the sum realised from disposal of such animals or their carcasses shall be allowed as
deduction in the PY in which such animals were disposed off.
S. 36 (1) (vii)
Expenditure eligible S. 36 (1) (vii) allows deduction in respect of amount of bad debts
for deduction written off as irrecoverable in the books of accounts of the assessee
for the relevant PY.
The bad debts for which deduction is claimed by the assessee should
have been considered in computing the income of the assessee
either for the PY of write off or earlier years. Otherwise, deduction is
not allowed U/s 36 (1) (vii).
Demonstrative For claiming deduction U/s 36 (1) (vii), what is required is writing off
infallible proof – not of bad debts in the books of accounts. There is no requirement that
required. the assessee should produce demonstrative and infallible proof to
establish bad debts.
Creation of provision Deduction U/s 36 (1) (vii) is not allowed if the assessee merely
– not enough. creates a provision for bad and doubtful debts.
Deemed write off Where a debt which was not recognised in the books of accounts as
per the requirement of accounting standards but has been taken into
account in computing the income of the assessee on the basis of
notified ICDS, has become irrecoverable, it can still be claimed as bad
debts U/s 36 (1) (vii) since it is deemed that the debt has been
written off as irrecoverable in the books of accounts of the assessee.

MT Educare –CA Inter Income Tax Classes – R.SOUMYANARAYANAN.FCA. GRAD CWA. Page-302
Chapter-11: PGBP

Recovery of bad debts If a deduction has been allowed in respect of a bad debt U/s 36 (1)
– tax implications. [S. (vii), and subsequently the amount recovered in respect of such debt
41 (4)]. is more than the amount due after the allowance had been made,
the excess shall be deemed to be the profits and gains of business or
profession and will be chargeable as income of the PY in which it is
recovered, whether or not the business or profession in respect of
which the deduction has been allowed is in existence at the time.
S. 36 (1) (ix)
Who is eligible for Corporate assessee.
deduction?
Expenditure eligible for S. 36 (1) (ix) allows deduction in respect of expenditure incurred
deduction in connection with promotion of family planning amongst
employees.
Classification of (a) Revenue expenditure; (b) Capital expenditure.
expenditure (supra)
Deductibility of revenue Revenue expenditure on promotion of family planning amongst
expenditure on promotion employees shall be allowed as deduction in the PY in which it is
of family planning. incurred.
Deductibility of capital It is allowed as deduction in 5 equal annual instalments
expenditure on promotion beginning with the PY in which it is incurred.
of family planning.
Tax treatment of Unabsorbed expenditure on promotion of family planning shall
unabsorbed expenditure on be treated on par with unabsorbed depreciation.
promotion of family
planning.
Carry forward allowed to Where amalgamation takes place, the unabsorbed expenditure
amalgamated company on promotion of family planning incurred by the amalgamating
company shall be allowed to be carried forward by the
amalgamated company (if it is an Indian company).
S. 36 (1) (xv)
The amount of STT paid by the assessee during the year in respect of taxable securities
transactions entered into in the course of business shall be allowed as deduction U/s 36 (1) (xv)
subject to the condition that such income from taxable securities transactions is included U/H
PGBP.
S. 36 (1) (xvi)
1 The Finance Act, 2013 has introduced a new tax called Commodities Transaction Tax (CTT)
to be levied on taxable commodities transactions entered into in a recognised association,
vide Chapter VII of the Finance Act, 2013.
2 For this purpose, a `taxable commodities transaction` means a transaction of sale of
commodity derivatives in respect of commodities, other than agricultural commodities,
traded in recognised associations.
3 CTT is to be levied at 0.01% on sale of commodity derivative. CTT is to be paid by the seller.
4 Consequently, S. 36(1) (xvi) provides that an amount equal to the CTT paid by the assessee
in respect of the taxable commodities transactions entered into in the course of his business
during the PY shall be allowable as deduction, if the income arising from such taxable
commodities transactions is included in the income computed U/H PGBP.

MT Educare –CA Inter Income Tax Classes – R.SOUMYANARAYANAN.FCA. GRAD CWA. Page-303
Chapter-11: PGBP

11. Deduction U/s 37 (1):

Nature of provisions of Residuary provisions.


S. 37 (1)
Conditions for availing The expenditure should not be of the nature described in sections
deduction U/s 37 (1) 30 to 36.
It should have been incurred by the assessee in the relevant PY.
It should be in respect of a business carried on by the assessee the
profits of which are being computed and assessed.
It must have been incurred after the business was set up.
It should not be in the nature of any personal expenses of the
assessee.
It should have been laid out or expended wholly and exclusively
for the purposes of such business.
It should not be in the nature of capital expenditure.
Explanation-1 to S. 37 The expenditure should not have been incurred by the assessee
(1) for any purpose which is an offence or is prohibited by law.
CBDT Circular 5/2012 Any expense incurred in providing freebees to medical
practitioner is in violation of the provisions of Indian Medical
Council (Professional Conduct, Etiquette and Ethics) Regulations,
2002 has clarified that the expenditure so incurred shall be
inadmissible U/s 37 (1), being an expense prohibited by the law.
The disallowance shall be made in the hands of such
pharmaceutical or allied health sector industry or other assessee
which has provided aforesaid freebees and claimed it as a
deductible expense in its accounts against income.
Further, a sum equivalent to value of freebees enjoyed by the
aforesaid medical practitioner or professional associations is also
taxable as business income or income from other sources, as the
case may be, depending on the facts of each case.
Explanation-2 to S. 37 CSR activity expenses are to be disallowed while computing
(1) income U/H PGBP.
Disallowance U/s 37 S. 37 (2B) disallows expenses on advertisement in political
(2B). magazines made by any person carrying on business or profession
in computing income U/H PGBP.

CSR activity – in brief:


1 S. 135 of the Companies Act, 2013 read with Schedule VII thereto and Companies
(Corporate Social Responsibility) Rules, 2014 are the special provisions under the new
company law regime imposing mandatory CSR obligations.
2 Every company, listed or unlisted, private or public, having a net worth of Rs. 500 crores
or more [Net worth criterion]; or turnover of Rs. 1,000 crores or more [Turnover
criterion]; or a net profit of Rs. 5 crores or more [Net Profit criterion] during any financial
year to constitute a CSR Committee of the Board;
3 CSR Committee has to formulate CSR policy and the same has to be approved by the

MT Educare –CA Inter Income Tax Classes – R.SOUMYANARAYANAN.FCA. GRAD CWA. Page-304
Chapter-11: PGBP

Board. Such company shall undertake CSR activities as per the CSR Policy. Such company
shall spend in every financial year, at least 2% of its average net profits made in the
immediately three preceding financial years, on the CSR activities specified in Schedule
VII to the Companies Act, 2013.
4 As per Rule 4 of the Companies (CSR) Rules, 2014, the following expenditure are not
considered as CSR activity for the purpose of section 135: (a) Expenditure on activities
undertaken in pursuance of normal course of business; (b) Expenditure on CSR activities
undertaken outside India; (c) Expenditure which is exclusively for the benefit of the
employees of the company or their families; and (d) Contributions to political parties.

D. Disallowances:

1. Disallowance U/s 40 (a) (i):

1 What is disallowed U/s 40 (a) (i)? Expense by way of interest, royalty, FTS or any
other sum chargeable to tax in the hands of the
recipient, payable outside India.
Expense by way of interest, royalty, FTS or any
other sum chargeable to tax in the hands of the
recipient being a FC/NCNR, payable in India.
2 When it is disallowed? Situation-1: Tax was not deducted at source
during the PY.
Situation-2: Tax was deducted at source during the
PY but it was not remitted to the credited of the
Government within the due date specified in S.
139 (1) for filing return for the PY.
3 Deduction in the PY of remittance. It Shall be Deemed that the assessee has
[Proviso to S. 40 (a) (i)]. deducted and paid the tax on such sum on the
date of the furnishing of return of income by the
payee → if assessee fails to deduct tax at source
and that payer is not and assessee in default
(Refer NOTE)
NOTE: -
i) has furnished his return of income under section 139;
ii) has taken into account such sum for computing income in such return of
income; and
iii) has paid the tax due on the income declared by him in such return of
income, and the payer
iv) furnishes a certificate to this effect from an accountant in such form as may
be prescribed,

MT Educare –CA Inter Income Tax Classes – R.SOUMYANARAYANAN.FCA. GRAD CWA. Page-305
Chapter-11: PGBP

2. Disallowance U/s 40 (a) (ia):

1 What is disallowed Expense by way of any sum payable to a resident, on which


U/s 40 (a) (i)? there is an obligation for the assessee under Chapter XVII-B to
deduct tax at source.
2 When it is disallowed? Situation-1: Tax was not deducted at source during the PY.
Situation-2: Tax was deducted at source during the PY but it
was not remitted to the credited of the Government within the
due date specified in S. 139 (1) for filing return for the PY.
3 Quantum disallowed 30% of the expense.
4 Deduction in respect Expense aforesaid to the extent disallowed U/s 40 (a) (ia) in the
of disallowed portion PY of incurrence, could be allowed as deduction in the PY of
in the PY of remittance of TDS. [Proviso-1 to S. 40 (a) (ia)].
remittance.
5 Illustration For instance, tax on royalty paid to Mr. A, a resident, has been
deducted during the PY 2018-19, the same has to be paid by
31st July/30th September 2019, as the case may be. Otherwise,
30% of royalty paid would be disallowed in computing the
income for AY 2019-20.
If in respect of such royalty, tax deducted during the PY 2018-
19 has been paid after 31st July/30th September, 2019, 30% of
such royalty would be allowed as deduction in the year of
payment.

Proviso-2 to S. 40 (a) (ia):

1 A sum (on which tax is deductible at source under Chapter XVII-B) is paid to a resident
without deducting tax at source.
2 However, the payee includes such sum in computing the returned income of the
relevant PY, files return for the PY U/s 139 and the tax on the returned income was fully
paid by way of advance tax or otherwise.
3 And the payer obtains a certificate in Form-26A from a CA to the effect that the payee
has complied with the conditions aforesaid.
4 Then, tax on the sum (supra) is deemed to have been deducted at source and remitted
to the credit of Government on the date of filing of return by the payee. [Proviso-2 to S.
40 (a) (ia)].
5 In this case, 30% of the sum is disallowed U/s 40 (a) (ia) in the hands of payer in the PY of
incurrence, it will be allowed in the PY in which the return in filed by the payee.

Q-1: Delta Ltd credited the following amounts to the account of resident payees in the
month of March, 2020 without deduction of tax at source. What would be the consequence
of non-deduction of tax at source by Delta Ltd on these amounts during the FY 2019-20,
assuming that the resident payees in all the cases mentioned below, have not paid the tax, if
any, which was required to be deducted by Delta Ltd?

MT Educare –CA Inter Income Tax Classes – R.SOUMYANARAYANAN.FCA. GRAD CWA. Page-306
Chapter-11: PGBP

Particulars Amount
(1) Salary to its employees (credited and paid in March, 2020) 12,00,000
Directors` remuneration (credited in March, 2019 and paid in April,
(2) 28,000
2020)
Would your answer change if Delta Ltd has deducted tax on directors` remuneration in April,
2020 at the time of payment and remitted the same in July, 2020?

Solution:

Non-deduction of tax at source on any sum payable to a resident on which tax is deductible
at source as per the provisions of Chapter XVII-B would attract disallowance U/s 40 (a) (ia).

Therefore, non-deduction of tax at source on any sum paid by way of salary on which tax is
deductible U/s 192 or any sum credited or paid by way of directors` remuneration on which
tax is deductible U/s 194J, would attract disallowance @ 30% U/s 40 (a) (ia). Whereas in
case of salary, tax has to be deducted U/s 192 at the time of payment, in case of directors`
remuneration, tax has to be deducted at the time of credit of such sum to the account of
the payee or at the time of payment, whichever is earlier.

Therefore, in both the cases i.e., salary and directors` remuneration, tax is deductible in the
PY 2019-20, since salary was paid in that year and directors` remuneration was credited in
that year.

Therefore, the amount to be disallowed U/s 40 (a) (ia) while computing business income for
AY 2020-21 is as follows –

Particulars Amount Disallowance


paid in Rs. u/s 40 (a)
(ia) @ 30%
(1) Salary [tax is deductible U/s 192] 12,00,000 3,60,000
(2) Directors` remuneration [tax is deductible U/s 194J 28,000 8,400
without any threshold limit]
Disallowance U/s 40 (a) (ia) 3,68,400

If the tax is deducted on directors` remuneration in the next year i.e., PY 2020-21 at the
time of payment and remitted to the Government, the amount of Rs. 8,400 would be
allowed as deduction while computing the business income of AY 2021-22.

Disallowance of any sum paid to a resident at any time during the PY without deduction of
tax U/s 40 (a) (ia) [ Palam gas service (2017) (SC) + Circular No.10/2013, dated 16.12.2013]

1 The provisions of S. 40 (a) (ia) would cover not only the amounts which are payable as on
31st March of a PY but also amounts which are payable at any time during the year.
2 The statutory provisions are amply clear and in the context of S. 40 (a) (ia), the term
"payable" would include "amounts which are paid during the PY".

MT Educare –CA Inter Income Tax Classes – R.SOUMYANARAYANAN.FCA. GRAD CWA. Page-307
Chapter-11: PGBP

Q-2: During the FY 2019-20, the following payments/expenditures were made/incurred by


Mr. Yuvan Raja, a resident individual (whose turnover during the year ended 31.3.2019 was
Rs. 99L):

(i) Interest of Rs. 12,000 was paid to Rehman & Co., a resident partnership firm, without
deduction of tax at source;
(ii) Rs. 3,00,000 was paid as salary to a resident individual without deduction of tax at
source;
(iii) Commission of Rs. 16,000 was paid to Mr. Vidyasagar on 02.07.2019 without
deduction of tax at source.

Briefly discuss whether any disallowance arises under the provisions of S. 40 (a) (ia).

Solution:

Disallowance U/s 40 (a) (ia) is attracted where the assessee fails to deduct tax at source as is
required under the Act, or having deducted tax at source, fails to remit the same to the
credit of the CG within the stipulated time limit.

(i) The obligation to deduct tax at source from interest paid to a resident arises U/s 194A in
the case of an individual, whose total turnover in the immediately preceding PY, i.e., PY
2018-19 exceeds Rs. 100L. Thus, in present case, since the turnover of the assessee is less
than Rs. 100L, he is not liable to deduct tax at source. Hence, disallowance U/s 40 (a) (ia) is
not attracted in this case.

(ii) The disallowance of 30% of the sums payable U/s 40 (a) (ia) would be attracted in
respect of all sums on which tax is deductible under Chapter XVII-B. S. 192, which requires
deduction of tax at source from salary paid, is covered under Chapter XVII-B. The obligation
to deduct tax at source U/s 192 arises, in the hands all assessee-employer even if the
turnover amount does not exceed Rs. 100L in the immediately preceding PY.

Therefore, in the present case, the disallowance U/s 40 (a) (ia) is attracted for failure to
deduct tax at source U/s 192 from salary payment. However, only 30% of the amount of
salary paid without deduction of tax at source would be disallowed.

(iii) The obligation to deduct tax at source U/s 194-H from commission paid in excess of Rs.
15,000 to a resident arises in the case of an individual, whose total turnover in the
immediately preceding PY, i.e., PY 2018-19 exceeds Rs. 100 lakhs. Thus, in present case,
since the turnover of the assessee is less than Rs. 100 lakhs, he is not liable to deduct tax at
source. Therefore, disallowance U/s 40 (a) (ia) is not attracted in this case.

3. Disallowance U/s 40 (a) (ii):

Any sum paid on account of tax or cess levied on profits on the basis of or in proportion to
the profits and gains of any business or profession shall be disallowed.

MT Educare –CA Inter Income Tax Classes – R.SOUMYANARAYANAN.FCA. GRAD CWA. Page-308
Chapter-11: PGBP

4. Disallowance U/s 40 (a) (iib):

1 What is disallowed Any amount paid by way of royalty, licence fee, service fee,
U/s 40 (a) (iib)? privilege fee, service charge, etc., which is levied exclusively on
a State Government undertaking, by the SG or
Amount appropriated, directly or indirectly, from a State
Government undertaking, by the SG.
2 Meaning of State (a) A corporation established by or under any Act of the SG;
Government (b) A company in which more than 50% of the paid up equity
undertaking. share capital is held by the SG;
(c) A company in which more than 50% of the paid up equity
share capital is held singly or jointly by (a) or (b);
(d) A company or corporation in which the SG has the right to
appoint the majority of directors or to control the management
or policy decisions;
(e) An authority, a board or an institution or a body established
or constituted by or under any Act of the SG or owned or
controlled by the SG.

5. Disallowance U/s 40 (a) (iii):

Any sum which is chargeable U/H `Salaries` if it is payable outside India or to a NR and if the
tax has not been paid thereon nor deducted therefrom under Chapter XVII-B shall be
disallowed.

6. Disallowance U/s 40 (a) (iv):

Any contribution to a provident fund or the fund established for the benefit of employees of
the assessee shall be disallowed, unless the assessee has made effective arrangements to
make sure that tax shall be deducted at source from any payments made from the fund
which are chargeable to tax U/H `Salaries`.

7. Disallowance U/s 40 (a) (v):


(i) Tax paid on non-monetary perquisites on behalf of employees is not deductible.
(ii) In case of an employee, deriving income in the nature of perquisites (other than
monetary payments), the amount of tax on such income paid by his employer is
exempt from tax in the hands of that employee.
(iii) Correspondingly, such payment is not allowed as deduction from the income of the
employer.
(iv) Thus, the payment of tax on perquisites by an employer on behalf of employee will be
exempt from tax in the hands of employee but will not be allowable as deduction in
the hands of the employer.

MT Educare –CA Inter Income Tax Classes – R.SOUMYANARAYANAN.FCA. GRAD CWA. Page-309
Chapter-11: PGBP

8. Disallowance of interest paid to partners U/s 40 (b):

If the following conditions are not fulfilled, interest paid to the partners shall not be allowed
as deduction in computing the business income of the firm:

1 Payment of interest to partners should be authorized by the partnership deed.


2 Payment of interest to partners should be in accordance with the terms of the
partnership deed.
3 Payment of interest should pertain to the period succeeding the date of authorisation
contained in the partnership deed.
4 Rate of interest should not exceed 12% p.a (computed on simple interest basis).

Partner in representative capacity and lender in individual capacity: (Explanation 1 to S. 40


(b)):

(i) Where an individual is a partner in a firm on behalf, or for the benefit, of any other
person, interest paid by the firm to such individual (otherwise than as partner in a
representative capacity), is not taken into account, for the purposes of S. 40 (b).
(ii) That means, the interest so paid shall be allowed as deduction in computing the
business income of the firm and the conditions laid down in S. 40 (b) shall not apply for
allowing it as deduction.

Partner in individual capacity and lender in representative capacity: (Explanation 2 to S. 40


(b)):

(i) Where an individual is a partner in a firm in his individual capacity, interest paid by the
firm to such individual will not be taken into account for the purpose of S. 40 (b), if such
interest is received by him on behalf or for the benefit of any other person.
(ii) That means, the interest so paid shall be allowed as deduction in computing the
business income of the firm and the conditions laid down in S. 40 (b) shall not apply for
allowing it as deduction.

9. Disallowance of remuneration paid to partners U/s 40 (b):

If the following conditions are not fulfilled, remuneration paid to the partners shall not be
allowed as deduction in computing the business income of the firm:
1 Payment of remuneration to partners should be authorized by the partnership deed.
2 Payment of remuneration to partners should be in accordance with the terms of the
partnership deed.
3 Payment of remuneration should pertain to the period succeeding the date of
authorisation contained in the partnership deed.
4 Remuneration shall be paid only to working partner.
5 Remuneration paid to all the partners put together shall not cross the ceiling.

MT Educare –CA Inter Income Tax Classes – R.SOUMYANARAYANAN.FCA. GRAD CWA. Page-310
Chapter-11: PGBP

Points requiring attention:

1 Meaning of working Working partner means an individual who is actively engaged


partner. in conducting the affairs of the business or profession of the
firm of which he is a partner. [Explanation 4 to S. 40 (b)].

2 Ceiling on remuneration
Book Profits Quantum of deduction
On the first Rs. 3L of book profit Rs. 1,50,000 or 90% of book profit,
or in case of loss whichever is higher
on the balance of book profit 60% of book profit
3 Meaning of book The net profit as shown in the profit and loss a/c for the
profits. relevant PY computed in accordance with the provisions for
computing income from profits and gains [Explanation 3 to S.
40 (b)].
The above amount should be increased by the remuneration
paid or payable to all the partners of the firm if the same has
been deducted while computing the net profit.

Q-3: A firm has paid Rs. 7,50,000 as remuneration to its partners for the PY 2019-20, in
accordance with its partnership deed, and it has a book profit of Rs. 10L. What is the
remuneration allowable as deduction?

Solution:

The allowable remuneration calculated as per the limits specified in S. 40 (b) (v) would be –

Particulars Rs.
On first Rs. 3L of book profit [Rs. 3,00,000 x 90%] 2,70,000
On balance Rs. 7L of book profit [Rs. 7,00,000 x 60%] 4,20,000
6,90,000

The excess amount of Rs. 60,000 (i.e., Rs. 7,50,000 – Rs. 6,90,000) would be disallowed as
per S. 40 (b) (v).

Q-4: Rao & Jain, a partnership firm consisting of two partners, reports a net profit of Rs.
7,00,000 before deduction of the following items:

(i) Salary of Rs. 20,000 each per month payable to two working partners of the firm (as
authorized by the deed of partnership).
(ii) Depreciation on plant and machinery U/s 32 (computed) Rs. 1,50,000.
(iii) Interest on capital at 15% per annum (as per the deed of partnership). The amount of
capital eligible for interest Rs. 5,00,000.
Compute:
(i) Book-profit of the firm U/s 40 (b).
(ii) Allowable working partner salary for the AY 2020-21 as per S. 40 (b).

MT Educare –CA Inter Income Tax Classes – R.SOUMYANARAYANAN.FCA. GRAD CWA. Page-311
Chapter-11: PGBP

Solution:

(i) As per Explanation 3 to S. 40 (b), "book profit" shall mean the net profit as per the profit
and loss account for the relevant PY computed in the manner laid down in Chapter IV-D as
increased by the aggregate amount of the remuneration paid or payable to the partners of
the firm if the same has been already deducted while computing the net profit.

In the present case, the net profit given is before deduction of depreciation on plant and
machinery, interest on capital of partners and salary to the working partners. Therefore, the
book profit shall be as follows:

Computation of Book Profit of the firm U/s 40 (b):

Particulars Rs. Rs.


Net Profit (before deduction of depreciation, salary and
interest) 7,00,000
Less: Depreciation U/s 32 1,50,000
Interest @ 12% p.a. [being the maximum allowable as per 60,000 2,10,000
S. 40 (b)] (Rs. 5,00,000 x 12%)
Book Profit 4,90,000

(ii) Salary actually paid to working partners = Rs. 20,000 x 2 x 12 = Rs. 4,80,000.

As per the provisions of S. 40 (b) (v), the salary paid to the working partners is allowed
subject to the following limits –

On the first Rs. 3,00,000 of book profit Rs. 1,50,000 or 90% of book profit,
or in case of loss whichever is more
On the balance of book profit 60% of the balance book profit

Therefore, the maximum allowable working partners` salary for the AY 2019-20 in this case
would be:

Particulars Rs.
On the first Rs. 3,00,000 of book profit [(Rs. 1,50,000 or 90% of Rs.
2,70,000
3,00,000) whichever is more]
On the balance of book profit [60% of (Rs. 4,90,000 – Rs. 3,00,000)] 1,14,000
Maximum allowable partners` salary 3,84,000
Hence, allowable working partners` salary for the AY 2020-21 as per the provisions of S. 40
(b) (v) is Rs. 3,84,000.

MT Educare –CA Inter Income Tax Classes – R.SOUMYANARAYANAN.FCA. GRAD CWA. Page-312
Chapter-11: PGBP

10. Disallowance of interest and remuneration paid by AOP or BOI to its members: S. 40
(ba):

1 What is disallowed U/s 40 (ba)? Any payment of interest, salary, commission, bonus or
remuneration made by an AOP or BOI to its members
will also not be allowed as a deduction in computing
the income of the AOP or BOI.
2 Net interest paid by AOP or BOI Where interest is paid by an AOP or BOI to a member
is to be disallowed. who has paid interest to the AOP/BOI, the amount of
[Explanation-1 to S. 40 (ba)]. interest to be disallowed U/s 40 (ba) shall be limited
to the net amount of interest paid by AOP/BOI to the
member.
3 Member in representative Where an individual is a member in an AOP/BOI on
capacity – lender in individual behalf, or for the benefit, of any other person,
capacity – No disallowance U/s interest paid by the AOP/BOI to such individual
40 (ba). [Explanation-2 to S. 40 (otherwise than as member in a representative
(ba)]. capacity), is not taken into account, for the purposes
of S. 40 (ba).
That means, the interest so paid shall be allowed as
deduction in computing the business income of the
AOP/BOI.
4 Member in individual capacity – Where an individual is a member in an AOP/BOI in his
lender in representative individual capacity, interest paid by the AOP/BOI to
capacity – No disallowance U/s such individual will not be taken into account for the
40 (ba). [Explanation-3 to S. 40 purpose of S. 40 (ba), if such interest is received by
(ba)]. him on behalf or for the benefit of any other person.
That means, the interest so paid shall be allowed as
deduction in computing the business income of the
AOP/BOI.

11. Disallowance U/s 40A (2):

1 What is disallowed Assessee incurs expenditure in connection with purchase of goods or


U/s 40A (2)? availing of services or facilities from specified persons.
The payment in this regard is excessive or unreasonable having regard
to (a) FMV of the commodity purchased or service availed; (b)
legitimate business needs of the assessee; (c) benefit derived by the
assessee as a result of this expenditure.
Such excessive or unreasonable portion of the payment is disallowed.
2 Meaning of specified See the table given below.
person.

MT Educare –CA Inter Income Tax Classes – R.SOUMYANARAYANAN.FCA. GRAD CWA. Page-313
Chapter-11: PGBP

Assessee Specified person Example


1. An Individual To a relative X purchases goods from his brother.
2. A company To a director of the company or any A, B and C are three directors of X
relative of the director Ltd. X Ltd. employs Mrs. A or Mrs. B
is paid for her tax advice by X Ltd.
3. A firm To a partner of the firm or a relative A and B are two partners of AB and
of the partners. Co., a firm. The firm purchases raw
material from the sister of B.
4. An AOP To a member of the association or a XY, an association of persons, has X
relative of the member. and Y as its two members. The
association purchases goods
manufactured by the son of X.
5. A HUF To a member of the family or relative X is the karta of X (HUF) with other
of such person. members being Y, Z and A. The family
appoints Mrs. A as selling agent.
6. Any assessee To an individual who has a X holds 20 per cent equity share
substantial interest in the business of capital in X Ltd. X Ltd. hires trucks
the taxpayer or a relative of such owned by the brother of X and pays
individual. rent.
7. Any assessee To a company which has a Y, Ltd. holds 20 per cent equity
substantial interest in the business of shares in X Ltd. A & B are directors of
the taxpayer, any director of such Y Ltd. X Ltd pays salary to Mrs. B.
company or a relative of such
director.
8. Any assessee To a firm/association of A and B are two partners of AB and
persons/Hindu undivided family who Co., a partnership firm. AB and Co.
has a substantial interest in the holds 20 per cent equity share capital
business of the taxpayer or in X Ltd. which appoints Mrs. B as
partner/member of such person or a software consultant.
relative of partner/member.
9. Any assessee To a company, one of whose A and B are directors of Y Ltd. A
directors has a substantial interest in holds 20 per cent equity share capital
the business of the taxpayer or in X Ltd. X Ltd. purchases material for
payment is made to any director of packaging from Mrs. B.
such company or any relative of such
director.

10. Any assessee To a firm/association of A and B are two partners of Y & Co.,
persons/HUF, one of whose a firm. B is 20 percent partner in X
partners/members has a substantial and Co., another firm. X and Co.
interest in business of the taxpayer appoints Mrs. A as its legal adviser
or payment is made to any other and pays fees.
partner/member of such
firm/association/HUF or any relative
of such person.

MT Educare –CA Inter Income Tax Classes – R.SOUMYANARAYANAN.FCA. GRAD CWA. Page-314
Chapter-11: PGBP

11. An individual
To a person in whose business the Mrs. X holds 20 per cent equity share
tax payer or any of his relative has a capital in Y Ltd. X purchases goods
substantial interest. from Y Ltd.
12. A company To a person in whose business the Mrs. A owns 20 per cent equity
tax payer/any director of capital in Y Ltd. A is a director in X
taxpayer/any relative of such Ltd., X Ltd. purchases raw material
director has a substantial interest. from Y Ltd.
13. A To a person in whose business the A’s brother B is a 30 per cent partner
Firm/AOP/HUF tax payer/any partner/member of in Y Co., a firm. A is a partner in X
the tax payer or any relative of such Co., another firm X Co. purchases
partner/member has a substantial goods from Y Co.
interest.

Meaning of relative:

Relative

Linear
Linear
Spouse Brother Sister diascendan
ascendant
t

Substantial interest:

Substantial interest

In company In any other concern

Beneficial ownership of Entitlement to atleast


equity share capital 20% of profits of the
carrying atleast 20% of concern at any time
voting power at any time during the previous year
during the previous year

12. Disallowance U/s 40A (3):

If the following conditions are fulfilled, an expenditure incurred by the assessee shall be
disallowed in computing the business income:

1 The assessee incurs an expenditure which is otherwise deductible in computing income


U/H PGBP.
2 The amount of expenditure exceeds Rs. 10000.
3 Payment in respect of this expenditure or part thereof in a day exceeds Rs. 10000.
4 Payment is made otherwise than by way of account payee cheque or account payee
bank draft or by use of ECS through a bank account.

Note 1: In case of payment made to transport operators for plying, hiring or leasing goods
carriages, the amount of Rs. 10000 should be replaced with Rs.35000

MT Educare –CA Inter Income Tax Classes – R.SOUMYANARAYANAN.FCA. GRAD CWA. Page-315
Chapter-11: PGBP

Note 2: However, Rule 6 DD provides circumstances under which there will be no


disallowance even if the payment exceeding Rs. 10000 is made otherwise than by way of
account payee cheque or account payee bank draft or by use of ECS through a bank
account.

Exceptions U/R 6DD:


1 Payments made to RBI
2 Payments made to a banking company.
3 Payments made to SBI or its subsidiaries.
4 Payments made to co-operative bank
5 Payments made to land mortgage bank
6 Payments made to any primary agricultural credit society
7 Payments made to LIC
8 Payments made to the Government and, under the rules framed by it, such payment is to be made
in legal tender.
9 Payment made by any letter of credit arrangement through a bank.
10 Payment made by a mail or telegraphic transfer through a bank.
11 Payment made by a bill of exchange made payable only to a bank.
12 Payment through the use of electronic clearing system through a bank account
13 Payment made through a credit card
14 Payment made through a debit card
15 Payment by way of adjustment against the amount of any liability incurred by the payee for any
goods supplied or services rendered by the assessee to payee.
16 Payment made for purchase of (a) agricultural or forest produce (b) the produce of animal
husbandry (including livestock, meat, hides and skins), dairy or poultry farming (c) fish or fish
products (d) the products of horticulture, to the cultivator, grower, or producer of such articles,
produce or products.
17 Payment made for the purchase of the products manufactured without the aid of power in a cottage
industry, to the producer of such products.
18 Payment in a village or town, which on the date of payment is not served by any bank, to any person
who ordinarily resides, or is carrying on any business or profession or vocation, in any such village or
town.
19 Payment made to an employee of the assessee or the heir of any such employee, on or in
connection with the retirement, retrenchment, resignation, discharge or death of such employee,
on account of gratuity, retrenchment compensation or similar terminal benefit and the aggregate of
such sums payable to the employee or his heir does not exceed Rs. 50000.
20 Payment made by an assessee by way of salary to his employee after deducting the income tax from
salary in accordance with S. 192 and when such employee-
(i) is temporarily posted for a continuous period of 15 days or more in a place other than his
normal place of duty or on a ship and
(ii) does not maintain any account in any bank at such place or ship.
21 Payment required to be made on a day on which the banks were closed either on account of holiday or strike.
22 Payment made by any person to his agent who is required to make payment in cash for goods or
services on behalf of such person.
23 Payment made by an authorized dealer or a money changer against purchase of foreign currency or
traveler’s cheque in the normal course of his business.

MT Educare –CA Inter Income Tax Classes – R.SOUMYANARAYANAN.FCA. GRAD CWA. Page-316
Chapter-11: PGBP

Note-3:

1 The expression `fish or fish products` referred to above would include `other marine
products such as shrimp, prawn, cuttlefish, squid, crab, lobster etc.`.
2 The `producers` of fish or fish products for the purpose of R. 6DD would include, besides
the fishermen, any headman of fishermen, who sorts the catch of fish brought by
fishermen from the sea, at the sea shore itself and then sells the fish or fish products to
traders, exporters etc.
3 However, the above exception will not be available on the payment for the purchase of
fish or fish products from a person who is not proved to be a `producer` of these goods
and is only a trader, broker or any other middleman, by whatever name called.

Cash payment made in excess of Rs.10,000 deemed to be the income of the subsequent
year, if an expenditure has been allowed as deduction in any PY on due basis: [S. 40A
(3A)].

In case of an assessee following mercantile system of accounting, if an expenditure has been


allowed as deduction in any PY on due basis, and payment has been made in a subsequent
year otherwise than by account payee cheque or account payee bank draft or use of ECS
through a bank account, then the payment so made shall be deemed to be the income of
the subsequent year if such payment or aggregate of payments made to a person in a day
exceeds Rs. 10,000 [S. 40A (3A)].

13. Disallowance U/s 40A (9):

1 Where an assessee (being an employer) contributes towards recognized provident fund /


approved superannuation fund / approved gratuity fund/ approved pension fund, it is
deductible subject to S. 36 (1) and S. 43B.
2 But where the assessee as an employer makes any payment for setting up any trust,
company, association or society, or contributes to any fund other than those referred to
above, and such contribution or payment is not required by the law, then such payment
or contribution is not allowed as deduction.

MT Educare –CA Inter Income Tax Classes – R.SOUMYANARAYANAN.FCA. GRAD CWA. Page-317
Chapter-11: PGBP

14. Disallowance U/s 43B:

The following sums are allowed as deduction only on the basis of actual payment within the
time limits specified in section 43B:

(a) Any sum payable by way of tax, duty, cess or fee, by whatever name called, under any
law for the time being in force.
(b) Any sum payable by the assessee as an employer by way of contribution to any
provident fund or superannuation fund or gratuity fund or any other fund for the
welfare of employees.
(c) Bonus or Commission for services rendered payable to employees.
(d) Any sum payable by the assessee as interest on any loan or borrowing from a deposit taking
non-banking financial company or systemically important non-deposit taking non-banking
financial company, in accordance with the terms and conditions of the agreement governing
such loan or borrowing (Refer Note)
(e) Any sum payable by the assessee as interest on any loan or borrowing from any PFI or
a State Financial Corporation or a State Industrial Investment Corporation.
(f) Interest on any loan or advance from a scheduled bank or co-opertive bank other than
a primary agricultural credit society or a primary co-operative agricultural and rural
development bank.
(g) Any sum paid by the assessee as an employer in lieu of earned leave of his employee.
(h) Any sum payable by the assessee to the Indian Railways for use of Railway assets.
The above sums can be paid by the assessee on or before the due date for furnishing the
ROI U/s 139 (1) in respect of the PY in which the liability to pay such sum was incurred.

Note: -
NBFC: The provision has been extended to any sum payable by the assessee as
interest on any loan or borrowing from a deposit taking non-banking financial
company or systemically important non-deposit taking non-banking financial
company, in accordance with the terms and conditions of the agreement governing
such loan or borrowing.
Systemically important non-deposit taking non-banking financial company means a
non-banking financial company which is not accepting or holding public deposits and
having total assets of not less than ` 500 crore as per the last audited balance sheet and
is registered with the RBI.
Deposit taking non-banking financial company means a non-banking financial
company which is accepting or holding public deposits and is registered with the RBI.

Conversion of interest into a loan or borrowing or advance or payable in other manner:

1 Explanation 3C & 3D clarifies that if any sum payable by the assessee as interest on any
such loan or borrowing or advance referred to in (d) and (e) above, is converted into a
loan or borrowing or advance, the interest so converted and not "actually paid" shall not
be deemed as actual payment, and hence would not be allowed as deduction. The
clarificatory explanations only reiterate the rationale that conversion of interest into a
loan or borrowing or advance does not amount to actual payment.

MT Educare –CA Inter Income Tax Classes – R.SOUMYANARAYANAN.FCA. GRAD CWA. Page-318
Chapter-11: PGBP

2 The manner in which the converted interest will be allowed as deduction has been
clarified in Circular No.7/2006 dated 17.7.2006. The unpaid interest, whenever actually
paid to the bank or financial institution, will be in the nature of revenue expenditure
deserving deduction in the computation of income. Therefore, irrespective of the
nomenclature, the deduction will be allowed in the PY in which the converted interest is
actually paid.

Q-: Hari, an individual, carried on the business of purchase and sale of agricultural
commodities like paddy, wheat, etc. He borrowed loans from Andhra Pradesh State
Financial Corporation (APSFC) and Indian Bank and has not paid interest as detailed
hereunder:

(i) Andhra Pradesh State Financial Corporation (PY 2018-19 & 2019-20) 15,00,000
(ii) Indian Bank (PY 2019-20) 30,00,000
45,00,000

Both APSFC and Indian Bank, while restructuring the loan facilities of Hari during the year
2019-20, converted the above interest payable by Hari to them as a loan repayable in 60
equal installments. During the year ended 31.3.2020, Hari paid 5 installments to APSFC and
3 installments to Indian Bank.

Q.4 Hari claimed the entire interest of Rs. 45,00,000 as an expenditure while computing the
income from business of purchase and sale of agricultural commodities. Discuss whether his
claim is valid and if not what is the amount of interest, if any, allowable.

Solution:

According to S. 43B, any interest payable on the term loans to specified financial institutions
and any interest payable on any loans and advances to scheduled banks shall be allowed
only in the year of payment of such interest irrespective of the method of accounting
followed by the assessee. Where there is default in the payment of interest by the assessee,
such unpaid interest may be converted into loan. Such conversion of unpaid interest into
loan shall not be construed as payment of interest for the purpose of S. 43B.

The amount of unpaid interest so converted as loan shall be allowed as deduction only in
the year in which the converted loan is actually paid.

In the given case of Hari, the unpaid interest of Rs. 15,00,000 due to APSFC and of Rs.
30,00,000 due to Indian Bank was converted into loan. Such conversion would not amount
to payment of interest and would not, therefore, be eligible for deduction in the year of
such conversion. Hence, claim of Hari that the entire interest of Rs. 45,00,000 is to be
allowed as deduction in the year of conversion is not tenable. The deduction shall be
allowed only to the extent of repayment made during the financial year. Accordingly, the
amount of interest eligible for deduction for the AY 2020-21 shall be calculated as follows:

Interest Number of Amount per Installments Interest


outstanding Installments installment paid allowable

MT Educare –CA Inter Income Tax Classes – R.SOUMYANARAYANAN.FCA. GRAD CWA. Page-319
Chapter-11: PGBP

(Rs.) (Rs.) (Rs.)


APSFC 15L 60 25,000 5 1,25,000
Indian
30L 60 50,000 3 1,50,000
Bank
Total amount eligible for deduction 2,75,000

E. Changes in rate of exchange of currency – S. 43A:

(1) The section provides that where an assessee has acquired any asset from a foreign
country for the purpose of his business or profession, and due to a change thereafter in the
exchange rate of the two currencies involved, there is an increase or decrease in the liability
(expressed in Indian rupees) of the assessee at the time of making the payment, the
following values may be changed accordingly with respect to the increase or decrease in
such liability:

(i) the actual cost of the asset U/s 43 (1)


(ii) the amount of capital expenditure incurred on scientific research U/s 35 (1) (iv).
(iii) the amount of capital expenditure incurred by a company for promoting family
planning amongst its employees U/s 36 (1) (ix).
(iv) the cost of acquisition of a non-depreciable capital asset falling U/s 48.

The amount arrived at after making the above adjustment shall be taken as the amount of
capital expenditure or the cost of acquisition of the capital asset, as the case may be.

(2) Where the whole or any part of the liability aforesaid is met, not by the assessee, but,
directly or indirectly, by any other person or authority, the liability so met shall not be taken
into account for the purposes of this section.

E. Presumptive taxation:

I. Special provisions for computing profits of business on presumptive basis – S. 44AD:

1 Who is eligible to come U/s Resident individuals, HUFs and partnership firms (but not
44AD for getting taxed on LLPs) who have not claimed deduction U/s 10AA and under
presumptive basis? Chapter VIA Part-C in the relevant AY.
2 Who are not eligible for A person carrying on profession as referred to in S. 44AA (1) i.e.,
coming/s 44AD? legal, medical, engineering or architectural profession or the
profession of accountancy or technical consultancy or interior
decoration or any other profession as is notified by the Board
(namely, authorized representatives, film artists, company
secretaries and profession of information technology);
A person earning income in the nature of commission or
brokerage;
A person carrying on any agency business.
3 Which income is subject to Profits and gains from eligible business.
presumptive taxation U/s
44AD?

MT Educare –CA Inter Income Tax Classes – R.SOUMYANARAYANAN.FCA. GRAD CWA. Page-320
Chapter-11: PGBP

4 What do we mean by eligible Any business (except the business of plying, hiring and
business? leasing goods carriages covered U/s 44AE) with total
turnover/gross receipts of up to Rs. 200L.
5 How do we compute profits By applying 6% on that portion of the turnover from eligible
and gains from the eligible business which is received by way of APC/APBD/by use of
business on a presumptive ECS through bank account during the PY or before the due
basis? date for filing return U/s 139 (1) for that PY and by applying
8% on the remaining portion of the turnover from the
eligible business.
6 No further deduction would All deductions allowable U/s 30 to 38 shall be deemed to
be allowed. have been allowed in full and no further deduction shall be
allowed.
7 WDV of the assets to be The WDV of any asset of eligible business shall be
updated as if depreciation calculated as if the assessee had claimed and had been
was allowed. actually allowed deduction in respect of depreciation for
each of the relevant AYs.
8 Relief from maintenance of Assessees opting for the presumptive scheme U/s 44AD are
books of accounts and audit. not required to maintain books of account U/s 44AA or get
them audited U/s 44AB.
9 Higher threshold for non- S. 44AB makes it obligatory for every person carrying on
audit of accounts for business to get his accounts of any PY audited if his total
assessees opting for sales, turnover or gross receipts exceed Rs. 100L. However,
presumptive taxation U/s if an eligible person opts for presumptive taxation scheme
44AD. as per S. 44AD (1), he shall not be required to get his
accounts audited if the total turnover or gross receipts of
the relevant PY does not exceed Rs. 200L .
10 Advance tax obligation. (xi) The eligible assessee is required to pay advance tax by 15th
March of the FY.
(xii)
(xiii) Where an eligible assessee declares profit for any PY in
11 Not eligible to opt for
presumptive taxation U/s accordance with the provisions of this section and he
44AD for 5 AYs. declares profit for any of the 5 consecutive AYs relevant to
the PY succeeding such PY not in accordance with the
provisions of S. 44AD (1), he shall not be eligible to claim
the benefit of the provisions of this section for 5 AYs
subsequent to the AY relevant to the PY in which the profit
has not been declared in accordance with the provisions of
S. 44AD (1). [S. 44AD (4)]. (See illustration given below).
(xiv)
(x) An eligible assessee to whom the provisions of S. 44AD (4)
12 Books of accounts and Audit
if S. 44AD (4) attracted. are applicable and whose TI exceeds the BEL has to
maintain books of account U/s 44AA and get them audited
and furnish a report of such audit U/s 44AB. [S. 44AD (5)].
(xi)

MT Educare –CA Inter Income Tax Classes – R.SOUMYANARAYANAN.FCA. GRAD CWA. Page-321
Chapter-11: PGBP

I. Presumptive taxation scheme for assessees engaged in eligible profession – S. 44ADA:

1 Who is eligible to come A resident assessee who is engaged in any profession


U/s 44ADA for getting referred to in S. 44AA (1) such as legal, medical, engineering
taxed on presumptive or architectural profession or the profession of accountancy
basis? or technical consultancy or interior decoration or any other
profession as is notified by the Board in the Official Gazette;
and whose total gross receipts does not exceed Rs. 50L in a
PY.
2 How do we compute By applying 50% on the total gross receipts.
profits and gains from
the eligible profession
on a presumptive basis?
3 No further deduction All deductions allowable U/s 30 to 38 shall be deemed to
would be allowed. have been allowed in full and no further deduction shall be
allowed.
4 WDV of the assets to be The WDV of any asset of eligible profession shall be
updated as if calculated as if the assessee had claimed and had been
depreciation was actually allowed deduction in respect of depreciation for each
allowed. of the relevant AYs.
5 Relief from maintenance Assessees opting for the presumptive scheme U/s 44ADA are
of books of accounts not required to maintain books of account U/s 44AA or get
and audit. them audited U/s 44AB.
6 Option to claim lower An assessee may claim that his profits and gains from the
profits. aforesaid profession are lower than the profits and gains
deemed to be his income U/s 44ADA (1); and if such TI
exceeds the BEL, he has to maintain books of account U/s
44AA and get them audited and furnish a report of such audit
U/s 44AB.
7 Advance tax obligation. (xv) The eligible assessee is required to pay advance tax by
15th March of the FY.

III. Special provisions for computing profits and gains of business of plying, hiring or
leasing goods carriages: S. 44AE:

1 Who is eligible to come Assessee who carries on the business of plying, hiring or
U/s 44AE for getting leasing goods carriages and who does not own more than 10
taxed on presumptive goods vehicles at any time during the PY.
basis?
2 How do we compute In case of heavy goods vehicle (that is, any goods carriage,
profits and gains from the gross vehicle weight of which exceeds 12000 kilograms),
this business on a the income is Rs. 1000 per tonne of gross vehicle weight for
presumptive basis? every month or part of a month during which such vehicle is
owned by the assessee for the PY.
In case of other vehicles, Rs. 7,500 for every month or part of
a month during which such vehicle is owned by the assessee
for the PY.

MT Educare –CA Inter Income Tax Classes – R.SOUMYANARAYANAN.FCA. GRAD CWA. Page-322
Chapter-11: PGBP

3 No further deduction All deductions allowable U/s 30 to 38 shall be deemed to


would be allowed. have been allowed in full and no further deduction shall be
allowed.
However, in the case of a firm, salary and interest would be
allowed as deduction subject to the conditions and limits
prescribed U/s 40 (b).
4 WDV of the assets to be The WDV of any asset of this business shall be calculated as if
updated as if the assessee had claimed and had been actually allowed
depreciation was deduction in respect of depreciation for each of the relevant
allowed. AYs.
5 Relief from maintenance Assessees opting for the presumptive scheme U/s 44AE are
of books of accounts and not required to maintain books of account U/s 44AA or get
audit. them audited U/s 44AB.
6 Option to claim lower An assessee may claim that his profits and gains from the
profits. aforesaid business are lower than the profits and gains
deemed to be his income U/s 44AE; and if such TI exceeds
the BEL, he has to maintain books of account U/s 44AA and
get them audited and furnish a report of such audit U/s 44AB.

F. Compulsory maintenance of accounts: [S. 44AA]:

S. 44AA Maintain the books of This section provides that every person carrying on the
(1) account and other legal, medical, engineering or architectural profession
documents by notified or accountancy or technical consultancy or interior
profession decoration or any other profession as has been notified
by the CBDT in the OG must statutorily maintain such
books of accounts and other documents as may enable
the AO to compute his TI in accordance with the
provisions of the Act.
Notified professions The professions notified so far are (a) the profession of
authorised representative; (b) the profession of film
artist (actor, camera man, director, music director, art
director, dance director, editor, singer, lyricist, story
writer, screen play writer, dialogue writer and dress
designer); (c) the profession of Company Secretary; and
(d) profession of information technology.
S. 44AA Maintain the books of Every person carrying on business or profession (other
(2) account and other than the professions specified above) must statutorily
documents if income / maintain such books of accounts and other documents
sales / turnover /gross as may enable the AO to compute his TI in accordance
receipts exceeds the with the provisions of the Act in the specified
prescribed limits. circumstances.
Specified Existing In cases where the income from the
circumstances (in case business or existing business or profession
of individual or HUF) profession exceeds Rs. 2,50,000 or the total sales
turnover or gross receipts, as the case
may be, in the business or profession

MT Educare –CA Inter Income Tax Classes – R.SOUMYANARAYANAN.FCA. GRAD CWA. Page-323
Chapter-11: PGBP

exceed Rs. 25,00,000 in any one of 3


years immediately preceding the PY;
or
Newly set In cases where the business or
up business profession is newly set up in any PY, if
or his income from business or
profession profession is likely to exceed Rs.
2,50,000 or his total sales turnover or
gross receipts, as the case may be, in
the business or profession are likely to
exceed Rs. 25,00,000 during the PY.
Specified Existing In cases where the income from the
circumstances (in case business or existing business or profession
of persons other than profession exceeds Rs. 1,20,000 or the total sales
individual or HUF) turnover or gross receipts, as the case
may be, in the business or profession
exceed Rs. 10,00,000 in any one of 3
years immediately preceding the PY;
or
Newly set In cases where the business or
up business profession is newly set up in any PY, if
or his income from business or
profession profession is likely to exceed Rs.
1,20,000 or his total sales turnover or
gross receipts, as the case may be, in
the business or profession are likely to
exceed Rs. 10,00,000 during the PY.
Showing lower income Where profits and gains from the business are
as compared to calculated on a presumptive basis U/s 44AE and the
income computed on assessee has claimed that his income is lower than the
presumptive basis U/s profits or gains so deemed to be the profits and gains of
44AE his business.
Where the provisions In cases where an assessee becomes ineligible to claim
of S. 44AD (4) are the benefit of the provisions of S. 44AD (1) for 5 AYs
applicable in his case subsequent to the AY relevant to the PY in which the
and his income > BEL profit has not been declared in accordance with the
in any PY. provisions of S. 44AD (1) and his income > BEL during
the PY.
S. 44AA Power to prescribe The CBDT has been authorised, having due regard to
(3) books of accounts and the nature of the business or profession carried on by
other documents to be any class of persons, to prescribe by rules the books of
kept. account and other documents including inventories,
wherever necessary, to be kept and maintained by the
taxpayer, the particulars to be contained therein and
the form and manner in which and the place at which
they must be kept and maintained.

MT Educare –CA Inter Income Tax Classes – R.SOUMYANARAYANAN.FCA. GRAD CWA. Page-324
Chapter-11: PGBP

S. 44AA Period for which the The CBDT has also been empowered to prescribe, by
(4) books of account and rules, the period for which the books of account and
other documents are other documents are required to be kept and
required to be kept maintained by the taxpayer.
and maintained

Points requiring attention:

1 Rules pertaining to maintenance of books of accounts & other documents – R. 6F (1):


(a) R. 6F contains the details relating to the books of account and other documents to be
maintained by certain professionals U/s 44AA.
(b) As per R. 6F, every person carrying on legal, medical, engineering, or architectural
profession or the profession of accountancy or technical consultancy or interior
decoration or authorised representative or film artist shall keep and maintain the
specified books of account and other documents in the following cases :

(a) if his gross receipts exceed Rs. 1,50,000 in all the 3 years immediately
preceding the PY; or
(b) if, where the profession has been newly set up in the PY, his gross receipts are
likely to exceed Rs. 1,50,000 in that year.
(c) It should be noted that professionals whose gross receipts are less than the specified
limits given above are also required to maintain books of account but these have not
been specified in the Rule.
(d) In other words, they are required to maintain such books of account and other
documents as may enable the AO to compute the total income in accordance with the
provisions of this Act.
2 Prescribed books of accounts and other documents – R. 6F (2):
(i) The following books of account and other documents are required to be maintained:
(a) a cash book;
(b) a journal, if accounts are maintained on mercantile basis ;
(c) a ledger;
(d) Carbon copies of bills and receipts issued by the person whether machine
numbered or otherwise serially numbered, in relation to sums exceeding Rs.
25;
(ii) Original bills and receipts issued to the person in respect of expenditure incurred by
the person, or where such bills and receipts are not issued, payment vouchers
prepared and signed by the person, provided the amount does not exceed Rs. 50.
Where the cash book contains adequate particulars, the preparation and signing of
payment vouchers is not required.
(iii) In case of a person carrying on medical profession, he will be required to maintain the
following in addition to the list given above:
(a) a daily case register in Form 3C.
(b) an inventory under broad heads of the stock of drugs, medicines and other
consumable accessories as on the first and last day of the previous year used
for his profession.

MT Educare –CA Inter Income Tax Classes – R.SOUMYANARAYANAN.FCA. GRAD CWA. Page-325
Chapter-11: PGBP

3 Place at which books to be kept and maintained: The books and documents shall be
kept and maintained at the place where the person is carrying on the profession, or
where there is more than one place, at the principal place of his profession. However, if
he maintains separate set of books for each place of his profession, such books and
documents may be kept and maintained at the respective places.
4 Prescribed period: The above books of account and documents shall be kept and
maintained for a minimum of 6 years from the end of the relevant AY.

Illustration:

Vinod is a person carrying on profession as film artist. His gross receipts from profession are
as under:

Financial year 2016-17 Rs. 115000


Financial year 2017-18 Rs. 180000
Financial year 2018-19 Rs. 210000

What is his obligation regarding maintenance of books of accounts for AY 2020-21 U/s
44AA?
Solution:

S. 44AA (1) requires every person carrying on any profession, notified by the Board in the
Official Gazette (in addition to the professions already specified therein), to maintain such
books of account and other documents as may enable the AO to compute his total income
in accordance with the provisions of the Income-tax Act, 1961.

A person carrying on a notified profession shall be required to maintain specified books of


accounts, only if:

(i) his gross receipts in all the three years immediately preceding the relevant PY has
exceeded Rs. 1,50,000; Or
(ii) it is a new profession which is setup in the relevant PY, it is likely to exceed Rs. 1,50,000
in that PY.

In the present case, Vinod is a person carrying on profession as film artist, which is a notified
profession. Since his gross receipts have not exceeded Rs. 1,50,000 in FY 2015-16, the
requirement U/s 44AA to compulsorily maintain the prescribed books of account is not
applicable to him. Mr. Vinod, however, required to maintain such books of accounts as
would enable the AO to compute his TI.

MT Educare –CA Inter Income Tax Classes – R.SOUMYANARAYANAN.FCA. GRAD CWA. Page-326
Chapter-11: PGBP

G. Audit of accounts of certain persons carrying on business or profession – S. 44AB:

(i) Who are required to get the accounts audited?


It is obligatory in the following cases for a person carrying on business or profession to
get his accounts audited before the "specified date" by a CA:
1 if the total sales, turnover or gross receipts in business exceeds Rs. 100L in any
PY; or
2 if the gross receipts in profession exceeds Rs. 50L in any PY; or
3 where the assessee is covered U/s 44AE and claims that the profits and gains
from business are lower than the profits and gains computed on a presumptive
basis. In such cases, the normal monetary limits for tax audit in respect of
business would not apply.
4 where the assessee is carrying on a notified profession U/s 44AA, and he claims
that the profits and gains from such profession are lower than the profits and
gains computed on a presumptive basis U/s 44ADA and his income > the BEL.
5 where the assessee is covered U/s 44AD (4) and his income exceeds the BEL.
(ii) Audit Report:
The person mentioned above would have to furnish by the specified date a report of
the audit in the prescribed forms. For this purpose, the Board has prescribed U/R 6G,
Forms 3CA/3CB/3CD containing forms of audit report and particulars to be furnished
therewith.
(iii) Accounts audits under other statutes are considered:
In cases where the accounts of a person are required to be audited by or under any
other law before the specified date, it will be sufficient if the person gets his accounts
audited under such other law before the specified date and also furnish by the said
date the report of audit in the prescribed form in addition to the report of audit
required under such other law.
Thus, for example, the provision regarding compulsory audit does not imply a second
or separate audit of accounts of companies whose accounts are already required to be
audited under the Companies Act, 2013. The provision only requires that companies
should get their accounts audited under the Companies Act, 2013 before the specified
date and in addition to the report required to be given by the auditor under the
Companies Act, 2013 furnish a report for tax purposes in the form to be prescribed in
this behalf by the CBDT.
(iv) Non-applicability:
The requirement of audit U/s 44AB does not apply to a person who declares profits
and gains on a presumptive basis U/s 44AD and his total sales, turnover or gross
receipts does not exceed Rs. 2 crore.
(v) Specified date:
The expression “specified date” in relation to the accounts of the PY (s) relevant to any
AY means the due date for furnishing the ROI U/s 139 (1).
(vi) Penal provision:
It may be noted that U/s 271B, penal action can be taken for not getting the accounts
audited and for not filing the audit report by the specified date.

MT Educare –CA Inter Income Tax Classes – R.SOUMYANARAYANAN.FCA. GRAD CWA. Page-327
Chapter-11: PGBP

MT Educare –CA Inter Income Tax Classes – R.SOUMYANARAYANAN.FCA. GRAD CWA. Page-328
Chapter-12: Filing of returns

A. Filing of returns – S. 139:

1. Voluntary return – S. 139 (1):

1 Filing of ROI by firms For each PY, every firm and company shall furnish ROI.
and companies – It shall be in the prescribed form. [Vide R. 12].
Mandatory. [S. 139 (1) It shall be verified in the prescribed manner.
(a)] It shall set forth prescribed particulars.
It shall be furnished within the due date specified in
Explanation-2 to S. 139 (1).
2 Mandatory filing of Any person other than a company or a firm, who is not
loss return by firms required to furnish a return under section 139(1), is required
and companies. to file income-tax return in the prescribed form and manner
[Proviso-3 to S. 139 on or before the due date if, during the previous year, such
(1)]. person –
a) has deposited an amount or aggregate of the amounts
exceeding ` 1 crore in one or more current accounts
maintained with a banking company or a co-operative bank;
or
b) has incurred expenditure of an amount or aggregate of
the amounts exceeding ` 2 lakh for himself or any other
person for travel to a foreign country; or
c) has incurred expenditure of an amount or aggregate of
the amounts exceeding ` 1 lakh towards consumption of
electricity; or
d) fulfils such other prescribed conditions.
3 Filing of ROI by other Every other person has obligation to file ROI only if the TI of the
persons. [S. 139 (1) PY > BEL.
(b)]. In such a case, ROI shall be filed in the prescribed form.
It shall be verified in the prescribed manner.
It shall set forth prescribed particulars.
It shall be furnished within the due date specified in
Explanation-2 to S. 139 (1).
4 Total income for the TI before giving effect to the provisions of Chapter VI-A or
purpose of S. 139 (1) Section 54 related exceptions under Capital Gains(Sec
(b). [Proviso-6 to S. 54,/54B/54D/54EC/54F).
139 (1)].
5 Due date for filing Assessees who have entered into 30th November of
return. [Explanation-2 international transactions or specified the AY
to S. 139 (1)]. domestic transactions.
Other assessees being (a) companies; (b) 30th September of
auditees; (c) working partner of an the AY
auditee-firm
Other assessees 31st July of the AY

MT Educare –CA Inter Income Tax Classes– R. SOUMYANARAYANAN. FCA. GRAD CWA. Page-329
Chapter-12: Filing of returns

Consequences of not filing return within the due date:


1. Interest U/s 234A
1 Circumstances in Return was filed beyond the time limit specified in S. 139 (1).
which interest is
levied U/s 234A.
2 Computation of Base for Tax on returned income – TDS – TCS –
interest computation of Advance tax – Self assessment tax paid
interest before the due date for filing return.
[Part of Rs. 100 ignored].
Interest rate 1% per month or part thereof.
Period for which Date immediately following the due date
interest is to the date of furnishing of return.
computed
2. Loss of tax holidays – S. 80AC
Provisions of S. 80AC No deduction is available under Chapter-VIA Part C, if the return
is not filed within the due date specified in Explanation-2 to S.
139 (1).
3. Fees U/s 234F
1 When it is levied? Where a person required to furnish ROI U/s 139, fails to do so
within the due date specified in Explanation-2 to S. 139 (1), he
shall pay fees U/s 234F.
2 Quantum of fees Situation-1 Return is filed beyond the due date but Rs.
payable. st
within 31 December of the AY. 5000
Situation-2 Return is filed beyond 31st December of Rs.
the AY. 10000
If the total income of the assessee does not exceed Rs. 5L, the
fee payable U/s 234F shall not exceed Rs. 1000.
3 Applicability of S. Provisions of S. 234F shall apply in respect of ROI required to be
234F. furnished for the AY 2018-19 and subsequent AYs.
4. Prosecution U/s 276CC
1 When S. 276CC shall Assessee having obligation to file return U/s 139 hasn’t filed
apply? return within the due date wilfully.
2 Consequence of Assessee may be punished with rigorous imprisonment and fine.
willful default.
3 Term of Situation-1 Tax that would have been Minimum
imprisonment evaded if the failure had not term = 6
been discovered > Rs. 25L months.
Maximum
term = 7
years.
Situation-2 Tax that would have been Minimum
evaded if the failure had not term = 3
been discovered does not Rs. months.
25L Maximum
term = 2
years.

MT Educare –CA Inter Income Tax Classes– R. SOUMYANARAYANAN. FCA. GRAD CWA. Page-330
Chapter-12: Filing of returns

5 Escape route (i.e. No ROI was filed before the end of the relevant AY.
prosecution U/s (Tax on assessed income – TDS – Advance tax) ≤ Rs. 3000.
276CC).

2. Filing of returns by beneficial owners of, or beneficiaries in, foreign assets: Proviso-4
and Proviso-5 to S. 139 (1):

1 Persons who have Resident and ordinarily resident who has no obligation to file
obligation to file return U/s 139 (1) and who:
return by virtue of (i) holds as a beneficial owner or otherwise, any asset
Proviso-4 to S. 139 (1) (including any financial interest in any entity) located outside
India; or
(ii) has a signing authority in any account located outside India;
(iii) is a beneficiary of any asset (including any financial interest
in any entity) located outside India.
shall file return in prescribed form, verified in prescribed
manner and setting forth prescribed particulars within the due
date specified in Expalantion-2 to S. 139 (1) (irrespective of
whether he has reported income or loss in the relevant PY).
2 Relaxation from filing An individual being a beneficiary of an asset outside India need
of return in respect of not file return by virtue of Proviso-4 to S. 139 (1), if the income
beneficiary. [Proviso-5 arising from such asset is includible in the income of the owner
to S. 139 (1)]. of the asset.
3 Beneficial owner. Beneficial owner (in respect of an asset) means an individual
[Expalantion-4 to S. who has provided, directly or indirectly, consideration for the
139 (1)]. assets for the immediate or future benefit, direct or indirect, of
himself or any other person.
4 Beneficiary. Beneficiary (in respect of an asset) means an individual who
[Explanation-5 to s. derives benefit from the asset during the PY and the
139 (1)]. consideration for such asset has been provided by any person
other than such beneficiary.

3. Loss returns – S. 139 (3):

1 Filing of return U/s 139 (3) Where assessee has sustained loss U/H PGBP or U/H CG
and wants such loss to be carried forward to the next AY
U/s 72, S. 73, S. 73A, S. 74 or S. 74A, he shall file a return
of loss U/s 139 (3) within the due date specified in
Explanation-2 to S. 139 (1).
2 Consequence of non-compliance Where a loss return referred to in S. 139 (3) is not filed
with S. 139 (3). [S. 80]. within the due date (supra), then the assessee shall not
be entitled to carry forward losses U/s 72, S. 73, S. 73A,
S. 74 or S. 74A.
S. 80 supersedes provisions of Chapter-VI (i.e. S. 70 – S.
80).
3 Carry forward and set off If a loss return is filed belatedly, carry forward and set
prohibited U/s 80 and not set off. off is only prohibited U/s 80 and not set off.

MT Educare –CA Inter Income Tax Classes– R. SOUMYANARAYANAN. FCA. GRAD CWA. Page-331
Chapter-12: Filing of returns

4 Certain carry forwards not Belated filing of loss return shall not affect the following:
prohibited U/s 80 1 Carry forward of unabsorbed loss U/H IFHP U/s
71B.
2 Carry forward of unabsorbed depreciation U/s 32
(2). [Govind Nagar Sagar Ltd (Del) + East Asiatic
Company India (P) Ltd (Mad)].
3 Carry forward of unabsorbed capital expenditure
on scientific research U/s 35 (4) and S. 32 (2).
4 Carry forward of unabsorbed expenditure on
promotion of family planning amongst employees
U/s 36 (1) (ix) Proviso-2 and S. 32 (2).
5 Filing loss return beyond the due S. 119 (2) (a) empowers the CBDT to relax a list of
date but within the time extended provisions which inter-alia includes S. 139.
by CBDT – will it affect carry In order to avoid stringency of law, the CBDT, in exercise
forward of loss? of this power, extends the due date for filing ROI.
If the loss return is furnished beyond the due date
referred to in Explanation-2 to S. 139 (1), however,
furnished within the time extended by the CBDT, it shall
be regarded as one filed within S. 139 (1) time limit.
The benefit of carry forward is unaffected.

4. Belated return – S. 139 (4):

1 Filing of belated Any person who has not furnished a return within the time allowed
return to him U/s 139 (1) may furnish the return for any PY at any time
before the end of the relevant AY or before the completion of
assessment, whichever is earlier.
2 Assessment = Best judgement assessment U/s 144.
referred to in S.
139 (4)

5. Revised return – S. 139 (5):

1 When a revised If a person having furnished a return U/s 139 (1) or S. 139 (4)
return could be discovers any omission or wrong statement therein, then he may
filed? furnish a revised return at any time before the end of the relevant
AY or before the completion of assessment, whichever is earlier.
2 Assessment = Scrutiny assessment U/s 143 (3) or Best judgement assessment U/s
referred to in S. 144.
139 (5)

MT Educare –CA Inter Income Tax Classes– R. SOUMYANARAYANAN. FCA. GRAD CWA. Page-332
Chapter-12: Filing of returns

Issues in revised return:

1 Revised return steps into the Revised return steps into the shoes of original return.
shoes of original return. It substitutes the original return since inception.
If the original return was filed within S. 139 (1) time
limit, then the revised return is also deemed to have
been filed within S. 139 (1) time limit.
Thus, if the assessee offers loss through the revised
return, such revised return becomes a return U/s 139
(3) and such loss could be carried forward. [Dhampur
sugar mills Ltd (All)].
2 Whether a loss return filed If a loss return is filed U/s 139 (3), all the provisions of
U/s 139 (3) could be revised? the Act shall apply to such return as if it were a return
required to be furnished U/s 139 (1).
Therefore, if the original return was a loss return filed
U/s 139 (3), even such return could be revised U/s 139
(5). [Periyar district co-operative milk producer union
Ltd (Mad)].
3 Whether a revised return Yes. [Dr. N. Srivatava (MP)].
could be revised U/s 139 (5)?
4 Is there any restriction on the No. Revision U/s 139 (5) is possible any number of times
number of time a return could subject to the time limit specified therein. There is no
be revise? need for nod of the AO for revising the return. [Waman
Padmanabh Dande Vs CIT 22 ITR 339 (Nag)].

6. Defective return – S. 139 (9):

1 When return could be If the annexures, statements and columns in the return
regarded as defective? relating to computation of income chargeable under each
head of income, computation of GTI and TI have not been
fully filled, then the return could be regarded as defective.
2 Procedure on finding the Where the AO considers that the ROI furnished by the
return to be defective. assessee is defective, he may intimate the defect to the
assessee and give him an opportunity to rectify the defect
within a period of 15 days from the date of such intimation.
The AO has the discretion to extend the time period beyond
15 days, on an application made by the assessee.
3 Consequences of not curing If the defect is not rectified within the period of 15 days or
the defect. such further extended period, then the return would be
treated as an invalid return. The consequential effect would
be the same as if the assessee had failed to furnish the
return.
4 Power to condone the delay Where, however, the assessee rectifies the defect after the
expiry of the period of 15 days or the further extended
period, but before assessment is made, the AO can condone
the delay and treat the return as a valid return.

MT Educare –CA Inter Income Tax Classes– R. SOUMYANARAYANAN. FCA. GRAD CWA. Page-333
Chapter-12: Filing of returns

B. PAN – S. 139A:

1 Persons who should Every person whose TI during any PY exceeded BEL.
apply for PAN Every person carrying on any business or profession whose
total sales, turnover or gross receipts exceeds or is likely to
exceed Rs. 5L in any PY.
Every person who is required to furnish a ROI U/s 139 (4A).
Every person being a resident, other than an individual,
which enters into a financial transaction of an amount
aggregating Rs. 250000 or more in a FY
Every person who is the MD, director, partner, trustee,
author, founder, karta, CEO, principal officer or office
bearer of the person referred to in (d) or any person
competent to act on behalf of the person referred to in (d),
Persons notified by the CG.
2 Others can also apply Any person, other than the persons (supra), may apply to the
AO for the allotment of a PAN and the AO shall allot a PAN to
such person immediately.
3 Quoting of PAN in PAN shall be mandatorily quoted in all returns, challans and
documents pertaining to documents pertaining to prescribed transactions.
prescribed transactions.

[Notification No. 59/2019, dated 30.8.2019]


Manner for allotment of PAN to a person who has not been allotted a PAN but
possesses Aadhaar number
1 The Finance (No. 2) Act, 2019, has inserted sub-section (5E) to section 139A,
w.e.f. 1.9.2019, to provide inter alia that every person who is required to furnish
or intimate or quote his PAN and who has not been allotted a PAN but possesses
the Aadhaar number, may furnish or intimate or quote his Aadhaar Number in
lieu of the PAN and such person would be allotted a PAN in such manner as may
be prescribed.
2 Rule 114(4) requires submission of application for allotment of PAN by the
applicant in the prescribed form accompanied by the prescribed documents as
proof of identity, address and date of birth of such applicant.
3 The CBDT has, vide this notification, inserted sub-rule (1A) to Rule 114 w.e.f.
1.9.2019 to provide that any person, who has not been allotted a PAN but
possesses the Aadhaar number and has furnished or intimated or quoted his
Aadhaar number in lieu of the PAN in accordance with section 139A(5E), shall
be deemed to have applied for allotment of PAN and he shall not be required
to apply or submit any documents under Rule 114.
4 Further, sub-rule (1B) has also been inserted in Rule 114 to provide that any
person, who has not been allotted a PAN but possesses the Aadhaar number
may apply for allotment of the PAN under section 139A by intimating his
Aadhaar number and he shall not be required to apply or submit any
documents under Rule 114.

MT Educare –CA Inter Income Tax Classes– R. SOUMYANARAYANAN. FCA. GRAD CWA. Page-334
Chapter-12: Filing of returns

Transactions notified by the CBDT requiring mandatory quoting of PAN:

SN Nature of transaction Value of transaction


1. Sale or purchase of a motor vehicle or vehicle, as defined in All such transactions.
the Motor Vehicles Act, 1988 which requires registration by a
registering authority under that Act, other than two wheeled
vehicles.
2. Opening an account [other than a time-deposit referred to at All such transactions.
SN 12 and a Basic Savings Bank Deposit Account] with a
banking company or a co-operative bank to which the
Banking Regulation Act, 1949 applies (including any bank or
banking institution referred to in S. 51 of that Act).
3. Making an application to any banking company or a co- All such transactions.
operative bank to which the Banking Regulation Act, 1949,
applies (including any bank or banking institution referred to
in section 51 of that Act) or to any other company or
institution, for issue of a credit or debit card.
4. Opening of a demat account with a depository, participant, All such transactions.
custodian of securities or any other person registered U/s 12
(1A) of the SEBI Act, 1992.
5. Payment to a hotel or restaurant against a bill or bills at any Payment in cash of an
one time. amount > Rs. 50,000.
6. Payment in connection with travel to any foreign country or Payment in cash of an
payment for purchase of any foreign currency at any one amount exceeding Rs.
time. 50,000.
7. Payment to a Mutual Fund for purchase of its units Amount exceeding
Rs. 50,000.
8. Payment to a company or an institution for acquiring Amount exceeding
debentures or bonds issued by it. Rs.50,000.
9. Payment to the Reserve Bank of India for acquiring bonds Amount exceeding
issued by it. Rs. 50,000.
10. Deposit with a banking company or a co-operative bank to Cash deposits - (i)
which the Banking Regulation Act, 1949, applies (including exceeding Rs. 50,000
any bank or banking institution referred to in S. 51 of that during any one day;
Act); or post office
11. Purchase of bank drafts or pay orders or banker’s cheques Payment in cash of an
from a banking company or a co-operative bank to which the amount exceeding
Banking Regulation Act, 1949 applies (including any bank or Rs. 50,000 during any
banking institution referred to in S. 51 of that Act). one day.
12. A time deposit with (i) a banking company or a co-operative Amount exceeding Rs.
bank to which the Banking Regulation Act, 1949 applies 50,000 or aggregating to
(including any bank or banking institution referred to in S. 51 more than Rs. 5L during a
of that Act); (ii) a Post Office; (iii) a Nidhi referred to in S. 406 financial year.
of the Companies Act, 2013; or (iv) a non-banking financial
company which holds a certificate of registration U/s 45-IA of
the RBI Act, 1934, to hold or accept deposit from public.

MT Educare –CA Inter Income Tax Classes– R. SOUMYANARAYANAN. FCA. GRAD CWA. Page-335
Chapter-12: Filing of returns

13. Payment for one or more pre-paid payment instruments, as Payment in cash or by way
defined in the policy guidelines for issuance and operation of of a bank draft or pay order
prepaid payment instruments issued by Reserve Bank of India or banker’s cheque of an
under the Payment and Settlement Systems Act, 2007, to a amount aggregating to
banking company or a co-operative bank to which the Banking more than Rs. 50,000 in a
Regulation Act, 1949, applies (including any bank or banking financial year.
institution referred to in section51 of that Act) or to any other
company or institution.
14. Payment as life insurance premium to an insurer as defined in Amount aggregating to
the Insurance Act, 1938. more than Rs. 50K in a FY.
15. A contract for sale or purchase of securities (other than Amount exceeding Rs. 1L
shares) as defined in S. 2(h) of the Securities Contracts per transaction.
(Regulation) Act, 1956.
16. Sale or purchase, by any person, of shares of a company not Amount exceeding Rs. 1L
listed in a recognised stock exchange. per transaction.
17. Sale or purchase of any immovable property. Amount exceeding Rs. 10L
or SDV exceeding Rs. 10L.
18. Sale or purchase, by any person, of goods or services of any Amount exceeding Rs. 2L
nature other than those specified at SN 1 to 17 of this Table, if per transaction:
any.

Minor to quote PAN of parent or guardian:

Where a person, entering into any transaction referred to in this rule, is a minor and who
does not have any income chargeable to income-tax, he shall quote the PAN of his father or
mother or guardian, as the case may be, in the document pertaining to the said transaction.

Declaration by a person not having PAN:

Further, any person who does not have a PAN and who enters into any transaction specified
in this rule, shall make a declaration in Form No.60 giving therein the particulars of such
transaction.

Non-applicability of Rule 114B:

Also, the provisions of this rule shall not apply to the following class or classes of persons,
namely: (i) the CG, the SG and the Consular Offices; (ii) the non-residents referred to in S. 2
(30) in respect of the transactions other than a transaction referred to at SN 1 or 2 or 4 or 7
or 8 or 10 or 12 or 14 or 15 or 16 or 17 of the Table.

PAN or Form No. 60, not furnished at the time opening of account with bank (other than
time deposit and savings account) or subsequently, to be furnished on or before
30.6.2017:

A person who has an account (other than a time deposit and a Basic Saving Bank Deposit
Account) maintained with a banking company or a co-operative bank to which the Banking
Regulation Act, 1949 applies (including any bank or banking institution referred to in section

MT Educare –CA Inter Income Tax Classes– R. SOUMYANARAYANAN. FCA. GRAD CWA. Page-336
Chapter-12: Filing of returns

51 of that Act) and has not quoted his permanent account number or furnished Form No.
60, as the case may be, at the time of opening of such account or subsequently, he shall
furnish his permanent account number or Form No. 60, as the case may be, to a manager or
officer of banking company or co-operative bank, as the case may be on or before the 30th
June, 2017.

This becomes relevant in view of RBI circular dated 15.12.2016 mandating that no
withdrawal shall be allowed from the accounts having substantial credit balance/deposits if
PAN or Form No. 60 is not provided in respect of such accounts.

Meaning of certain phrases:

Phrase Inclusion
(i) Payment in connection with Payment towards fare, or to a travel agent or a tour
travel operator, or to an authorized person as defined in S.
2 (c) of the Foreign Exchange Management Act, 1999.
(ii) Travel agent or tour A person who makes arrangements for air, surface or
operator maritime travel or provides services relating to
accommodation, tours, entertainment, passport, visa,
foreign exchange, travel related insurance or other
travel related services either severally or in package.
(iii) Time Deposit Any deposit which is repayable on the expiry of a
fixed period.

C. Quoting of Aadhar number [S. 139AA]:


1 Mandatory quoting of Every person who is eligible to obtain Aadhar Number is required to
Aadhar Number. [S. 139AA mandatorily quote Aadhar Number, on or after 1st July, 2017: (a) in
(1)]. the application form for allotment of PAN; (b) in the ROI.
2 Mandatory quoting of If a person does not have Aadhar Number, he is required to quote
Enrolment Id, where person Enrolment ID of Aadhar application form issued to him at the time
does not have Aadhar of enrolment in the application form for allotment of PAN or in the
Number. [Proviso to S. ROI furnished by him.
139AA (1)]. Enrolment ID means a 28 digit Enrolment Identification Number
issued to a resident at the time of enrolment.
3 Intimation of Aadhar Every person who has been allotted PAN as on 1st July, 2017, and
Number to prescribed who is eligible to obtain Aadhar Number, shall intimate his Aadhar
Authority. [S. 139AA (2)]. Number to prescribed authority on or before a date as may be
notified by the CG.
4 Consequences of failure to In order to protect validity of transactions previously carried out
intimate Aadhar Number. through such PAN, the scheme of Section 139AA has been
[Proviso to S. 139AA (2)]. modified (with effect from September 1, 2019) to provide that if a
person fails to intimate the Aadhaar number, the PAN allotted to
such person shall be made inoperative in the prescribed manner.
5 Provision not to apply to The provisions of S. 139AA relating to quoting of Aadhar Number
certain person or class of would, however, not apply to such person or class or classes of
persons. [S. 139AA (3)]. persons or any State or part of any State as may be notified by the
CG.

MT Educare –CA Inter Income Tax Classes– R. SOUMYANARAYANAN. FCA. GRAD CWA. Page-337
Chapter-12: Filing of returns

Supreme Court Judgement on Aadhar-PAN linkage: [Press release dated 10.06.2017]

(i) Every person who is eligible to obtain Aadhar Number is required to mandatorily quote
Aadhar Number or Aadhar enrollment ID, on or after 1st July, 2017: (a) in the
application form for allotment of PAN; (b) in the ROI.
(ii) Every person who has been allotted PAN as on 1st July, 2017, and who is eligible to
obtain Aadhar Number, shall intimate his Aadhar Number to prescribed authority on
or before a date as may be notified by the CG for the purpose of linking PAN and
Aadhar.
(iii) However, for non-compliance of the point (ii) above, only a partial relied by the Court
has been given to those who do not have Aadhar and who do not wish to obtain
Aadhar for the time being, that their PAN will not be cancelled so that the other
consequences under the IT Act for failing to quote PAN may not arise.

Notification dated 11.05.2017:

The provisions of S. 139AA shall not apply to an individual who does not possess the Aadhar
number or the Enrollment ID and is:
(i) Residing in the states of Assam, J&K and Meghalaya;
(ii) A non-resident as the IT Act;
(iii) Of the age of 80 years or more at any time during the PY;
(iv) Not a citizen of India.

Notification No. 75/2019, dated 28th September 2019


Date for intimation of Aadhaar number to the prescribed authority extended
1 As per Section 139AA, every person who has been allotted Permanent Account
Number (PAN) as on 1st July, 2017, and who is eligible to obtain Aadhaar Number,
shall intimate his Aadhaar Number to prescribed authority on or before a date as
may be notified by the Central Government.
2 Accordingly, the Central Government has, vide Notification No. 31/2019, dated
31.03.2019, notified that every person who has been allotted permanent account
number as on 1st July, 2017, and who is eligible to obtain Aadhaar number, shall
intimate his Aadhaar number to the Principal DGIT (Systems) or Principal Director of
Income-tax (Systems) on or before 30th September, 2019.
3 The Central Government has, vide this notification extended the date from 30th
September, 2019 to 31st December, 2019.

D. Scheme for submission of returns through tax return preparers: [S. 139B]:

1 This section provides that, for the purpose of enabling any specified class or classes of
persons to prepare and furnish their returns of income, the CBDT may notify a Scheme
to provide that such persons may furnish their returns of income through a Tax Return
Preparer authorised to act as such under the Scheme.
2 The Tax Return Preparer shall assist the persons furnishing the return in a manner that
will be specified in the Scheme, and shall also affix his signature on such return.

MT Educare –CA Inter Income Tax Classes– R. SOUMYANARAYANAN. FCA. GRAD CWA. Page-338
Chapter-12: Filing of returns

3 A Tax Return Preparer can be an individual, other than: (i) any officer of a scheduled
bank with which the assessee maintains a current account or has other regular
dealings; (ii) any legal practitioner who is entitled to practice in any civil court in India:
(iii) a chartered accountant; (iv) an employee of the ‘specified class or classes of
persons’.
4 The “specified class or classes of persons” for this purpose means any person other
than a company or a person whose accounts are required to be audited U/s 44AB (tax
audit) or under any other existing law, who is required to furnish a ROI under the Act.
5 The Scheme notified under the said section may provide for the following -
(i) the manner in which and the period for which the Tax Return Preparers shall be
authorised,
(ii) the educational and other qualifications to be possessed, and the training and
other conditions required to be fulfilled, by a person to act as a Tax Return
Preparer,
(iii) the code of conduct for the Tax Return Preparers,
(iv) the duties and obligations of the Tax Return Preparers,
(v) the circumstances under which the authorisation given to a Tax Return Preparer
may be withdrawn, and
(vi) any other relevant matter as may be specified by the Scheme.
6 Every Scheme framed by the CBDT under this section shall be laid before each House of
Parliament while it is in session to make the same effective.
7 If both the houses decide in making any modification of Scheme, then, the Scheme will
have effect only in such modified form.
8 Similarly, if both the Houses decide that any Scheme should not be framed, then such
Scheme will thereafter be of no effect.
9 However, such modification or annulment should be without prejudice to the validity of
anything previously done under that scheme.
10 Accordingly, the CBDT has, in exercise of the powers conferred by this section, framed
the Tax Return Preparer Scheme, 2006, which came into force from 1.12.2006.
11 As per this scheme, Tax Return Preparer means any individual who has been issued a
Tax Return Preparer Certificate and a Unique Identification Number by the Partner
Organisation to carry on the profession of preparing the returns of income in
accordance with the provisions of this Scheme. However, persons referred to in clause
(ii) or clause (iii) or clause (iv) of section 288(2), namely, any officer of a Scheduled Bank
with which the assessee maintains a current account or has other regular dealings, any
legal practitioner who is entitled to practice in any civil court in India and an accountant
are not eligible to act as Tax Return Preparers.
12 It may be noted that as per S. 139B (3), an employee of the “specified class or classes of
persons” is not authorized to act as a Tax Return Preparer. Therefore, it follows that
employees of companies and persons whose accounts are required to be audited U/s
44AB or any other law for the time being in force, are eligible to act as Tax Return
Preparers.

MT Educare –CA Inter Income Tax Classes– R. SOUMYANARAYANAN. FCA. GRAD CWA. Page-339
Chapter-12: Filing of returns

E. Power of CBDT to dispense with furnishing documents etc with the return and filing of
return in electronic form. [S. 139C and S. 139D]:

1 S. 139C provides that the CBDT may make rules providing for a class or classes of persons
who may not be required to furnish documents, statements, receipts, certificate, reports
of audit or any other documents, which are otherwise required to be furnished along
with the return under any other provisions of this Act.
2 However, on demand, the said documents, statements, receipts, certificate, reports of
audit or any other documents have to be produced before the AO.
3 S. 139D empowers the CBDT to make rules providing for –
(a) the class or classes of persons who shall be required to furnish the ROI in
electronic form;
(b) the form and the manner in which the ROI in electronic form may be furnished;
(c) the documents, statements, receipts, certificates or audited reports which may
not be furnished along with the ROI in electronic form but have to be produced
before the AO on demand;
(d) the computer resource or the electronic record to which the ROI in electronic
form may be transmitted.

F. Persons authorised to verify ROI – S. 140:

This section specifies the persons who are authorized to verify the ROI U/s 139 of the Act.

Assessee Circumstance Authorised Persons


1. Individual (i) In circumstance s not covered - the individual himself
under (ii), (iii) & (iv) below
(ii) where he is absent from - The individual himself; or
India - Any person duly authorised by him
in this behalf holding a valid power
of attorney from the individual
(Such power of attorney should be
attached to the ROI)
(iii) Where he is mentally - His guardian; or
incapacitated from attending to his - Any other person competent to
affairs act on his behalf
(iv)where, for any other reason, it is - any person duly authorised by
not possible for the individual to him in this behalf holding a valid power
verify the return of attorney from the individual (Such
power of attorney should be attached to
the ROI)
2. HUF (i) in circumstances not - the karta
covered under (ii) and (iii) below
(ii) where the karta is absent - any other adult member of the
from India HUF
(iii) where the karta is mentally - any other adult member of the
incapacitated from attending to his HUF
affairs

MT Educare –CA Inter Income Tax Classes– R. SOUMYANARAYANAN. FCA. GRAD CWA. Page-340
Chapter-12: Filing of returns

3. Company (i) in circumstances not - the managing director of the


covered under (ii) to (v) below company
(ii) (a) where for any - Any director of the company
unavoidable reason such managing
director is not able to verify the
return; or
(b) where there is no MD - Any director of the company
(iii) Where the company is not - a person who holds a valid power of
resident in India attorney from such company to do so
(such power of attorney should be
attached to the return).
(iv)(a) Where the company is - Liquidator
Being wound up (whether
under the orders of a court or
otherwise);or
(b) where any person has been
appointed as the receiver of any - Liquidator
assets of the company
(v) Where the management of - the principal officer of the
the company has been taken over company
by the Central Government or any
State Government under any law
4. Firm (i) in circumstances not - the managing partner of the firm
covered under (ii) below
(ii) (a) where for any
Unavoidable reason such - Any partner of the firm, not being
managing partner is not able to a minor
verify the return; or
(b) where there is no managing
partner. - Any partner of the firm, not being
a minor
5. LLP (i) in circumstances not - Designated partner
covered under (ii) below
(ii) (a) where for any - Any partner of the LLP
Unavoidable reason such
designated partner is not able to
verify the return; or
(b) where there is no
designated partner. - Any partner of the LLP
6. Local - - the principal officer
authority
7. Political - - the chief executive officer of
party such party (whether he is known as
[referred to secretary or by any other designation)
in S. 139
(4B)]

MT Educare –CA Inter Income Tax Classes– R. SOUMYANARAYANAN. FCA. GRAD CWA. Page-341
Chapter-12: Filing of returns

8. Any other - - any member of the association


association or the principal officer of such
association
9. Any other - - that person or some other
person person competent to act on his behalf.

Verification of return – S. 140 (Amendments):

1 S. 140 provides for verification of ROI by specified persons in case of different assessees.
2 To facilitate implementation of the Insolvency and Bankruptcy Code, 2016 (IBC), it is provided that
the insolvency professional appointed by the adjudicating authority would be the authorised
signatory for verification of ROI of the specified company, with effect from AY 2018-19.
3 Specified company means company in whose case an application for corporate insolvency
resolution process is admitted by Adjudicating Authority; and such application may be admitted U/s
7 or S. 9 or S. 10 of IBC.
4 "insolvency professional" means a person enrolled U/s 206 with an insolvency professional agency
as its member and registered with the Board as an insolvency professional U/s 207;"
5 National Company Law Tribunal is Adjudicating Authority under IBC.
6 In case of such companies, in terms of the provisions of IBC, the Board and the directors are not
functional and the responsibilities are conferred on insolvency professional. Accordingly, the
provision is amended to facilitate verification and signature of such Company's ROI by insolvency
professional.

G. Self-assessment – S. 140A:
1 Payment of self- Where any tax is payable on the basis of any return required to be
assessment tax etc. [S. furnished under, inter alia, S. 139, after taking into account –
140A (1)]. (i) the amount of tax, already paid
(ii) any tax deducted or collected at source;
the assessee shall be liable to pay such tax together with interest and fee
payable under any provision of this Act for any delay in furnishing the
return or any default or delay in payment of advance tax before furnishing
the return.
2 Order of adjustment of Where the amount paid by the assessee U/s 140A (1) falls short of the
amount paid by the aggregate of the tax, interest and fee as aforesaid, the amount so paid
assessee. [Explanation shall first be adjusted towards the fee payable and thereafter towards
to S. 140A (1)]. interest and the balance, if any, shall be adjusted towards the tax payable.
3 Assessee-in-default for If any assessee fails to pay the whole or any part of such of tax or interest
non-payment of sum or fees, he shall be deemed to be an assessee in default in respect of such
referred to in S. 140A tax or interest or fees remaining unpaid and all the provisions of this Act
(1). [S. 140A (3)]. shall apply accordingly.
Penalty could be levied to the extent of tax outstanding, if the assessee is
assessee-in-default in respect of tax. [S. 221].
Collection and recovery proceedings could be initiated and the sum due
could be recovered in accordance with modes of recovery specified in S.
222 and S. 226.

MT Educare –CA Inter Income Tax Classes– R. SOUMYANARAYANAN. FCA. GRAD CWA. Page-342
Chapter-13: Provisions relating to advance tax

A. Provisions relating to advance tax:

1 Requirement to pay tax in Though income of the PY is assessed to tax in the AY, the
advance. [S. 207 (1)]. recovery of tax on such income is not postponed to the AY.
It is recovered in the same PY in which the income is
earned. One of the mechanisms through which the tax is
recovered in the same PY is the advance tax mechanism.
2 Steps in computation of Estimation of TI of the assessee for the relevant PY.
advance tax liability. Estimation of the tax liability using the tax rates specified in
Part-III of Schedule I of the Finance Act and special rates
applicable for specified income.
Advance tax liability = Tax liability estimated (inclusive of
surcharge (if applicable) & education cess) – TDS – TCS.
3 No advance tax obligation Case-1: If the advance tax liability < Rs. 10000, there is no
in certain cases. advance tax obligation. [S. 208].
Case-2: If the assessee = senior citizen (i.e. resident + Age ≥
60 years) and he does not have income U/H PGBP, then
there is no advance tax obligation. [S. 207 (2)].
4 Advance tax installments Due date of Instalment amount payable
and due date. [S. 211]. instalment
On or before Not less than 15% of advance tax
15th June liability.
On or before Not less than 45% of advance tax
15th September liability, as reduced by the amount, if
any, paid in earlier instalment.
On or before Not less than 75% of advance tax
15th December liability, as reduced by the amount, if
any, paid in earlier instalments.
On or before The whole of advance tax liability, as
15th March reduced by the amount, if any, paid in
earlier instalments.
5 Advance tax payment and The assessees who have opted for presumptive taxation U/s
due date applicable for 44AD or S. 44ADA shall pay the whole of the advance tax
assessees who have opted liability on or before 15th March.
for presumptive taxation
U/s 44AD or S. 44ADA.
6 Payment made before the Any amount paid by way of advance tax on or before 31st
end of the PY to be March shall also be treated as advance tax paid during the
treated as advance tax. FY for all purposes of the Act.
7 Income on which tax is For example, employee receives salary without TDS U/s 192.
deductible but not Then, the employee shall pay the tax on his salary in
deducted – Tax on such advance. Otherwise, the employee exposes himself to
income shall be paid by interest U/s 234B. This so even if the employer is punished
way of advance tax. by way of levying interest U/s 201 (1A) for non-deduction of
tax at source.

MT Educare –CA Inter Income Tax Classes– R. SOUMYANARAYANAN. FCA. GRAD CWA. Page-343
Chapter-13: Provisions relating to advance tax

B. Interest U/s 234B:

1 When interest is levied U/s Assessee has obligation to pay tax in advance. But has
234B? not paid any advance tax.
Advance paid by the assessee < 90% of the assessed tax.
2 Meaning of assessed tax Tax on returned income – TDS – TCS.
3 Base for computation of Assessed tax – Advance tax paid. [Part of Rs. 100
interest ignored].
4 Interest rate 1% per month or part thereof.
5 Period for which interest is 1st day of the AY to date of payment of self-assessment
computed tax U/s 140A.

C. Interest U/s 234C:

SN Default Interest U/s 234C


1 Advance tax paid on or before 15th June of [Tax due on returned income – Advance
the PY < 12% of tax due on the returned tax paid on or before 15th June] * 1% * 3
income. months.
th
2 Advance tax paid on or before 15 [Tax due on returned income – Advance
September of the PY < 36% of tax due on tax paid on or before 15th September] *
the returned income. 1% * 3 months.
th
3 Advance tax paid on or before 15 [Tax due on returned income – Advance
December of the PY < 75% of tax due on tax paid on or before 15th December] * 1%
the returned income. * 3 months.
th
4 Advance tax paid on or before 15 [Tax due on returned income – Advance
December of the PY < 75% of tax due on tax paid on or before 15th March] * 1% * 1
the returned income. month.

Points requiring attention:

1 Tax due on returned income = Tax on returned income – TDS – TCS.


2 Computation of interest in case of [Tax due on returned income – Advance tax
assessees who have opted for paid on or before 15th March] * 1% * 1 month.
presumptive taxation scheme referred
to in S. 44AD or S. 44ADA.
3 Provisions of S. 234C not to apply in The provisions of S. 234C shall not apply to any
certain cases. shortfall in the payment of the tax due on
returned income where such shortfall is on
account of under estimation or failure to
estimate:
(a) The amount of capital gains.
(b) Income by way of winnings from
lotteries, cross word puzzles, races, card
games, gambling etc.;
(c) Income U/H PGBP in cases where the
income accrues or arises under the said
head for the first time; or

MT Educare –CA Inter Income Tax Classes– R. SOUMYANARAYANAN. FCA. GRAD CWA. Page-344
Chapter-13: Provisions relating to advance tax

(d) Income of the nature referred to in s.


115BBDA. i.e. dividend from domestic
companies exceeding Rs. 10L.

However, the assessee has to pay the whole of


the amount of tax payable in respect of sums
referred to in (a) to (d) above as part of the
remaining instalments of advance tax which are
due or where no such instalment is due, by 31st
March of the FY.

Illustration:

For the AY 2020-21, Mr. X anticipates on 15.06.2018 that his income will be Rs. 720000. On
10th December, 2019 he was forced because of certain contingencies to sell some capital
assets on which he earned a STCG (not being one covered by S. 111A) of Rs. 2L. Rs. 2L was
not anticipated by him on15.06.2019. Mr. X has paid advance tax as under:

On 15.06.2019 Rs. 8000


On 15.09.2019 Rs. 9000
On 15.12.2019 Rs. 14000
On 15.03.2020 Rs. 30000

Compute the interest payable U/s 234C.

Computation of interest U/s 234C:

I Advance tax payable up to Actual advance tax paid Interest U/s 234C = Nil
15.06.2019 = Rs. 58760 * up to 15.06.2019 = Rs.
12% = Rs. 7051. 8000
II Advance tax payable up to Actual advance tax paid Interest U/s 234C = [(45% of
15.09.2019 = Rs. 58760 * up to 15.09.2019 = Rs. Rs. 58760) – Rs. 17000] *1% *
36% = Rs. 21154. 17000 3 months = Rs. 282.
III Advance tax payable up to Actual advance tax paid Interest U/s 234C = [(75% of
15.12.2019 = Rs. 100360 * up to 15.12.2019 = Rs. Rs. 100360) – Rs. 31000] *1%
75% = Rs. 75270. 31000 * 3 months = Rs. 1326.
IV Advance tax payable up to Actual advance tax paid Interest U/s 234C = [Rs.
15.03.2020 = Rs. 100360. up to 15.03.2020 = Rs. 100360 – Rs. 61000] *1% * 1
61000 months = Rs. 393.
Total interest U/s 234C (I + II + III + IV) Rs. 2001

Note:

1 TI of Mr. X (including capital gains) = Rs. 920000 Tax thereon = Rs. 100360.
2 TI of Mr. X (excluding capital gains) = Rs. 720000 Tax thereon = Rs. 58760.

MT Educare –CA Inter Income Tax Classes– R. SOUMYANARAYANAN. FCA. GRAD CWA. Page-345
Chapter-14: Tax Collection at Source

Applicability and Rate [S. 206C (1)/(1C)/(1F)]:

S. 206C (1):

U/s 206C (1), sellers of certain goods are required to collect tax from the buyers at the
specified rates. The specified % for collection of tax at source is as follows:

Nature of Goods TCS %

(i) Alcoholic liquor for human consumption 1%


(ii) Tendu leaves 5%
(iii) Timber obtained under a forest lease 2.5%
(iv) Timber obtained by any mode other than (iii) 2.5%
(v) Any other forest produce not being timber or tendu leaves 2.5%
(vi) Scrap 1%
(vii) Minerals, being coal or lignite or iron ore 1%

S. 206C (1C):

S. 206C (1C) provides for collection of tax by every person who grants a lease or a licence or
enters into a contract or otherwise transfers any right or interest in any –

(a) Parking lot or


(b) toll plaza or
(c) a mine or a quarry

to another person (other than a PSC) for the use of such parking lot or toll plaza or mine or
quarry for the purposes of business. The tax shall be collected as provided, from the licensee
or lessee of any such licence, contract or lease of the specified nature, at the rate of 2%.

Note:

1 Mining and quarrying excludes mining and quarrying of mineral oil.


2 Mineral oil includes petroleum and natural gas.
3 Thus, mining and quarrying excludes mining and quarrying of petroleum and natural gas.
4 Consequently, the oil exploration and incidental services are relieved from the
applicability of TCS provisions, since these services are in the organized sector.

S. 206C (1F):

S. 206C (IF) provides that every person, being a seller, who receives any amount as
consideration for sale of a motor vehicle of the value exceeding Rs. 10L, shall collect tax
from the buyer @ 1% of the sale consideration.

MT Educare –CA Inter Income Tax Classes– R. SOUMYANARAYANAN. FCA. GRAD CWA. Page-346
Chapter-14: Tax Collection at Source

Term Meaning
Buyer (For the A person who obtains in any sale, by way of auction, tender, or any other
purpose of S. mode, goods of the nature specified in the Table in S. 206C (1) or the
206C (1) and right to receive any such goods but does not include –
(1C)) (A) a PSC, the CG, a SG, and an embassy, a high commission, legation,
commission, consulate and the trade representation, of a foreign
State and a club, or
(B) a buyer in the retail sale of such goods purchased by him for
personal consumption.
Buyer (For the A person who obtains in any sale, goods of the nature specified therein,
purpose of S. but does not include –
206C (1F). (A) the CG, a SG and an embassy, a High Commission, legation,
commission, consulate and the trade representation of a foreign
State; or
(B) a local authority as defined in Explanation to S. 10 (20); or
(C) a public sector company which is engaged in the business of
carrying passengers.
Seller (i) The CG,
(ii) a SG or
(iii) any local authority or
(iv) corporation or
(v) authority established by or under a Central or State Act
(vi) any company or
(vii) firm or
(viii) co-operative society
Seller also includes an individual or a HUF whose total sales, gross
receipts or turnover from the business or profession carried on by him
exceed the monetary limits specified under clause (a) or clause (b) of S.
44AB during the FY immediately preceding the FY in which the goods of
the nature specified in the Table in S. 206C (1) are sold.
Scrap Waste and scrap from the manufacture or mechanical working of
materials which is definitely not usable as such because of breakage,
cutting up, wear and other reasons;

CBDT Clarification relating to certain issues with respect to section 206C(1F):

These amendments in S. 206C have given rise to certain issues relating to the scope and
applicability of the provisions.

Accordingly, the CBDT has, vide Circular No. 22/2016 dated 8.6.2016 and Circular
No.23/2016 dated 24.6.2016, clarified the following issues in "Question & Answer (Q&A)”
format.

Q.1 Whether TCS@1% is on sale of motor vehicle at retail level or also on sale of motor
vehicles by manufacturers to dealers/ distributors?

MT Educare –CA Inter Income Tax Classes– R. SOUMYANARAYANAN. FCA. GRAD CWA. Page-347
Chapter-14: Tax Collection at Source

A. To bring high value transactions within the tax net, S. 206C has been amended to provide
that the seller shall collect the tax @ 1% from the purchaser on sale of motor vehicle of the
value exceeding Rs. 10L.

This is brought to cover all transactions of retail sales and accordingly, it will not apply on
sale of motor vehicles by manufacturers to dealers/distributors.

Q.2 Whether TCS @ 1% on sale of motor vehicle is applicable only to luxury cars?

A. No, as per S. 206C (1F), the seller shall collect tax @ 1% from the purchaser on sale of any
motor vehicle of the value exceeding Rs. 10L.

Q.3 Whether TCS @ 1% is applicable in the case of sale to Government Departments,


Embassies, Consulates and United Nation Institutions, of motor vehicle or any other goods
or provision of services?

A. Government, institutions notified under United Nations (Privileges and Immunities) Act
1947, and Embassies, Consulates, High Commission, Legation, Commission and trade
representation of a foreign State shall not be liable to levy of TCS @ 1% U/s 206C (1F).

Q.4 Whether TCS is applicable on each sale of motor vehicle or on aggregate value of sale
during the year?

A. Tax is to be collected at source @ 1% on sale consideration of a motor vehicle exceeding


Rs. 10L. It is applicable to each sale and not to aggregate value of sale made during the year.

Q.5 Whether TCS @ 1% on sale of motor vehicle is applicable in case of an individual?

A. The definition of "Seller" as given in clause (c) of the Explanation below S. 206C (11) shall
be applicable in the case of sale of motor vehicles also.

Accordingly, an individual who is liable to audit as per the provisions of S. 44AB during the
FY immediately preceding the FY in which the motor vehicle is sold shall be liable for
collection of tax at source on sale of motor vehicle by him.

Q.6 How would the provisions of TCS on sale of motor vehicle be applicable in a case
where part of the payment is made in cash and part is made by cheque?

A. The provisions of TCS on sale of motor vehicle exceeding Rs. 10L is not dependent on
mode of payment. Any sale of motor vehicle exceeding Rs. 10L would attract TCS @ 1%.

Time of Collection of tax [Section 206C(1)/(1C)/(1F)]:

The tax should be collected at the time of debiting of the amount payable by the buyer or
licensee or lessee, as the case may be, to his account or at the time of receipt of such
amount from the buyer or licensee or lessee, as the case may be, in cash or by the issue of a
cheque or draft or any other made, whichever is earlier.

MT Educare –CA Inter Income Tax Classes– R. SOUMYANARAYANAN. FCA. GRAD CWA. Page-348
Chapter-14: Tax Collection at Source

In case of sale of a motor vehicle of the value exceeding Rs. 10L, tax shall be collected at the
time of receipt of such amount.

Non-applicability of TCS [S. 206C (1A)]:

No collection of tax shall be made in the case of a resident buyer, if such buyer furnishes to
the person responsible for collecting tax, a declaration in writing in duplicate in the
prescribed form and verified in the prescribed manner to the effect that goods referred to in
column (2) of the Table in (1) above are to be utilised for the purpose of manufacturing,
processing or producing articles or things or for the purposes of generation of power and
not for trading purposes.

Furnishing of copy of declaration within specified time [S. 206C (1B)]:

The person responsible for collecting tax under this section shall deliver or cause to be
delivered to the CCIT or CIT one copy of the declaration referred to S. 206C (1A) on or
before 7th of the month next following month in which the declaration is furnished to him.

TCS to be paid within prescribed time [S. 206C (3)]:

Any amount collected U/s 206C (1) or (1C) shall be paid within the prescribed time to the
credit of the CG or as the Board directs.

Time limit for paying tax collected to the credit of the CG [R. 37CA]:

Circumstance Period within which such


Person collecting sums in sum should be paid to the
accordance with section credit of the Central
206C(1)/(1C) Government
(1) An office of the (i) where the on the same day
Government tax is paid without
production of an
income-tax challan
(ii) where tax on or before 7 days from
is paid the end of the month in
accompanied by which the collection is
an income-tax made
challan
(2) Collectors other than an within one week from the
office of the Government last day of the month in
which the collection is
made

The entire TCS process can be understood at a glance from the diagram given in the next
page. The reference to Rules and Forms are only for the information of Students. They are,
however, not required to memorize the Rule numbers and Form numbers for examination
purposes.

MT Educare –CA Inter Income Tax Classes– R. SOUMYANARAYANAN. FCA. GRAD CWA. Page-349
Chapter-14: Tax Collection at Source

MT Educare –CA Inter Income Tax Classes– R. SOUMYANARAYANAN. FCA. GRAD CWA. Page-350
Chapter-15: TDS (Chapter XVII-B)

1. Recovery of tax in the same FY: S. 190:

1 Though income of the PY is assessed to tax in the AY, the tax on such income is sought to
be recovered in the same PY.
2 The recovery is done through three mechanisms (a) TDS; (b) TCS; (c) Advance tax.
3 The provisions relating to TDS mechanism are contained in Chapter XVII-B. [S. 192 to S.
206AA].

2. Direct payment by the assessee – S. 191:

In the following circumstances, the person earning income shall pay the tax direct:

1 Relevant income is not liable to TDS under Chapter XVII-B.


2 It is liable to TDS but was not subjected to TDS.

3. Overview of TDS chapter:

Sections Provisions
S. 192 TDS on salary
S. 192A TDS on payment of accumulated balance due to an employee
S. 193 TDS on interest on securities
S. 194A TDS on interest other than interest on securities
S. 194B TDS on winnings from lottery, crossword puzzle etc.
S. 194BB TDS on winnings from horse race
S. 194C TDS on payment to contractors
S. 194D TDS on insurance commission
S. 194DA TDS on payment in respect of life insurance policy
S. 194E TDS on payments to non-resident sportsmen or sports association
S. 194G TDS on commission etc on the sale of lottery tickets
S. 194H TDS on commission or brokerage
S. 194I TDS on rent
S. 194IA TDS on payment for transfer of certain immovable property other than agricultural
land
S. 194IB TDS on payment of rent by certain individuals or HUF.
S. 194IC TDS on payment under specified agreement.
S. 194J TDS on fee for professional services, technical services etc.
S. 194LA TDS on payment of compensation on acquisition of certain immovable property.
S. 194M Payment made by an individual or a HUF for contract work or by way of fees for
professional services or commission or brokerage
S. 194n TDS on cash withdrawal in excess of ` 1 crore - w.e.f 01/09/2019
S. 195A TDS on income payable net of tax
S. 196 No TDS on interest or dividend or other sums payable to Government, RBI or
certain corporations.
S. 197 Certificate for deduction at lower rate
S. 197A No deduction in certain cases
S. 198 TDS is income received

MT Educare –CA Inter Income Tax Classes– R. SOUMYANARAYANAN. FCA. GRAD CWA. Page-351
Chapter-15: TDS (Chapter XVII-B)

S. 199 Credit for tax deducted


S. 200 Duty of person deducting tax
S. 200A Processing of statement of TDS
S. 201 Consequences of failure deduct tax at source.
S. 202 Deduction only one mode of recovery
S. 203 Certificate for tax deducted
S. 203A Tax deduction and collection account number
S. 205 Bar on direct demand on assessee
S. 206AA Requirement to furnish PAN

4. Applicability of surcharge and educate cess on rates of TDS:

Payee Payment Surcharge HEC


Resident Salary ≤ Rs. 50L - 4%
Rs. 100L ≥ Salary > Rs. 50L 10% 4%
Salary > Rs. 100 15% 4%
Other than salary - -
Non-resident being foreign Any payment ≤ Rs. 100L - 4%
companies Rs. 1000L ≥ Any payment > Rs. 100L 2% 4%
Any payment > Rs. 1000L 5% 4%
Non-resident being Individual/ Any payment ≤ Rs. 50L - 4%
HUF/AJP/AOP/BOI Rs. 100L ≥ Any payment > Rs. 50L 10% 4%
Any payment > Rs. 100L 15% 4%
Other non-residents Any payment ≤ Rs. 100L - 4%
Any payment > Rs. 100L 12% 4%

5. Impact of absence of PAN – S. 206AA:

1 If the payee does not have PAN, the minimum TDS rate is 20%. [S. 206AA (1)].
2 When 20% is used for deducting tax at source as per S. 206AA, it shall not be
incremented by education cess. [CBDT circular].

6. TDS on Salary – S. 192:

1 Person responsible for making Every person responsible for paying any income
payment and the payment chargeable to tax U/H `Salaries` to deduct tax on the
covered by the section. [S. 192 amount payable.
(1)]. He may be individual, HUF or any other person.
Liability to tax audit in the immediately preceding FY
is not relevant.
2 Payee. [S. 192 (1)]. Resident or non-resident.
3 Manner of deduction of tax. [S. TDS has to be calculated at the average rate of
192 (1)]. income-tax computed on the basis of the rates in
force for the relevant FY in which the payment is
made, on the estimated TI of the assessee. [See
illustration-1 & 2].

MT Educare –CA Inter Income Tax Classes– R. SOUMYANARAYANAN. FCA. GRAD CWA. Page-352
Chapter-15: TDS (Chapter XVII-B)

4 What do we mean by average Average rate of income-tax means the rate arrived
rate of income-tax? [S. 2 (10)]. at by dividing the amount of income-tax calculated
on the TI, by such TI.
5 Timing of TDS. [S. 192 (1)]. Tax shall be deducted at source at the time of
payment of salary.
6 Employer undertaking the The employer may, at his option, pay tax on the
obligation to pay tax on non- non-monetary perquisites enjoyed by the employee,
monetary perquisites enjoyed by in lieu of deduction of tax at source from salary
the employee. [S. 192 (1A) & payable to the employee.
(1B)]. Such tax will have to be worked out at the average
rate applicable to aggregate salary income of the
employee and payment of tax will have to be made
by the employer every month out of his funds, along
with tax deducted at source on monetary payment
of salary, allowances etc. [Illustration-3].
The employee is going to get credit even in respect
of tax on non-monetary perquisites borne by the
employer.
4 Tax treatment of tax paid by the It is income in the hands of the employee, since it is
employer on the non-monetary a payment made by the employer to discharge the
perquisites enjoyed by the obligation of the employee. However, it is exempt
employee. in the hands of the employee U/s 10 (10CC).
Though it is a business expense for the employer, it
shall be disallowed in view of S. 40 (a) (v).
5 Multiple employment In cases where an assessee is simultaneously
/successive employment – employed under more than one employer or the
manner of tax deduction. [S. 192 assessee takes up a job with another employer
(2)]. during the FY after his resignation or retirement
from the services of the former employer, he may
furnish the details of the income U/H "Salaries” due
or received by him from the other employer, the tax
deducted therefrom and such other particulars to
his current employer.
Thereupon, the subsequent employer should take
such information into consideration and then
deduct the tax remaining payable in respect of the
employee`s remuneration from both the employers
put together for the relevant FY. [See illustration-4].
6 Considering relief while In respect of salary payments to employees of
estimating TDS. [192 (2A)]. Government or to employees of companies, co-
operative societies, local authorities, universities,
institutions, associations or bodies, deduction of tax
at source should be made after allowing relief U/s
89 (1), where eligible.
In this regard, the employee has to submit form-10E
to the employer.

MT Educare –CA Inter Income Tax Classes– R. SOUMYANARAYANAN. FCA. GRAD CWA. Page-353
Chapter-15: TDS (Chapter XVII-B)

7 Considering income under other A tax payer having salary income in addition to
heads and loss under head IFHP. other income chargeable to tax for that FY, may
[S. 192 (2B)]. send to the employer, the following:
(a) particulars of such other income;
(b) particulars of any tax deducted under any
other provision;
(c) loss, if any, U/H IFHP.
The employer shall take the above particulars into
account while calculating tax deductible at source.
8 Consideration of other income However, the tax deductible from salary should not
and TDS thereon not to result in get reduced as a result of considering income under
reducing TDS U/s 192. [Proviso other heads and tax deducted thereon. But there is
to S. 192 (2B)]. no issue on account of reduction of TDS due to
considering LFHP. [See illustration-5].
9 Furnishing of statement of Employer shall furnish to the employee, a statement
particulars of perquisites or giving correct and complete particulars of
profits in lieu of salary by perquisites or profits in lieu of salary provided to
employer to employee. [S. 192 him and the value thereof.
(2C)]. The statement shall be in Form-12BA. This
requirement is applicable only where the salary
paid/payable to an employee exceeds Rs. 150000.
For other employees, the particulars of
perquisites/profits in lieu of salary shall be given in
Form-16 itself.
10 Compliance with the circulars Every year, the CBDT issues a circular giving details
issued by the CBDT. and direction to all employers for the purpose of
deduction of tax from salaries payable to the
employees during the relevant financial year. These
instructions should be followed.
11 Requirement to obtain evidence Responsibility is cast on the person responsible for
/ proof / particulars of claims paying any income chargeable U/H "Salaries” to
from the employee by the obtain from the assessee, the evidence or proof or
employer. [S. 192 (2D)]. particulars of prescribed claims (including claim for
set-off of loss) under the provisions of the Act in the
prescribed form and manner, for the purposes of –
(i) estimating income of the assessee; or
(ii) computing tax deductible U/s 192 (1).
12 Provisions of R. 26C R. 26C requires furnishing of evidence of the
following claims by an employee to the person
responsible for making payment U/s 192 (1) in Form
No.12BB for the purpose of estimating his income or
computing the tax deduction of tax at source:

MT Educare –CA Inter Income Tax Classes– R. SOUMYANARAYANAN. FCA. GRAD CWA. Page-354
Chapter-15: TDS (Chapter XVII-B)

S.No. Nature of Claim Evidence or


particulars
1. House Rent Name, address and
Allowance PAN of the
landlord(s) where
the aggregate rent
paid during the PY >
Rs. 1L.
Leave Travel
Evidence of
2. Concession or
expenditure
Assistance
3. Deduction of Name, address and
interest U/H IFHP PAN of the lender
4. Deduction under Evidence of
Chapter VI-A investment or
expenditure.
13 Salary in foreign currency – For purposes of deduction of tax out of salaries
manner of tax deduction. [S. 192 payable in a foreign currency, the value of salaries in
(6)]. terms of rupees should be calculated at the
prescribed rate of exchange as specified in R. 26.
[TTBR on the date of payment].
14 Adjustments to the TDS It is possible for the employer to increase or
instalments to cover previous decrease the tax to be deducted to cover the
shortfalls or excess. [S. 192 (3)]. previous deficiency or surplus in deduction.

Illustration-1:

Mr. X worked with X Ltd on a monthly salary of Rs. 50000 till 31.07.2019. From 01.08.2019
he joined Y Ltd on a monthly salary of Rs. 60000. Determine the tax to be deducted at
source by X Ltd and Y Ltd.

Computation of tax to be deducted by X Ltd:

1 Total salary (50000*12) Rs. 600000


2 Tax thereon (including HEC) Rs. 33800
3 Tax to be deducted each month (33800/12) Rs. 2817

Month Tax deducted at source by X Ltd


April 19 Rs. 2817
May 19 Rs. 2817
June 19 Rs. 2817
July 19 Total salary from X Ltd was only Rs. 200000 for the aforesaid 4 months which is
less than BEL. There is no need to deduct tax at source, although tax was
deducted at source for the first 3 months.

MT Educare –CA Inter Income Tax Classes– R. SOUMYANARAYANAN. FCA. GRAD CWA. Page-355
Chapter-15: TDS (Chapter XVII-B)

Computation of tax to be deducted at source by Y Ltd on the assumption that details of


the salary drawn from X Ltd have been submitted in Form 12B:

1 Salary drawn from X Ltd Rs. 200000


2 Salary to be drawn from Y Ltd (60000*8) Rs. 480000
3 Income from salary Rs. 680000
4 Tax thereon (including HEC) Rs. 50440
5 Tax already deducted by X Ltd (Rs. 2817*3) Rs. 8451
6 Total tax to be deducted by Y Ltd (4-5) Rs. 41989
7 Tax to be deducted each month by Y Ltd (Rs. 41989/8) Rs. 5249

Illustration-2:

R is employed with X Ltd and drawing a salary of Rs. 60000 p.m. He has contributed Rs.
36000 towards the RPF. He has submitted the following details of other income to the
employer duly verified by him:

Gross receipts from profession Rs. 140000


Tax deducted at source @ 10% Rs. 14000
Expenses incurred on earning the professional income Rs. 120000
Loss from house property let out (Rs. 73000)
TDS by the tenant on the house property (rental income of Rs. 230000 @ 10%) Rs. 23000

Compute the tax deductible by R Ltd in the aforesaid case.

Determination of TDS:

SN Particulars Tax deductible by including Tax deductible by including


loss from HP and income loss from HP but excluding
from consultancy income from consultancy
1 Salary Rs. 720000 Rs. 720000
2 Loss from HP (Rs. 73000) (Rs. 73000)
3 Net (1-2) Rs. 647000 Rs. 647000
4 Income from Rs. 20000
consultancy
5 GTI (3+4) Rs. 667000 Rs. 647000
6 Deduction U/s 80C (Rs. 36000) (Rs. 36000)
7 Total income Rs. 631000 Rs. 611000
8 Tax on total income Rs. 40248 Rs. 36088
9 TDS on rent Rs. 23000 Rs. 23000
10 TDS on professional Rs. 14000
receipts
11 TDS Rs. 3248. Rs. 13088.

Note: Tax to be deducted every month = Rs. 13088 / 12 = Rs. 1091.

MT Educare –CA Inter Income Tax Classes– R. SOUMYANARAYANAN. FCA. GRAD CWA. Page-356
Chapter-15: TDS (Chapter XVII-B)

7. TDS on premature withdrawal from EPF – S. 192A:

(a) Compliance with rule 9 of part A of Fourth Schedule : Certain concerns:

1 Under the Employees Provident Fund and Miscellaneous Provisions Act, 1952 (EPF &
MP Act, 1952), certain specified employers are required to comply with the Employees
Provident Fund Scheme, 1952 (EPFS).
2 However, these employers are also permitted to establish and manage their own
private provident fund (PF) scheme subject to fulfillment of certain conditions.
3 The provident funds established under a scheme framed under EPF & MP Act, 1952 or
Provident Fund exempted U/s 17 of the said Act and recognised under the Income-tax
Act, 1961 are termed as Recognised Provident fund (RPF) under the Act.
4 Part A of the Fourth Schedule to the Income-tax Act, 1961 contains the provisions
relating to RPFs.
5 Under the existing provisions of R. 8 of Part A of the Fourth Schedule, the withdrawal of
accumulated balance by an employee from the RPF is exempt from taxation.
6 For the purpose of discouraging pre-mature withdrawal and promoting long term
savings, if the employee makes withdrawal before continuous service of five years
(other than the cases of termination due to ill health, contraction or discontinuance of
business, cessation of employment etc.) and does not opt for transfer of accumulated
balance to new employer, the withdrawal would be subject to tax.
7 R. 9 of Part A of the Fourth Schedule provides the manner of computing the tax liability
of the employee in respect of such pre-mature withdrawal.
8 In order to ensure collection of tax in respect of such pre-mature withdrawals, Rule 10
of Part A of the Fourth Schedule casts responsibility on the trustees of the RPF to
deduct tax as computed in Rule 9 at the time of payment.
9 R. 9 provides that the tax on withdrawn amount is required to be calculated by re-
computing the tax liability of the years for which the contribution to RPF has been
made by treating the same as contribution to unrecognized provident fund.
10 The trustees of private provident fund schemes, are generally a part of the employer
group and hence, have access to or can easily obtain the information regarding
taxability of the employee making pre-mature withdrawal for the purposes of
computation of the amount of tax liability U/R 9.
11 However, it may not always be possible for the trustees of EPFS to get the information
regarding taxability of the employee such as year-wise amount of taxable income and
tax payable for the purposes of computation of the amount of tax liability U/R 9.

(b) Provisions of S. 192A:

1 Applicability and S. 192A provides for deduction of tax @ 10% on premature taxable
TDS rate withdrawal from employees provident fund scheme.
Accordingly, in a case where the accumulated balance due to an
employee participating in a RPF is includible in his TI owing to the
provisions of Rule 8 of Part A of the Fourth Schedule not being
applicable, the trustees of the EPF Scheme, 1952 or any person
authorized under the scheme to make payment of accumulated
balance due to employees are required to deduct tax @10%.

MT Educare –CA Inter Income Tax Classes– R. SOUMYANARAYANAN. FCA. GRAD CWA. Page-357
Chapter-15: TDS (Chapter XVII-B)

2 Timing of taxTax should be deducted at the time of payment of accumulated


deduction. balance due to the employee
3 Non-applicability No tax deduction is to be made this section if the amount of such
of TDS U/s 192A payment or aggregate amount of such payment to the payee is less
than Rs. 50,000.
4 Deduction at MMR Any person entitled to receive any amount on which tax is
in case of non- deductible under this section has to furnish his PAN to the person
submission of PAN. responsible for deducting such tax. In case he fails to do so, tax
would be deductible at the MMR(*Note)

*Note: -
MMR (Maximum Marginal Rate) is now 42.744% [Tax Rate 30% + Surcharge 37% +
H&EEC 4%] as highest surcharge rate for individual is now 37% instead of 15%.

8. TDS on Interest on Securities [S. 193]:

1 Person responsible for This section casts responsibility on every person responsible for
deduction of tax at paying any income by way of interest on securities.
source
2 Payee Resident.
3 Meaning of interest on It means: (a) interest on any security of the CG or SG; (b) interest on
securities. [S. 2 (28B)].
debentures or other securities for money issued or on behalf of a
local authority or a company or a corporation established by a
Central or State Act.
4 Rate of TDS 10%
5 Time of tax deduction. Tax should be deducted at the time of credit of such income to the
account of the payee or at the time of payment thereof, whichever
is earlier.
6 Non-applicability of No tax shall be deducted at source on any interest payable to any
TDS U/s 193. insurance company in respect of any securities owned by it or in
which it has full beneficial interest.
No tax shall be deducted on interest payable on any security of the
CG or SG. However, tax shall be deducted on interest on 7.75%
Savings (Taxable) Bonds 2018 exceeding Rs. 10000.
No TDS on interest payable on any security issued by a company,
where such security is in demat form and is listed on a RSE in India.
No tax shall be deducted from any interest payable to an Individual
or HUF, who is resident in India, on debentures issued by a
company in which the public are substantially interested, if: (a) the
interest is paid by the company by an APC; and (b) the amount of
such interest or, as the case may be, the aggregate of the amounts
of such interest paid or likely to be paid during the FY by the
company to such individual or HUF does not exceed Rs. 5000.

MT Educare –CA Inter Income Tax Classes– R. SOUMYANARAYANAN. FCA. GRAD CWA. Page-358
Chapter-15: TDS (Chapter XVII-B)

9. TDS on interest other than interest on securities – S. 194A:

1 Payment covered. [S. 194A (1)]. Interest other than interest on securities.
2 Payee. [S. 194A (1)]. Resident.
3 Person responsible for making Any person.
payment. [S. 194A (1)]. However, an individual or HUF has TDS
obligation only when they were liable to tax
audit in the immediately preceding FY. [Proviso
to S. 194A (1)].
4 TDS rate. [S. 194A (1)] 10%
5 Time of tax deduction. [S. 194A (1)] At the time of credit to the account of the payee
in the books of accounts of the payer or at the
time of payment (whichever is earlier).
6 Threshold limit. [S. 194A (1)]. Rs. 5000.
7 Cases where there is no TDS S. 194A (3) (iii): Payee =
obligation. [S. 194A (3)]. (a) Banking company
(b) Co-operative land mortgage bank
(c) Co-operative society engaged in banking
business
(d) Statutory corporation
(e) LIC
(f) UTI
(g) Company or co-operative society
engaged in business of insurance.
(h) Any other person notified by the CG in
the OG. [Example: National skill
development fund].
S. 194A (3) (iv): No TDS on interest paid by the
firm to its partners.
S. 194A (3) (vi): No TDS obligation in respect of
interest payable on sum deposited under:
(a) Kissan vikas patra
(b) Indira Vikas Patra
(c) NSC
(d) Post office monthly income account
(e) Post office time deposit account
(f) Post office recurring deposit account
S. 194A (3) (viii): No TDS on Interest on refund
granted under IT Act.
S. 194A (3) (x): No TDS on discount of ZCB.
S. 194A (3) (xi): No TDS on interest payable by
the SPV to the Business Trust, since it is exempt
U/s 10 (23FC).
S. 194A (3) (vii): No TDS on interest on deposits
(other than time deposits) with bank.

MT Educare –CA Inter Income Tax Classes– R. SOUMYANARAYANAN. FCA. GRAD CWA. Page-359
Chapter-15: TDS (Chapter XVII-B)

S. 194A (3) (ix) + S. 194A (3) (ixa): Motor


accident claims tribunal awards compensation
and interest thereon for the delay in awarding it
to the motor accident victim or his legal heir.
The person causing accident pays compensation
and interest thereon to the motor accident
victim or his legal heir. It is taxable in the PY of
receipt U/H IFOS. [S. 145A (b) & S. 56 (2) (viii)].
50% is deductible U/s 57 (iv). The TDS obligation
arises in respect of interest only upon payment.
That too, only when the threshold limit of Rs.
50000 is crossed by the interest paid during a FY.
S. 194A (3) (i) (c): No TDS on interest paid by the
Post office on deposits made under senior
citizen scheme if the threshold of Rs. 40000 (Rs.
50000 in case of senior citizen) is not crossed.
S. 194A (3) (i) (d): No TDS on interest on
deposits (time deposit or otherwise) paid by a
HFC to a resident, if the threshold of Rs. 5000 is
not crossed.
HFC = Public company whose main object is to
provide long-term financial assistance for
construction or purchase of RHP in India and is
eligible for deduction U/s 36 (1) (viii).
If the HFC has adopted core banking solutions
(CBS), then the threshold of Rs. 5000 is in
respect of interest paid by the HFC as a whole.
If CBS was not adopted, then the threshold limit
is in respect of interest paid by each branch.
S. 194A (3) (i) (a): No TDS on interest paid to
resident by a banking company on time
deposits, if the threshold of Rs. 40000 (Rs. 50000
in case of senior citizen) is not crossed.
Time deposits shall include recurring deposits.
If the banking company has adopted core
banking solutions (CBS), then the threshold of
Rs. 40000 (Rs. 50000 in case of senior citizen) is
in respect of interest paid by the banking
company as a whole.
If CBS was not adopted, then the threshold limit
is in respect of interest paid by each branch.
S. 194A (3) (viia) (a): No TDS on any interest on
any deposit paid to any one by:
(a) Primary agricultural credit society
(b) Primary credit society
(c) Co-operative land mortgage bank
(d) Co-operative land development bank

MT Educare –CA Inter Income Tax Classes– R. SOUMYANARAYANAN. FCA. GRAD CWA. Page-360
Chapter-15: TDS (Chapter XVII-B)

S. 194A (3) (v): No TDS on interest on time


deposits paid by a co-operative bank (not being
one covered by S. 194A (3) (viia) (a)) to another
co-operative society.
S. 194A (3) (i) (b): No TDS on interest on time
deposits paid by a co-operative bank (not being
one covered by S. 194A (3) (viia) (a)) to a
member or non-member if the THL of Rs. 40K
(Rs. 50K in case of senior citizen) is not crossed.
If the co-operative bank has adopted core
banking solutions (CBS), then the threshold of
Rs. 40000 (Rs. 50000 in case of senior citizen) is
in respect of interest paid by the co-operative
bank as a whole.
If CBS was not adopted, then the threshold limit
is in respect of interest paid by each branch.
S. 194A (3) (viia) (b): No TDS on interest on
savings account paid by a co-operative bank to
its resident depositors.
S. 194A (3) (v): No TDS on any interest paid by a
co-operative society to its members or another
co-operative society.
No TDS on any interest paid by a co-operative
society to non-members if the threshold of Rs.
5000 is not crossed.
S. 197A (1D): No TDS on interest paid by off-
shore banking units on borrowings made from or
deposits accepted from non-residents or
resident but not ordinarily residents.
Off-shore banking units are branches of Indian
banks established in SEZ which deals only with
SEZ units in Foreign exchange. It makes available
international funds at international rates.
8 Avoidance of TDS through form-15G If the payee of interest has PAN, his TI is not
and form-15H. likely to exceed BEL and the interest is not going
to exceed the BEL, Form-15G could be issued to
avoid TDS U/s 193 and S. 194A. [S. 197A (1A), S.
197A (1B)].
If the payee = senior citizen, has PAN and his
estimated TI is not likely to cross the BEL, Form-
15H could be issued to avoid TDS U/s 193 and S.
194A. [S. 197A (1C)].
Form-15G or form-15H is invalid if PAN is not
quoted in it. [S. 206AA (2)].

MT Educare –CA Inter Income Tax Classes– R. SOUMYANARAYANAN. FCA. GRAD CWA. Page-361
Chapter-15: TDS (Chapter XVII-B)

Applicability of provisions for deduction of tax at source under section 194A on interest on
fixed deposit made in the name of the Registrar General of Court or the depositor of the
Fund on directions of Courts [Circular No.23/2015, dated 28-12-2015]:

(a) S. 194A stipulates deduction of tax at source (TDS) on interest other than interest on
securities if the aggregate of amount of such interest credited or paid to the account of
the payee during the financial year exceeds the specified amount.
(b) In the case of UCO Bank in Writ Petition, the Delhi HC has held that the provisions of S.
194A do not apply to fixed deposits made in the name of Registrar General of the Court
on the directions of the Court during the pendency of proceedings before the Court.

In such cases, till the Court passes the appropriate orders in the matter, it is not known
who the beneficiary of the fixed deposits will be. Amount and year of receipt is also
unascertainable.
(c) The deposits kept with the bank under the orders of the Court were essentially funds in
the custody of the Court.
(d) The interest on that account, although credited in the name of Registrar General, was
also part of funds under the custody of the Court.
(e) The Registrar General is not the recipient of the income represented by interest that
accrues on the deposits made in his name. The credit of interest is not a credit to the
account of a person who is liable to be assessed to tax.
(f) The actual payee is not ascertainable. The litigant depositing the money is not to be
understood as the account holder with the Bank or is not the recipient of income
represented by the interest accruing thereon.
(g) The litigant who is asked to deposit money in the Court ceases to have control or
proprietary right over these funds.
(h) The person to whom the funds would be paid ultimately is determined by the Court
and at that stage tax would be required to be deducted at source to the credit of
recipient.
(i) However, the litigant who deposits the funds can’t be stated to be the recipient of
income.
(j) The person who is ultimately granted the funds would be determined by orders that
are passed subsequently. At that stage, undisputedly, tax would be required to be
deducted at source to the credit of the recipient.
(k) Till the Court decides, the machinery provisions for deduction of tax U/s 194A is non-
operational, since payee is not known.
(l) The CBDT has accepted the aforesaid judgment. Accordingly, it is clarified that interest
on FDRs made in the name of Registrar General of the Court or the depositor of the
fund on the directions of the Court, will not be subject to TDS till the matter is decided
by the Court. However, once the Court decides the ownership of the money lying in the
fixed deposit, the provisions of S. 194A will apply to the recipient of the income.

MT Educare –CA Inter Income Tax Classes– R. SOUMYANARAYANAN. FCA. GRAD CWA. Page-362
Chapter-15: TDS (Chapter XVII-B)

10. TDS on winnings from lotteries etc – S. 194B:

1 Payment covered Any income by way of winnings from any lottery or crossword
puzzle or card game and other game of any sort (including any
game show, an entertainment programme on TV or electronic
mode in which people compete to win prizes).
2 Payee Resident or non-resident.
3 Threshold limit Rs. 10000
4 TDS rate 30%
5 Duty of the payer Where the winnings are wholly in kind or partly in cash and
where the winnings are partly in kind but the part in cash is not sufficient to meet the
in kind. [Proviso to S. liability of deduction of tax in respect of whole of the
194B]. winnings, the person responsible for paying shall, before
releasing the winnings, ensure that tax has been paid in
respect of the winnings.
6 Consequences of not Penalty is levied for the non-compliance with the
discharging the duty requirements of Proviso below S. 194B. Levying authority is
referred to in Proviso to JCIT. Quantum of penalty = Tax not paid as per the Proviso to
S. 194B. S. 194B. [S. 271C (1) (b) (ii)]. If there is a reasonable cause for
the default, no penalty shall be levied. [S. 273B].
Rigorous imprisonment for non-compliance with the Proviso
to S. 194B: Minimum = 3 months; Maximum = 7 years. Also
fine could be levied. [S. 276B (b) (ii)].

Illustration:

Cadbury Ltd organized a cross word puzzle. Mr. X, a non-resident wins a Maruti car in this
cross word puzzle on 13.02.2019. The market price of the car is Rs. 240000.
(a) What is the procedure to be TDS rate applicable for cross word puzzle
adopted before handing over the winnings is 30%. Since, the payee is a NR, the TDS
Maruti car to Mr. X? rate shall be incremented to include education
cess. The effective TDS rate is 31.2%.
Since the winning is wholly in kind, the proper
procedure is that Mr. X shall deposit Rs. 74880
with the company and then the company can
release the car.
Alternatively, Mr. X may pay advance tax of Rs.
74880 and give an affidavit to the company
stating the advance tax paid is in respect of the
winnings from cross word puzzle. [Proviso to S.
194B].
(b) Suppose, in (a), Mr. X in addition to TDS works out to Rs. 99840. Rs. 80000 cash
car, also wins a cash prize of Rs. winnings shall not be release and it shall be
80000. What will be your answer? appropriated towards TDS. The balance Rs. 19840
shall be deposited by Mr. X for getting the car
released. [Proviso to S. 194B].

MT Educare –CA Inter Income Tax Classes– R. SOUMYANARAYANAN. FCA. GRAD CWA. Page-363
Chapter-15: TDS (Chapter XVII-B)

11. TDS on winnings from horse races – S. 194BB:

1 Payment covered Income way of horse race winnings.


2 Payer Bookmaker or person licensed by the Government for
racing in a race course or for arranging for wagering or
betting in a race course.
3 Payee Resident or non-resident.
4 Threshold limit Rs. 10000
5 TDS rate 30%
6 Time of deduction of tax At the time of payment.

12. Payments to contractors and sub-contractors – S. 194C:

1 Payments covered. [S. Any sum for carrying out any work (including supply of labour for
194C (1)]. carrying out any work) payable in pursuance of a contract.
2 Payee. [S. 194C (1)]. Resident contractor.
3 Person responsible for The CG or any SG; or
making payment. any local authority; or
[Clause (i) of any corporation established by or under a Central or State Act;
Explanation S. 194C]. or
any company; or
any co-operative society; or
Any authority dealing with housing accommodation; or
any society registered under the Societies Registration Act,
any trust; or
any university; or
any Government of a foreign State or a foreign enterprise or any
association or body established outside India; or
any firm; or
an individual or a HUF or an AOP or a BOI, if such person is liable
to audit of accounts U/s 44AB during the FY immediately
preceding the FY in which such sum is credited or paid to the
account of the contractor.
4 TDS rate. [S. 194C (1)]. Payee TDS rate
Individual or HUF 1%
Others 2%
5 Time of tax deduction. At the time of credit to the account of the contractor in the
[S. 194C (1)]. books of accounts of the payer or at the time of payment
(whichever is earlier).
6 Meaning of work. Work includes:
[Clause (iv) of (a) advertising;
Explanation to S. (b) broadcasting and telecasting including production of
194C]. programmes for such broadcasting or telecasting;
(c) carriage of goods or passengers by any mode of transport
other than by railways;
(d) catering;

MT Educare –CA Inter Income Tax Classes– R. SOUMYANARAYANAN. FCA. GRAD CWA. Page-364
Chapter-15: TDS (Chapter XVII-B)

(e) manufacturing or supplying a product according to the


requirement or specification of a customer by using
material purchased from such customer, but does not
include manufacturing or supplying a product according
to the requirement or specification of a customer by
using material purchased from a person, other than such
customer.
7 Meaning of contract. It includes sub-contract.
[Clause (iii) of
Explanation to S.
194C].
8 Threshold limit. [S. No deduction shall be made from the amount of any sum
194C (5)]. credited or paid to the contractor, if such sum does not exceed
Rs. 30000:
However, where the aggregate of the amounts of such sums
credited or paid or likely to be credited or paid during the FY
exceeds Rs. 100000, the person responsible for paying such
sums shall be liable to deduct tax at source. [Proviso to S. 194C
(5)]. [See illustration-1].
9 Sum on which tax is to Where any sum is paid or credited for manufacturing or
be deducted at source supplying a product according to the requirement or
where payments are specification of a customer by using material purchased from
made to the such customer, tax shall be deducted at source—
contractor for (i) on the invoice value excluding the value of material, if
manufacturing of such value is mentioned separately in the invoice; or
goods according to (ii) on the whole of the invoice value, if the value of material
the specifications of is not mentioned separately in the invoice.
the payer using the
materials purchased (See illustration-2).
from the payer. [S.
194C (3)].
10 No TDS on payments No individual or HUF shall be liable to deduct tax on the sum
made for personal credited or paid to the account of the contractor where such
purpose. [S. 194C (4)]. sum is credited or paid exclusively for personal purposes of such
individual or any member of HUF. [See illustration-3].
11 No TDS on payments No deduction is required to be made from the sum credited or
made to contractors paid or likely to be credited or paid during the previous year to
in transport business. the account of a contractor, during the course of the business of
[S. 194C (6)]. [See plying, hiring or leasing goods carriages, if he furnishes his PAN
illustration-4 & 5]. to the deductor.
This relaxation from the requirement to deduct tax at source
shall only be applicable to the payment in the nature of
transport charges (whether paid by a person engaged in the
business of transport or otherwise) made to a contractor, who
fulfills the following three conditions cumulatively –

MT Educare –CA Inter Income Tax Classes– R. SOUMYANARAYANAN. FCA. GRAD CWA. Page-365
Chapter-15: TDS (Chapter XVII-B)

Points requiring attention:

1 Payments to non- The deduction of income-tax at source from payments made to


resident contractors. non-resident contractors will be governed by the provisions of
S. 195 and not by the provisions of S. 194C.
2 Illustrative list of Payment made under a contract for
payments covered by construction/repair/renovation/alteration of building.
S. 194C. [CBDT Payment made under a contract for laying roads.
Circular 681]. Payment made under a contract for laying airfields.
Payment made under a contract for laying railway lines.
Payment made under a contract for erection and installation of
plant and machinery.
3 Deduction of tax at In case, the Owner/Seller of the gas sells as well as transports
source on payment the gas to the purchaser till the point of delivery, where the
of gas transportation ownership of gas to the purchaser is simultaneously
charges by purchaser transferred, the manner of raising the sale bill (whether the
of natural gas to transportation charges are embedded in the cost of gas or
seller. [CBDT Circular shown separately) does not alter the basic nature of such
9/2012]. contract which remains essentially a ‘contract for sale’ and not
a ‘works contract’ as envisaged in S. 194C.
Therefore, in such circumstances, the provisions of Chapter
XVIIB are not applicable on the component of Gas
Transportation Charges paid by the purchaser to the
Owner/Seller of the gas. Further, the use of different modes of
transportation of gas by Owner/Seller will not alter the
position.
However, transportation charges paid to a third party
transporter of gas, either by the Owner/Seller of the gas or
purchaser of the gas or any other person, shall continue to be
governed by the appropriate provisions of the Act and tax shall
be deductible at source on such payment to the third party at
the applicable rates.

MT Educare –CA Inter Income Tax Classes– R. SOUMYANARAYANAN. FCA. GRAD CWA. Page-366
Chapter-15: TDS (Chapter XVII-B)

4 Applicability of TDS
The issue under consideration is whether payments made by
provisions on
the broadcaster/telecaster to production houses for production
payments by
of content/programme are payments under a work contract
broadcasters or
liable for tax deduction at source U/s 194C or a contract for
Television Channelsprofessional or technical services liable for tax deduction at
to production houses
source U/s 194J.
for production of In this regard, the CBDT has clarified that while applying the
content or
relevant provisions of TDS on a contract for content
programme for
production, a distinction is required to be made between:
telecasting [Circular(i) a payment for production of content/programme as per
No. 04/2016, dated the specifications of the broadcaster/telecaster; and
29-2-2016]. (ii) a payment for acquisition of broadcasting/ telecasting
rights of the content already produced by the
production house.
In the first situation where the content is produced as per the
specifications provided by the broadcaster/ telecaster and the
copyright of the content/programme also gets transferred to
the telecaster/ broadcaster, such contract is covered by the
definition of the term `work` in S. 194C and, therefore, subject
to TDS U/s 194C.
However, in a case where the telecaster/broadcaster acquires
only the telecasting/ broadcasting rights of the content already
produced by the production house, there is no contract for
``carrying out any work”, as required in S. 194C (1). Therefore,
such payments are not liable for TDS U/s 194C. However,
payments of this nature may be liable for TDS under other
sections of Chapter XVII-B (i.e. S. 194J). [See illustration-7].
5 No TDS on GST Where in terms of agreement between the payer and the
component. [CBDT payee, the component of GST comprised in the amount
Circular 23/2017]. payable to a resident is indicated separately, tax shall be
deducted at source on the amount paid or payable without
including the GST component.

Illustration-1:

Case Contract Payer Payee Contract Date of Amount TDS TDS


amount payment paid rate amount
(Rs.)
1 A X Ltd Contractor- 50000 01.08.2019 Rs. 50000 1% Rs. 500
P
2 B X Ltd Contractor- 50000 01.08.2019 Rs. 20000 - -
Q 01.09.2019 Rs. 30000 - -
3 C X Ltd Contractor- 120000 01.08.2019 Rs. 30000 1% Rs. 300
R 01.10.2019 Rs. 30000 1% Rs. 300
01.12.2019 Rs. 30000 1% Rs. 300
01.02.2020 Rs. 30000 1% Rs. 300

MT Educare –CA Inter Income Tax Classes– R. SOUMYANARAYANAN. FCA. GRAD CWA. Page-367
Chapter-15: TDS (Chapter XVII-B)

4 D X Ltd Contractor- 30000 01.08.2019 Rs. 30000 - -


E S 30000 14.08.2019 Rs. 30000 - -
F 30000 01.12.2019 Rs. 30000 - -
G 30000 01.02.2020 Rs. 30000 1% Rs.
1200
(Rs.
120000
* 1%)

Illustration-2:

Blackberry Ltd gives cloth to Mr. X and asks Mr. X to stitch shirts as per the specifications
given by Blackberry Ltd. Mr. X Charges in his invoice Rs. 200 per shirt for stitching 10000
shirts and raises a bill of Rs. 20L.

(a) Discuss the liability for tax deduction at source.


(b) Suppose Blackberry Ltd sells cloth of Rs. 50L to Mr. X and asks Mr. X to stitch shirts as
per the specifications. Discuss the TDS liability in the following cases:

Case-1: Mr. X raises a bill on Blackberry Ltd as under:


Cloth Rs. 50L
Stitching charge Rs. 20L
Total Rs. 70L

Case-2: Mr. X raises consolidated bill of Rs. 70L as under:


Sale of 10000 shirts @ Rs. 700 per shirt Rs. 70L

Answer:

(a) Tax shall be deducted @ 1% on Rs. 20L.


(b) Case-1: Tax shall be deducted @ 1% on Rs. 20L.
Case-2: Tax shall be deducted @ 1% on Rs. 70L.

Illustration-3:

During the FY 2019-20, Mr. A, a leading advocate without roaring practice, makes the
following payments to the following resident contractors:

Contract Name of the Description of contract Amount paid or


contractor credited in the books
1 Mr. X Construction of residential house for his Rs. 3500000
own occupation
2 Mr. Y Catering contract in connection with Rs. 800000
daughter’s wedding

Examine the liability to deduct tax.

MT Educare –CA Inter Income Tax Classes– R. SOUMYANARAYANAN. FCA. GRAD CWA. Page-368
Chapter-15: TDS (Chapter XVII-B)

Answer:

(a) As per S. 194C (5), no individual or HUF shall be liable to deduct tax on the sum
credited or paid to the account of the contractor where such sum is credited or paid
exclusively for personal purposes of such individual or any member of HUF.
(b) In view of the provisions of S. 194C (5), Mr. A has not TDS obligation U/s 194C on the
above payments.

Illustration-4:

XYZ Ltd paid a sum of Rs. 180000 to ABT parcel service and Rs. 10L to Indian Railways
towards freight charges. Examine the TDS implications.

Answer:

1 Work includes carriage of goods by any mode of transport other than by railways.
2 Therefore, the freight payments made to railways is not subject to TDS.
3 Payments made to ABT parcel service is also not liable to TDS in view of provisions of S.
194C (6) provided it has furnished its PAN to XYZ Ltd and gives a declaration that it does
not own more than 10 good carriages.
4 However, if PAN is not furnished, tax is to be deducted at source @ 20%. [S. 206AA].

Illustration-5:

Mr. T owns 5 goods carriages from 01.04.2019 to 31.10.2019. On 01.11.2019, he purchased


6 more goods carriages. On 01.01.2020, he sold 8 goods carriages.

Mr. P makes the following payments of transportation charges to Mr. T during the FY 18-19:

Payment Date of payment Payment


1 15.04.2019 Rs. 35000
2 15.07.2019 Rs. 40000
3 15.11.2019 Rs. 20000
4 15.12.2019 Rs. 20000
5 15.02.2020 Rs. 50000

Discuss the TDS implications in the hands of Mr. P.

MT Educare –CA Inter Income Tax Classes– R. SOUMYANARAYANAN. FCA. GRAD CWA. Page-369
Chapter-15: TDS (Chapter XVII-B)

Answer:

Payment Date Amount Number TDS implications Remarks


of
vehicles
at the
time of
payment
1 15.04.2019 Rs. 5 No tax is deductible Mr. T has to furnish
35000 a declaration that he
does not own more
than 10 good
carriages during the
FY. He shall also
furnish his PAN.
2 15.07.2019 Rs. 5 No tax is deductible Mr. T has to furnish
40000 a declaration that he
does not own more
than 10 good
carriages during the
FY. He shall also
furnish his PAN.
3 15.11.2019 Rs. 11 No tax is deductible. Sum paid < Rs.
20000 30000. Aggregate
payment till date <
Rs. 100000.
4 15.12.2019 Rs. 11 Tax is deductible. Sum paid < Rs.
20000 TDS = Rs. 115000 * 1% 30000. But
= Rs. 1150. aggregate payment
It shall be deducted till date > Rs.
out of Rs. 20000. 100000.
5 15.02.2020 Rs. 3 Tax is deductible even Sum paid > Rs.
50000 though the number of 30000. Also
vehicles owned on the aggregate payment
date of payment is till date > Rs.
less than 10. 100000.
This is because at
some time during the
FY, the number of
vehicles exceeded 10.
TDS = Rs. 50000 * 1%
= Rs. 500. It shall be
deducted out of Rs.
50000.

MT Educare –CA Inter Income Tax Classes– R. SOUMYANARAYANAN. FCA. GRAD CWA. Page-370
Chapter-15: TDS (Chapter XVII-B)

Illustration-6:

Moon TV, a television channel, made payment of Rs. 50L to a production house for
production of programme for telecasting as per the specifications given by the channel. The
copyright of the programme is also transferred to Moon TV. Would such payment be liable
for tax deduction at source U/s 194C? Discuss.

Also, examine whether the provisions of tax deduction at source U/s 194C would be
attracted if the payment was made by Moon TV for acquisition of telecasting rights of the
content already produced by the production house.

Answer:

(a) In this case, since the programme is produced by the production house as per the
specifications given by Moon TV, a television channel, and the copyright is also
transferred to the television channel, the same falls within the scope of definition of
the term `work` U/s 194C. Therefore, the payment of Rs. 50L made by Moon TV to the
production house would be subject to tax deduction at source U/s 194C.
(b) If, however, the payment was made by Moon TV for acquisition of telecasting rights of
the content already produced by the production house, there is no contract for
``carrying out any work”, as required in S. 194C (1). Therefore, such payment would not
be liable for tax deduction at source U/s 194C.

13. TDS on insurance commission – S. 194D:

1 Payment covered Any income by way of remuneration or reward, whether by way of


commission or otherwise, for soliciting or procuring insurance
business (including the business relating to the continuance,
renewal or revival of policies of insurance).
2 Payee Resident insurance agent.
3 Person Insurance companies.
responsible for
making payment.
4 TDS rate 5%
5 Threshold limit Rs. 15000.
6 Time of tax
At the time credit to the account of the payee in the books of
deduction accounts of the payer or at the time of payment (whichever is
earlier).
7 Non-deduction S. 197A is amended by the Finance Act 2017 to provide that the
through Form- recipient of commission can furnish declaration U/s 197A in Form-
15G/H. 15G/H for non-deduction of tax at source.

MT Educare –CA Inter Income Tax Classes– R. SOUMYANARAYANAN. FCA. GRAD CWA. Page-371
Chapter-15: TDS (Chapter XVII-B)

14. TDS on payment in respect of life insurance policy – S. 194DA:

1 Payment covered Any sum received under a life insurance policy, including the sum
allocated by way of bonus on such policy (if not exempt U/s 10
(10D)).
2 Payee Resident
3 Payer Insurance companies
4 TDS rate 5%
5 Time of tax At the time of payment.
deduction
6 Threshold limit Rs. 1L
The obligation to deduct tax arises only when the aggregated
payment made is Rs. 1L or more.
7 Non-deduction S. 197A is amended by the Finance Act 2017 to provide that the
through Form- recipient of commission can furnish declaration U/s 197A in Form-
15G/H. 15G/H for non-deduction of tax at source.

Provisions of S. 10 (10D):

(i) Any sum received under life insurance policy including sum allocated by way of bonus
on such policy is exempt U/s 10 (10D).
(ii) However, the following are not exempt:
(a) Key man insurance policy proceeds.
(b) Life insurance policy proceeds, if the premium payable for any of the years
during the term of the policy exceeds
If the policy was issued before 01.04.2012 20% of the
actual capital
sum assured.
If the policy was issued on or after 01.04.2012 10% of the
actual capital
sum assured.
If the policy was issued on or after 01.04.2013 [In case of 15% of the
persons with disability or person with severe disability as actual capital
per S. 80U or suffering from disease or ailment as specified sum assured.
in S. 80DDB]
[However, if the sum is payable upon death of the insured, S. 10 (10D)
exemption is available].
(c) Sum received U/s 80DD (3)

MT Educare –CA Inter Income Tax Classes– R. SOUMYANARAYANAN. FCA. GRAD CWA. Page-372
Chapter-15: TDS (Chapter XVII-B)

Sum received U/s 80DD (3):

1 A resident individual may deposit an amount under a scheme framed by LIC or other
insurer or UTI and approved by CBDT, for the maintenance of disabled dependent, which
may provide for payment of annuity or lumpsum amount for the benefit of disabled
dependent, in the event of the death of the depositor.
2 For the sum so deposited, deduction is allowed to the depositor U/s 80DD.
3 However, if the disabled dependent predeceases the depositor, any sum received by the
depositor under the scheme (supra) shall be charged to tax in his hands in the PY of
receipt.

Illustration:

Examine the applicability of the provisions for tax deduction at source U/s 194DA in the
following cases –

(a) Mr. X, a resident, is due to receive Rs. 4.50L on 31.03.2020, towards maturity proceeds
of LIC policy taken on 01.04.2015, for which the sum assured is Rs. 4L and the annual
premium is Rs. 1,25,000.
(b) Mr. Y, a resident, is due to receive Rs. 2.75L on 31.3.2020 on LIC policy taken on
31.03.2012 for which the sum assured is Rs. 2.50L and the annual premium is Rs.
35,000.
(c) Mr. Z, a resident, is due to receive Rs. 95,000 on 1.10.2019 towards maturity proceeds
of LIC policy taken on 1.10.2012 for which the sum assured is Rs. 90,000 and the annual
premium was Rs. 15,000.

Answer:

(a) Since the annual premium exceeds 10% of sum assured in respect of a policy taken
after 31.03.2012 the maturity proceeds of Rs. 4.50L are not exempt U/s 10 (10D) in the
hands of Mr. X. Therefore, tax is required to be deducted @ 5% U/s 194DA on the
maturity proceeds of Rs. 4.50L payable to Mr. X.
(b) Since the annual premium is less than 20% of sum assured in respect of a policy taken
before 1.4.2012, the sum of Rs. 2.75L due to Mr. Y would be exempt U/s 10 (10D) in his
hands.
Hence, no tax is required to be deducted at source U/s 194DA on such sum payable to
Mr. Y.
(c) Even though the annual premium exceeds 10% of sum assured in respect of a policy
taken after 31.3.2012, and consequently, the maturity proceeds of Rs. 95,000 would
not be exempt U/s 10 (10D) in the hands of Mr. Z, the tax deduction provisions U/s
194DA are not attracted since the maturity proceeds are less than Rs. 1L.

MT Educare –CA Inter Income Tax Classes– R. SOUMYANARAYANAN. FCA. GRAD CWA. Page-373
Chapter-15: TDS (Chapter XVII-B)

15. Payments to non-resident sportsmen or sports association [S. 194E]:

SN Payee Payments covered TDS Time of tax


rate deduction
1 Non-resident Income by way of participation in India 20% At the time of
foreign national in any game (other than a game the credit or
sportsmen winnings wherefrom are taxable U/s payment
(including athlete) 115BB) or sport; (whichever is
Income by way of advertisement; earlier)
Income by way of contribution of
articles relating to any game or sport in
India in newspapers, magazines or
journals;
2 Non-resident Income from performance in India. 20%
foreign national
entertainer
3 Non-resident Any amount guaranteed to be paid or 20%
sports payable to such association or
association. institution in relation to any game
(other than a game the winnings
wherefrom are taxable U/s 115BB) or
sport played in India;

16. TDS on Commission etc. on the sale of lottery tickets – S. 194G:

1 Payment covered Any income by way of commission, remuneration or prize (by


whatever name called) on lottery tickets
2 Payee Lottery agent (Resident or non-resident)
3 TDS rate 5%
4 Threshold limit Rs. 15000.
5 Time of deduction At the time of credit to the account of the payee or at the time of
of tax payment, whichever is earlier.

19. TDS on commission or brokerage – S. 194H:


1 Payment covered Commission (not being one covered U/s 194D) or brokerage.
2 Payee Resident
3 Person responsible for Any person.
making payment. However, if the payer is an individual or HUF, the obligation
to deduct tax at source under this section arises, only if such
individual or HUF was liable to tax audit in the immediately
preceding FY. [Proviso-2 to S. 194H].
4 TDS rate 5%
5 Threshold limit. Rs. 15000.
[Proviso-1 to S. 194H]

MT Educare –CA Inter Income Tax Classes– R. SOUMYANARAYANAN. FCA. GRAD CWA. Page-374
Chapter-15: TDS (Chapter XVII-B)

6 Time of deduction of tax At the time of credit to the account of the payee or at the
at source. time of payment, whichever is earlier.
7 Meaning of commission "Commission or brokerage" includes any payment received
or brokerage. or receivable, directly or indirectly, by a person acting on
behalf of another person for services rendered, or for any
services in the course of buying or selling of goods, or in
relation to any transaction relating to any asset, valuable
article or thing, other than securities.
8 Non-applicability of S. This section is not applicable to professional services. For
194H for payments that S. 194J shall apply.
made for professional “Professional Services” means services rendered by a person
services. in the course of carrying on legal, medical, engineering or
architectural profession or the profession of accountancy or
technical consultancy or interior decoration or such other
profession as notified by the CBDT for the purpose of
compulsory maintenance of books of account U/s 44AA.
9 No TDS on payments There would be no requirement to deduct tax at source on
made to PCO franchises. commission or brokerage payments made by BSNL or MTNL
[Proviso-3 to S. 194H]. to their public call office (PCO) franchisees.
10 Significance of ‘other Brokerage or commission paid to stock brokers or
than securities’ in underwriters is outside the ambit of S. 194H.
Commission or
brokerage definition.

Applicability of TDS provisions on payments by television channels and publishing houses


to advertisement companies for procuring or canvassing for advertisements [Circular No.
05/2016, dated 29-2-2016]:

(a) There are two types of payments involved in the advertising business: (i) Payment by
client to the advertising agency, and (ii) Payment by advertising agency to the
television channel/newspaper company
(b) The applicability of TDS on these payments has already been dealt with in Circular No.
715, where it has been clarified in Q-1 & Q-2 that while TDS U/s 194C (as work
contract) will be applicable on the first type of payment, there will be no TDS U/s 194C
on 2nd type of payment e.g. payment by advertising agency to the media company.
(c) However, another issue has been raised in various cases as to whether the
fees/charges taken or retained by advertising companies from media companies for
canvassing/booking advertisements (typically 15% of the billing) is `commission` or
`discount` for attracting the provisions of S. 194H.
(d) The CBDT has clarified that no TDS is attracted on payments made by television
channels/newspaper companies to the advertising agency for booking or procuring of
or canvassing for advertisements.
(e) This is because the relationship between the media company and the advertisement
agency is that of a ‘Principal-to-principal’.

MT Educare –CA Inter Income Tax Classes– R. SOUMYANARAYANAN. FCA. GRAD CWA. Page-375
Chapter-15: TDS (Chapter XVII-B)

Illustration:

Mudra Adco Ltd., an advertising company, has retained a sum of Rs. 15L, towards charges
for procuring and canvassing advertisements, from payment of Rs. 1 crore due to Cloud TV,
a television channel, and remitted the balance amount of Rs. 85L to the television channel.
Would the provisions of tax deduction at source U/s 194H be attracted on the sum of Rs.
15L retained by the advertising company?

Answer:

(a) The issue of whether fees/charges taken or retained by advertising companies from
media companies for canvassing/booking advertisements (typically 15% of the billing)
is `commission` or `discount` to attract the provisions of tax deduction at source has
been clarified by the CBDT vide its Circular No.5/2016 dated 29.2.2016.
(b) The relationship between the media company and the advertising agency is that of a
`principal-to-principal` and, therefore, not liable for TDS U/s 194H. In view of the same,
the CBDT has clarified that no liability to deduct tax is attracted on payments made by
television channels to the advertising agency for booking or procuring of or canvassing
for advertisements.
(c) Accordingly, in view of the clarification given by CBDT, no tax is deductible at source on
the amount of Rs. 15L retained by Mudra Adco Ltd., the advertising company, from
payment due to Cloud TV, a television channel.

18. TDS on rent – S. 194-I:

1 Payment covered. Any income by way of rent.


2 Payee Resident
3 Person responsible for making Any person.
payment However, if the payer is an individual or HUF, the
obligation to deduct tax at source arises only if such
individual or HUF was liable to tax audit in the
immediately preceding FY. [Proviso-2 to S. 194-I].
4 Meaning of rent. [Explanation “Rent” means any payment, by whatever name called,
to S. 194-I]. under any lease, sub-lease, tenancy or any other
agreement or arrangement for the use of (either
separately or together) any,—

(a) land; or
(b) building; or
(c) machinery; or
(d) plant; or
(e) equipment; or
(f) furniture; or
(g) fittings,
whether or not any or all of the above are owned by
the payee;

MT Educare –CA Inter Income Tax Classes– R. SOUMYANARAYANAN. FCA. GRAD CWA. Page-376
Chapter-15: TDS (Chapter XVII-B)

5 Threshold limit. [Proviso-1 to Rs. 240000


S. 194-I].
6 TDS rate. 2% on rent for the use of any machinery or plant or
equipment; and
10% on rent for the use of any land or building or
furniture or fittings.
7 Time of deduction of tax At the time of credit to the account of the payee or at
the time of payment (whichever is earlier).
8 No TDS on rent payable to No deduction shall be made U/s 194-I from the rent
REIT. [Proviso-3 to S. 194-I]. credited or paid to a business trust, being a REIT, in
respect of any real estate asset owned directly by it.

Issues in S. 194-I:

(1) Applicability of TDS provisions U/s 194-I to payments made by the customers on
account of cooling charges to the cold storage owners:

1 The CBDT Circular No. 1/2008 dated 10.1.2008 provides clarification regarding
applicability of provisions of S. 194-I to payments made by the customers on account of
cooling charges to the cold storage owners.
2 The main function of the cold storage is to preserve perishable goods by means of a
mechanical process, and storage of such goods is only incidental in nature.
3 The customer is also not given any right to use any demarcated space/place or the
machinery of the cold store and thus does not become a tenant. Therefore, the
provisions of 194-I are not applicable to the cooling charges paid by the customers of the
cold storage.
4 However, since the arrangement between the customers and cold storage owners are
basically contractual in nature, the provision of S. 194-C will be applicable to the
amounts paid as cooling charges by the customers of the cold storage.

(2) No TDS on GST component – [CBDT Circular 23 / 2017 – dated 19.07.2017]:

1 S. 194-I speaks of ‘income by way of rent’.


2 GST paid by the tenant does not partake the nature of ‘income’ of the landlord.
3 The landlord only acts as a collecting agency for Government for collection of GST.
4 Therefore, TDS will be made on rental income without including GST.

(3) Clarification on applicability of TDS provisions of S. 194-I on lumpsum lease premium


paid for acquisition of long term lease [Circular No 35/2016, dated 13-10-2016]:

(a) The issue of whether or not TDS U/s 194-I is applicable on lump sum lease premium or
one-time upfront lease charges" paid by an assessee for acquiring long-term leasehold
rights for land or any other property has been examined by the CBDT.
(b) The CBDT has clarified that lump sum lease premium or one-time upfront lease
charges, which are not adjustable against periodic rent, paid or payable for acquisition
of long-term leasehold rights over land or any other property are not payments in the

MT Educare –CA Inter Income Tax Classes– R. SOUMYANARAYANAN. FCA. GRAD CWA. Page-377
Chapter-15: TDS (Chapter XVII-B)

nature of rent within the meaning of S. 194-I. Therefore, such payments are not liable
for TDS U/s 194-I.

(4) Whether landing & parking charges paid to AAI should be subjected to TDS U/s 194-I?

(a) The issue as to whether the charges fixed by the Airport Authority of India (AAI) for
landing and parking facility for the aircraft are for the “use of the land” by the airline
company came up before the SC in Japan Airlines Co. Ltd and in Singapore Airlines Ltd.
(2015) 377ITR 372.
(b) The SC observed that the charges which are fixed by the AAI for landing and take-off
services as well as for parking of aircrafts are not for the "use of the land". These
charges are for services and facilities offered in connection with the aircraft operation
at the airport which include providing of air traffic services, ground safety services,
aeronautical communication facilities, installation and maintenance of navigational
aids and meteorological services at the airport.
(c) There are various international protocols which mandate all authorities manning and
managing these airports to construct the airport of desired standards which are
stipulated in the protocols. The services which are required to be provided by these
authorities, like AAI, are aimed at passengers` safety as well as for safe landing and
parking of the aircrafts. Therefore, the services are not restricted to merely permitting
"use of the land" of airport.
(d) On the contrary, it encompasses all the facilities that are to be compulsorily offered by
the AAI in tune with the requirements of the protocol.
(e) The SC observed that the charges levied on air-traffic includes landing charges, lighting
charges, approach and aerodrome control charges, aircraft parking charges, aerobridge
charges, hangar charges, passenger service charges, cargo charges, etc. Thus, when the
airlines pay for these charges, treating such charges as charges for "use of the land"
would tantamount to adopting a totally simplistic approach which is far away from the
reality.
(f) The SC opined that the substance behind such charges has to be considered and when
the issue is viewed from this angle, keeping the larger picture in mind, it becomes very
clear that the charges are not for use of the land per se and, therefore, it cannot be
treated as "rent" within the meaning of S. 194-I.
(g) The SC, thus, concurred with the view taken by the Madras High Court in Singapore
Airlines case and overruled the view taken by the Delhi High Court in United
Airlines/Japan Airlines case.

(5) No requirement to deduct tax at source U/s 194-I on remittance of Passenger Service
Fees (PSF) by an Airline to an Airport Operator [Circular No. 21/2017, dated 12.06.2017]

(a) S. 194-I requires deduction of tax at source at specified percentage on any income
payable to a resident by way of rent. Explanation to this section defines the term “rent”
as any payment, by whatever name called, under any lease, sub-lease, tenancy or
any other agreement or arrangement for the use of any (a) land; or (b) building; or (c)
land appurtenant to a building; or (d) machinery; (e) plant; (f) equipment (g) furniture; or
(h) fitting, whether or not any or all of them are owned by the payee.
(b) On the issue of whether payment of PSF by an airline to an Airport Operator qualifies as

MT Educare –CA Inter Income Tax Classes– R. SOUMYANARAYANAN. FCA. GRAD CWA. Page-378
Chapter-15: TDS (Chapter XVII-B)

rent to attract TDS U/s 194 -I, the Bombay HC relied on the SC ruling in Japan Airlines and
Singapore Airlines case, wherein it was observed that the primary requirement for any
payment to qualify as rent is that the payment must be for the use of land and building
and mere incidental/minor/insignificant use of the same while providing other facilities
and service would not make it a payment for use of land and buildings so as to attract S.
194-I.
(c) Accordingly, the Bombay HC declined to admit the ground relating to applicability of the
provisions of S. 194 -I on PSF charges holding that no substantial question of law arises.
(d) The CBDT, accepting the view of the Bombay HC, has clarified that the provisions of S.
194-I shall not be applicable on payment of PSF by an airline to Airport Operator.

19.TDS on transfer of certain immovable property other than agricultural land [Sec. 194-
IA]
1 Payment covered Every transferee responsible for paying any sum as consideration
for transfer of immovable property other than rural agricultural
land (land, other than agricultural land, or building or part of
building)
2 Payee Resident
3 Payer Any transferee
4 TDS rate 5%
5 Threshold limit Tax is not required to be deducted at source where the total
amount of consideration for the transfer of immovable
property is less than ` 50 lakh.
6 Time of deduction of tax The deduction is to be made at the time of credit of such sum to
the account of the resident transferor or at the time of payment
of such sum to a resident transferor, whichever is earlier.
7 Non-Applicability of S. Since tax deduction at source for compulsory acquisition of
194IA immovable property is covered under section 194LA, the
provisions of section 194-IA do not get attracted in the hands of
the transferee in such cases.
8 No requirement to The provisions of S. 203A containing the requirement of
obtain TAN. obtaining TAN shall not apply to the person required to
deduct tax in accordance with S. 194-IB.
9 Meaning of consideration Consideration for transfer of immovable property include all
for transfer of immovable charges of the nature of club membership fee, car parking fee,
property electricity or water facility fee, maintenance fee, advance fee or
any other charges of similar nature, which are incidental to
transfer of the immovable property.

20. Payment of rent by certain individuals or Hindu undivided family: [S. 194-IB]:
1 Payment covered Any income by way of rent.
2 Payee Resident
3 Payer An individual or a HUF, other than those individual or HUF
who was liable to tax audit in the immediately preceding FY.
Can be resident or non-resident.
4 Effective date of this 01.06.2017
provision.

MT Educare –CA Inter Income Tax Classes– R. SOUMYANARAYANAN. FCA. GRAD CWA. Page-379
Chapter-15: TDS (Chapter XVII-B)

5 TDS rate 5%
6 Threshold limit Tax has to be deducted at source only if the amount of such
rent exceeds Rs. 50,000 for a month or part of a month
during the PY.
7 Time of deduction of tax This deduction is to be made at the time of credit of such
rent, for the last month of the PY or the last month of
tenancy, if the property is vacated during the year, as the
case may be, to the account of the payee or at the time of
payment, whichever is earlier.
In other words, deduction of tax U/s 194-IB is only one time
in a year. (i.e. Tax deduction is annually in the last month of
the PY. In case the tenant is vacating the premises during the
PY, then tax should be deducted in the month when the
tenant vacates the premises.
8 No requirement to The provisions of S. 203A containing the requirement of
obtain TAN. obtaining TAN shall not apply to the person required to
deduct tax in accordance with S. 194-IB.
9 Meaning of “Rent”. “Rent” means any payment, by whatever name called, under
any lease, sub-lease, tenancy or any other agreement or
arrangement for the use of any land or building or both.
10 Deduction not to exceed S. 206AA requires providing of PAN of the deductee to the
rent for last month. deductor, failing which tax shall be deducted at a higher rate
(i.e., higher of the rate provided in the relevant section,
rates in force and 20%).
Where the tax is required to be deducted as per the
provisions of S. 206AA, such deduction shall not exceed the
amount of rent payable for the last month of the PY or the
last month of the tenancy, as the case may be.

Points requiring attention:

1 Location of property isIt is irrelevant as to where the land and building is situated. It
immaterial. may be in India or abroad.
2 TDS only on component Tax shall be deducted only on the component of rent paid for
attributable to land and
the use of land or building. Rent attributable to any other
building. thing taken on rent (example: furniture) shall not be subjected
to TDS U/s 194-IB.
3 Purpose of usage – The land or building taken by tenant can be used for any
immaterial. purpose. (i.e. it can be commercial or residential).
4 Time limit for Tax deducted under this section shall be remitted within 30
remittance of TDS. days from the end of the month in which deduction is made.

MT Educare –CA Inter Income Tax Classes– R. SOUMYANARAYANAN. FCA. GRAD CWA. Page-380
Chapter-15: TDS (Chapter XVII-B)

21. TDS on FPS, FTS, royalty, non-compete fee, fee for exclusivity of rights and director’s
remuneration – S. 194J:

SN Payments Person responsible for Payee Threshold TDS


covered making payment limit. rate
[Proviso-1 to
S. 194J].
1 Fee for Any person (including Resident Rs. 30000 10%
professional individual and HUF liable
services to tax audit in the
immediately preceding
FY). [Proviso-2 to S. 194J].
2 Fee for Any person (including Resident Rs. 30000 2%
technical individual and HUF liable (engaged only in
services to tax audit in the the business of
immediately preceding operation of call
FY). [Proviso-2 to S. 194J]. centre)
Other residents Rs. 30000 10%
3 Royalty Any person (not being Resident Rs. 30000 10%
individual and HUF)
4 Non-compete Any person (not being Resident Rs. 30000 10%
fees individual and HUF)
5 Fee for Any person (not being Resident Rs. 30000 10%
exclusivity of individual and HUF)
rights
6 Director’s Company Resident - 10%
remuneration

Points requiring attention:

1 Director = employee Remuneration shall be subjected to TDS U/s 192 and not U/s 194J.
2 Timing of deduction of At the time of credit to the account of the payee in the books of
tax. accounts of the payer or at the time of payment, whichever is
earlier.
3 Meaning of professional “Professional services” means services rendered by a person in the
services. [Explanation to course of carrying on legal, medical, engineering or architectural
S. 194J]. profession or the profession of accountancy or technical consultancy
or interior decoration or advertising or such other profession as is
notified by the Board for the purposes of S. 44AA or of S. 194J;
4 Professions notified for 1 Film artist – (a) an actor; (b) a cameraman; (c) a director,
the purpose of S. 44AA. including an assistant director; (d) a music director, including
an assistant music director; (e) an art director, including an
assistant art director; (f) a dance director, including an
assistant dance director; (g) an editor; (h) a singer; (i) a lyricist;
(j) a story writer; (k) a screen-play writer; (l) a dialogue writer;
(m) a dress designer;
2 Company secretary

MT Educare –CA Inter Income Tax Classes– R. SOUMYANARAYANAN. FCA. GRAD CWA. Page-381
Chapter-15: TDS (Chapter XVII-B)

3 Information technology professional


4 Authorised representative
5 Professions notified for Sports person,
the purpose of S. 194J. Umpires
Referees
Coaches
Trainers
Team Physicians
Physiotherapists
Event managers
Commentators
Anchors
Sports Columnists
6 No TDS on fees received The term "profession”, as such, is of a very wide import. However,
by other professionals. the term has been defined in this section exhaustively. For the
purposes of TDS, therefore, all other professions would be outside
the scope of S. 194J. For example, this section will not apply to
professions of teaching, sculpture, painting etc. unless they are
notified.
7 No TDS if professional No individual or a HUF shall be liable to deduct tax on the sum by
fee is paid exclusively way of fees for professional services in case such sum is credited or
for personal purpose. paid exclusively for personal purposes of such individual or any
[Proviso-3 to S. 194J]. member of HUF.
8 Illustration-1 on point-7 X, a CA with roaring practice, pays Rs. 2L as legal fees to Y, an
advocate, in connection with securing divorce from his wife. X, in
view of Proviso-3 to S.194J has no obligation to deduct tax at source
U/s 194J.
9 Illustration-2 on point-7 X, a leading advocate, underwent heart surgery and in this
connection he paid Rs. 5L to Apollo Hospital. X, in view of Proviso-3
to S.194J has no obligation to deduct tax at source U/s 194J.

TPAs liable to deduct tax U/s 194J on payment to hospitals on behalf of insurance
companies: [CBDT Circular 8/ 2009]:

(a) The CBDT has, through Circular No.8/2009 dated 24.11.2009, clarified that TPAs (Third
Party Administrator`s) who are making payment on behalf of insurance companies to
hospitals for settlement of medical/insurance claims etc. under various schemes
including cashless schemes are liable to deduct tax at source U/s 194J on all such
payments to hospitals etc.
(b) This is because the services rendered by hospitals to various patients are primarily
medical services and, therefore, the provisions of S. 194J are applicable to payments
made by TPAs to hospitals etc.
(c) It is to be noted that for invoking provisions of S. 194J, there is no stipulation that the
professional services have to be necessarily rendered to the persons who makes
payment to hospital.

MT Educare –CA Inter Income Tax Classes– R. SOUMYANARAYANAN. FCA. GRAD CWA. Page-382
Chapter-15: TDS (Chapter XVII-B)

Examination question:
Siddharth Hospitals (P) Ltd, the assessee, has recently been accorded recognition by several
insurance companies to admit and treat patients on cashless hospitalization basis. Payment
to the assessee hospital will be made by Third party administrators (TPA) who will process
the claim of the patients admitted and make the payments to the various hospitals including
the assessee. All TPAs are corporate entities. The assessee wants to know whether the TPAs
are bound to deduct tax at source. Advise suitably.

Analysis:
1 The services rendered by hospitals to various patients are primarily medical services and
therefore, S. 194J is applicable on payments made by TPAs to hospitals.
2 Further, for invoking the provisions of S. 194J, there is no stipulation that the
professional services have to be necessarily rendered to the person who makes
payments to hospitals.
3 Therefore, TPAs who are making payment on behalf of insurance companies to hospitals
for settlement of medical or insurance claim etc under various schemes including
cashless schemes are liable to deduct tax at source U/s 194J on all such payments to
hospitals. [CBDT Circular 8/2009].

No TDS on GST component – CBDT Circular 23/2017:

1 Wherever in terms of the agreement or contract between the payer and the payee, the
component of 'GST on services' comprised in the amount payable to a resident is
indicated separately, tax shall be deducted at source on the amount paid or payable
without including such 'GST on services' component.
2 GST shall include Integrated Goods and Services Tax, Central Goods and Services Tax,
State Goods and Services Tax and Union Territory Goods and Services Tax.

Exemption from multi-level TDS on payments for acquisition of software U/s 194J:

(a) S. 194J prescribes TDS @ 10% on payments made to a resident by way of royalty. This
deduction is to be made by any person, not being an individual or a HUF, who is
responsible for paying to a resident any royalty.
(b) Clause (ba) of Explanation to S. 194J clarifies that for the purposes of S. 194J, the
expression 'royalty' shall have the same meaning as in Explanation 2 to S. 9 (1) (vi).
(c) Explanation 4 has been inserted by Finance Act, 2012 to S. 9 (1) (vi). It clarifies that the
transfer of all or any rights in respect of any right, property or information includes
transfer of all or any right to use computer software (including granting of a licence)
irrespective of the medium through which such right is transferred.
(d) It means that payment for acquiring software constitutes royalty for the purpose of S. 9
(1) (vi), which was earlier regarded as payment made for purchase of goods.
(e) Thus, the payments for acquisition of software have been brought within the ambit of
'royalty' for S. 9 (1) (vi) purposes and accordingly, on software payments tax has to be
deducted U/s 194J/S. 195.
(f) The software is made available to the end-users by the software developers in two
ways: (a) directly; (b) through distributors.

MT Educare –CA Inter Income Tax Classes– R. SOUMYANARAYANAN. FCA. GRAD CWA. Page-383
Chapter-15: TDS (Chapter XVII-B)

(g) In a direct arrangement between the software developer and the end-user, the
standard End-User Licence Agreement specifically prevents the end-user from resale of
acquired software and therefore, the question of multiple level of deduction of tax will
not arise in such a scenario.
(h) However, in the case of software-distributor-end-user chain, it results in multiple-level
TDS on payments made for acquisition of software.
(i) In order to remove the cascading effect on account of tax deduction at multiple levels,
a notification (Notification No.21/2012 dated 13.6.2012, effective from 1st July, 2012)
has been issued by the CG in exercise of its power U/s 197A (1F).
(j) The notification (supra) exempts TDS U/s 194J from certain software payments.
(k) Accordingly, where payment is made by the transferee for acquisition of software from
a resident-transferor, the provisions of S. 194J would not be attracted if –
(i) the software is acquired in a subsequent transfer without any modification by
the transferor;
(ii) tax has been deducted either U/s 194J or U/s 195 on payment for any previous
transfer of such software; and
(iii) the transferee obtains a declaration from the transferor that tax has been so
deducted along with the PAN of the transferor.

22. NEW SECTION 194M - Payment made by an individual or a HUF for contract work
or by way of fees for professional services or commission or brokerage

1 Applicability Section 194M, inserted with effect from 1.9.2019, provides for
and rate of deduction of tax at source @5% by an individual or a HUF
TDS
responsible for paying any sum during the financial year to any
resident –
i) for carrying out any work (including supply of labour for
carrying out any work) in pursuance of a contract; or
ii) by way of commission (not being insurance commission
referred to in section 194D) or brokerage; or
iii) by way of fees for professional services.
It may be noted that only individuals and HUFs (other than those
who are required to deduct income-tax as per the provisions of
section 194C or 194H or 194J) are required to deduct tax in respect
of the above sums payable during the financial year to a resident.
2 Time of The tax should be deducted at the time of credit of such sum or
deduction at the time of payment of such sum, whichever is earlier.
3 Threshold No tax is required to be deducted where such sum or, as the case
limit may be, aggregate amount of such sums credited or paid to a
resident during the financial year does not exceed R s . 50,00,000.

MT Educare –CA Inter Income Tax Classes– R. SOUMYANARAYANAN. FCA. GRAD CWA. Page-384
Chapter-15: TDS (Chapter XVII-B)

4 Non- An individual or a Hindu undivided family is not liable to deduct tax at


applicability source under section 194M if –
of TDS i) they are required to deduct tax at source under section
under 194C for carrying out any work (including supply of
section labour for carrying out any work) in pursuance of a
194M contract i.e., an individual or a HUF who is subject to tax
audit under section 44AB(a)/(b) in the immediately
preceding financial year and such amount is not
exclusively credited or paid for personal purposes of such
individual or HUF.
ii) they are required to deduct tax at source under Section
194H on commission (not being insurance commission
referred to in section 194D) or brokerage i.e., an
individual or a HUF whose total sales, gross receipts or
turnover from the business or profession carried on by him
exceed the monetary limits of R s . 1 crore and R s . 50 lakhs,
respectively, specified under section 44AB during the
immediately preceding financial year.
iii) they are required to deduct tax at source under section
194J on fees for professional services i.e., an individual or
a HUF whose total sales, gross receipts or turnover from
the business or profession carried on by him exceed the
monetary limits of Rs.1 crore and Rs.50 lakhs, respectively,
specified under section 44AB during the immediately
preceding financial year and such amount is not
exclusively credited or paid for personal purposes of such
individual or HUF.
5 No The provisions of section 203A containing the requirement of obtaining Tax
requirement deduction account number (TAN) shall not apply to the person required to
to obtain
deduct tax in accordance with the provisions of section 194M.
TAN

23. NEW SECTION 194N - TDS on cash withdrawal in excess of ` 1 crore - w.e.f
01/09/2019 Finance (No. 2) Act, 2019]
(With a view to encourage the cashless economy, this section is inserted)

1 Applicability Section 194N, inserted with effect from 1.9.2019, provides that every person,
and rate of being
TDS ✓ a banking company to which the Banking Regulation Act, 1949 applies
(including any bank or banking institution referred under section 51 of
that Act)
✓ a co-operative society engaged in carrying on the business of banking, or
✓ post office
who is responsible for paying, in cash, any sum or aggregate of sums exceeding
Rs 1 crore during the previous year to any person from one or more accounts
maintained by such recipient-person with it, shall deduct tax at source @2% of
sum exceeding Rs 1 crore [TDS @2% on amount in excess of R s 1 crore].

MT Educare –CA Inter Income Tax Classes– R. SOUMYANARAYANAN. FCA. GRAD CWA. Page-385
Chapter-15: TDS (Chapter XVII-B)

2 Time of This deduction is to be made at the time of payment of such sum.


deduction
3 Non- Liability to deduct tax at source under section 194N shall not be
applicability applicable to any payment made to-
of TDS ✓ the Government
under ✓ any banking company or co-operative society engaged in carrying
section on the business of banking or
194N ✓ a post-office
✓ any business correspondent of a banking company or co-operative
society engaged in carrying on the business of banking, in
accordance with the RBI guidelines
✓ any white label ATM operator of a banking company or co-
operative society engaged in carrying on the business of banking,
in accordance with the authorisation issued by the RBI under the
Payment and Settlement Systems Act, 2007
✓ such other person or class of persons notified by the Central
Government in consultation with the RBI.

4 Other Point: Tax deducted u/s 194N is not considered as deemed receipt of income

Clarification as to the applicability of Section 194N and manner of computing the


threshold limit of R s. 1 crore thereunder, where cash withdrawals have taken
place prior to 1.9.2019 [Press Release dated 30.8.2019]
The CBDT has, vide Press Release dated 30.8.2019, clarified that section 194N is to come
into effect from 1st September, 2019. Hence, any cash withdrawal prior to 1st
September, 2019 will not be subjected to the TDS under section 194N. However, since
the threshold of ` 1 crore is with respect to the previous year 2019-20, calculation of
amount of cash withdrawal for triggering deduction under section 194N shall be
counted from 1st April, 2019.
Hence, if a person has already withdrawn ` 1 crore or more in cash upto 31st August,
2019 from one or more accounts maintained with a banking company or a cooperative
bank or a post office, TDS@2% shall apply on all subsequent cash withdrawals

24. Tax Deducted is income received [Section 198]


1) All sums deducted in accordance with the foregoing provisions shall, for the
purpose of computing the income of an assessee, be deemed to be income
received.
2) However, the following tax paid or deducted would not be deemed to be
income received by the assessee for the purpose of computing the total income–
(i) the tax paid by an employer under section 192(1A) on non-monetary
perquisites provided to the employees
(ii) tax deducted under section 194N [Finance (No. 2) Act, 2019]

MT Educare –CA Inter Income Tax Classes– R. SOUMYANARAYANAN. FCA. GRAD CWA. Page-386
Chapter-15: TDS (Chapter XVII-B)

25. Credit for tax deducted at source [S. 199]:

1 Tax deducted at source and paid to the credit of the CG shall be treated as payment of
tax on behalf of the person from whose income the deduction was made;
2 Any sum referred to in S. 192 (1A) and paid to the CG, shall be treated as the tax paid on
behalf of the person in respect of whose income, such payment of tax has been made.
3 The CBDT is empowered to frame rules for the purpose of giving credit in respect of tax
deducted or tax paid under Chapter XVII.
4 The CBDT also has the power to make rules for giving credit to a person other than the
persons mentioned in (1) and (2) above. Further, the CBDT can specify the AY for which
such credit may be given.

Rule 37BA - Credit for tax deducted at source for the purposes of S. 199:
1 R. 37BA (1) provides that credit for tax deducted at source and paid to the CG shall be
given to the person to whom the payment has been made or credit has been given (i.e.,
the deductee) on the basis of information relating to deduction of tax furnished by the
deductor to the income-tax authority or the person authorized by such authority.
2 Rule 37BA (2) (i) provides that where under any provisions of the Act, the whole or any
part of the income on which tax has been deducted at source is assessable in the hands
of a person other than the deductee, credit for the whole or any part of the tax
deducted at source, as the case may be, shall be given to the other person and not to the
deductee.
3 For example, if tax is deducted at source on the interest income of minor, the credit of
TDS shall be allowed to the parent in whose name such interest shall be clubbed.
4 However, the deductee should file a declaration with the deductor and the deductor
should report the tax deduction in the name of the other person in the information
relating to deduction of tax referred to in R. 37BA (1).
5 Credit for tax deducted at source and paid to the CG shall be given for the AY for which
such income is assessable.
6 For example, if builder deducts tax at source on cash paid to the assessee in joint
development agreement on 01.01.2019 and capital gains are assessable in the year in
which completion certificate is received and completion certificate is received on
01.01.2022, the TDS will be allowed in the PY 21-22.
7 Where tax has been deducted at source and paid to the CG and the income is assessable
over a number of years, credit for tax deducted at source shall be allowed across those
years in the same proportion in which the income is assessable to tax.

Deduction of tax at source on interest income accrued to minor child, where both the
parents have deceased: [Notification no. 5/2017]:

1 It has been brought to the notice of CBDT that in cases of minors whose both the parents
have deceased, tax deductors or banks are clubbing the interest income accrued to the
minor in the hands of grandparents and issuing TDS certificates to the grandparents,
which is not in accordance with the law as the Act envisages clubbing of minor’s income
with that of the parents only and not any other relative.
2 Ideally in such situations, the income should be assessed in the hands of minor and the
ROI shall be filed by the minor through the guardian.

MT Educare –CA Inter Income Tax Classes– R. SOUMYANARAYANAN. FCA. GRAD CWA. Page-387
Chapter-15: TDS (Chapter XVII-B)

3 It is clarified that in case of minors where both the parents have deceased, the TDS on
the interest income accrued to the minor is required to be deducted and reported
against PAN of the minor child.

Deduction of tax at source on interest on deposits made under Capital Gains Accounts
Scheme, 1988 where depositor has deceased: [Notification No. 08/2017, dated
13.09.2017]:

The PDGIT (Systems) has, in exercise of the powers delegated by the CBDT U/R 31A (5), vide
this notification, specified that in case of deposits under the Capital Gains Accounts Scheme,
1988 where the depositor has deceased:
(i) TDS on the interest income accrued for and upto the period of death of the depositor is
required to be deducted and reported against PAN of the depositor, and
(ii) TDS on the interest income accrued for the period after death of the depositor is
required to be deducted and reported against PAN of the legal heir,

unless a declaration is filed U/R 37BA (2) that credit for tax deducted has to be given to
another person.

26. Duty of person deducting tax [S. 200]:

1 The persons responsible for deducting the tax at source should deposit the sum so
deducted to the credit of the CG within the prescribed time [S. 200 (1) + R. 30].
2 Further, an employer paying tax on non-monetary perquisites provided to employees in
accordance with S. 192 (1A), should deposit within the prescribed time, the tax to the
credit of the CG or as the Board directs [S. 200 (2) + R. 30].
3 For the purpose of improving the reporting of payment of TDS made through book entry
and to make existing mechanism enforceable, S. 200 (2A) has been inserted. Accordingly,
where the tax deducted or tax referred to in S. 192 (1A) has been paid without the
production of a challan, the PAO/TO/CDDO or any other person, by whatever name
called, who is responsible for crediting such sum to the credit of the CG, shall deliver or
cause to be delivered within the prescribed time a statement in the prescribed form,
verified in the prescribed manner and setting forth prescribed particulars to the
prescribed income-tax authority or the person authorised by such authority.
4 The deductor or employer, as the case may be shall file a quarterly statement of TDS to
DGIT (systems) in the prescribed form, setting forth prescribed particulars within the
prescribed time. [S. 200 (3) + R. 31A].
5 Statement of TDS U/s 192 in Form No.24Q. Statement of TDS U/s 193 to 196D in Form
No. 26Q in respect of all deductees other than a deductee being a NCNR or a FC or RNOR
in which case the relevant form would be Form No.27Q.
6 The proviso to S. 200 (3) permits the deductor or employer to deliver to the prescribed
authority a correction statement for rectification of any mistake or to add, delete or
update the information furnished in the statement delivered U/s 200 (3) in such form
and verified in such manner as may be specified by the authority.

MT Educare –CA Inter Income Tax Classes– R. SOUMYANARAYANAN. FCA. GRAD CWA. Page-388
Chapter-15: TDS (Chapter XVII-B)

Rule 30 – Prescribed time and mode of payment to Government account of TDS or tax
paid U/s 192 (1A):

SN Deductor Time-limit for remittance


1 Office of the Government (payment of tax Same day on which tax was deducted.
is without production of income-tax
challan)
2 Office of the Government (payment of tax On or before 7 days from the end of the
is accompanied by an income-tax challan) month in which the deduction is made or
income-tax is due U/s 192 (1A).
3 Deductors other than a Government office On or before 30th April.
[Where the income or amount is credited
or paid in the month of March].
4 Deductors other than a Government office On or before seven days from the end of
[In any case other than one covered in 3]. the month in which the deduction is
made or income-tax is due U/s 192 (1A).

Note:
1 In special cases, the AO may, with the prior approval of the JCIT, permit quarterly
payment of the tax deducted U/s 192/ 194A/194D/194H on or before 7th of the month
following the quarter, in respect of first three quarters in the financial year and 30th
April in respect of the quarter ending on 31st March.
2 The dates for quarterly payment would, therefore, be 7th July, 7th October, 7th January
and 30th April, for the quarters ended 30th June, 30th September, 31st December and
31st March, respectively.
3 Tax deducted U/s 194-IA and 194-IB have to be remitted within 30 days from the end of
the month of deduction. A challan-cum-statement in Form 26QB/26QC has to be
furnished within 30 days from the end of the month of deduction.

Time limit for submission of quarterly statements – R. 31A:

Date of ending of the quarter Due date


SN of the FY
1 30th June 31st July of the FY
2 30th September 31st October of the FY
3 31st December 31st January of the FY
4 31st March 31st May of the FY immediately following the FY in
which the deduction is made.

27. Correction of arithmetic mistakes and adjustment of incorrect claim during


computerized processing of TDS statements [S. 200A]:

1 At present, all statements of tax deducted at source are filed in an electronic mode,
thereby facilitating computerised processing of these statements.
Therefore, in order to process TDS statements on computer, electronic processing on the
same lines as processing of income- tax returns has been provided in S. 200A.

MT Educare –CA Inter Income Tax Classes– R. SOUMYANARAYANAN. FCA. GRAD CWA. Page-389
Chapter-15: TDS (Chapter XVII-B)

2 The following adjustments can be made during the computerized processing of


statement of tax deducted at source or a correction statement -
(i) any arithmetical error in the statement; or
(ii) an incorrect claim, if such incorrect claim is apparent from any information in the
statement.
3 The term "an incorrect claim apparent from any information in the statement" shall
mean such claim on the basis of an entry, in the statement, -
(i) of an item, which is inconsistent with another entry of the same or some other
item in such statement;
(ii) in respect of rate of deduction of tax at source, where such rate is not in
accordance with the provisions of the Act.
4 The interest, if any, has to be computed on the basis of the sums deductible as
computed in the statement;
5 The fee, if any, has to be computed in accordance with the provision of section 234E.
6 The sum payable by, or the amount of refund due to, the deductor has to be determined
after adjustment of interest and fee against the amount paid U/s 200 or S. 201 or S. 234E
and any amount paid otherwise by way of tax or interest or fee.
7 Intimation will be prepared and generated and sent to the deductor, specifying his tax
liability or the refund due, within 1 year from the end of the FY in which the statement is
filed. The refund due shall be granted to the deductor.
8 For this purpose, the CBDT is empowered to make a scheme for centralized processing of
statements of TDS to determine the tax payable by, or refund due to, the deductor.

28. TDS violation consequences:

1 Declaring deductor or Where there is non-deduction or short deduction or non-


employer as AID. [S. 201 remittance or non-discharge of obligation assumed U/s
(1)]. 192 (1A), then the deductor or employer, as the case
may be, may be declared as AID through an order passed
U/s 201 (1).
2 When deductor will not be (i) Payee = Resident or Non Resident
regarded as AID even if he (ii) Payee has filed ROI U/s 139 for the relevant AY.
has made non-deduction or (iii) Payee has included the sum received without
short-deduction? [Proviso-1 TDS while computing the RI.
to S. 201 (1)]. (iv) Tax due on RI has been paid either by way of
advance tax or SAT.
(v) The deductor has obtained a certificate in Form-
26A from a CA to the effect that the payee has
complied with the aforesaid conditions.
3 Recovery not possible when Even if the payee is a non-resident to whom payment
payee has paid tax directly. was made without TDS, the deductor cannot be
[Explanation to S. 191 and regarded as AID, if the tax had been paid directly by the
S. 201 (1)]. payee. Accordingly, nothing could be recovered from the
deductor in such cases.

MT Educare –CA Inter Income Tax Classes– R. SOUMYANARAYANAN. FCA. GRAD CWA. Page-390
Chapter-15: TDS (Chapter XVII-B)

4 Tim-limit for passing order No order U/s 201 (1), deeming a person to be an AID for
U/s 201 (1). [S. 201 (3)]. failure to deduct the whole or any part of the tax from a
person resident in India, shall be passed at any time after
the expiry of 7 years from the end of the FY in which the
payment is made or credit is given.
No time-limit is specified in case of non-remittance and
in case of non-discharge of obligation assumed U/s 192
(1A).
No time-limit is specified in case of short deduction or
non-deduction in relation to payment made to non-
resident.
5 Disallowance Vide S. 40 (a) (i), S. 40 (a) (ia) and S. 40 (a) (iii) in the
PGBP Chapter.

29. Deduction not only one mode of recovery [S. 202]:

(i) Recovery of tax through deduction at source is only one method of recovery.
(ii) The AO can use any other prescribed methods of recovery in addition to tax deducted
at source.

30. Certificate for tax deducted [S. 203]:

(i) Every person deducting tax at source shall issue a certificate to the effect that tax has
been deducted and specifies the amount so deducted, the rate at which tax has been
deducted and such other particulars as may be prescribed.
(ii) Every person, being an employer, referred to in S. 192 (1A) shall, within such period, as
may be prescribed, furnish to the person in respect of whose income such payment of
tax has been made, a certificate to the effect that tax has been paid to the CG, and
specify the amount so paid, the rate at which the tax has been paid and such other
particulars as may be prescribed.

Certificate of TDS to be furnished U/s 203 [R. 31]:

1 The certificate of deduction of tax at source to be furnished U/s 203 shall be in Form
No.16 in respect of tax deducted or paid U/s 192 and in any other case, Form No.16A.
2 Form No.16 shall be issued to the employee annually by 15th June of the FY immediately
following the FY in which the income was paid and tax deducted. Form No.16A shall be
issued quarterly within 15 days from the due date for furnishing the statement of TDS
U/R. 31A.

MT Educare –CA Inter Income Tax Classes– R. SOUMYANARAYANAN. FCA. GRAD CWA. Page-391
Chapter-16: Alternate Minimum Tax

Provisions of alternate minimum tax – Chapter XII-BA [S. 115JC – S. 115JF]:

1 Rationale behind Profit linked deductions and investment linked deductions have
this chapter not only eroded the tax base of corporate-assessees but also
eroded that of the non-corporate assessees.
Therefore, just like corporate assessees contribute a minimum tax
under an alternate route (i.e. book profit route) every year, the
non-corporate assessees shall also be made to contribute a
minimum tax every year under an alternate route. Therefore,
Chapter XII-BA was inserted.
2 Applicability of Assessee Applicability
Chapter XII-BA. [S. of Chapter
115JEE]. XII-BA
Company No
Non-corporate assessee (not claiming deductions No
U/s 10AA, Chapter VI-A part-C and S. 35AD)
Non-corporate assessee (claiming deduction U/s Yes
10AA, Chapter VI-A part-C and S. 35AD) being
firm/LLP/Local authority.
Non-corporate assessee (claiming deduction U/s No
10AA, Chapter VI-A part-C and S. 35AD) being
Individual/HUF/AOP/BOI/AJP whose Adjusted TI
≤ Rs. 20L
Non-corporate assessee (claiming deduction U/s Yes
10AA, Chapter VI-A part-C and S. 35AD) being
Individual/HUF/AOP/BOI/AJP whose Adjusted TI
> Rs. 20L
Non-corporate assessee being a co-operative No
society (claiming deduction only U/s 80P and not
under other sections of Chapter VI-A Part-C / S.
10AA / S. 35AD).
Non-corporate assessee being a co-operative Yes
society (claiming deduction U/s 80P as well as
under other sections of Chapter VI-A Part-C / S.
10AA / S. 35AD).
3 Computation of 1 TI computed in accordance with the provisions of this ****
adjusted TI. [S. Act without giving effect to Chapter-XII-BA
115JC (2)]. 2 Deduction U/s 10AA ****
3 Deduction under Chapter VI-A Part-C (not being S. ****
80P)
4 Deduction claimed if any U/s 35AD as reduced by the ****
amount of depreciation allowable in accordance with
the provisions of S. 32, as if no deduction U/s 35AD
was allowed in respects of the assets on which
deduction U/s 35AD is claimed.
5 Adjusted TI (1+2+3+4) ****

MT Educare –CA Inter Income Tax Classes– R. SOUMYANARAYANAN. FCA. GRAD CWA.Page-392
Chapter-16: Alternate Minimum Tax

4 Determination of Where the regular income tax (RIT) payable for a PY by the
route of taxation. assessee is less than the AMT payable for such PY, then the ATI
shall be deemed to be the TI of that PY and the assessee shall be
liable to pay income tax on such TI @ 18.50%.
5 Meaning of RIT It means the income-tax payable for a PY by a person on his
TI in accordance with the provisions of this Act other than
the provisions of this Chapter.
6 Meaning of AMT It means the amount of tax computed on ATI at a rate of
18.5%.
7 Computational SN RIT > AMT AMT > RIT
procedure 1 RIT **** AMT ****
2 Surcharge (if **** Surcharge (if ****
applicable) applicable)
3 RIT (including **** AMT (including ****
surcharge) [1+2]. surcharge) [1+2].
4 Marginal relief (if (****) Marginal relief (if (****)
available) available)
5 RIT (after **** AMT (after ****
marginal relief) marginal relief)
[3-4] [3-4]
6 HEC *** HEC ***
7 Final RIT [5+6] **** Final AMT [5+6]
8 Report of CA. [S. Every person to whom S. 115JC applies shall obtain a report, in
115JC (3)]. Form 29C, from an accountant, certifying that the ATI and the
AMT have been computed in accordance with the provisions of
this Chapter and furnish such report on or before the DD for
furnishing of ROI U/s 139 (1).
9 Tax credit for AMT If a person pays tax U/s 115JC for a PY, he shall be entitled to
[S. 115JD (1) & credit in respect of such tax.
(2)]. Amount of credit = (Final AMT paid – Final RIT payable).
10 No interest on No interest shall be payable on tax credit allowed U/s 115JD (1).
AMT credit. [S.
115JD (3)].
11 Carry forward of The amount of tax credit determined U/s 115JD (2) shall be C/F &
credit. [S. 115JD set off in accordance with the provisions of S. 115JD (5) & (6) but
(4)]. such C/F shall not be allowed beyond 15th AY immediately
succeeding the AY for which tax credit becomes allowable U/s
115JD (1).
12 Year and extent of If, an AY, the RIT exceeds the AMT, the tax credit shall be allowed
set off. [S. 115JD to be set off to the extent of the excess of RIT over the AMT and
(5)]. the balance of the tax credit, if any, shall be C/F.
13 Variation to AMT If the amount of RIT or the AMT is reduced or increased as a result
credit pursuant to of any order passed under this Act, the amount of tax credit
orders passed allowed under this section shall also be varied accordingly. [S.
under this Act. 115JD (6)].

MT Educare –CA Inter Income Tax Classes– R. SOUMYANARAYANAN. FCA. GRAD CWA.Page-393
Chapter-16: Alternate Minimum Tax

14 Illustration on Assessee = Mr. X


AMT credit. Particulars AY 2019-20 AY 2020-21 AY 2021-22
RIT Rs. 8L Rs. 9L Rs. 11L
AMT Rs. 12L Rs. 11L Rs. 8L
AMT credit Rs. 4L Rs. 2L -
Set off of credit - - Rs. 3L
Life of credit AY 2033-34 AY 2034-35 -
Route of taxation Adjusted TI Adjusted TI Normal TI
Tax liability Rs. 12L (AMT) Rs. 11L (AMT) Rs. 8L (AMT)
15 Tax Credit The credit for tax paid U/s 115JC shall be allowed in accordance
allowable even if with the provisions of S. 115JD, even if the assessee has not
adjusted TI does claimed any deduction U/s 10AA or S. 35AD or Chapter VI-A Part-C
not exceed Rs. 20L in any PY and the Adjusted TI of that year does not exceed Rs. 20L.
in the year of set-
off or even if profit
linked deductions
or investment
linked deductions
are not claimed in
the PY of set off.
[S. 115JEE (3)].
16 Restriction on the Where the amount of tax credit in respect of any income-tax paid
quantum of AMT in any country or specified territory outside India U/s 90 or S. 90A
credit. [Proviso to or S. 91, allowed against the AMT payable, exceeds the amount of
S. 115JD (2)]. [See the tax credit admissible against the RIT payable by the assessee,
illustration-5)]. then, while computing the amount of credit U/s 115JD (2), such
excess amount shall be ignored.
In other words, the amount of tax credit in respect of AMT shall
not be allowed to be carried forward to subsequent year to the
extent such credit relates to the difference between the amount
of foreign tax credit (FTC) allowed against AMT and FTC allowable
against the regular tax payable by the assessee.

MT Educare –CA Inter Income Tax Classes– R. SOUMYANARAYANAN. FCA. GRAD CWA.Page-394

You might also like